You are on page 1of 489

Copyrighhts © 2016 by

b the Deparrtment of Clin


nical Pharma
acology at Fa
aculty of Meedicine, Mnas
soura
Universitty, Egypt.

Previouss editions copyright © 20


015, 2014, 20
013, 2012, 2011, 2010, 2009,
2 2008, 22000 by the
Departm
ment of Cliniccal Pharmacoology at Facu cine, Mnasou
ulty of Medic ura Universitty, Egypt.

No part of this book may be reprroduced or d distributed in


n any form orr by any meaans, or stored
d in a
databasee or retrievall system, witthout the prio
or written pe
ermission of the
t copyrighhts owner,
Departm
ment of Cliniccal Pharmaco ology at Facuulty of Mediccine, Mansouura Universitty.

This is a copyrightedd work and is s protected b


by the Egypttian Intellectu
ual Property Law 82 of 2002. Use
of this w
work is subjec
ct to this law w. The Departrtment of Clin
nical Pharmaacology at MMansoura Fac culty of
Medicine e reserves alll rights in an
nd to the worrk.

2000 ‫سنة‬
‫ لس‬1456 :‫رقم اإليداع بدار الككتب‬
‫م‬
2000/9/6 ‫بتاريخ‬
Preface

C
linical training for undergraduate students often focuses on diagnostic rather than
therapeutic skills. Sometimes students are only expected to copy the prescribing
behavior of their clinical teachers, or existing standard treatment guidelines, without
explanation as to why certain treatment is chosen. Books may not be much help either.
Pharmacology reference works and formularies are drug-centered, and although clinical
textbooks and treatment guidelines are disease-centered and provide treatment
recommendations, they rarely discuss why these therapies are chosen. Different sources
may give contradictory advice.
This book in primarily intended for under graduate medical students who are about to
enter the clinical phase of their studies. It will provide step by step guidance to the process
of rational prescribing together with many illustrative examples. It teaches also skills that are
necessary throughout a clinical career. Postgraduate students and practicing doctors may
also find it a source of straightforward information.
I wish to acknowledge the ongoing efforts of my contributing authors, and we are deeply
grateful to all those who have with such good grace given us their time and energy to supply
valuable facts and opinions, they principally include:

 Prof. Hussein El-Beltagi who took over the preparation of all books since the 1st edition
in 1995 including the revision process, printing control, distribution and selling control.
 Assist. Prof. Mohamed-Hesham Daba who took over the revision process and
amendments of the last two editions.
 Assist. Prof. Abdel-Motaal Fouda who prepared the last two editions in a readable up-
to-date text to provide essential information necessary throughout the clinical career.
 Dr. Sameh Abdel-Ghany who assisted in the revision process.

Much of any merit this book may have is due to the


generosity of those named above.

Gamal M. Dahab (MD, PhD)

Professor Emeritus in Clinical Pharmacology


Mansoura Faculty of Medicine

iii
Miss
sion and
a Vis
sion

Our mission
The Clinnical Pharm macology Department
D is seeking excellence
e and leadeership in fou
ur major
core acctivities: edu
ucation, ressearch, com
mmunity serrvice, and faculty
f and staff development.
We are e connectin ng basic medical
m sc iences with clinical care
c througgh innovattive and
disciplin
ned teachin ng of clinica
al pharmaco
ology in an integrative
i manner
m

Our v
vision
The dep partment off Clinical Ph
harmacolog gy is aiming
g to be a premier acad demic model in the
field of pharmacoloogy and the erapeutics in Egypt annd Middle East
E throug h promoting use of
the bestt therapeutics and dev veloping new
wer experimmental and clinical reseearch proje
ects.

Value
es

The gu
uiding prin
nciples and beliefs ffor the dep
partment

 Excellence, creativity, innovation, fairness, honesty, transparenncy, collab


boration,
teammwork and lifelong learning
 Reccognition tha
at our stude
ent comes ffirst
 All m
members ofo our department musst see them mselves as integral to the successs of our
misssion and ouur departmeent as integ ral to their personal
p su
uccess.
 As we subscrribe to the ese values,, we shall be profes ssionals in the profes
ssion of
education.

iv
Contributers

Effat A. Haroun MD, PhD Somaya A. Mokbel MD, PhD


Prof. of Clin Pharmacology Prof. of Clin Pharmacology
Mansoura Faculty of Medicine Mansoura Faculty of Medicine

Elhamy M. El-Kholy MD, PhD Amany A. Shalaby MD, PhD


Prof. of Clin Pharmacology Prof. of Clin Pharmacology
Mansoura Faculty of Medicine Mansoura Faculty of Medicine

Gamal M. Dahab MD, PhD, MSc (Int.Med) Amal Abdel-Hamid MD, PhD
Prof. of Clin Pharmacology Prof. of Clin Pharmacology
Mansoura Faculty of Medicine Mansoura Faculty of Medicine

Farida M. El-Banna MD, PhD Essam A. Ghyati MD, PhD


Prof. of Clin Pharmacology Assist. Prof. of Clin Pharmacology
Mansoura Faculty of Medicine Mansoura Faculty of Medicine

Aly M. Gaballah MD, PhD, MSc (int.Med) Mohamed-Hesham Y. Daba MD, PhD
Prof. of Clin Pharmacology Assist. Prof. of Clin Pharmacology
Mansoura Faculty of Medicine Mansoura Faculty of Medicine

Layla T. Hanna MD, PhD Abdel-Motaal M. Fouda MD, PhD


Prof. of Clin Pharmacology Assist. Prof. of Clin Pharmacology
Mansoura Faculty of Medicine Mansoura Faculty of Medicine

Mohamed Kheriza MD, PhD, MSc (Int.Med) Vivian Boshra MD, PhD
Prof. of Clin Pharmacology Assist. Prof. of Clin Pharmacology
Mansoura Faculty of Medicine Mansoura Faculty of Medicine

Abdel-Rahman A. Yassin MD, PhD Hala A. Al-Ashri MD, PhD


Prof. of Clin Pharmacology Assist. Prof. of Clin Pharmacology
Mansoura Faculty of Medicine Mansoura Faculty of Medicine

Mohmmad A. Attia MD, PhD Nageh Rizk MD, PhD


Prof. of Clin Pharmacology Lecturer in pharmacology
Mansoura Faculty of Medicine Mansoura Faculty of Medicine

Mohamed Abdel-Ghani MD, PhD Elsayed A. Hassan MD, PhD


Prof. of Clin Pharmacology Lecturer in Clin Pharmacology
Mansoura Faculty of Medicine Mansoura Faculty of Medicine

Hussien M. El-Beltagi MD, PhD Mohamed Abdel-Monem MD, PhD


Prof. of Clin Pharmacology Lecturer in Clin Pharmacology
Mansoura Faculty of Medicine Mansoura Faculty of Medicine

Karawan M. Abdel-Rahman MD, PhD Mahmoud A. Naga MD, PhD


Prof. of Clin Pharmacology Lecturer in Clin Pharmacology
Mansoura Faculty of Medicine Mansoura Faculty of Medicine

v
Ahmad Hassan MD, PhD Mohamed Abou El-khair MD, PhD
Lecturer in Clin Pharmacology Lecturer in Clin Pharmacology
Mansoura Faculty of Medicine Mansoura Faculty of Medicine

Ahlam El-masry MD, PhD Sameh A. Abdel-Ghani MSc.


Lecturer in Clin Pharmacology Assist. Lecturer in Clin Pharmacology
Mansoura Faculty of Medicine Mansoura Faculty of Medicine

Rehab Hamdy MD, PhD


Lecturer in Clin Pharmacology
Mansoura Faculty of Medicine

vi
Table of Contents

CHAPTER 1: GENERAL PRINCIPLES

Part 1: Pharmacodynamics 1
Receptors 2
Ion channels 7
Enzymes 8
Carrier molecules 8
Part 2: Factors affecting the dose-response relationship 8
Factors related to the drug 8
Factors related to the patient 10
Part 3: Clinical pharmacokinetics 13
Absorption of drugs 13
Distribution of drugs 15
Elimination of drugs 16
Metabolism of drugs 20
Part 4: Adverse drug reactions 23
Drug induced liver injury 24
ADR on pregnancy 25
Part 5: Principles of drug interactions 26
Pharmacokinetic interactions 26
Pharmacodynamic interactions 28
Review questions 30

CHAPTER 2: AUTONOMIC PHARMACOLOGY

Part 1: Basic information 39


Part 2: Adrenergic agonists 46
Direct acting sympathomimetic drugs 46
Indirect acting sympathomimetic drugs 51
Mixed acting sympathomimetic drugs 52
Part 3: Adrenergic receptor antagonists 53
Alpha adrenergic blockers 53
Beta adrenergic blockers 58
Part 4: Sympathoplegic drugs 62
Centrally acting sympathoplegic drugs 62
Adrenergic neuron blockers 63
Part 5: Parasympathomimetic drugs 64
Direct acting parasympathomimetics 64
Indirect acting parasympathomimetics 67

vii
Part 6: Muscarinic antagonists 71
Part 7: Ganglion blocking drugs 74
Part 8: Neuromuscular blockers 74
Review questions 78

CHAPTER 3: DIURETIC AGENTS AND VOLUME BALANCE

Part 1: Basic information 85


Part 2: Diuretic classes and agents 87
Loop diuretics 88
Thiazide diuretics 89
Potassium sparing diuretics 90
Osmotic diuretics 92
Part 3: Advantages and disadvantages of diuretics in some edematous 94
conditions
Congestive heart failure 94
Chronic kidney disease 94
Liver cirrhosis 95
Lower limb edema due to pregnancy 95
Part 4: Volume depletion and fluid replacement 96
Part 5: Disorders of serum sodium and potassium 97
Part 6: Manipulation of the urine pH 100
Review questions 102

viii
█ Intro
oductory definitions

Medica al pharma acology is a basic c science.. It the science


s deealing with
h small
molecu ules used tot prevent, diagnose , or treat diseases.
d
Clinicaal pharma acology is s the scieence concerned with the raational, sa afe and
effectivve use of drugs
d in hu
umans. It ccombines elements of o basic phharmacoloogy with
clinical medicine e; in other words, it involves the
t complex interacction betweeen the
drug and the pattient.
A drug g is any chemical molecule that can interact with w bodyy systems at the
molecu ular level and produc ce effect.

The drrug-body interaction


ns

Part 1
1: Ph
harmaco
odynamiics (Mec
chanism of drug a
action)

Pharmaacodynam mics is summmarized a


as what a drug does to the b
body; a drug may
produc
ce its effectts through:
■ Interraction witth body co
ontrol systtems (regulatory prote
eins):
(a
a) Receptorrs (b) Ion
n channelss
(c
c) Enzymess (d) Caarrier molec
cules

■ Direc
ct chemica
al or physic
cal mecha
anisms.
■ Interraction with certain metabolic
m p
pathways.

1
A. REC
CEPTORS
S

Receptors: they are protein macrom molecules. When


W they
y combine with a dru ug, they
may be e activated d or blocke
ed.
Ligand d: is any molecule tha at can com
mbine with the recepptors. A ligaand that ac
ctivates
the rec ceptor is called agonist.
a A ligand that
t block
ks the reeceptor is called
antago onist.
y: it is the empathy of
Affinity o the rece
eptor to the ligand. It determinnes the nummber of
receptoors occupied by the drug.

█ Typ
pes of rec
ceptors

■ Ion channel--linked re
eceptors (direct
ligand-gated
d ion channnels):
- The receptor is an ion
n channell consists
s of 5
transsmembran ne subunits
s (α1, α2, β,, γ, δ).
- Bindding of thee agonist to
t the extrracellular part
p of
the rreceptor causes
c opeening of th he channe el for a
speccific ion.
- The response of these re eceptors iss very fast and their duration
d is very shortt.

- Exam
mples:
 N
Nicotinic Ach e ion channnel opens for Na+
d-plate: the
A recepttors in the motor end
ions in ressponse to stimulation
s n by Ach.
 The Gama a aminobuteric acid (GABA) re n the brainn: the ion channel
eceptors in c
-
opens for Cl ions in response to stimulation by GAABA.

■ G-protein-lin
nked rece
eptors:
- The receptor co
onsists of 7 membranne subunits.
- Bind
ding of the
e agonist too the extra
acellular part
of the receptor ca auses ac ctivation of
acellular G--protein.
intra
- Wheen the G-p protein is activated,
a unit
its α subu
bindds to GTP to be pho osphorylateed and bring
stimulatory or inhibitory response.
- Their responsse is slow wer than ion chann nel
receeptors but their
t duration is long
ger.

- Stim
mulatory G-protein
G (Gs) leadss to increa
ase
enyl cyclase enzyme → ↑ cAMP
ade P → activation
of specific proteins s (proteinn kinase es).
Exa
amples of Gs-couple
G ed receptorrs are the β1
and
d β2-adrenergic receptors.

2
- Inhibitory G-protein (G
Gi) leads to
o decrease yclase enzzyme → ↓ cAMP
e adenyl cy c →
inhibition of protein kinases. Exxamples of
o Gi-coup pled recep
ptors are the
t α2-
adreenergic rec
ceptors an
nd M2 musscarinic rec
ceptors.

- Gq--coupled receptorrs: they increase inositol triphosp phate (IP3 3) and


2+
diac
cylglyceroll (DAG). IP3
3 increasess free intrac
cellular Ca . Exampl es of Gq-ccoupled
rece
eptors are the α1-ad drenergic reeceptors, M1M and M3 3 muscarinnic recepto
ors.

■ Tyro
osine kinase (TK)--linked re
eceptors::
- The receptor consists of 2 largee domainss: an
extraacellular hormone-b
h binding doomain and d an
intra
acellular TKK-binding domain c onnected by a
transsmembran ne segment.
- Bindding of the e agonist to the hoormone-binnding
dommain cause es activattion of th
he intrace
ellular
dommain to ac ctivate TK enzyme → activatio on of
seveeral protein
ns known as
a “signalin
ng proteinss”.

- Exam
mples: insu
ulin recepttors.

■ Intra
acellular recepto
ors:
- Theyy are located inside the
t cell eith her in the cytoplasm
c or directlyy on the DNA.
D
- Theyy regulate transcripti
t on of genees in the nuucleus or the mitoch ondria.
- Their agonist must
m enter inside the
e cell to reaach them.
- Theyy have twoo importantt features:
 Their response is slo ow (time iss required for
f synthes sis of new proteins).
 Their effeccts persist for long tiime after the agonistt is removeed.
- Exam
mples: receptors for corticoste
eroids, sexx hormone
es, thyroxinn, etc.

Types
s of drug
g-recepto
or bonds
s

■ The ionic bon nd:


It is an electric
cal attraction betwee
en two opp
posing cha
arges. It
is strong but reeversible.

■ The hydrogen n bond:


It is an attraction betwe
een two h
hydrogen bonds.
b It is weak
and reversible.

■ The covalent bond:


Veryy strong an
nd irreversible.
If oc
ccurred bettween drugg and rece
eptor, the receptor
r be
ecomes
permmanently blocked.
b

3
█ BIOL
LOGICAL RESPONS
SE TO DRU
UG-RECE
EPTOR BIN
NDING
(Dose
e-response relatiionship s
studies)

When a drug com


mbines with a recepto
or, this ma
ay lead to one
o of the following:
■ Agoonist effect: meanss that the drug com mbines with the recceptor andd gives
resp
ponse.
■ Anttagonist effect:
e mea
ans that thhe drug combines with
w the reeceptor bu ut gives
NO response e, and prev
vents the re
eceptor fro
om binding
g to anotheer drug.

▌Agon
nist effe
ect

Accord
ding to the “dose-rresponse relations es”, theree are 2 ty
ship curve ypes of
responses to drugs:

Graded respon
nse Qu
uantal res ponse

- The response is increase


ed - The
T respon
nse does nnot increas
se
prop portionally y to the do
ose of the proportiona
p ally to the agonist bu
ut it is
agon nist e.g. the response e of the he
eart all-or-none
a e responsee e.g. prevvention
to addrenaline. of
o convulsions by anttiepileptic drugs
- It is tthe respon
nse to mos st drugs. - Itt is responnse to few drugs.
- The response could be tested in on ne - The
T respon nse could nnot be testted in
or mmore anima als. one
o animal and mustt be tested d in a
group
g of animals.

Effecttiveness
s and saffety

■ Effic
cacy
- It is tthe ability of a drug to
t produce
e response
e (effect) affter binding
g to the receptor.
- It is measured d by the Emax (the m
maximal re
esponse that a drug g can eliciit at full
conc
centration)):

4
■ Full agonist is the d drug that gives maximal
m rresponse at full
conceentration (aat full occu
upancy).
■ al agonistt is that ag
Partia gonist givees submax
ximal resp
ponse even
n at full
conceentration i..e. never g ives Emax

■ Pote
ency
- ED5 ve Dose) is the dose
50 (Effectiv e of the dru
ug that giv
ves 50% o
of the Emaxx, or it is
the d
dose that gives
g the desired
d effe
ect in 50%
% of a test populationn of subjec
cts.
- A drug that givves ED50 byb smaller doses is described
d as
a “potentt” drug.
- Poteency of dru
ugs is gene erally less clinically importantt than efficcacy becauuse you
can increase the dose of o a less po otent drug to obtain the effect of a moree potent
one (provided that it is not toxic).

■ Safe
ety
- TD5
50 (Toxic Dose)
D is th
he dose o f the drug needed to cause a harmful effect
e in
50%
% of a test population
n of subjec
cts.

- LD5
50 (Lethal Dose) is th
he dose ne
eeded to cause
c deatth in 50% of a test group
g of
anim
mals. It is experiment
e al term tha
at can be determined
d d in animalls.

5
- Therrapeutic in
ndex (TI) LD50/ED5
L 50:
- It is the ratio between
b th
he LD50 an
nd the ED5
50. It is a measure
m off safety; if there is
a larrge differe
ence betweeen the do ose of a drug
d that produces
p tthe desired
d effect
and the dose that
t producces a toxic
c effect, it is said tha
at the drug has a large TI.
- Druggs with higgh TI are more
m safe ffor clinical use, and vice
v versa (e.g. warfa
arin has
arrow TI and requires careful th erapeutic monitoring
a na g).

▌Anta
agonist effect
e

 Anta
agonist is the ligand
d that com
mbines with
h the recep
ptor and d
does not activate
a
it. Itt has no intrinsic activity, bu ut may cause a pha armacolog gical respoonse by
inhibbiting the actions
a of endogenou
e us substan nces or othher drugs.
 If thee antagoniist binds to
o the same e site of thhe agonist on the recceptor, it is
s called
com mpetitive antagonist
a t. If the an
ntagonist binds
b to an
nother sitee on the re
eceptor,
and prevented d the action
n of the aggonist, it is called nonn-competiitive antag gonist.
 Com m may be reversible
mpetitive antagonism
a e or irreve
ersible:
 R e antagonist makess weak bo
Reversible ond with th
he recepto
or so as you
y can
o
overcome the
t block byb giving h high doses
s of the ago
onist, and even you can get
th
he maximaal response in preseence of the
e antagonis mountable effect).
st (i.e. surm
T
The duratio
on of block
k is short b
because the antagonist can be easily wasshed off
th
he recepto
ors.
 Irrreversible nist make s covalen
e antagon nt bond with the recceptor so as you
ccannot oveercome the block o r get the maximal responser b
by increas
sing the
ddose of the
e agonist (ii.e. non-su
urmountable effect). TheT occup
pied recepttors are
ppermanently blocked d, so the d duration off block is long, and the bodyy has to
ssynthesize new recepptors to reg gain the orriginal state
e.

6
Other types of drug anttagonism

■ Che
emical anttagonism: e.g. one acidic dru
ug when added
a to a basic drrug can
causse precipita
ation of ea
ach other’ss
Exammple: the addition of
o gentamyycin (basic drug) to carpenicilli
c in (acidic drug)
d in
the ssame syrin
nge causes
s chemical complex.

■ Physsical antaagonism: antagonissm betwee


en two
drug
gs carrying opposite charges.
c
Exammple: pro otamine is
s used ffor treatm ment of
hepa dose because protam
arin overd mine carries +ve
charrge while heparin
h ca
arries –ve charge. One
O mg
of prrotamine can
c neutrallize 100 un
nits of hepa
arin.

■ Phys siological antagonism: antaggonism be etween


two drugs producing opposite effe
ects by acttivation
of diifferent rec
ceptors.
Exammple: adre
enaline is the physio ological an
ntagonist of
o histaminne becausse while
hista
amine cauuses hypo otension a and bronchoconstric ction throuugh activa
ation of
hista
amine H1 receptors,, adrenalin ne causes hypertens sion and bronchodiilatation
throu ation of adrrenergic α & β recepttors respec
ugh activa ctively.

■ Pharmacokinetic antag gonism: (seee drug intteractions)).


- One drug mayy prevent absorptio
a on of anoth her drug e.g. antacidds ↓ absorption of
iron & aspirin.
- One drug maay increas se metabo olism of another
a drrug e.g. ri fampicin induces
i
hepaatic enzymmes and ↑ metabolism
m m of oral co
ontraceptivve pills.
- One drug mayy ↑ excretion of anoother drug e.g. NaHC CO3 causee alkalinizaation of
e and ↑ exc
urine cretion of acidic
a drug
gs like asppirin.

B. ION
N CHANNE
ELS

How drugs could modulatte ion cha annels?


 Phyysical block: e.g. blocking of Na+
channels by lo ocal anesth
hetics.
 Thee ion chaannel mayy be parrt of the
receeptor e.g. ion
i channe el-linked re
eceptors.
 Thee ion chann nel may be modulatted by G-
prottein linked receptors.
 Ion channels may be modulated
m by intracellular ATPP e.g. ATPPase senssitive K+
channels in the pancre eatic β cellls, rise off intracellu
ular ATP ccauses clo
osure of
+
pancreatic K channels.

7
C. ENZ
ZYMES

How drugs could affect enzymes?


 Thee drug mayy act as a competitivve inhibito or of the enzyme e.g
g. neostigmmine on
cholinesterasee enzyme.
 Thee drug mayy act as irrreversible inhibitor of
o the enzy yme e.g. o
organophos sphates
on ccholinesterrase enzym
me.
 Thee drug mayy act as a fa
alse subst rate for the
e enzyme e.g.
e α-metthyldopa is s a false
substrate for dopa
d arboxylase..
deca
 Thee drug mayy induce orr inhibit hep
patic microosomal enzymes acttivity (see later).

D. CARRIER MO
OLECULES

 Theese are sm
mall protein
n molecule c moleculees across the cell
es that carrry organic
memmbrane whhen they arre too large
e or too po
olar.
 Drug
gs could affect
a carrie
er molecul es by bloc
cking their recognitio
on site.

Part 2
2: Fac
ctors afffecting dose-re
esponse relation
nship

A. FAC
CTORS RE
ELATED TO
O THE DR
RUG

1. Drug
g shape (s
stereoisom
merism):
- Mosst drugss have
multtiple stere eoisomers
(enaantiomers) (e.g. L-
thyrooxin an
nd D-
thyrooxin). The receptor
site is usually sensitive
for one sterreoisomer
and not suittable for
anotther, like the hand
and the glo ove. This
mea ans that on ne isomer
mayy be hundrred times
more e potent than the
otheer. In other instances one isome er is benefficial while the other is toxic.
- This phenomeenon may explain hhow a sin ngle drug could actt as agon nist and
antaagonist (i.ee. partial agonist)
a b ecause many
m drugss are pressent in “ra acemic
mixttures” rath her than asa pure isoomers; or how one isomer is effective and a the
otheer isomer iss toxic.

8
2. Molecular weight (MW):
- Most drugs have MW between 100-1000 Da. Drug particles larger than MW 1000
Da cannot be absorbed or distributed. They should be given parenterally.
- Drug particles larger than MW 1000 Da cannot cross placental barrier.

3. Time of drug administration (Chronopharmacology):


- Many body functions (e.g. liver metabolism, RBF, blood pressure, HR, gastric
emptying time, etc.) have daily circadian rhythm. Some enzymes responsible for
metabolism of drugs are active in the morning or evening.
- Also many diseases (e.g. asthma attacks, myocardial infarction, etc.) are circadian
phase dependent.
- Chronopharmacology is the science dealing with tailoring drug medication
according to the circadian rhythm of the body to get better response and/or to
avoid possible side effects.
- Examples:
- Episodes of acute bronchial asthma are common at night due to circadian
variation of cortisol and other inflammatory mediators, so it is better to give the
anti-asthmatic medications in the evening.
- Blood pressure is at its peak during afternoon, so it is better to give the
antihypertensive medications at morning.

4. Drug cumulation:
Cumulation occurs when the rate of drug administration exceeds the rate of its
elimination (especially in patients with liver or renal disease). Some drugs are
cumulative due to their slow rate of elimination e.g. digoxin.

5. Drug combination:
Drug combination is very common in clinical practice. When two or more drugs
are combined together, one of the following may occur:

a) Summation or addition:
- Summation means that the combined effect of two drugs is equal to the sum
of their individual effects (i.e. 1+1=2). It usually occurs between drugs having
the same mechanism, for example, the use of two simple analgesics together.

b) Synergism and potentiation:


- Synergism means that the combined effect of two drugs is greater than the
sum of their individual effects (i.e. 1+1=3). The two drugs usually have
different mechanisms of action, for example, the use of penicillin with
aminoglycosides to exert bactericidal effect.

9
- Potentiation is similar to synergism but, in potentiation, the effect of one drug
itself is greatly increased by intake of another drug without notable effect (i.e.
1+0=2), for example, Phenobarbitone has no analgesic action but it can
potentiate the analgesic action of aspirin.

c) Antagonism:
One drug abolishes the effect of the other i.e. 1+1=0 (see before).

B. FACTORS RELATED TO THE PATIENT

1. Age, sex, and weight.

2. Pathological status:
Liver or kidney diseases significantly alter the response to drugs due to altered
metabolism. Also the failing heart is more sensitive to digitalis than the normal heart.

3. Pharmacogenetic factors (idiosyncrasy):


It is abnormal response to drugs due to genetic abnormality in drug metabolism.
These are some examples:

▌Examples of heritable conditions causing EXAGGERATED drug response:

a) Pseudocholinestrase deficiency:
Succinylcholine is a neuromuscular blocker metabolized by pseudo-
cholinestrase enzyme. Some individuals with deficient PsChE, when they take
succinylcholine, severe muscle paralysis occurs due to lack of succinylcholine
metabolism, and may lead to death from respiratory paralysis (succinylcholine
apnea).

b) Glucose-6-phosphate dehydrogenase (G6PD) deficiency:


- G6PD is the most common human enzyme defect. G6PD enzyme catalyzes
the reduction of NADP+ into NADPH which maintains glutathione in the RBCs
in its reduced form. Reduced glutathione keeps Hb in the reduced (ferrous)
form and prevent formation of methemoglobin and cell membrane injury
(hemolysis) by oxidizing drugs.
- Individuals with deficiency of G6PD may suffer acute hemolysis if they are
exposed to oxidizing drugs e.g. nitrates, antimalarial drugs, and others.

c) Thiopurine methyltransferase (TPMT) deficiency:


- Thiopurine methyltransferase (TPMT) is an enzyme that methylates thiopurine
anticancer drugs (e.g. 6-mercaptopurine and 6-thioguanine) into less toxic
compounds.

10
- G eficiency in TPMT lea
Genetic de ads to incre
eased conversion of parent thiopurine
d
drugs into more toxic compou unds, leadiing to seveere myelottoxicity an
nd bone
m
marrow supppression which mayy be fatal.
- T
TPMT defiiciency prrevalence is 1:300. Screening for TPM MT deficieency is
n
necessary in patients
s treated byy thiopurin
ne anticanc
cer drugs.

d) Acetylator phenotyp
p es:
M
Many drugs are me etabolized in the liver by ac cetylation (e.g. isoniazid).
Ac
cetylation reaction is under g genetic co ontrol and
d people ccan be classified
ac
ccording to
o their rate
e of acetyla
ation into rapid
r and slow acetyylators:
- In
n rapid ac
cetylators: excessive
e isoniazid toxic mettabolites aaccumulate
e in the
liver causin
ng hepatoc cellular nec
crosis.
- In
n slow acetylators s: isoniazzid accum mulates in
p
peripheral tissues causing
c peeripheral neuropathy
d
due to inteerference with
w pyrido oxine metaabolism, (so
p
pyridoxine “vit B6” iss added to o isoniazid therapy to
p
prevent neu urotoxicity
y).
- S
Some drug gs that are metabolize ed by acettylation can
c
cause systtemic lupus erythem matosis-like
e syndrome
(S
SLE) in slo
ow acetylattors (see bbox).

▌Exa
amples off heritable
e condition
ns causing
g DECREA
ASED drug
g respons
se:

a) Resistance
e to couma
arin (warfa
arin) antic
coagulants
s:
- In
n normal individuals, warfarin anticoagu
ulant acts by
b inhibitinng the enzzyme vit
K epoxide reductase
r ble for redu
responsib uction of th
he oxidized
d vit K (inac
ctive) to
itts reduced
d form (active).
- S
Some indivviduals havve another variant of this enzymme making g them nee eding 20
times the usual
u dose of coumarrin to get the response.

b) R
Resistance
e to vit D (v
vit D-resisstant ricke
ets):
C
Children witth vit D-res
sistant rickkets need huge
h doses
s of vit D to
o be treate
ed.

c) Re
esistance
e to mydria
atics:
Dark eyes are
a genetically less re
esponsive to the effect of mydrriatics.

4. Hypo
oreactivity
y to drugs
s:
(Tolera
ance; tach
hyphylaxiss; drug res
sistance)

Tolerance mean ns progressive decre ease in drug respon nse with suuccessive admin-
istration. The sam
me respons se could b
be obtained d by highe
er doses. Itt occurs ovver long
period.. Tachyphy ylaxis is an acute typ
pe of tolerance that occurs
o verry rapidly.

11
Mechanism of tolerance:
 Pharmacodynamic tolerance: may occur due to:
 Receptor desensitization: prolonged exposure to the drug leads to slow
conformational changes in the receptors by which the receptor shape
becomes no longer fitted well with the drug.
 Receptor down-regulation: prolonged exposure to the drug leads to decrease
number of the functional receptors.
 Exhaustion of mediators: e.g. depletion of catecholamines by amphetamine.

 Pharmacokinetic tolerance:
 Due to ↑ metabolic degradation of a drug by induction of hepatic enzymes
e.g. with chronic administration of ethanol.

 Behavioral tolerance:
 It occurs by a drug independent learning of the brain how to actively
overcome a certain drug-induced effect through practice e.g. with
psychoactive drugs.

5. Hyperreactivity to drugs:
(Rebound and withdrawal effect)

Rebound effect: is recurring of symptoms in exaggerated form when a drug is


suddenly stopped after a long period of administration.

Mechanism: prolonged administration of the antagonist leads to up-regulation


(increase number) of receptors. When the antagonist is suddenly stopped, severe
reaction occurs e.g. severe tachycardia and arrhythmia occurs after sudden stopping
of beta-blockers.

Withdrawal effect (syndrome) is recurring of symptoms in exaggerated form plus


addition of new symptoms when a drug is suddenly stopped e.g. withdrawal effects
that occur after sudden stopping of opioids in opioid addicts.

N.B. Some examples of drugs should not be stopped suddenly:

Drug Sudden withdrawal can lead to:


Beta-blockers : Severe tachycardia, arrhythmia, and even myocardial infarction.
Clonidine : Severe hypertension (hypertensive crisis).
Cimetidine : Severe hyperacidity and even peptic ulceration.
Corticosteroids : Acute Addisonian crisis.
Morphine : Withdrawal symptoms (see CNS).
Warfarin : Thrombotic catastrophes

12
Part 3: Cllinical pharmac
cokinetic
cs

Definittion: it is th
he journey of the dru g inside th
he body. It includes 4 processe
es:

Abssorption ution
Distribu m
Metabolism
M Excreetion

█ ABS
SORPTION
N OF DRUG
GS

Definittion: it is th
he passage
e of drug f rom the sitte of administration tto the plassma.
The ma ain routes s of administration: o
oral, sublin
ngual, recta
al, inhalatio
on, injection, etc.

Factorrs affectiing drug absorptio


a on:

A. Facttors relate
ed to the drug
d

- Mollecular sizee: small mo olecules arre absorbe


ed than large molecuules.
- Dosse: absorpttion increa ases with in
ncreasing the dose (up to limit)).
- Drug formulattions: e.g. sustained--release ta ablets are slow
s in abssorption.
- Loccal effects of the drugg: e.g. druggs producing VC ↓ thheir own ab bsorption.
- Drug combina v C ↑ abssorption off iron.
ation: e.g. vit
- Lipid solubilityy, drug ion
nization, an
nd the pKaa of the dru
ug.

B. Facttors relate
ed to the absorbing
a g surface:

- Rouute of administration: i.v. route is the fastest while rectal


r is thee slowest.
- Inte
egrity of the
e absorbinng surface: may ↑ or ↓ absorptio on.
- Loccal blood flow: ischemmia ↓ abso orption.
- Speecific facto
ors: e.g. ap
poferritin syystem for iron, etc.

The pK
Ka and drug ioniz
zation

Princip
ples
- Ionized (polarr; charged d) drugs a are poorly absorbed d,
while unionized (non-p polar, non--charged) drugs are e
more absorbe ed.
- Mosst drugs are a weak acids or b bases. Theey become e
ionizzed or non--ionized ac
ccording to the pH aroound them.
- Acid dic drugs (e.g.
( aspirin) are morre ionized in alkaline pH and vicce versa.
- Bassic drugs (ee.g. amphe etamine) a re more ionized in ac cidic pH annd vice verrsa.
- pKa
a of a drug
g: is the pH at whic h 50% of the drug is ionized and 50% is non-
ioniized. (W
Where p = inverse log
g; Ka = association/d
dissociatio
on constant).

13
Examp
ple of pH variation
v and
a drug k
kinetics with
w aspirin
n:

Aspirin is an acid
dic drug; its
s pKa = 3.5
5
The pHH of the sto
omach is 1.5 Th e pH of the intestine
e is 8.5

►Wheen aspirin is put in th he stomac ch:


 Asppirin is acid
dic drug annd become es more abbsorbable in acidic p
pH.
 Logg (Unionize ed /Ionized)* = pKa – pH = 3.5 – 1.5 = 2 (log
( 2 =1022 ).
 Thiss means th hat the rattio of union
nized: ionized = 100/1 (or accuurately 0.9
99 parts
are absorbed and 0.01 parts are n non-absorbbed).

►Wheen aspirin is put in thhe intestin


ne:
 Asp
pirin is acid
dic drug annd become es less abssorbable in
n alkaline p
pH.
 Log
g (Unionize ed/Ionized)* = pKa – ppH = 3.5 – 8.5 = – 5 (log -5 =1 0–5 ).
 Thiss means thhat the ratio of union
nized/ionize
ed = 1/1000000 (or acccurately 0.00001
0
partts are absoorbed and 0.99999 p parts are non-absorb bed).

The above rule applies only to a


*N.B. T acidic drugs
s like aspirrin. For bassic drugs, the
t ratio
would b
be reversed
d.

►Ion ttrapping of
o aspirin:
In the stomach, aspirin is more abssorbable in nto
stomacch cells buut once entered the cells, the pH
changees from 1.5 outside to 7.4 insside the cell.
So asppirin becom mes ionize
ed inside tthe cells and
a
can’t diffuse outsside them again
a → ga
astric ulcer.

al significa
Clinica ance of pK
Ka
 Know
wing the site of drug
g absorptio
on from the
e GIT (see principles)).
 Treaatment of drug
d toxicitty:
- Tooxicity witth acidic drugs
d (e.g . aspirin) could be treated byy alkaliniza
ation of
urrine, which
h renders this drug m more ionize ed in urine and less reeabsorbab
ble.
- Tooxicity with basic drrugs (e.g. aamphetam mine) couldd be treateed by acidiification
off urine, which renderrs this drugg more ionized in urinne and lesss reabsorb
bable.
 Ion ttrapping in
n breast milk:
- Thhe pH of the
t breast milk is 7 i.e. it is coonsidered acidic in rrelation to plasma
(p
pH 7.4).
- Basic drugss (with pKa
a > 7.2) ten
nd to be ionized, and thus trappped, insidee breast
m
milk more thhan acidic
c drugs; heence, the milk/plasm
m a ratio (M//P ratio) would be
hiigh.

14
█ DIST
TRIBUTIO
ON OF DRU
UGS

Sites o
of drug disstribution
 Plassma: 3 liters
 Extrracellular water:
w 9 liters
 Intraacellular water:
w 29
2 liters

▌Volum
me of dis
stribution
n (Vd)

Definittion: The apparent


a volume
v of water into
o which
the druug is distributed in the body after distrribution
equilibrrium.

Calcula
ation:
Total amount
a off the drug in the body y
Vd
d = ————————————— —————————— ————— L
Plassma conc of the drugg (after dis
stribution equilibrium)
e )

Clinica
al significa
ance:
 Deteermination of the site
e of drug d on e.g.:
distributio
- A total Vd < 5 L: means that the e drug is confined
c to
o the vascuular compartment
an
nd can be removed by b dialysiss.
- A total Vd 5-15 L: mea ans that thhe drug is restricted
r to
t the ECF F.
- A total Vd > 41 L: me eans that tthe drug is s highly bo
ound to tisssue prote
eins and
ca
annot be reemoved by y dialysis.
 Calcculation of the total amount o of drug in the body y by singlee measuremment of
plasma concen ntration (from the eq quation).
 Calcculation off the loading dosse (LD) needed to attain a desired plasma
conccentration (Cp): LD = Vdd x Cp.
 Calcculation of drug cleaarance:

▌Binding of dru
ugs to pla
asma pro
oteins

 Mosst drugs wh
hen introdu
uced into tthe body are
a bound tot plasma proteins.
 Albu
umin: the most
m impo
ortant plasmma protein
n and it ca
an bind –vve or +ve charged
c
drug
gs.

Clinica
al significa
ance:
 The pharmaco
ological efffect of the
e drug is related only
y to its fre
ee part no
ot to its
bounnd part (th
he bound part
p acts o
only as a re
eservoir fro
om which tthe drug is
s slowly
relea
ased).

15
 Bind
ding of drugs to plasm
ma protein
ns prolong
gs their effe
ects.
 Wheen the drugg has high
h plasma p nding (e.g. 99% for warfarin), the
protein bin t free
part that exertts the phaarmacologiic effect is s 1%. Anyy small dissplacement of the
boun nd part by another drug
d (say fo
or example e another1% is displlaced) can lead to
drammatic toxic
city (double
es the amo ount of the free part in plasma)..
 Man ny disease e states (e.g. chron nic liver disease, pregnancy, renal failuure) can
affec
ct the level of albumiin and the nature of plasma pro oteins, thuus causing serious
probblems with some drugs.

█▌EXC
CRETION AND ELIM
MINATION
N OF DRUG
GS

ation of dru
Elimina ugs may fo
ollow one o
of 2 proces
sses (orderrs):

First-ord
der elimin
nation Zero-order elim
mination

- Occcurs to mo ost drugs. - Occurs


O to limited nuumber of drugs.
- Connstant ratioo (%) of th
he drug is - Constant
C amount
a off the drug is
i
elim
minated per unit time i.e. the ratte of eliminated
e per unit ti me i.e. the
e rate of
elim
mination is proportional to plassma elimination
e n is not pro oportional to
conncentrationn. The higher the plasma
p concentratio on. A familiar
conncentrationn, the greatter the rate
e of example
e is
s ethanol, cconcentrattions of
elim
mination. which
w decline at a co onstant ratte of
approxima
a ately 15 mg g/100 mL/h h.
- Elim
mination do oes not deepend on - Elimination
E n depends on satura able
satuurable enz zyme system. enzyme
e sy
ystem.
- Thee t1/2 of the drug is co
onstant. - The
T t1/2 of the drug iss not consttant.
- Drug cumulation is not common
c - Drug
D cumu ulation is ccommon

Examp ated by zerro-order: prednisolon


ples of drugs elimina p ne, theophyylline.

N.B. S
Some drug gs are elim
minated b
by first-ord
der elimina
ation in low
w doses and by
zero-orrder elimination in hig
gh doses e.g. aspirin and phenytoin.

16
Clinica
al significa
ance of ze
ero-order
elimina
ation:
 Mod
dest chan
nge in dru
ug dose may
prodduce unexxpected toxxicity.
 Elim
mination off drugs or attainmennt of
Cpsss takes lo
ong time.
 Chaanges in drug form mulation may
prodduce adveerse effects
s.
 Drug cumulattion and innteractionss are
commmon.

▌Elimination half-life (t1/2)

Definittion: It is the time taken forr the


concen ntration of a drug in n blood to
o fall
half to its originall value.

Calcula
ation:
 From
m the plasm ntration ve rsus
ma concen
time
e curve.
 From
m the equaation:

Clinica
al significa
ance:
 Deteermination of inter-d
dosage intterval: dru
ugs are giv
ven every t1/2 to avo
oid wide
flucttuations off the peakk level (the highest plasma con ncentrationn of the drug) and
trouugh level (the lowest plasma co oncentratio on).
 Time e-course of drug accumulat
a tion: if a drug
d is started as a constant infusion,
i
the CCp will accuumulate to approach ssteady-statte after 4-5 5 t1/2.
 Time e-course of drug elimination
e n: If a drug g is stopped after aan infusion, the Cp
will d
decline to re
each comp plete eliminaation after 4-5 t1/2.
 Drug gs having long t1/2 could be give en once daily to imp prove patieent compliiance.

▌Stead
dy-state plasma concentra
c ation (Cps
ss)

Definittion: the steady le evel of d drug in plasma achieved whhen the rate of
adminisstration eq
quals the ra
ate of elim ination.

The rule of 5:
 Thee Cpss is re
eached aftter 4-5 t1/2.

17
 If w
we changedd the dose, the new CCpss is reaached afte
er 4-5 t1/2.
 If do
osing stop
ps, complete eliminattion of drug
g from plasma occurrs after 4-5
5 t1/2.

18
▌Therrapeutic drug
d mon
nitoring (T
TDM)

Definittion: monittoring of se
erum drug concentra ations to optimize druug therapy
y.
 Serum drug samples
s are
a usuallyy taken wh hen the drrug has reeached the e CPSS
(e.g
g. at the tro
ough level, just beforre the next dose).
 TDM M can be done by monitoring g drug effect rather than con centration e.g. in
warrfarin theraapy, TDM is a monitoring the INR (see blood
s done via d).

Clinica
al significa
ance:
 To a
avoid toxiicity in the
e followingg situation ns:
- Drugs withh a low ‘the erapeutic i ndex’ e.g. lithium, diigoxin, andd warfarin.
- PPresence of disease e states (e..g. liver or renal dysffunction) thhat can afffect the
d
drug’s pha
armacokine
etics.
 To improve efficacy
e of drugs ha
aving pharrmacokinettic problem
ms e.g. ph
henytoin
and
d other drugs with no
on-linear kinetics.
 Diffferentiatio en drug ressistance an
on betwee nd patient non-comp
pliance.

▌Clearance as a channe
el of elim
mination

Definittion: plasm
ma clearan
nce of a su
ubstance means
m the
e volume o
of plasma cleared
from th
his substannce per min
nute.

Calcula
ation:

Clinica
al significa
ance of renal cleara
ance:
If the drug is clea
ared by the kidney, cle earance caan help to determine whether th his drug
is eliminated by reenal filtrattion or sec
cretion: a drug
d that is
s eliminateed only by filtration
f
cannot exceed 12 27 ml/min. If clearanc ce > 127 ml/min
m → th
he drug is eeliminated also by
tubularr secretion.

Routess of eliminnation:
 Kidn ney (the major
m route)).
 Bilee and liver.
 Lunngs, intestine, milk, saliva and ssweat.

Clinica
al importance of kno
owing the
e route of eliminatio
e on:
 Help
p to adjustt the dose to avoid c
cumulation.
 Avo
oid drugs eliminated
e by a disea
ased organ
n.
 Targ
geting theerapy: e.g g. drugs eliminatedd by the lung couuld be used as
exp
pectorants..

19
█ MET
TABOLISM
M OF DRUGS (biotra
ansformattion)

 e liver is the major site of drug metabolism but


The b other organs ca an also
mettabolize drrugs e.g. kidney, lung
gs, and adrenal gland ds.
 Manny lipid soluble drugs must bee converted d into a wa ble form (p
ater-solub polar) to
be e
excreted.
 Som
me drugs area not me etabolized a
at all and excreted
e unchanged (hard dru
ugs).
 Mettabolism of
o drugs may
m lead tto:
- Conversion of activ
ve drug in
nto inactiv olites → terrmination of drug
ve metabo
effect.
- Conversion of activ ve drug in nto active metabolittes → prol ongation of drug
effect e.g. codeine (a
active drug
g) is metab
bolized to morphine
m ((active pro
oduct).
- Conversion of inactive drug in nto active metabolites (prodru ugs) e.g. enalapril
e
(inactive drug) is mettabolized tto enalaprilat (active metabolitee).
- Conversion of non--toxic dru ug into toxxic metab
bolites (e.g
g. paracetamol is
converted into the to
oxic produ ct N-acety
ylbenzoquinone).

Bioch
hemical reactions
r s involve
ed in dru
ug metab
bolism

The drug must enter


e phasse I of che emical rea
actions be excreted as water--soluble
compoound. If thee drug is not
n liable tto convers sion into water-solub
w ble compo ound by
phase I, it must enter
e phase II to incrrease solub
bility and enhance
e el imination.

▌Phas
se I reac
ctions

- Pha
ase I reactions include oxidation,
duction, an
red nd hydroly
ysis.
- Enzzymes cataalyzing pha
ase I react ions includ
de
cyto
ochrome P450, ald dehyde a and alcoh hol
deh
hydrogenasse, deam minases, esterase es,
amiidases, and
d epoxide hydratase es.
- Thee majority of phase I reactionss is done by b
the cytochro ome P450 (CYP45 50) enzym me
systtem locate ed primarily inside mmembranou us
vesicles (micrrosomes) on o the surrface of th he
smo ooth endoplasmiic retic
culum of
pareenchymal liver cells s. CYP450 0 activity is
also
o present in otherr tissue e e.g. kidneey,
testtis, ovariess and GIT.

20
- Although this class has more than 50 enzymes, six of them metabolize 90% of
drugs. The most important subfamily is CYP3A4 which is responsible for
metabolism of over 50% of drugs.
- Genetic polymorphism of several clinically important CYP450 enzymes is a
source of variability of drug metabolism in humans.
- Drugs may be metabolized by only one CYP450 enzyme (e.g. metoprolol by
CYP2D6) or by multiple enzymes (e.g. warfarin).
- Some drugs and environmental substances can induce (increase activity) or
inhibit certain CYP450 enzymes leading to significant drug interactions.
- Other examples of non-microsomal oxidation include xanthine oxidase
(converts xanthine to uric acid) and monoamine oxidase (MAO) (oxidizes
catecholamines and serotonin). Only the microsomal enzymes are subjected to
induction or inhibition by drugs.

Microsomal enzyme induction Microsomal enzyme inhibition

 Microsomal inducers ↑ rate of  Microsomal inhibitors ↓ rate of


metabolism of some drugs leading to metabolism of some drugs leading to
↓ their serum levels and therapeutic ↑ their serum levels and toxicity.
failure.  Enzyme inhibition can occur after
 Induction usually requires prolonged short period of exposure to the
exposure to the inducing drug. inhibiting drug.
 Examples of inducing agents:  Examples of inhibiting agents:
phenytoin, phenobarbitone, macrolide antibiotics (e.g.
carbamazepine, rifampicin, smoking, erythromycin), ciprofloxacin,
chronic alcohol intake, St John's cimetidine, ketoconazole, ritonavir,
Wort, grapefruit juice.
 Clinical examples:  Clinical examples:
- Rifampicin accelerates metabolism of - Ciprofloxacin inhibits metabolism of
contraceptive pills leading to failure of warfarin (anticoagulant) leading to
contraception. accumulation of warfarin and
- Phenytoin accelerates metabolism of bleeding.
cyclosporine-A leading to graft - Erythromycin inhibits metabolism of
rejection. theophylline leading to toxicity of
theophylline (cardiac arrhythmia).

▌Phase II reactions (conjugation)

- It involve coupling of a drug or its metabolite to water-soluble substrate (usually


glucuronic acid) to form water-soluble conjugate.
- Glucuronyl transferase is a set of enzymes that is responsible for the majority
of phase II reactions. This set of enzymes is also located inside liver

21
miccrosomes and is the e only phasse II reactio
on that is inducible b
by drugs and
a is a
posssible site of drug in
nteractionss e.g. phen nobarbital induces gglucuronida
ation of
thyrroid hormoone and reduces theiir plasma levels.
- Som me glucuroonide conjjugates seecreted in bile can be
b hydrolyyzed by in ntestinal
bac cteria and the free drug
d can bbe reabsorbed again (enteroheepatic circu
ulation),
thiss can exten
nd the actio
on of somee drugs.
- Oth
her examples of non njugation reactions include sulphate
n-glucuro unide con s
con
njugation (steroids), glycine conjugatio
on (salicylic acid), and gluttathione
con
njugation (e
ethacrynic acid).

▌Firstt-pass me
etabolism stemic elimination
m (pre-sys n)

Definittion: metaabolism off drugs a at the sitee of administration before re eaching


system he liver aft er oral administratio
mic circulattion e.g. th on, the lung
g after inh
halation,
the skin
n after topical adminnistration, e
etc.

Hepatiic first-pas
ss metabo
olism:
 Com
mplete: lido ocaine.
 Parttial: propraanolol, morphine,
nitro
oglycerine
 Non ne: atenolo
ol and mon
nonitrates

How to
o avoid?
- By iincreasing the dose of the drugg.
- By ggiving the drug throu
ugh other rroutes e.g. sublingua
al, inhalatio
on, or i.v.

▌Bioav
vailability
y

Definittion: it is the
t fraction of the d
drug becom me availab ble for sysstemic effe
ect after
adminisstration. The bioavailability of d
drugs given i.v. is 100%.

22
Factors affecting bioavailability:
 Factors affecting absorption.
 Factors affecting metabolism.
 First-pass metabolism.

Part 4: Adverse drug reactions (ADR)

An ADR is any response to a drug which is noxious, unintended, and occurs at


doses used in man for prophylaxis, diagnosis or therapy.

Predisposing factors:
 Multiple drug therapy.
 Extremes of age: due to age related changes in pharmacokinetics and dynamics.
 Associated disease: e.g. impaired renal or hepatic function.
 Genetics: can affect the pharmacokinetics.

Classification:

▌Type A (Augmented):

These reactions are predictable from the known pharmacology of the drug. They
may result from an exaggerated response (e.g. hypotension from an
antihypertensive) or non-specificity (e.g. anticholinergic effects with tricyclic
antidepressants).

Prevention
 Take a careful history for predisposing factors.
 Use the smallest dose of the drug adequate for the desired effect.
 Adjust dosage to therapeutic end-points, e.g. blood pressure or INR.
 Adjust dosage to optimum plasma concentrations, e.g. digoxin.
 Adjust dosage in relation to renal function, hepatic function, or other drugs.

▌Type B (Bizarre):

These are less common, less predictable, and may be severe. Examples are:
 Immunologic: penicillin allergy
 Genetic: haemolysis in G6PD deficiency
 Disease: amoxycillin rash in glandular fever
 Idiosyncratic: malignant hyperpyrexia in anesthesia.

Prevention
 Take a careful drug history, especially of allergies

23
 Fam
mily historyy: allergies or genetic
c disease
 Avo
oid drugs susceptibl
s e to ADRss in particular diseas
se states, e.g. cloza
apine in
bon
ne marrow depressio
on.

Type A (Augmen
nted) Ty
ype B (Biz
zarre)
- Pred
dictable - Unpredictable
- Dosee-dependeent - Dose-independent
- High
h incidence
e - Low incid
dence
- Mayy respond to
t dose ad
djustment - Generally
y need to sstop the drrug

█ Drug
g-induced
d liver injjury (DILII)

DILI ac
ccounts forr up to 10%
% of all advverse drug
g reactionss and may be fatal.
It may be classifie
ed into:
 Acccording to time courrse: acute or chronic.
 Acccording to mechanis sm: dose-d dependentt, idiosynchhratic, or im
mmune me ediated
 Acccording to histologic g: hepatoce
cal finding ellular, cho
olestatic, o r mixed picture.

Hepato
ocellular (cytotoxic)
( ) DILI Cholestatic
C c DILI
Featurres: Features:
F
 The e drug or its metaboliites affectss  The drug or its meetabolites affect
a
pareenchymal liver cells leading
l to the bilia
ary canalicuuli leading to
cell necrosis and
a initiatioon of narrowin ng or dest ruction of biliary
infla
ammatory process. passage es.
 It m
may be spo otty, zonal, or diffuse..  Clinically it resemb bles obstruuctive
 Clinnically it ressembles viral hepatittis jaundice e with prurritus and ↑ALP.
A
withh ↑ ALT and AST.

Commmon drugs:: Common


C drugs:
d
Parace
etamol – methyldopa – Chloroprom
C mazine – suulfonylureaas –
amioda
arone – iso
oniazid – va
alproic acid
d oral
o contrac ceptive pil ls – anabo
olic
steroids
s – macrolides
m s – co-amo oxiclav

24
█ ADR on pregnancy

Key facts:

 Fetal birth defects represent 2-3% of all births, the majority of which are related
do drugs.
 Some fetal defects may be impossible to identify, or can be delayed e.g. the use
of diethylstilbesterol (estrogenic compound) during pregnancy is associated with
development of adenocarcinoma of girls’ vagina at teen age.
 Three factors determine the risk of teratogenicity: dose of the drug; duration of
administration; and stage of pregnancy.
 Most drugs with a MW <1000 can cross the placental barrier.
 All drugs should be considered harmful until proven otherwise.

Mechanism of teratogenicity according to pregnancy stage:

 Before implantation: (0-17 day): The effect is all-or-none i.e. either death of the
embryo (abortion) or no effect.

 Early pregnancy: (3-10 weeks):


- It is the most dangerous period because it is period of organogenesis.
- Selective interference can produce characteristic anatomical abnormality e.g.
aminoglycosides cause damage to 8th cranial nerve.

 Late pregnancy:
- Gross anatomical abnormalities are less liable to occur.
- Functional defects rather than anatomical abnormalities can occur especially
in organs having delayed formation e.g. brain, testes, and bone.

Examples of teratogenic drugs:

 ACE inhibitors and angiotensin receptor blockers cause fetal pulmonary and
renal dysfunction.
 Antithyroid drugs can cause fetal goiter and hypothyroidism.
 Tetracycline antibiotics inhibit growth of fetal bones and stain teeth.
 Aminoglycosides cause fetal 8th cranial nerve damage.
 Warfarin can cause fetal intracerebral bleeding.
 NSAIDs cause premature closure of ductus arteriosus.
 Benzodiazepines cause cleft lip and palate.
 Sex hormones can cause inappropriate virilization or feminization.
 Antiepileptic drugs cause neural tube defect (spina bifida).
 Cytotoxic drugs can cause multiple structural damage.

25
The FDA pregnancy categories:

Category Definition Animal Human


Adequate studies in animal and human did
A not show a risk to the fetus either in the  
first or in the late trimesters.
Animal studies did not show risk to the  ?
fetus but there are no adequate studies in
human.
B
or: Animal studies showed a fetal risk, but
adequate studies in human did not show a  
risk to the fetus.
Animal studies showed a risk to the fetus
but there are no adequate studies in  ?
C humans; the benefits from the use of the
drug in pregnant women may by
acceptable despite its potential risks. Benefit > Risk

There is evidence of human fetal risk, but  


D the potential benefit of the drug may
outweigh its potential risk. Benefit > Risk
Studies in animals and humans showed  
evidence of fetal risk. The potential risk of
X
use in pregnant women clearly outweighs
Risk > Benefit
any potential benefit.

Part 5: Principles of drug-drug interactions

Classification:
■ Pharmacokinetics interactions.
■ Pharmacodynamic interactions.

█ PHARMACOKINETIC INTERACTIONS

Drug interactions in vitro:


e.g. antipseudomonal penicillins and aminoglycosides form complexes in the infusion
fluid (see chemotherapy).

26
Drug interactions in vivo:

Absorption

 Formation of complexes:
- Tetracycline forms complexes with Ca2+, Mg2+ and Al3+
- Cholestyramine forms complexes with digitalis and thyroxin.

 Absorption can be blocked:


- Adrenaline ↓ absorption of local anesthetics due to VC.
- Colchicine ↓ absorption of vitamin B12

 Change in intestinal motility:


- Anticholinergic drugs ↓intestinal motility → ↑ absorption of some drugs.
- Prokinetic drugs ↑ intestinal motility → ↓ absorption of some drugs.

 Changes in gastric pH:


- Antacids ↓absorption of salicylates.
- Ketoconazole is poorly absorbed in absence of gastric acidity.

Distribution

- Sulfonamides displace bilirubin from pl pr in premature infants → kernicterus.


- Phenylbutazone displaces warfarin → excessive bleeding.

Metabolism

- Inhibition or induction of microsomal metabolism (see before).


- Inhibition of non-microsomal enzymes:
- MAO inhibitors ↓ metabolism of some drugs e.g. benzodiazepines, serotonin
and norepinephrine.
- Disulfiram inhibits acetaldehyde dehydrogenase enzyme → ↓ metabolism of
acetaldehyde → accumulation of acetaldehyde causes flushing, nausea,
vomiting, and tachycardia.

Excretion

 Reduction in urinary elimination:


- Probenecid ↓ renal excretion of penicillin.
- Quinidine ↓ renal excretion of digoxin.

 Changes in urinary pH:


- Alkalinization of urine (e.g. sodium bicarbonate) ↑ excretion of weak acids
- Acidification of urine (e.g. ammonium chloride) ↑ excretion of weak bases.

27
 Changes in urinary volume:
Diuretics can increase toxicity of some drugs by reducing plasma volume e.g.
thiazide can increase lithium toxicity.

 Stimulation of biliary excretion:


Phenobarbital ↑ biliary excretion of many drugs by increasing both bile flow and
the synthesis of conjugating proteins.

██ PHARMACODYNAMIC INTERACTIONS

 Antagonism: competitive, non-competitive, chemical, physical, etc.


 Synergism: e.g. MAO inhibitors can cause toxic synergism with TCA.
 Potentiation: e.g. ethanol can enhance CNS depression caused by opioids
 Changes in the intracellular or extracellular environment: e.g. diuretic-
induced hypokalemia can ↑ digitalis toxicity.

28
29
30
Review Questions

Define the following pharmacokinetic parameters:


 Volume of distribution
 pKa of drugs
 Elimination half life
 First-pass metabolism
 Bioavailability

Mention the clinical significance of each of the following:


 Volume of distribution
 pKa of drugs
 Plasma protein binding of drugs
 Elimination half-life
 Zero-order elimination
 Microsomal enzyme induction
 Hepatic conjugation of drugs

Mention the main differences between:


 Reversible and irreversible antagonism.
 Graded response and quantal response.
 First order elimination and zero order elimination.
 Potency and efficacy.
 Physical and physiological antagonism.
 Habituation and addiction.
 Oxidation and conjugation of drugs.

Discuss 2 pharmacogenetic conditions associated with toxic drug response


Discuss 2 pharmacogenetic conditions associated with reduced drug response
Write short account on antagonism between drugs

31
Of each of the following questions, D. Sex hormones act on these types of
select ONE BEST answer: receptors
E. Corticosteroids act on these types of
1. A drug may act by all the following receptors
mechanisms EXCEPT:
A. Interaction with protein 5. The following statements are true
macromolecules embedded in the cell for graded dose-response relationship
membranes EXCEPT:
B. Interaction with cell membrane ion A. It is the response to most drugs
channels B. The response is directly proportional
C. Interaction with intracellular enzymes to drug concentration (linear relation)
D. Interaction with cell membrane C. It could be tested in one animal
phospholipids D. It can be used for comparing the
E. Interaction with gene functions potencies and efficacies of drugs
E. It can be used for calculation of the
2. Ion-channel-linked receptors (direct LD50 of drugs
ligand-gated ion channels) are
characterized by: 6. The following statements are true
A. They are the type of receptors for quantal dose-response relationship
principally present in autonomic EXCEPT:
ganglia and skeletal ms motor end A. It is the response to anticonvulsant
plate and antiarrhythmic drugs
B. They are the type of receptors B. The response to the drug is not
principally present in vascular directly proportional to drug
endothelium concentration (all-or-none)
C. They are rosette-shaped structures C. It could be tested in one animal
consist of 7 membrane subunits D. It helps in calculation of the ED50 and
D. Their response is slower than other LD50 of drugs
receptors E. It helps in estimation of the degree of
E. Activation of these receptors leads to drug safety
activation of a second messenger
7. When a drug has a steep dose-
3. Which of the following is classified response curve, this means:
as belonging to the tyrosine kinase A. The drug is lethal
family of receptors:
B. The drug is expensive
A. GABA receptors
C. The drug is efficacious
B. β-Adrenergic receptors
D. The drug is safe
C. Insulin receptors
E. Minimal change in the dose can lead
D. Nicotinic acetylcholine receptors to dramatic effect.
E. Hydrocortisone receptors
8. The following statements are true
4. All the following are true for for drug’s therapeutic index EXCEPT:
intracellular (DNA-linked) receptors A. It is the relation between the lethal
EXCEPT: dose in 50% of animals to the curative
A. They regulate transcription of genes dose in 50% of them
inside the nucleus B. The lower the TI, the safer will be the
B. Their response is very fast but drug.
persists for long time C. It should be done to any drug before
C. Their agonists must enter inside the it’s being approved for human use
cell to reach them inside the nucleus

32
D. For theoretically useful drugs, it must E. Is described as addition if the action
be greater than 1 of one drug abolishes the effects of
E. It could be applied in animal testing another

9. The following is true for competitive 13. A drug may interact with ion
antagonism: channels by all of the following
A. It never occurs with enzymes mechanisms EXCEPT:
B. Is the same as physiological A. The drug may change the ion channel
antagonism structure
C. The agonist can never abolish the B. The drug may block the channel
effect of the antagonist physically
D. Is best exemplified by the use of C. The drug may change an intracellular
neostigmine to treat curare toxicity ATP on which the channel depends
E. Best described as non-surmountable D. The ion channel may be part of ion
process channel-linked receptors
E. The ion channel may be modulated by
10. A drug is said to be reversible G-protein linked receptors
antagonist when:
A. It blocks the receptors by making 14. Failure of the patient to breath after
covalent bonds with them surgical operation may be due to:
B. The duration of blockade is too long A. Pseudocholinestrase deficiency
C. Increasing the dose of the agonist will B. Methemoglobin reductase deficiency
reverse the block C. G-6-PD deficiency
D. The response curve of the agonist in D. Vitamin K epoxide reductase
presence of this drug is not parallel to deficiency
that of the agonist alone E. Monoamine oxidase deficiency
E. Termination of the drug effect dep-
ends on synthesis of new receptors 15. Hemolysis that may occur with
sulfonamides therapy may be due to:
11. The interaction that may occur A. Pseudocholinestrase deficiency
between acidic and basic drugs is B. Methemoglobin reductase deficiency
called: C. G-6-PD deficiency
A. Chemical antagonism D. Vitamin K epoxide reductase
B. Physical antagonism deficiency
C. Physiological antagonism E. Monoamine oxidase deficiency
D. Biological antagonism
E. Receptor antagonism 16. Severe myelosuppression following
6-mercaptopurine therapy is most likely
12. The following is true for interactions due to:
between drugs: A. Pseudocholinestrase deficiency
A. Is not harmful if occurred between B. Methemoglobin reductase deficiency
drugs having steep dose-response C. G-6-PD deficiency
curves D. Vitamin K epoxide reductase
B. Is not harmful if occurred between deficiency
drugs having narrow therapeutic E. Thiopurine methyltransferase
ratios deficiency
C. Is not harmful if occurred between
drugs undergoing zero-order kinetics 17. Hepatic toxicity that may
D. May lead to valuable therapeutic accompany isoniazide therapy may be
effects due to:

33
A. Defective oxidation reaction D. Phenobarbital (pKa = 7.4)
B. Defective conjugation reaction E. Propranolol (pKa = 9.4)
C. Defective deamination reaction
D. Slow acetylation reaction 22. The following statements are true
E. Rapid acetylation reaction for Vd of drugs EXCEPT:
A. It can exceed the volume of water in
18. Failure of some children with rickets the body
to respond to therapeutic doses of B. Drugs with large Vd can be removed
vitamin D is most likely to be due to: by dialysis
A. Differences in sex C. Would be expected to be 5L if the
B. Differences in body weight drug is confined to the blood.
C. Genetic variation D. Highly lipid-soluble drugs would be
expected to have large Vd
D. Tolerance
E. Intolerance E. It can help in the calculation of the
total amount of the drug in the body
19. The following are true for overshot
phenomenon (drug intolerance)
23. The plasma half-life (t1/2) of drugs:
EXCEPT: A. Is expressed as the percentage that
remains ½ hour after administration
A. It occurs due to down-regulation of
receptors B. Will be short if the drug gets into the
enterohepatic circulation
B. It occurs after sudden stoppage of
some drugs given for long time C. Cannot be calculated if the drug is
excreted through the bile
C. It may lead to serious withdrawal
effects D. Is constant for drugs having zero-
order elimination
D. It can be avoided by gradual
cessation of drugs E. Can be prolonged by slowing the rate
of drug elimination
E. It is best exemplified by occurrence of
severe tachycardia after sudden
stopping of beta blockers. 24. The bioavailability of a drug:
A. Is defined as the actual blood
20. The following statements are true concentration required to produce a
for pKa of drugs EXCEPT: pharmacological effect
A. Ionized drugs are poorly absorbed B. Will be unaffected by changes in
while unionized drugs are more formulation
absorbed C. May be affected by liver damage
B. Ionization of most drugs depends on D. Must be 100% for a drug given by
the pH of the medium around them mouth and is completely absorbed
C. pKa of drugs can help knowing the E. Is a term applied only to oral
site of drug absorption. administration
D. Acidic drugs become more
absorbable in alkaline pH 25. All the following are phase I
E. Basic drugs become more biotransformation reactions EXCEPT:
reabsorbable in alkaline urine A. Sulfate conjugation
B. Xanthine oxidation
21. Which of the following drugs will be C. Nitroreduction
absorbed to the LEAST extent in the D. Ester hydrolysis
stomach: E. Oxidative deamination
A. Ampicillin (pKa = 2.5)
B. Aspirin (pKa = 3.5)
C. Warfarin (pKa = 5.0)

34
26. Metabolism (biotransformation) of E. Drug X will have a shorter duration of
drugs can lead to all the following action than drug Y because less of
results EXCEPT: drug X is present for a given effect.
A. Conversion of active compound into
inactive metabolites 30. Which of the following terms best
B. Conversion of active compound into describes the antagonism of
active metabolites leukotriene’s bronchoconstrictor effect
C. Conversion of inactive compound into (mediated at leukotriene receptors) by
active metabolites terbutaline (acting at adrenoceptors) in
D. Conversion of non-toxic compound a patient with asthma?
into toxic metabolites A. Pharmacologic antagonist.
E. Conversion of water-soluble B. Partial agonist.
compound into lipid-soluble C. Physiologic antagonist.
metabolites D. Chemical antagonist.
E. Noncompetitive antagonist.
27. All the following statements are true
for First-order kinetics EXCEPT: 31. Which of the following provides
A. Apply to most drugs in clinical use information about the variation in
B. Apply to salicylate (aspirin) sensitivity to the drug within the
metabolism within small dose. population studied?
C. The concentration versus time curve A. Maximal efficacy.
is non-linear. B. Therapeutic index.
D. The rate of elimination depends on C. Drug potency.
plasma concentration of the drug D. Graded dose-response curve.
E. Steady state plasma concentration E. Quantal dose-response curve.
can be reached after 5 half lives
32. Which of the following provides
28. All the following statements are true information about the largest response
for zero-order kinetics EXCEPT: a drug can produce, regardless of
A. Elimination rate is independent of the dose?
dose A. Drug potency.
B. Elimination depends on saturable B. Maximal efficacy.
enzyme system
C. Mechanism of receptor action.
C. Plasma concentration of the drug D. Therapeutic index.
cannot be expected at any time
E. Therapeutic window.
D. The t1/2 of the drug is not constant
E. There is no fear from drug cumulation 33. A pro-drug is:
or interactions
A. The prototype member of a class of
drugs.
29. Drugs X and Y have the same
mechanism of diuretic action. Drug X in B. The oldest member of a class of drugs
a dose of 5mg produces the same C. An inactive drug that is transformed in
magnitude of diuresis as 500 mg of the body to an active metabolite.
drug Y. This suggests that: D. A drug that is stored in the body
A. Drug Y is less efficacious than drug X tissues and is then gradually released
in the circulation.
B. Drug X is about 100 times more
potent than drug Y. E. Ionized drug trapped in breast milk.
C. Toxicity of drug X is less than that of
drug Y. 34. If the rate of infusion of a drug were
doubled, what response in the steady
D. Drug X is a safer drug than drug Y.

35
state concentration would be E. About 99%
expected?
A. Remain unchanged 40. Concerning the renal excretion of
B. Doubled drugs:
C. Increase 50% A. Drugs that are ionized in the renal
D. Decrease 50% tubules are more likely to undergo
E. Decrease 100% passive reabsorption.
B. Low MW drugs are much more likely
35. Half-life of a drug may be helpful to to be actively secreted than filtered.
determine: C. Only the fraction of the drug that is
A. Elimination of the drug unbound (free) to plasma proteins is
filtered by the glomerulus.
B. Level of absorption
C. Rate of absorption through the GIT
D. Decreasing urinary pH enhance
excretion of weakly acidic drugs.
D. Time to reach the steady state
E. Renal clearance cannot exceed the
E. Distribution into body systems. GFR (125 ml/min).

36. What determines the degree of 41. In which of the following cases
movement of a drug between body could a graded dose-response curve be
compartments? constructed?
A. Partition constant A. Prevention of convulsions
B. Degree of ionization B. Prevention of arrhythmias
C. pH C. Reduction of death
D. Molecular size D. Reduction of fever
E. All of the above E. Relief of insomnia
37. For intravenous (IV) dosages, what 42. Which of the following can be used
is the bioavailability assumed to be? as a relative indicator of the margin of
A. 0% safety of a drug?
B. 25% A. T.I.
C. 50% B. LD50
D. 75% C. ED50
E. 100% D. EC50
E. TD50
38. Which of the following can produce
a therapeutic response? A drug that is: 43. Flurazepam has a pKa of 8.2. What
A. Bound to plasma albumin percentage of flurazepam will be
B. Concentrated in the bile ionized at a urine pH of 5.2?
C. Concentrated in the urine A. 0.1%
D. Not absorbed from the GI tract B. 1.0%
E. Unbound to plasma proteins C. 50%
D. 99%
39. Aspirin is a weak organic acid with E. 99.9%
a pKa of 3.5. What percentage of a
given dose will be in the lipid-soluble 44. Which route of administration is
form at a stomach pH of 1.5? most likely to subject a drug to first
A. About 1% pass metabolism?
B. About 10% A. Intravenous
C. About 50% B. Sublingual
D. About 90% C. Oral

36
D. Inhalation E. Phase I metabolism of Drug A will
E. Intramuscular increase its intracellular access and
actions
45. If a drug was given by a constant
infusion rate, which of the following 48. The FDA assigns the letters A, B, C,
factors determines how long it will take D, and X to drugs approved for human
for the drug to reach a steady-state use. To which of the following does this
concentration (Cpss) in the blood? classification apply?
A. Apparent volume of distribution A. Amount of dosage reduction needed
B. Bioavailability as serum creatinine clearances fall
C. Clearance B. Fetal risk when given to pregnant
women
D. Half-life
E. Infusion rate (mg of drug/min) C. Amount of dosage reduction needed
in presence of liver dysfunction
46. Which of the following best D. Relative margins of safety/therapeutic
index
describes what the term
“tachyphylaxis” means? E. The number of unlabeled uses for a
drug
A. An increase in the rate of the
response, for example, an increase of
the rate of muscle contraction 49. Which effect may lead to toxic
reactions when a drug is taken
B. Immediate hypersensitivity reactions
continuously or repeatedly?
(i.e., anaphylaxis)
C. Prompt conformational changes of the A. Refractoriness
receptor such that agonists, but not B. Cumulative effect
antagonists, are able to bind and C. Tolerance
cause a response D. Tachyphylaxis
D. Quick and progressive rises in the E. Intolerance
intensity of drug response, with
repeated administration, even when 50. Tolerance and drug resistance can
the doses are unchanged be a consequence of:
E. Rapid development of tolerance to the A. Change in receptors, loss of them or
drug’s effects exhaustion of mediators
B. Increased receptor sensitivity
47. Drug A undergoes a series of Phase C. Decreased metabolic degradation
I metabolic reactions before being D. Decreased renal tubular secretion
eliminated. Which of the following
statements best describes the
E. Activation of a drug after hepatic first-
pass
characteristics of Drug A, or the role of
Phase I reactions in its metabolism?
51. If two drugs with the same effect,
A. Complete metabolism of Drug A by taken together, produce an effect that
Phase I will yield products that are
is equal in magnitude to the sum of the
less likely to undergo renal tubular
effects of the drugs given individually, it
reabsorption
is called as:
B. Drug A is a very polar substance
A. Antagonism
C. Drug A will be biologically inactive
until it is metabolized B. Potentiation
D. Phase I metabolism of Drug A C. Synergism
involves conjugation with glucuronic D. Additive effect
acid or sulfate E. Supersensitivity

37
52. All of the following statements
about efficacy and potency are true
EXCEPT:
A. Efficacy is usually a more important
clinical consideration than potency
B. Efficacy is the maximum effect of a
drug
C. Potent drugs usually given in small
dose.
D. Potency is a comparative measure,
refers to the different doses of two
drugs that are needed to produce the
same effect
E. The ED50 is a measure of drug’s
efficacy

Answers

1D 11 A 21 E 31 E 41 D
2A 12 D 22 B 32 B 42 A
3C 13 A 23 E 33 C 43 E
4B 14 A 24 C 34 B 44 C
5E 15 C 25 A 35 D 45 D
6C 16 E 26 E 36 E 46 E
7E 17 E 27 C 37 E 47 A
8B 18 C 28 E 38 E 48 B
9D 19 A 29 B 39 E 49 B
10 C 20 D 30 C 40 C 50 A
51 D
52 E

38
Part 1
1: Basiic inform
mation

The au utonomic nerv-


ous sy ystem co ontrols
involunntary ac
ctivity
(Fig 1, ttable 1).

Sympa athetic nervous


systemm (SNS)
‒ Sho ort prega anglio-
nic axons orig ginate
from
m thoracic c and
lum
mbar areas of the
spinnal cord and
synapse in ganglia
locaated closse to
the spinal corrd.
‒ Thee adrenal me-
dulla is consiidered
ammodified ganglion and d is innerva
ated by sym
mpathetic fibers.
‒ Theermoregulaatory swea at glands aare anatommically sym mpathetic, but the postgan-
glio
onic nerve fibers
f relea
ase acetylccholine (AC
Ch) (i.e. sym
mpathetic cholinergic).

Parasyympathetic c nervouss system (PPNS)


‒ Lon ng pregangglionic axo
ons originaate from cranial
c and
d sacral arreas of thee spinal
cordd and synnapse in ganglia
g loc
cated clos se to or within
w the innervatedd organ
(with few exce
eptions).
‒ Sho ort postgan
nglionic ax
xons innervvate many tissues an
nd organs aas the SNS S.
‒ Parasympatheetic innervvation preddominates over symp pathetic in nervation of most
orgaans exceppt blood veessels (ha ve only sym
mpathetic supply).

39
Entericc nervous system (E ENS)
‒ Thee ENS is co onsidered the
t third ddivision of the
t ANS.
‒ It iss a collectiion of neurons insid e the wall of the GIT
T that conntrols the motility,
m
exo
ocrine and endocrine secretionss of the GI tract.
‒ Nerrve terminaals contain peptides and purine es as neuro
otransmitteers.
‒ Thiss system functions
f in
ndependen ntly of the CNS and is modulaated by bo oth SNS
andd PNS.

omatic nerrvous systtem contro


The so ols volunta
ary activity::
‒ Lonng axons originate in the spinal cord and directly
d innervate ske cles (no
eletal musc
gan
nglia).
‒ Nerrve termin nals in thet neurromuscula ar junctio on releasse Ach as the
neu
urotransmittter.

Neuro
otransmiitters of the ANS
S

1. Norrepinephriine and epinephrin


ne

They arre catecho


olamines, having
h cate
echol nucleu
us.

Biosyn
nthesis of catechola amines:
‒ In n
nerve endin ngs, tyrosinne is hydro b tyrosine hydroxylasse to form (dopa);
oxylated by
dop
pa is then n decarboxylated to o form doopamine which
w is hhydroxylated into
nore
epinephrinne inside sttorage vessicles.
‒ In ccertain areaas of the brain and in the ad drenal
meddulla, noreepinephrinee is meth hylated by
y N-
metthyltransferrase to form epineph hrine.

Storagge and rele


ease:
‒ Norrepinephrinne is stored in vessicles in nerve
n
term
minals.
‒ Norrepinephrinne also exxists in a non-vesicular
cyto
oplasmic pool
p that is released
d by indirrectly
acting symp pathomime etics (e.gg., tyrammine,
amp phetaminee).

Termin
nation:
■ R e (80%): mainly in the
Re-uptake e form of:
‒ Neuronal uptake (in
nto neuron al cytoplas sm).
‒ Granular uptake (into storagee vesicles).

■ M
Metabolism
m (18-20%
%):
Figure
F 2. Syynthesis and termination
of
o norepinephhrine

40
‒ Monoam
mine oxidas enzyme: metabolizes
se (MAO) e m s norepine phrine in neuronal
n
cytoplasmm.
 MAO--A: present in the braain and pe eripheral tis
ssues (e.g. liver & inte
estine).
 MAO--B: presen nt mainly iin the Bra
ain and mo ore active on dopam mine. It
has litttle effect on
o norepinnephrine annd seroton nin.
‒ Catecho
ol-O-methy
yl transfe OMT): metabolizes nnorepineph
erase (CO hrine in
synaptic space.

N.B. Th he end-prooduct of catecholami


c ine metabo olism is
vanillyl mandelic acid
a (VMA).. The norm
mal urinary level of
VMA iss 4-8 mg//day. Higher levels indicate tu umor in
suprare enal medulla
a (pheochroomocytom ma, Fig 3).

Clin
nical
corrrelates:
Deppression is s asso-
ciateed with dec creased
activvity of NA and/or
sero
otonin at th he level
of ssynapse. Tricyclic
T
antiddepressant drugs ac ct by inhibbition of neuronal
n
uptaake of NA and
a seroton nin while MA
AO inhibitin
ng drugs
act bby inhibition
n of their metabolism.
m . Both mechanisms
lead to accum mulation off NA and serotonin at the
synaaptic levels.

Park kinsonism is associated witth deficie ency of


dopaamine in the
t negrostriatal pathhway. Inhib
bition of
MAO O-B enzym me (selegiline) and COMT enzyme
(tolc apone) lea
ads to acccumulation of dopamine and
imprrovement off bradykinesia.

2. Ace
etylcholine
e (ACh)
‒ ACh
h is synthe
esized in nerve termiinals from acetyl co--A and cho
oline. Synthesized
AChh is stored in vesicless in nerve terminal.
‒ Bottulinum tooxin blocks s Ach releaase and ca auses skele
etal musclee paralysis
s.
‒ Thee main fatee of ACh iss rapid hyddrolysis byy cholinestterase (ChhE) enzyme; there
are two isofoorms:

True Ch
hE Pseudo ChE
C
‒ Present in CNS, ganglia, NMJ.
N ‒ Presennt in plasmaa and liver.
‒ Speccific for Ach
h. ‒ Not sp
pecific for A
Ach
‒ Defic
ciency is fattal. ‒ Deficie
ency is Nott fatal
‒ Regeenerates in 2-3 monthss. ‒ Regennerates in 2 -3 weeks.

41
Clinica
al correla
ates:
Congen
nital PsChE
E deficienc
cy (succinyylcholine ap
pnea):
Succinyylcholine is a neuromuscular blo ocker that is
i metabolized by
PsChE enzyme. Some S indiv
viduals havve congen nital deficie
ency of
PsChE, when theyy take succinylcholine tto produce muscle relaxation
before surgery, severe
s musscle paralyysis occurs
s due to lack of
succinyylcholine metabolism.
m Death maay occur from
f paralyysis of
respirattory musc cles. Urgent blood transfusio on and artificial
a
respirattion may be
e required.

3. Co-ttransmittters
A number of Non n-adrenerrgic-Non-c cholinergiic (NANC) transmitteers may be e found
ociation wiith NA or Ach
in asso A in the e autonomic nerve teerminals. T They are re
eleased
with th
he primaryy transmittter to plaay a regu ulatory function. Exxamples include:
i
neuroppeptide Y; encephalin
e ; histamine
e; 5HT; ATP
P; PGs; and
d nitric oxid
de (NO).

42
Table 1. Distribution and functions of autonomic receptors.

SYMPATHETIC PARASYMPATHETIC
Tissue
R Effect R Effect

Heart β1 ↑ all cardiac properties M2 ↓ SA node activity and AV


(tachycardia, ↑A-V conduction, conduction (NOT atrial
↑contractility, etc) conduction)
α1 VC of most BV VD of most BV (through release
Blood of EDRF). N.B. Most vascular M3
β2 VD of skeletal muscle BVs, and M3*
vessels receptors are non-innervated
coronary artery.
Bronchi
Smooth ms β2 Relaxation (Bronchodilatation) M3 Contraction (Bronchoconstriction)
Glands
α1 ↓ Bronchial secretion M3 ↑ Bronchial secretion
GIT
Wall α, β2 Relaxation (↓ motility) M3 Contraction (↑ motility)
Sphincters α1 Contraction M3 Relaxation
Salivary gld α1 ↑ enzyme secretion (viscid saliva) M3 ↑ water secretion (salivation)
Liver β2 Glycogenolysis −
Stomach HCl − M1 ↑ HCl secretion
U bladder
Detrusor ms β2 Relaxation M3 Contraction
Sphincter α1 Contraction (urine retention) M3 Relaxation (urine flow)
Uterus β2 Relaxation −
α1 Contraction
♂ organs α1 Ejaculation M3 Erection
Kidney β1 ↑ Renin secretion −
Skeletal ms β2 Tremors and enhancement of −
neuromuscular transmission
Eye
Iris ms α1 Pupil dilatation (mydriasis) M3 Pupil constriction (miosis)
Ciliary ms β2 Relaxation (distant vision) M3 Contraction (near vision)
IOP β2 ↑ aq humor secretion (↑ IOP) M3 ↑ aq humor drainage (↓ IOP)
Lacrimal gld − M3 ↑ lacrimal secretion
Sweat gld α1 ↑ sympathetic sweating M3 ↑ Thermoregulatory sweating
(forehead & palms) (cholinergic sweating)
Fat cells β3 Lipolysis −
Mast cells β2* ↓ histamine release −
Plasma K+ β2 Decrease plasma K+ −
Nerve α2 ↓ NA release −
terminals
β2 ↑ NA release

* = Non-innervated receptors i.e. receptors are found in the organ but have no autonomic nerve
supply. They can respond only to circulating or administered agonists.
EDFR = endothelial derived relaxing factor = nitric oxide (NO).

43
44
Table 2. Summary of adrenergic receptors

α1 α2 β1 β2 β3
2nd msngr Gq (↑ IP3 & ↑ DAG) →↑ Ca2+ Gi (↓ cAMP) Gs (↑ cAMP) Gs (↑ cAMP) Gs (↑cAMP)
Sites and 1. VC of most bl vessels 1. Presynaptic nerve 1. ↑ all cardiac 1. Presynaptic nerve endings (↑ ↑ lipolysis
function (α1A) endings (↓ NA properties NA release) (adipose
2. Contraction of all release) 2. ↑ renin release 2. Central: ↑ central sympathetic tissue)
sphincters (GIT, urinary). 2. Central: ↓ central (kidney) outflow
3. Contraction of dilator sympathetic outflow 3. VD of sk ms bl vessels and
pupillae ms (mydriasis) 3. Relaxation of GIT & coronary artery
4. Contraction of uterus UB walls 4. Bronchodilatation
5. Relaxation of GIT & UB 5. Relaxation of GIT & UB walls
walls 6. Relaxation of uterus
6. Adrenergic sweating 7. Skeletal muscle tremors
(forehead & palm)
8. ↑ aqueous humor secretion
9. ↑ liver glycogenolysis
10.↓ plasma K+

Sel. agonist Phenylepherine Clonidine Dobutamine Salbutamol


Selective Prazosin Yohimbine Atenolol Butoxamine (not used clinically)
antagonist
Non-selec Adrenaline Ephedrine
agonist
Non-selec Phenoxybenzamine Ergot alkaloids Propranolol Timolol
antagonist

■ β4 and β5 are also present but still under investigation.


■ In most smooth muscles, the α1 receptors mediate contraction through activation of Ca2+ dependent myosin light chain kinase but in the
GIT smooth muscles, they mediate relaxation through hyperpolarization caused by opening of Ca2+ dependent K+ channels.
■ α1 receptors have 3 subtypes, A, B, and D; α2 receptors have three subtypes: A, B, and C.
Table 3. Summary of cholinergic receptors

M1 M2 M3 Nn Nm
2nd msngr Gq (↑ IP3 & ↑ DAG) Gi (↓cAMP) Gq (↑ IP3 & ↑ DAG) →↑ Ca2+ Ion channel Ion channel
→↑ Ca2+

Sites and 1. CNS ↓↓ SAN activity and 1. VD of most BV through synthesis of All autonomic NMJ → skeletal
function AV conduction (not endothelial-derived relaxing factor (EDRF) ganglia and muscle
2. Stomach → ↑
atrial conduction) → ↓ blood pressure. adrenal medulla contraction
HCL secretion
2. Contraction of all wall smooth muscles
(bronchi, GIT, UB) and relaxation of all
sphincters.
3. ↑↑ all body secretions (sweating,
salivation, lacrimation, etc).
4. Eye: → miosis & ciliary muscle contraction
(accommodation for near vision).

Selective
Pyrenzepine Gallamine --------- Trimetaphan d-tubocurarine
antagonist
Non-selec
Acetylcholine
agonist
Non-selec
Atropine - hyoscine
antagonist

N.B.
 M4 and M5 are also present in the CNS.
 M1 – M3 – M5 are linked to Gq (↑ IP3 & ↑ DAG).
 M2 – M4 are linked to Gi (↓ cAMP)

45
Part 2
2: Adrrenergic
c agonis
sts (Sym
mpathom
mimetics
s)

Actions and che


emical stru
ucture

■ Thesse drugs act


a either directly
d or indirectly (by release of norep
pinephrine stored),
to aactivate adrenergic
a c receptorrs and mimicm the
e effects of endo ogenous
cate
echolaminees.
■ Not all sympa athomimetiic drugs a activate the adrenergic recepttors by the same
degrree; some e drugs have hig her affinitty toward ds certainn receptor class
(sele
ectivity) depending
d on the u ltrastructu ure of the
ese recepttors; howeever, in
large
e doses, this
t selecttivity is los
st.
■ Chemically, these drugs may conttain catech
hol nucleus
s, (i.e.
cate
echolamin nes) or not (non-cate
echolaminnes).
■ Cateecholamine es (e.g. epinep phrine, norepinephrine,
isoprroterenol, and dopammine) are ccharacteriized by:
 Thhey are noot absorbe nd cannott pass the BBB.
ed orally an Figure 5. Catechol
C
 Thhey have very
v short duration due to rappid inactiv
vation nucleus
byy MAO and d COMT.

█ DIRE
ECT- ACT
TING SYMP
PATHOMIIMETIC DR
RUGS

1. Epinephrine
e (Adrenaline)

Chemistry and pharmaco


p kinetics
 Epin
nephrine is a natural catechola
amine neurrotransmitter. Syntheetic epinep
phrine is
poorly absorb bed from thhe GI tractt and does s BBB to aany extent.
s not pass
 It is administereed parente
erally (i.m. and s.c.). The bsorption iss slow due to local
T s.c. ab
VC.
 Nebulized form ms (inhalation) are alsso available.
 It is metabolizeed rapidly by COMT and MAO. Metabolittes are exccreted in urine.

Mecha
anism and pharmac
cological e
effects
 Epin
nephrine acctivates alll α and β-a adrenocepptors.
 β recceptors meediate their effects thhrough inc
crease intra
acellular cA
AMP.
 α1 reeceptors mediate
m the
eir effects through in
ncrease intracellular IIP3 and DA
AG.

CVS  Increase rate


r (chro notropic effect)
e and
d force (innotropic efffect) of
(The major the cardia
ac muscle (β1); it ma
ay precipittate anginaa in patien
nts with
action) coronary insufficienc
cy.
 Increases c pressurre due to
s systolic t positive
ve inotrop
pic and

46
chronotropic effectts (β1), and
a decreeases dia astolic prressure
(because VD of skeeletal musc cle blood vessels
v (β22) overcommes the
VC producced by α1 rreceptors in skin and splanchnicc vascular beds).
 At high do
oses, VC ((α1) of all vascular beds
b dominates leading
pred
to increas
se both syystolic and diastolic BP.
B
 Increase coronary blood flo
ow due to
o increaseed cardiac work
ation of me
(accumula etabolites),, and β2 stimulation ((VD).
Respirratory  Relaxation
n of bronch
hial smooth muscle (β2).
(
system
m  Decrease bronchial secretionss (α1).
Eye  Mydriasis due to conntraction of
o dilator pupillae muuscle (α1)
Metabo olic  Hyperglyccemia due to stimulattion of heppatic glyco
ogenolysis (β2).
effects
s  Lipolysis and
a increasse free fattty acids in blood.

Therap es (emerge
peutic use ency condittions)

■ Anaphylactic shock:
 It is a life-tthreatening g conditio on (acute
hyp persensitivity reactio on) resultting from
masssive release of histamin ne from
inflaammatory cells in res sponse to exposure
to allergic substance e (e.g. penicillin).
Histtamine cau uses severe hypoten nsion and
bronchoconsttriction by b its eeffect on
histtamine (H1) receptors s.
 Inje
ection of ep pinephrine immediate ely dilates the bronchi (β2), deccreases brronchial
secretions (α α1), and elevates
e B
BP (VC); so, epine ephrine iss considerred the
“phhysiologica e” of hista
al antidote amine as itt can reverrse all its eeffects by actions
on d different re
eceptors.
 It iss given 0.5 5 ml (1:100
00) i.m. (id eally in the
e lateral th
high musclles) and could be
repe eated /5 min
m if no ressponse.

■ Acutte bronch hospasm: within m minutes affter s.c. administrat


a ation, epinephrine
indu
uces bronc chodilation
n (β2) and decrease airway ed dema (α1);; however, short-
actin
ng selectivve β2 agonists could be used with
w fewer CVSC side eeffects.

■ Carddiac arres st: early i.v.


i epinep
phrine adm ministration, during cardiopulmonary
resu
uscitation (CPR),
( cann restore c
cardiac activity (β1) and impro ove vascullar tone
colla
apse (α1). Recomme
R nded dosee is 1 ml off 1:1000 i.v
v. or 10 ml 1:10000 i..v.

■ Withh local anesthetics


a s: diluted concentrrations of epinephrrine (1:100000) or
noreepinephrinee are som
metimes ad dded to loc
cal anesthhetics to p
produce lo
ocal VC;
this will prolo
ong the duration off the anes ue to ↓ aabsorption of the
sthetic (du
anessthetic) and
d reduce bleeding.
b

47
Adverse effects
 Cerebral hemorrhage: due to marked elevation of BP.
 Anginal pain from excessive cardiac work and strain.
 Cardiac arrhythmia: especially if given with digoxin, or if given i.v. (i.v. epinephrine
can produce fatal ventricular fibrillation).
 Acute pulmonary edema: due to increase both systemic and pulmonary
pressures, this cause acute rise of intrapulmonary hydrostatic pressure with
hydrostatic flux of fluid.

Contraindications

 Presence of hypertension: as it may cause cerebral hemorrhage.


 Ischemic heart disease.
 Cardiac arrhythmia or with digitalis: epinephrine can worsen the arrhythmia
and even precipitate ventricular fibrillation.
 Cardiac outflow obstruction (e.g. severe aortic stenosis, severe pulmonary
hypertension, hypertrophic obstructive cardiomyopathy). In these conditions,
epinephrine will increase contractility of the cardiac muscle against a narrow
outlet, so it can precipitate acute pump failure.
 Thyrotoxicosis: because patients with thyrotoxicosis have increased
sympathetic overactivity (tachycardia, tremors, anxiety, etc) due to increased
sensitivity of β receptors.

2. Norepinephrine

 It activates α (mainly) and β1- The baroreceptor reflex


receptors leading to increase both  It is a physiological reflex
systolic and diastolic BP with reflex concerned with regulation of BP.
bradycardia.  Sudden increase of BP →
 It has little activity on β2-receptors. activation of stretch receptors in
 It is used as VC (by slow i.v. infusion) the carotid sinus and aortic arch →
in acute hypotensive states. afferent impulses through the
vagus to the CVS centers in the
medulla→ decrease HR and
3. Dopamine vascular tone. Hypotension has
opposite effect.
 It is a natural catecholamine given by i.v.  These receptors begin to respond
infusion because it has very short at pressures ≥170 mm Hg.
duration of action (2 min).  In chronic hypertension, this set
 In low doses: stimulates dopamine point is shifted to higher level.
D1 receptors in renal and mesenteric  Baroreflex is blocked by atropine.
vascular beds leading to VD and
increase renal and hepatic blood flow.

48
 In intermediate doses: stimulates cardiac β1 receptors leading to increase
contractility and COP.
 In large doses: stimulates vascular α1 receptors leading to VC and ↑↑ BP.

Therapeutic uses
■ Shock states:
 Shock is a complex state of hypotension associated with impaired tissue
perfusion of the vital organs (brain, liver, kidney, etc.).
 Dopamine, given by continuous i.v. infusion, restores adequate tissue perfusion
by increasing COP (β1), and increasing renal blood flow (RBF) and glomerular
filtration rate (GFR; D1). In high doses, it also stimulates vascular α1 receptors
leading to improvement of vascular tone collapse and elevation of BP.
 N.B. If vasopressors (e.g. noradrenaline) are given alone, they will elevate BP but
aggravate tissue ischemia due to VC.

4. β-adrenoceptor agonists:

a. Selective β1 agonist: Dobutamine

 Dobutamine is a synthetic catecholamine that is related to dopamine. It is


administered by i.v. infusion because of its short duration (2 min).
 It activates mainly cardiac β1-receptors with no effect on dopamine receptors
leading to increase COP with little or no vascular effects.
 It is given by continuous i.v. infusion in cardiogenic shock (a complication of left
ventricular infarction), to reverse myocardial depression and increase COP (β1).

Dopamine Dobutamine
‒ Natural catecholamine ‒ Synthetic catecholamine
‒ Stimulates D > β1 > α1 ‒ Stimulates β1 only
‒ Used for treatment of most cases ‒ Used mainly for treatment of
of shock cardiogenic shock

b. Selective β2 agonists:
Salbutamol, terbutaline, salmetrol, formoterol, ritodrine

 They are synthetic non-catecholamines.


 Salbutamol and terbutaline have short duration – salmetrol and formoterol have long
duration.
 They have greater selectivity at β2 receptors leading to relaxation of bronchial
and uterine smooth muscles, and decrease peripheral resistance (VD).
 In high doses, selectivity on β2 receptors is lost, leading tachycardia and even
arrhythmia (β1).

49
Therapeutic uses:
 Treatment of bronchial asthma (see respiratory for more details).
 Ritodrine is commonly used to induce uterine relaxation and delay preterm labor.

Adverse effects
 High doses can cause hypotension (from VD), tachycardia and arrhythmia due to
loss of selectivity.
 Tremors: β2 receptors in skeletal muscles neuromuscular junction facilitate
neuromuscular transmission and induce tremors.

c. Non-selective β agonist: Isoproterenol (Isoprenaline)

 Synthetic catecholamine that predominantly stimulates both β1& β2 receptors.


 It increases HR and contractility, and relax bronchial smooth muscles.
 It is used as a bronchodilator; however, its nonselectivity is one of the major
drawbacks that makes it rarely used therapeutically.

5. α-adrenoceptor agonists:

a. Phenylephrine, methoxamine, and medodrine

 They are non-catecholamines having long duration of action.


 They selectively stimulate α1-receptors leading to VC, and increased both
systolic and diastolic pressures with reflex bradycardia.
 They are used as vasopressors to correct hypotension.
 They could be used locally as eye or nose drops to produce VC and relieve
congestion (i.e. nasal decongestants).

b. Xylometazoline and oxymetazoline

 These drugs stimulate both α1 and α2 receptors but with slight selectivity toward
α1 receptors.
 They are primarily used locally as eye or nose drops to produce VC (nasal
decongestants).

Adverse effects and precautions


‒ Sufficient concentrations may be absorbed systemically and produce severe
hypertension (even stroke); so they must be avoided in hypertensive patients.
‒ Prolonged and continuous use (> 3 weeks) may lead to atrophy of the nasal
mucosa (due to VC); so they should be used for the minimum duration.
‒ Repeated local application leads to rebound severe congestion, so they must be
used with the minimal dose and for the shortest duration.

50
c. Clonidine

 It is a centrally acting α2 agonist leading to decrease central sympathetic


outflow and blood pressure (see page 55).

d. Tizanidine

 It is another centrally acting α2 agonist (congener of clonidine) with greater effect


on presynaptic α2 in the spinal cord, so it inhibits neurotransmission and
reduces muscle spasm with minimal effect on blood pressure.
 It is used as skeletal muscle relaxant in various spastic conditions such as
multiple sclerosis, back pain, and spine injuries.

6. Dopamine receptor agonists: Fenoldopam

 It stimulates peripheral dopamine (D1) receptors in renal and mesenteric arteries,


leading to VD and decrease peripheral resistance.
 It is used parenterally as a rapid-acting vasodilator to treat emergency
hypertension in hospitalized patients.

█ INDIRECT- ACTING SYMPATHOMIMETIC DRUGS

1. Amphetamine and its derivatives

 Amphetamine and its derivatives are


indirect-acting adrenergic drugs. They Attention deficit
mediates their primarily action by blocking hyperactivity disorder
neuronal uptake of dopamine and (ADHD)
norepinephrine and promoting their It is a neuropsychiatric disorder
release from store sites in the CNS and appears after the age 6 years.
synapses. The child is hyperkinetic and
 Amphetamine stimulates the entire CNS, lacks attention in addition to
cortex, brainstem, and medulla. This leads poor school performance.
to increased alertness, decreased fatigue, The amphetamine derivative
depressed appetite, and insomnia. methylphenidate is a CNS
stimulant that can improve
attention spans allowing better
Therapeutic uses
function in school, and reduces
‒ Methylphenidate is an amphetamine hyperkinesia associated with this
derivative used in the treatment of syndrome.
attention-deficit/ hyperactivity disorder
(ADHD) of children.
‒ Modafenil is another derivative used for treatment of narcolepsy.

51
Adverse effects
Narcolepsy is a chronic
‒ High doses cause anxiety, seizures, neurological disorder character-
hypertension, chest pain, and life- ized by intermittent, uncontrolla-
threatening arrhythmia. ble episodes of falling asleep
‒ Psychosis, hallucinations, and drug during the daytime. These
dependence. sudden sleep attacks may occur
during any type of activity at any
time of the day.
2. Cocaine

 Cocaine is an alkaloid derived from the coca plant. It is widely abused as a


recreational stimulant.
 It inhibits neuronal uptake of norepinephrine, dopamine, and serotonin leading
to their accumulation in the synaptic spaces with profound CNS stimulation.
 Adverse effects: similar to amphetamine
 Manifestations of cocaine toxicity is managed by benzodiazepines.

█ MIXED-ACTING SYMPATHOMIMETIC DRUGS

Ephedrine

 Ephedrine is a plant alkaloid effective


Chronic orthostatic
orally and, unlike catecholamines,
hypotension
penetrates the brain and can produce
CNS stimulation. On standing, venous return is re-
 Ephedrine acts by both: duced by the effect of gravity. Nor-
mally, BP decrease is prevented by
‒ Release of norepinephrine from nerve
reflex sympathetic activation with
endings.
increased HR, and peripheral VC.
‒ Direct stimulation of α and β
Impairment of autonomic reflexes
receptors (weak and prolonged).
that regulate BP can lead to chronic
 Tolerance develops to ephedrine after orthostatic hypotension. This might
continuous administration. be caused by drugs that impair
autonomic function (e.g., tricyclic
Therapeutic uses antidepressants and α blockers),
‒ Ephedrine is used as bronchodilator and diabetes, and autonomic neuropa-
thy.
CNS stimulant, but its clinical use is
now declining because of the Drugs that activate α1 receptors
availability of better, more potent agents (e.g. Midodrine) can be used for
this indication. Ephedrine can be
with fewer side effects.
also used (rarely).
‒ Pseudoephedrine is one of the four
isomers of ephedrine. It is present in
many nasal decongestant mixtures.

52
Table 4. Selected therapeutic uses of adrenoceptor agonists (sympathomimetics)

Clinical condition Agonist Receptor


Cardiac arrest Epinephrine β1 – α1
Anaphylactic shock Epinephrine β2 – α1
Shock (most types) Dopamine D1 - β1
Cardiogenic shock Dobutamine β1
Chronic orthostatic hypotension Midodrine, phenylephrine α1
Bronchial asthma Salbutamol, terbutaline, salmetrol β2
Premature uterine contractions Ritodrine β2
Running nose (rhinitis) Oxymetazoline, xylometazoline α1
ADHD Methylphenidate ?
Narcolepsy Modafenil ?

Part 3: Adrenergic receptor antagonists

These drugs interact with either α- or β-adrenoceptors to prevent or reverse the


actions of endogenously released catecholamines or exogenously administered
sympathomimetics.

█ α- ADRENERGIC BLOCKERS

■ Non-selective α-receptor blockers: phenoxybenzamine, phentolamine


■ Selective α1-receptor blockers: prazosin , terazosin, doxazosin
■ Selective α2-receptor blockers: yohimbine
■ The ergot alkaloids.

1. Non-selective α- blockers: Phenoxybenzamine

 Phenoxybenzamine is a noncompetitive, irreversible antagonist at both α1 and


α2 receptors. It binds covalently with α receptors, resulting in long-lasting blockade
(15 –50 h).
 Blockade of α receptors leads to orthostatic hypotension and reflex tachycardia.

Therapeutic uses
■ Management of pheochromocytoma
 Phenoxybenzamine or phentolamine are used for long-term management of
inoperable tumors.

53
 β-receptor antagonists are
often given after α-blockers Pheochromocytoma
to prevent the cardiac effects It is tumor of the adrenal medulla that secretes
of excessive catecholamines excess catecholamines → headache,
(see box). hypertension, palpitations, sweating, and
dyspnea. 10% of the tumors are malignant.
Adverse effects Diagnosis:
 Orthostatic hypotension and  CT scan.
 High levels of VMA in urine
reflex tachycardia.
 Impairment of ejaculation. Treatment:
 Miosis.  Surgical excision of the tumor.
 Combined  and  lockers to block all
adrenergic receptors:
2. Selective α1- blockers: ‒ Phenoxybenzamine 100 mg /day +
Prazosin, terazosin, propranolol 50 mg/day.
doxazosin, tamsulosin ‒ Start first with phenoxybenzamine to
control BP then add -blocker.
 Prazosin is the prototype drug. ‒ If α-blockers are used alone, the
 All of these agents decrease elevated catecholamines will act on
peripheral resistance and unopposed β-receptors leading to severe
lower arterial BP by: palpitations, arrhythmia, etc.
‒ If β-blockers are used alone, the
 α1- receptor blockade.
elevated catecholamines will act on
 Direct VD of both arterial unopposed α-receptors leading to severe
and venous smooth hypertension.
muscles.  Labetalol is a combined  and  lockers
 They cause minimal changes that could be used alone.
in COP, RBF, and the GFR.
 They don’t trigger reflex tachycardia by the same degree as the non-selective
blockers.
 They improve plasma lipid profile and decrease LDL and TGs.
 Doxazosin has the longest duration of action (22 h).

Therapeutic uses
 Treatment of mild-to-moderate hypertension: especially in patients with renal
failure because it does not decrease RBF or GFR.
 Treatment of congestive heart failure because they decrease both the afterload
and preload through combined arteriolo- and veno- dilatation (see CVS).
 Benign prostatic hyperplasia (BPH) and impaired bladder emptying because
blockade of α1 receptors in smooth muscles of the bladder neck and prostate
leads to decrease resistance to urine flow. The old drug prazosin is no longer
recommended for this indication.

54
Tamsulosin is the most commonly used
Salt and water retention
for treatment of BPH because:
induced by BP lowering
‒ It has high affinity for α1A & α1D, the 2 drugs
receptor subtypes responsible for  It occurs as a compensatory
mediating smooth muscle contraction response after long duration of
in prostatic tissue. antihypertensive therapy in the
‒ It has little effect on standing BP form of ankle edema and slight
compared with other α1-blockers. weight gain.
 Hypotension leads to reflex
Adverse effects stimulation of the renin-
angiotensin-aldosterone system
 First dose hypotension (syncope) which causes fluid retention.
‒ Occurs more frequently with prazosin.
 Diuretics are often prescribed
It starts 30-90 min after the first dose. with BP lowering drugs to
‒ It occurs more frequently in salt and minimize this effect.
water depleted patients.
‒ Prevention: start with a small dose at bedtime then increase the dose gradually.
 Fluid retention (salt and water retention): (see box).
 False positive test for antinuclear factor of rheumatoid arthritis.
 α- blockers can worsen incontinence in women with pelvic floor pathology.

3. Selective α2- blockers: Yohimbine

 Selective presynaptic α2-blocker that leads to increase norepinephrine release.


 It is sometimes used as aphrodisiac (enhance sexual desire) without clinical
evidence.

4. The ergot alkaloids

Chemistry and pharmacokinetics

 Ergots are a wide variety of compounds that are produced by the fungus
Claviceps purpurea. These agents have a strong structural similarity to
norepinephrine, dopamine, and serotonin.
 They may be natural or semi-synthetic:

Natural alkaloids Semi-synthetic alkaloids


 Ergotamine  Dihydroergotamine
 Ergometrine  Methylergometrine
 Ergotoxine: a mixture of three other alkaloids. It  Dihydroergotoxin
is very toxic and not used clinically.  Bromocryptine

55
 Ergots may be administered parenterally, rectally, or orally, and vary widely in
their degree and speed of absorption.
 The absorption of ergotamine is increased by caffeine.
 Ergots are extensively metabolized to compounds of varying activity and half-life.

Mechanism and pharmacological effects

 Ergots act as agonist, antagonist or partial agonists at three receptor types: α-,
dopamine, and serotonin receptors.
 The pharmacologic use of ergots is determined by the relative effect of each
member on these receptors.

Drug Receptor Main effect Main uses


Ergotamine Partial agonist at Direct VC with greater Acute migraine
α- and 5-HT effect on cerebral BVs attacks
receptors
Methlyergometrine α-receptors Uterine smooth Postpartum
agonist muscle contraction hemorrhage
Dihydroergotoxin α-receptors VD with greater effect Senile cerebral
antagonist on cerebral BVs insufficiency
Bromocryptine Dopamine agonist Pituitary and ne- Suppress lac-
grostriatal pathway tation and ttt of
parkinsonism

Therapeutic uses

■ Ergotamine: acute migraine


‒ The major effect of ergotamine is cerebral VC by a direct action; it reverses the
rebound VD that is the probable cause of pain.
‒ It should be given at the start of aura because it has slow onset. It is better to
be combined with caffeine because caffeine increases its absorption.
‒ If given before aura (e.g. for prophylaxis), it can induce cerebral VC and
precipitates the acute attack (i.e. contraindicated).
‒ The VC induced by ergotamine is long-lasting and cumulative; therefore, pat-
ients must not take more than 6 mg of the oral preparation for each attack.

■ Methylergometrine: postpartum hemorrhage


‒ It causes prolonged and forceful contraction of uterine smooth muscles.
‒ Ergots should not be used to induce labor. It should be given at the time of
placenta delivery (3rd stage of labor) and never before that. If it is given before
delivery of the placenta, it causes severe spasm of uterine smooth muscles
and retained placenta.

56
■ Brommocriptine e: hyperprrolactinem mia
‒ Bromocriptin ne is a dopamine rec ceptor agoonist that causes
c inhiibition of prolactin
p
seecretion (high prolactin levels c
can induce infertility and
a ameno orrhea in women).
w
‒ It is used to suppre ess norma al lactation
n and as a dopam ine alterna ative in
Paarkinson’ss disease

■ Dihy oxin: used


ydroergoto d as cerebrral vasodila
ator in senile cerebraal insufficie
ency.

Advers
se effects of ergot alkaloids
a
 Nauusea and vomiting
v due to stim ulation of
CTZZ.
 High doses ca ause VC of
o small artterioles of
fing
gers leadinng to coldd hands a and even
ganngrene (erg
gotism).
 VC of coronarry artery with
w anginall pain.
 Uteerine contrraction and
d abortionn if given
duriing pregna
ancy.

█ MIGR
RAINE AND
D SOME DR
RUG TREAT
ATMENTS

ne is severre unilatera
Migrain al periodic headache characterized by:

- The stage of aura:


a occu
urs in 15-3 0% of
casees. Suddenn release of
o serotonin n (and
otheer mediatoors) of unnknown ettiology
caussing VC ofo cerebra al BV → vvisual,
olfac
ctory or au
uditory hallucinationss.
- The stage of pain: prolonged VC leads
accuumulation of waste e metaboliites →
seveere VD → perivascu ular edem a and
seveere headacche.

▌Drugs
s used in the
t acute attack
■ Ergo
otamine and dihydro mine (see before).
oergotam
■ Triptans: sum
matriptan, zolmitript
z an
– Theey are agon
nists at 5-H
HT1D and 5
5-HT1B rece
eptors.
– Activation off 5-HT1D receptorss inhibits inflammation of meningess, pain
tran
nsmission, and release of VD ssubstances citonin genne-related peptide
s e.g. calc
in trigeminal neurons. Activation
A of 5-HT1BB receptorss causes V VC of the dilated
cereebral vesssels. Abou ut 50%–800% of pattients repo ort relief frrom pain within
w 2
houurs after oral administration.

57
– 5-HT1B activity can cause coronary spasm so; these drugs are contraindicated
in patients with ischemic heart disease (IHD).
– Like ergotamine, they are also contraindicated for prophylaxis of migraine or in
combination with ergotamine because severe hypertension and coronary spasm
may occur.

▌Drugs used for prophylaxis


 Propranolol: the most commonly used drug for prophylaxis.
 Ca2+ channel blockers: verapamil (unclear mechanism).
 Clonidine: (unclear mechanism).
 Tricyclic antidepressant (TCA) drugs.

█ β- ADRENERGIC BLOCKERS

The β-receptor–blocking drugs differ in their relative affinities for β1 and β2


receptors; however, the selectivity is dose-related and tends to diminish at higher
doses.

■ Non-selective β-blockers: e.g. propranolol, pindolol, timolol, sotalol, nadolol


■ Cardio-selective β1-blockers: e.g. atenolol, metoprolol, bisoprolol, nebivolol.
■ β-blockers with additional VD action: e.g. dilevalol, carvedilol
N.B. the VD action comes from either: blocking the vascular α1 receptors;
increasing PGE2 and PGI2 synthesis; or by release of endothelial NO.

Chemistry and pharmacokinetics

 Propranolol is the prototype β-adrenoreceptor antagonist.


 β-blockers are absorbed well after oral administration, many have low
bioavailability because of extensive first-pass metabolism.
 Lipophilic β-blockers (e.g. propranolol) can pass readily to the CNS and are
cleared by hepatic metabolism. Hydrophilic β-blockers (e.g. atenolol) have
limited penetration to the CNS and are excreted primarily by the kidney with little
hepatic metabolism.
 β-blockers which undergo hepatic metabolism usually require multiple daily
dosing. Drugs eliminated via the kidney are suitable for once daily administration.

Mechanism and pharmacological effects

CVS effects  They block cardiac β1 receptors and decrease all cardiac
properties (↓ contractility and COP, ↓ A-V conduction
"bradycardia", ↓ excitability, and automaticity).

58
 They block the β2-mediated VD in peripheral vessels leading to ↓
blood flow to most tissues.
 They decrease blood pressure through:
 ↓↓ COP by their –ve inotropic and chronotropic effects.
 ↓↓ renin release from the kidney (β1).
 ↓↓ norepinephrine release and central sympathetic outflow (by
blocking presynaptic β2).
 They cause resetting of baroreceptors to a lower level (see
before).
 Some β-blockers block also vascular α1 receptors.
 Some β-blockers enhance synthesis of vasodilator PGE2 and
PGI2.

Respiratory  β-blockers produce bronchospasm as a result of β2-receptor


blockade; this effect is more significant in asthmatic patients.
 Even selective β1 blockers can interact with β2 receptors in high
doses and produce bronchospasm, especially in asthmatic
patients.

Eye  ↓ IOP by decreasing aqueous humor secretion from the ciliary


epithelium (timolol and betaxolol have excellent effect).
 Sufficient timolol can be absorbed after topical application to
increase airway resistance and decrease HR and contractility.

Metabolic  Inhibition of glycogenolysis in the liver (β2) leading to aggravation


effects of hypoglycemic effect of insulin and other hypoglycemic drugs.
 ↑ plasma K+ (hyperkalemia) in patients with renal failure.
 ↑ plasma triglycerides and ↓ HDL.

CNS  Antianxiety effects.


 Night mares, vivid dreams, and depression.
 Sexual dysfunction through combined central and peripheral
mechanisms.

Skeletal ms  ↓ essential tremors due to blocking of β2 in skeletal muscles.

Other  Propranolol has local anesthetic (membrane stabilizing) action i.e.


specific it can inhibit excitability of the cardiac muscle.
properties  Pindolol is a partial agonist i.e. it doesn’t cause excessive
bradycardia.
 Esmolol is ultrashort acting (t1/2 = 10min) because of extensive
hydrolysis by plasma esterases; it is administered by i.v. infusion to
control arrhythmia during surgery and emergency situations.
 Labetalol blocks β-receptors and α1-receptors (mixed blocker).

59
 Carvedilol has additional antioxidant action.
 Nebivolol is the most selective β1 blocker.

Therapeutic uses

■ Treatment of hypertension: (see the mechanisms above). All beta-blockers,


irrespective of their properties, lower BP to a similar extent.

■ Ischemic heart disease (classic angina and acute MI)


‒ They decrease contractility, HR, and systolic BP, thus decrease myocardial work
and O2 demand (in acute myocardial infarction (AMI), -blockers given within 6-
12 h can decrease the infarct size).
‒ They increase the diastolic (coronary) filling time.
‒ They cause redistribution of blood to the ischemic (subendocardial) regions.
‒ β-blockers improve myocardial metabolism through metabolic switching from
fat utilization to carbohydrates utilization (cytoprotective effect).

■ Cardiac arrhythmias (especially thyrotoxic and supraventricular arrhythmia).


‒ They decrease A-V conduction, lengthen the refractory periods of the SA node,
and suppress automaticity.
‒ Propranolol decreases excitability through its membrane stabilizing action.
‒ Acute arrhythmia during surgery is treated by i.v. esmolol.

■ Hypertrophic obstructive cardiomyopathy


‒ It is a congenital thickening of the ventricular wall and interventricular septum.
It is the most common cause of sudden death in young athletes. Thickening of
the interventricular septum impairs blood flow through the aortic outlet
especially during exercise.
‒ Drugs having –ve inotropic effect such as β-blockers and verapamil decrease
HR and contractility, thus decrease the outflow tract resistance. Drugs with +ve
inotropic effects (e.g. digoxin) have the opposite effect.

■ Hyperthyroidism: propranolol is used to control tachycardia, anxiety, and


tremors due to sympathetic overactivity in hyperthyroidism. It also prevents
peripheral conversion of T4 into T3.

■ Esophageal varices due to liver cirrhosis: propranolol is used to decrease


portal and hepatic blood flow through combined decrease of COP and inducing
VC in the splanchnic vascular bed (through an unopposed α-action).

■ Open angel glaucoma: topical timolol or betaxolol ↓ aq humor secretion.

■ Other uses: pheochromocytoma (must be combined with alpha-blockers); and


prophylaxis of migraine (propranolol has an unclear mechanism).

60
Adverse effects
Heart block
 Tiredness and fatigue (the most comm-
Heart block means block of the
on side effect) due to reduced COP and
electrical conduction at any point in
block of β2-mediated VD in skeletal the conducting system e.g. SA nodal
muscles (mainly non-selective agents). block, AV nodal block, or bundle
 Bradycardia and impairment of branch block.
myocardial contractility, so they can
precipitate heart failure or heart block
N.B.
in patients with compromised cardiac
Excessive myocardial depression
function. caused by overdose of β-blockers
 Bronchospasm in susceptible can be reversed by i.m. glucagon.
individuals due to blockade of β2- This is because β-blockers decrease
receptors which mediate dilation in the intracellular cAMP making all β-
agonists acting through cAMP is
bronchi. Asthma is an absolute
useless. Glucagon increases
contraindication for all beta-blockers.
contractility by a mechanism
 Aggravation of peripheral ischemia unrelated to cAMP.
and cold extremities (mainly non-
selective agents). (selective β-blockers
are the preferred class if there is associated peripheral vascular disease).
 In diabetic patients, β-blockers (mainly non-selective) can potentiate the
hypoglycemic effect of insulin and oral hypoglycemic drugs (because they block
glycogenolysis), and mask tachycardia & tremors resulting from severe
hypoglycemia.
 CNS effects: vivid dreams, night mares, and depression.
 Sudden withdrawal can increase the risk of angina and arrhythmias due to
adrenoceptor “supersensitivity”. Gradual withdrawal is recommended.

Contraindications

■ Absolute contraindications
 Bronchial asthma.
 Any degree of heart block.
 Prinzmetal’s (vasospastic) angina (see CVS).
 Sudden withdrawal after long-term use.

■ Relative contraindications
 Acute – or severe chronic heart failure.
 Peripheral vascular diseases (PVD).
 Diabetes mellitus.
 In athletes involved in strenuous sports because beta-blockers can interfere
with the ability to perform strenuous physical activities.

61
Part 4: Sympathoplegic drugs

█ CENTRALLY-ACTING SYMPATHOPLEGIC DRUGS

1. Alpha-methyldopa

Mechanism and pharmacological effects


In the CNS, α-methyldopa competes with dopa for dopa decarboxylase enzyme
leading to formation of α-methylnorepinephrine, (and also α-methyldopamine), which
is a false transmitter. α-methylnorepinephrine stimulates central α2 receptors
leading to reduced central sympathetic outflow and decreased BP.

Therapeutic uses
Methyldopa is the drug of choice to treat arterial hypertension in pregnancy
because of its long and reliable track record.

Adverse effects
 The most common side effect is sedation, nightmares, and mental depression
due to central deficiency of norepinephrine.
 Mild hyperprolactinemia and extrapyramidal manifestations due to central
deficiency of dopamine.
 Positive Coombs test and autoimmune hemolytic anemia.
 Autoimmune hepatitis is rare.

2. Clonidine

Mechanism and pharmacological effects


 Clonidine is a central α2-receptor agonist leading to decreased central
sympathetic outflow and reduction in the total peripheral resistance.
 Reduction of BP is not associated with changes of the RBF or GFR.

Therapeutic uses
 It is mainly used in the management of hypertension complicated by renal disease.
 To reduce anxiety accompanying opiate withdrawal or surgical operations.

Adverse effects
 Sedation and dry mouth (central effect).
 Sudden withdrawal of the drug can lead to rebound hypertension.
 Salt and water retention so it is usually combined with diuretics.

62
█ ADR
RENERGIC NEURON BLOCKERS
B S

Reserpine

Mecha
anism and pharmac
cological e
effects
 Reseerpine is a plant alkaloid thaat blocks vesicular
v
ake of th
upta he neurottransmitterrs norepinephrine,
dopaamine, an nd seroto onin in bboth cen ntral and
perip
pheral neurons, as well
w as adre enal medullla.
 Thesse transm mitters acccumulate in the neuronal
cyto
oplasm an nd are de egraded bby MAO enzyme,
leading finally to deple etion of thhe nervouss system
from
m these bio ogenic amines.
 The effect of reeserpine iss slow and
d persists for many
dayss after disc
continuatio
on.

Therap
peutic use
es
Trea
atment of mild-to
m mooderate hyypertension; howeve
er, it is now
w not con nsidered
amoong the firsst or second line drug
gs becausee of numerrous adverrse effects.

Advers
se effects
 The most impo ortant side
e effect is s
sedation, nightmare es, and meental deprression
(2%)) due to ceentral deple
etion of booth norepin
nephrine and serotonnin.
 Hype erprolactin
nemia and extrapyra amidal man nifestations
s (parkinso
onism) may occur
due to central depletion of dopamiine.
 GIT symptomss (abdomin nal crampss, mild diarrhea, increase HCl) are comm mon due
to ovverpredomminance of parasymp pathetic activity.

63
Part 5: Parasympathomimetic drugs (Cholinomimetics)

■ The parent compound of all cholinomimetic drugs is acetylcholine.


■ ACh is the natural neurotransmitter in the following sites:
‒ All autonomic ganglia whether sympathetic or parasympathetic.
‒ Parasympathetic nerve endings to involuntary organs and exocrine glands.
‒ Sympathetic nerve endings to thermoregulatory sweat glands.
‒ Sympathetic nerve endings to adrenal medulla.
‒ Skeletal muscle motor end plate.
‒ Certain tracts within the CNS
■ ACh acts on both muscarinic and nicotinic receptors to produce all the effects
listed in table 1, and table 3 (see before).
■ ACh is not used clinically because: (1) it has very short duration of action (seconds)
due to rapid hydrolysis by AChE enzyme; and, (2) it lacks selectivity.
■ Cholinomimetic drugs are drugs that produce effects similar to ACh or cholinergic
nerve stimulation, but with more selectivity and fewer side effects than ACh.

Classification of cholinomimetic drugs

Direct-acting cholinomimetics Indirect-acting cholinomimetics


They act by direct stimulation of They act by inhibition of AChE enzyme
cholinergic receptors leading to accumulation of ACh.

■ Muscarinic agonists ■ Reversible ChE inhibitors:


Bethanecol, carbachol Physostigmine, neostigmine,
Pilocarpine, cevimeline pyridostigmine, donepezil

■ Nicotinic agonists ■ Irreversible ChE inhibitors:


Nicotine, lobeline Organophosphate compounds

█ DIRECT-ACTING PARASYMPATHOMIMETICS

█ Muscarinic agonists:

Pharmacological effects:

CVS  ↓ AV conduction and HR (M2).


 VD of all vascular beds through release of EDRF (M3).
Respiratory  Contraction of bronchial smooth muscle (M3).
effects  ↑ bronchial secretions (M3).

64
Eye  Miosis due to contraction of constrictor pupillae muscle (M3).
 Accommodation for near vision due to contraction of ciliary
muscle (M3).
 ↓ IOP (contraction of the ciliary muscle causing opening of the
trabecular meshwork and facilitates drainage of aq humor).
GI tract  ↑ motility and relaxation of sphincters (M3).
 Salivation (M3) and increase HCl secretion (M1).
Urinary tract  Contraction of bladder smooth muscles (M3).
 Relaxation of sphincters (M3).
Exocrine gld  ↑ all exocrine secretions, salivation, lacrimation, sweating, etc.

1. Carbachol

 It is choline ester but resistant to hydrolysis by AChE enzyme.


 It stimulates both muscarinic and nicotinic receptors.
 It is used as local eye drops to ↓ IOP in glaucoma. It contracts the ciliary ms
causing opening of the trabecular meshwork and facilitates drainage of aq humor.

2. Bethanecol

 It is a choline ester but resistant to hydrolysis by AChE enzyme, so it has long


duration of action (2-3 h) as compared to Ach.
 It stimulates muscarinic receptors with no activity on nicotinic receptors.
 It is used to reverse post-operative urine retention and paralytic ileus (in absence
of organic obstruction).

N.B.
 Bethanechol is administered orally or s.c., not by i.v. or i.m., because parenteral
administration may cause cardiac arrest.
 Bethanechol is contraindicated to treat urine retention due to mechanical
obstruction of the bladder or intestine because increasing contraction against a
closed outlet can lead to rupture of the viscus.

3. Cevimeline and pilocarpine

 Cevimeline is synthetic drug – pilocarpine is a natural plant alkaloid.


 Both drugs act as muscarinic agonists with no nicotinic effects.
 Both drugs can be given orally to increase salivary secretion and decrease
symptoms of dry mouth (xerostomia) associated with Sjögren syndrome.
 Pilocarpine is used as local eye drops to ↓ IOP in glaucoma.

65
se effects of musca
Advers arinic agon
nists
‒ Mosst importaant side effects
e inc
clude nausea, vomiting, sweeating, salivation,
nchoconsttriction, hypotension
bron n, and diarrhea; all of
o which caan be bloc
cked by
atro
opine.

Contra
aindication
ns of muscarinic ag
gonists
‒ Pepttic ulcer
‒ Bron
nchial asthhma
‒ Hearrt block.

█ Nico
otinic ago
onists:

1. Nicotine

 It is a compon
nent of cig
garette sm
moke. It
is a poison with
w many adverse e effects
and no therap peutic benefit.
 The overall effects of o nicotin e are
commplex and d result from mixed
stimulation and inhibition o of all
autoonomic gan nglia:
‒ Sm mall dos ses stimu ulate autoonomic
gaanglia le eading to o hyperte ension,
ta
achycardia a, increase
e GIT perisstalsis,
in
ncrease HClH secrettion, and CNS
sttimulation.
‒ To oxic doses lead to hypotensio
h on and
C
CNS depre ession du ue to ga anglion
bllockade.
 Nicootine is adddictive substance. TTrans-
dermmal patch hes containing nicotiine are
usedd to help sm
mokers stopp smoking..

2. Varrenicline

 It is nicotinic receptor partial ag


gonist
used
d for smok king cessattion.
 Headache an nd nauseaa are the most
com
mmon adve erse effects
s.
 Conttraindicateed in pregnancy
p y and
brea
ast feeding.

66
█ INDIRECT-ACTING PARASYMPATHOMIMETICS
(Cholinesterase inhibitors)

Mechanism and pharmacological effects

 Indirect-acting parasympathomimetics inhibit AChE enzyme resulting in


accumulation of ACh and stimulation of both muscarinic and nicotinic receptors.
 They are classified, according to nature and duration of AChE inhibition, into
reversible and irreversible inhibitors.

█ Reversible AChE inhibitors:

They interact with AChE enzyme by making reversible bond allowing duration of
inhibition lasting from minutes to hours.

1. Physostigmine

 Natural plant alkaloid (tertiary amine) that is well-absorbed from the GIT and can
pass to CNS.
 It can reversibly inhibit AChE enzyme for 3-4 hours, leading to:
 Muscarinic effects: hypotension, bradycardia, salivation, lacrimation,
increased GIT peristalsis (diarrhea and colic), miosis, etc.
 Nicotinic effects: skeletal muscle contraction.
 Central effects: headache, insomnia, excitation, and convulsions.

Therapeutic uses
 Because of lack of selectivity and harmful CNS effects, it is usually used as local
eye drops to produce miosis and treat chronic glaucoma.
 Physostigmine can be used to reverse the central and peripheral manifestations
of atropine poisoning.

2. Neostigmine

 Synthetic drug (quaternary amine) that is poorly absorbed from the GIT and
cannot pass to CNS.
 It is similar to physostigmine in mechanism and effects but it has no CNS
actions.

Therapeutic uses
 To reverse postoperative urine retention and paralytic ileus. It is contraindicated
if there is mechanical obstruction (to avoid rupture of the bladder or intestine).

67
 To reverse postoperative muscle Myasthenia gravis
paralysis resulting from the use of non-
 Myasthenia gravis is an
depolarizing neuromuscular blockers.
autoimmune disease in which
 Treatment of myasthenia gravis: antibodies complex with
 Neostigmine not only increases ACh nicotinic receptors at the
level in the neuromuscular junction neuromuscular junction to
but also can directly stimulate nicot- cause skeletal muscle weakness
inic receptors at the motor end plate.  AChE inhibitors, such as
 Atropine could be given with pyridostigmine, are used to
neostigmine to block the unwanted increase ACh levels at the
muscarinic effects caused by neuromuscular junction to fully
activate the remaining
excessive ACh.
receptors.
 Myasthenia gravis can be
3. Pyridostigmine diagnosed using the Tensilon
test, which can also assess the
 Reversible AChE inhibitor similar to adequacy of treatment with
neostigmine. AChE inhibitors.
 It is more preferred than neostigmine in
the chronic treatment of myasthenia
Alzheimer’s disease
gravis because:
‒ It has more selective action on  Alzheimer’s disease is chronic
neuromuscular junction (fewer degenerative disease
characterized by progressive
unwanted muscarinic effects).
impairment of memory and
‒ It has longer duration of action than cognitive functions.
neostigmine.
 Pathologic changes include
increased deposits of amyloid
4. Edrophonium β peptide and abnormal
protein (tau protein) in the
 It acts as the same of neostigmine and cerebral cortex, leading to
pyridostigmine but has very short cerebral vascular lesions, and
progressive loss of
duration of action (5-15 minutes).
cholinergic neurons.
 It is used in the diagnosis of
myasthenia gravis and to differentiate  Although evidence for the
benefit of AChE inhibitors is
between muscle weakness due to
statistically significant, the
insufficient treatment of myasthenia, or clinical benefit from these
due to excessive treatment with AChE drugs is mild and temporary.
inhibitors (Tensilon test).

 Tensilon test: small doses of edrophonium improve muscle strength in


untreated patients with myasthenia, but worsen muscle weakness if it was due to
excessive dose of AChE inhibitors (excessive ACh stimulation at the neuro-
muscular junction results in muscle weakness due to maintained depolarization).

68
5. Don
nepezil an
nd rivastigmine

 Theyy are AChE


E inhibitors
s that act m
more selec
ctively on central
c ChE enzym
AC me.
 Theyy are used
d to increas
se ACh levvels in the CNS and thus imprrove memoory and
cogn
nitive defic
cit associatted with A s disease (see box).
Alzheimer’s

█ Irrev
versible ChE
C inhib
bitors:
Organnophosphate comp pounds

 Theyy include:
 D
Drugs: echhothiophate eye dropps
 Insecticide
es: parathiion and maalathion
 N
Nerve gasses: sarin and
a soman n

 Orgaanophosph
hates are highly lip id soluble
e and rapidly absorb
bed by alll routes
inclu
uding the skin.
s Their CNS pene
etration is rapid and high.
 Theyy interact with AChE enzymee by makin
ng irreversible (covvalent) bo
ond (i.e.
phossphorylatio
on of the enzyme).
 As time passe es, the stre
ength of th
he bond inncreases, (a processs called “a
aging”),
and AChE bec comes irreversibly inhibited. (With mos st types o
of organopphosph-
ates,, 50% of th
he enzymee undego aaging after 3 hrs and 95% after 12 hrs).
 Once AChE is
i inhibited, ACh a
accumulate
es throughout the nervous system,
s
caussing musca
arinic and nicotinic ssymptoms.
 Echo
othiophatte is the only
o non-a
absorbable hosphate. It is available as
e organoph
miottic eye dro
ops for gla
aucoma. It s effect in the eye lasts for weeeks.

Manife
estations of
o organop
phosphate
e toxicity:
‒ CVSS: hypoten
nsion, bradycardia, sw
weating.
‒ Resspiratory: bronchosp
pasm, increease bronc
chial
secrretions, resspiratory ms
m paralys is.
‒ GITT: abdominal colic, diarrhea, an nd salivatioon.
‒ Eyee: severe miosis
m point pupil)), lacrimatiion.
(pinp
‒ CNS S: hallucinations, con nvulsions, and coma a.
‒ Skeeletal ms: twitches
t and
a fascicu ulation.
 Thee cause of death is re espiratory fa
ailure (block
ked
airw
way, paralyzzed respirattory ms & in
nhibited RC C).

Manag
gement
 Ensu
ure patent airway and d artificial respiration
n.
 Gasttric lavage
e and skin wash
w to re
emove the toxin.

69
 Intra
avenous no
ormal saline to raise BP.
 The triad: atro
opine – pralidoxime
e – diazepa
am

opine (2 mg
Atro g i.v. bolus
s)
- A
Atropine is non-selec
ctive musc
carinic blocker and can
c cross BBB to block
b all
m
muscarinic manifestaations of exxcess ACh centrally and perip pherally.
- C
Check pulsse and BP after 5 min n; if no res
sponse, rep peat the doose of atro
opine till
th
he HR is > 80 bpm and
a systolic c BP > 80 mmHg.
- T
The patien nt should be main ntained atropinized
a for 24-448 hrs because
b
o
organophossphates arre highly li pid solublee. So, it may
m dissolvve in body fat and
re
eleased ag
gain over tiime.

Pralidoxime (P
PAM; 2 gm
m i.v. over 2
20-30 min))
- Itt is also avvailable as ready-to-uuse autoinjjector.
- Iff given eaarly (before e aging), iit can rea
activate
(d
dephospho orylate) AC ChE enzy me espec cially at
th
he neurom muscular junction.
- P
Pralidoxime e is only efffective in o
organopho osphate
to
oxicity (i.e. it does not
n have a an effect if
i AChE
e
enzyme iss carbam mylated, a as occurs with
n
neostigmine or physo ostigmine).

zepam (10
Diaz ontrol convulsions.
0 mg i.v. orr i.m.): to co

5. Selective therapeu
Table 5 utic indicattions of pa homimeticss
arasympath

Clinica
al conditio
on D
Drug Recep
ptor
Postop
perative urine retentio
on B
Bethanechool (direct) M3
and pa
aralytic ileus N
Neostigmine (indirect)) M&N
Glauco
oma P
Pilocarpine, M3
C
Carbachol, physostigmine M&N
Xerosto
omia C
Cevemeline
e M3
Alzheim
mer’s disea
ase D
Donepezil, rivastigmin
ne M&N
Myasth
henia gravis N
Neostigmine, pyridosttigmine Nm
Diagno
osis of mya
asthenia E
Edrophoniu
um Nm
Atropin
ne toxicity P
Physostigm
mine M&N

70
Part 6: Muscarinic antagonists

Actions and chemical structure


They are either tertiary amine alkaloids or quaternary amines:
 Plant alkaloids: atropine is found in Atropa belladonna and scopolamine
(hyoscine) is found in Hyoscyamus niger. They are tertiary amines (i.e. well
absorbed and can pass to CNS).

 Synthetic derivatives: are either tertiary or quaternary amines (limited CNS


penetration):
 Drugs used mainly as bronchodilators: Ipratropium
 Drugs used mainly as antispasmodics: Hyoscine butylbromide
 Drugs used mainly to decrease HCl secretion: Pirenzepine
 Drugs used mainly for genitourinary system: Oxybutynin, tolterodine
 Drugs used mainly as mydriatics: Homatropine, tropicamide
 Drugs used mainly to treat parkinsonism: Benztropine

Mechanism and pharmacological effects


Muscarinic-receptor antagonists are competitive antagonists of ACh at all
muscarinic receptors.

CVS effects  They block M2 receptors in the SA node and increase HR.
 No significant effect on the force of contraction because there
are no muscarinic receptors, or parasympathetic innervation of the
ventricles.
 Blockade of vascular M3 receptors has no significant clinical
value.
 High doses cause toxic VD in the facial blush area (atropine flush)
which is not related to the antagonistic action.
Respiratory  Bronchodilatation and decrease mucus secretion.
GIT  Decrease salivation and HCl secretion.
 Decrease motility (antispasmodic action).
Urinary  Relaxation of the bladder smooth muscles and contraction of the
bladder sphincters leading to urine retention.
Sweat  Blocking of muscarinic receptors in thermoregulatory sweat glands
glands (cholinergic) leading to dry skin and elevation of body temperature
(atropine fever).
 Children are more sensitive to this effect.
Eye  Passive mydriasis due to paralysis of constrictor pupillae muscle.

71
 Cycloplegia (paralysis of ciliary muscle) leading to loss of
accommodation for near vision.
 Increase IOP due to mydriasis (decrease aqueous humor
drainage).
CNS  Tertiary amines can produce sedation, amnesia, delirium, and
hallucinations.

Therapeutic uses

CVS:
■ Bradycardia: parenteral atropine is the standard drug for most cases of
bradycardia including reflex bradycardia caused by vasopressor drugs.

Respiratory:
■ Bronchial asthma: Ipratropium is a quaternary amine. It has greater selectivity
for the bronchial tissue and limited CNS effects. It is given by inhalation to dilate
the bronchi and reduce secretions in asthma and chronic obstructive pulmonary
disease (COPD).
■ Preanesthetic medication: Preanesthetic injection of atropine is used in order
to:
 Prevent bronchoconstriction and reduce bronchial secretions caused by
excessive vagal stimulation during anesthesia.
 Protect the heart from excessive vagal tone (bradycardia) occurred during
anaesthesia.

GIT disorders:
■ Peptic ulcer: pirenzepine has greater selectivity for blocking M1 receptors in the
stomach and reduce HCl secretion; however, it is now rarely used because of
the availability of new and more potent drugs.
■ Diarrhea: the classic combination of atropine with diphenoxylate, (a congener
of meperidine), is available under many names (e.g, Lomotil). They decrease
hypermotility and secretions.
■ Abdominal colic: e.g. hyoscine butylbromide (Buscoban).

Urinary disorder:
■ Acute cystitis: oxybutynin is used to decrease bladder spasm and urinary
urgency associated with inflammatory bladder disorders.
■ Urine incontinence in adults: tolterodine is a new muscarinic antagonist used
for this indication because it has greater selectivity for bladder M3 receptors
and has long duration of action.

72
Eye:
■ Funduscopic examination: muscarinic antagonists are used as eye drops
(cyclopentolate; tropicamide) to produce mydriasis and cycloplegia and
facilitate retinal examination; however, phenylephrine (α-agonist) is preferred for
simple fundus examination due to its short duration.
■ Iridocyclitis: inflammation of the iris can cause adhesions between the iris and
lens (synechia). Long acting atropine eye drops is used to produce complete
cycloplegia and mydriasis (M3) to prevent this adhesion.

CNS:
■ Parkinson’s disease: benztropine has Motion sickness
greater selectivity for blocking the  It is a very common disturbance
muscarinic receptors in the basal of the inner ear that is caused by
ganglia and decrease the excitatory repeated motion such as from the
effect of ACh. movement of a car or ship.
 Bizarre head movement affects the
■ Motion sickness: scopolamine
organs of balance and equilibrium
(hyoscine) is the standard drug used for (vestibulocerebellar apparatus)
this indication. It blocks muscarinic causing nausea and vomiting.
receptors in the vestibulocerebellar  Overactivity of muscarinic
pathway that are responsible partially receptors is suspected to play an
for the nausea and vomiting. important role in this condition.

Other:
■ Organophosphate toxicity: atropine is the standard drug (see before).

Adverse effects
 Blurred vision (due to mydriasis and cycloplegia).
 Rise of IOP (glaucoma).
 Dryness of all body secretions: dry mouth, dry skin, dry eyes, etc..
 Urine retention especially in patients with senile enlarged prostate.
 Tachycardia.
 In children: atropine fever (due to blockade of thermoregulatory sweating
resulting in hyperthermia) and flush . Children are more sensitive to this effect.

Contraindications
 Narrow angle glaucoma
 Obstructive diseases of the GIT (e.g. pyloric stenosis), paralytic ileus, intestinal
atony of the elderly, etc.
 Urine retention due to senile enlarged prostate
 It should be used with caution in children.

73
Part 7: Ganglion blocking drugs

■ Trimethaphan and mecamylamine are competitive blockers of ACh at nicotinic


receptors at both sympathetic and parasympathetic ganglia.
■ Because of lack of selectivity and numerous adverse effects, they are used
rarely in the clinical setting hypertensive emergencies).

Part 8: Neuromuscular blockers

Non-depolarizing NMBs Depolarizing NMBs


members ■ Tubocurarine (prototype) is ■ Succinylcholine
rarely used clinically at this (It is ester of ACh)
time
■ Semisynthetic derivatives:
 Mivacurium
 Atracurium and
Cisatracurium
 Vecuronium
Absorption  All members are not absorbed orally because they are polar
and compounds (quaternary amines), they must be given parenterally.
distribution  All cannot cross BBB or placental barrier

Metabolism  Atracurium: spontaneous  By plasma pseudo ChE enzyme


plasma hydrolysis. Breakdown
products may cause seizures.
 Vecuronium: liver.
Duration  Mivacurium: 10-20 min  Few minutes.
 Atracurium: 20-30 min
 Vecuronium: 30 -40 min
Mechanism  Competitive block of Ach at  Depolarizing block of Nm
of action Nm receptors → muscle receptors → ms paralysis
paralysis. through 2 phases:
 Muscle paralysis can be  Phase 1: initial depolarization →
reversed by excess Ach transient ms contraction
(AChE inhibitors). followed by paralysis due to
maintained depolarization

74
(depolarizationn block).
 Phasse 2: the m muscle beccomes
repo
olarized agaain but remmains
insen
nsetive to sstimulation
n by
ds ↑ Ach to be
Ach (i.e. it need
stimu
ulated) (deesensitization
block
k).

Reverssal of  Neostigm
mine can re
everse the  Neosstigmine inncreases muscle
m
block block by in
ncreasingg Ach level paralysis during g phase 1 (due to
at NMJ annd displace
e increase muscl e depolarization),
competitiv
ve blockerss from the but itt can reverrse the bloock in
receptors. phas se 2 (becauuse the rec ceptor
is rela
atively inseensetive an
nd
needs excess A Ach to be
stimuulated).
 Freshh blood traansfusion.
peutic  To induce sk ms rela
Therap axation  The same
s but ssuccinylcholine is
uses during surrgical ope
erations. prefe
erred for shhort proced
dures
 To control convulsioons during e.g. endotrach
e heal intubaation
electrocon
nvulsive (E
ECT) (has shorter
s duaation).
therapy
Adversse  Histamine
e release lleading to  Sudd
den rise off IOP due to
t
effects
s hypotensio
on and contrraction of tthe extraoc
cular

75
bronchospasm. muscles in phase 1.
 Respiratory paralysis in high  Acute hyperkalemia (which
doses. may be dangerous and life-
threatening). It is due to efflux of
muscle K+ during depolarization.
 Postoperative muscle pain.
 Bradycardia due to stimulation
of cardiac muscarinic receptors
(similar to ACh).
 Prolonged respiratory
paralysis (apnea) may result
from congenital deficiency of
PsChE enzyme (treaed by
artificial respiraion and blood
ransfusion).
C/I and  Bronchial asthma: why?  Glaucoma or recent eye
precautions  Myasthenia gravis. surgery: why?
 With aminoglycosides or  Congenital deficiency of PsChE
quinidine (they can aggravate enzyme: why?
ms paralysis).

76
77
78
Review Questions

Mention the pharmacodynamic principles underlying the use of:

 Dopamine in shock.
 Adrenaline in acute anaphylactic shock.
 Ritodrine to delay premature labor.
 Sumatriptan in acute migraine.
 Ergometrine in postpartum hemorrhage.
 Tamsulosin in senile enlarged prostate.
 Propranolol (beta-blockers) in hypertension.
 Beta-blockers in obstructive cardiomyopathy.
 Alpha-methyldopa in hypertension of pregnancy.
 Neostigmine in myasthenia gravis.
 Pralidoxime (PAM) in organophosphate toxicity.
 Atropine before surgical operations.
 Tolterodine in urine incontinence in adults.
 Succinylcholine before endotracheal intubation.

Mention the pharmacodynamic principles underlying the contraindication of:

 Ergometrine (or ergotamine) during pregnancy.


 Alpha-blockers (or beta-blockers) alone in pheochromocytoma.
 Propranolol (beta-blockers) in bronchial asthma.
 Reserpine in parkinsonism.
 Bethanechol in urine retention due to senile enlarged prostate.
 Neostigmine to treat ms paralysis of organophosphate intoxicated patient.
 Atropine in closed angle glaucoma.
 Atropine in old patient with senile enlarged prostate.
 Atracurium in bronchial asthma.

Mention the rational of the following combinations:

 Alpha-blockers with beta-blockers in pheochromocytoma


 Atropine with neostigmine in myasthenia gravis.
 Caffeine with ergotamine in acute migraine.

Mention the main differences between:

 Dopamine and dobutamine.


 Ergotamine and ergometrine.
 Propranolol and atenolol.

79
Of each of the following questions, 5. Cholinergic stimulation causes:
select ONE BEST answer: A. Urine retention
B. Bronchodilatation
1. Regarding adrenergic α1 receptors, C. Sweating
all are true EXCEPT:
D. Tachycardia
A. Molecular techniques revealed the
presence of a number of subclasses.
E. Reduced gut motility
B. Their stimulation can contract the
pregnant human uterus.
6. Nicotinic acetylcholine receptors
are found in all the following sites
C. Their stimulation can increase EXCEPT:
peripheral resistance
A. Sympathetic ganglia
D. Their effect is more potent and shorter
duration than β2 receptors. B. Presynaptic nerve endings
E. Their activation leads to increase C. Central nervous system
intracellular calcium D. Skeletal muscles motor end plate
E. Vascular endothelium
2. Regarding adrenergic β2 receptors,
all are true EXCEPT: 7. Increased urinary levels of vanilyl
A. Their stimulation can relax the non- mandelic acid (VMA) above 8 mg/24
pregnant human uterus. hours is diagnostic marker of the
B. Their activation on mast cells leads to following tumors:
stabilization of mast cell membrane. A. Pheochromocytoma
C. Their activation leads to increase B. Carcinoid tumor
intracellular cAMP. C. Leukemia
D. Their selective antagonists have no D. Lymphoma
clinical uses. E. Astrocytoma
E. Continuous and prolonged stimulation
can lead to down-regulation 8. The actions of norepinephrine at
adrenergic receptors are terminated by
3. Stimulation of cardiac M2 which of the following:
cholinoceptors cause which of the A. Metabolism by MAO in the liver
following: B. Reuptake into the nerve terminal
A. Decrease myocardial contractility C. Conversion into 5-HIAA
B. Decrease SA nodal activity and heart D. Conversion to dopamine
rate E. None of the above
C. Decrease conduction velocity through
the Purkinje fibers 9. The following is true for true
D. Decrease coronary blood flow cholinesterase:
E. All of the above. A. Is found in autonomic ganglia and
myoneural junctions
4. Physiological events mediated by B. Is found in plasma and liver
stimulation of β1 adrenoceptors include C. It needs 2 weeks to be regenerated
all the following EXCEPT:
D. It can metabolize acetylcholine as well
A. Increase insulin secretion as other choline esters
B. Increase systolic blood pressure E. Its presence is not necessary for life
C. Shorten myocardial cell refractoriness
D. Increase outflow resistance in patients 10. Which of the following drugs acts
with obstructive cardiomyopathy indirectly by releasing norepinephrine?
E. Increase renin release by A. Angiotensin
juxtaglomerular cells of the kidney B. Dopamine

80
C. Phenylephrine E. Phenylepherine causes rise of blood
D. Amphetamine pressure with bradycardia
E. Isoprenaline
15. For the treatment of acute
11. Attention-deficit hyperactivity anaphylactic shock, adrenaline must be
disorder in children can be treated by: given by the following route:
A. Ephedrine A. Inhalation
B. Modafinil B. Subcutaneous
C. Tizanidine C. Intravenous
D. Methylphenidate D. Intramuscular
E. Midodrine E. Intracardiac

12. The following is correct about the 16. Regarding reflex bradycardia
action of sympathomimetics: induced by administration of
vasopressor drugs:
A. Adrenaline has almost exclusively β-
adrenoceptor agonist actions A. It starts to work as a compensatory
B. Noradrenaline has an approximately response after long time of
equal mix of α-and β-adrenoceptor vasopressor use
agonist actions B. Reflex is mediated through stretch
C. Isoprenaline has predominantly α- receptors in the left pulmonary artery
adrenoceptor agonist actions C. The receptors begin to respond at
D. Phenylepherine has predominantly β- pressure ≥ 150 mmHg
adrenoceptor agonist actions D. In chronic hypertension the set point
E. Dopamine acts on specific D- is shifted to a higher level
receptors as well as other E. Beta blockers readjust the set point to
adrenoceptors. a higher level

13. Epinephrine, all are true EXCEPT: 17. Dobutamine is best indicated for
A. It is a polar (ionized) compound. management of which the following
shock:
B. Is synthesized from norepinephrine
within the adrenal medulla A. Septic shock
C. Cannot be administered orally. B. Cardiogenic shock
D. It is available as eye drops for C. Anaphylactic shock
ophthalmic use. D. Hypovolemic shock
E. The final product of metabolism is E. Neurogenic shock
vanillylmandelic acid (VMA).
18. Ritodrine hydrochloride can be used
14. The following statements about the in the management of:
action of sympathomimetics (i.v.) are A. Parkinson’s disease
correct EXCEPT: B. Bronchial asthma
A. Adrenaline infusion causes rise in both C. Depression
systolic and diastolic blood pressure D. Premature labor
with tachycardia E. Bradycardia
B. Noradrenaline infusion causes rise in
both systolic and diastolic blood 19. Selective α2 agonists that is used to
pressure with bradycardia relieve muscle spasm associated with a
C. Dopamine infusion causes decrease variety of neurological conditions is:
in renal blood flow and GFR. A. Clonidine
D. Salbutamol causes fall of blood B. Tizanidine
pressure with tachycardia
C. Ritodrine

81
D. Midodrine 24. Which of the following drugs will
E. Alpha methyldopa decrease heart rate in a patient with a
normal heart but will have no effect on
20. Nasal decongestants carry the risk heart rate in a cardiac transplant
of cerebral stroke in which of the recipient?
following conditions: A. Epinephrine
A. Arterial hypertension B. Salbutamol
B. Allergic rhinitis C. Norepinephrine
C. Epistaxis D. Phenylephrine
D. Benign prostatic hypertrophy E. Dopamine
E. Sinusitis
25. False +ve test for antinuclear factor
21. Oxymetazoline has which of the may be caused by:
following actions: A. Phenoxybenzamine
A. Bronchodilation B. Prazosin
B. Vasoconstriction C. Reserpine
C. Hyperglycemia D. Yohimbine
D. Tachycardia E. Ergotamine
E. Inhibition of ejaculation
26. The following alpha blocker is best
22. Chronic orthostatic hypotension prescribed to decrease symptoms of
due to impaired autonomic reflexes can urine retention due to senile enlarged
be managed by: prostate:
A. Midodrine A. Prazosin
B. Ritodrine B. Tremazosin
C. Amphetamine C. Phenoxybenzamine
D. Modafenil D. Terazosin
E. Cocaine E. Tamsulosin

23. 65 year old male requires extensive 27. Alpha blockers can worsen which of
dental work. In your first session with the following urinary problems:
him you inject lidocaine (2%) plus l: A. Urine retention due to senile enlarged
100,000 epinephrine. Although there prostate
was initial anesthesia, you are B. Urine retention due to atonic bladder
surprised to discover that after 15 C. Urine retention with over flow due to
minutes the patient grimaces with pain spinal cord injuries
when you work in the affected area. D. Urine incontinence due to pelvic floor
What is the best possible explanation? pathology in women
A. The patient is a chronic complainer E. Dysuria and frequency associated
B. The injection missed the appropriate with bladder inflammation
nerves
C. The patient metabolizes lidocaine 28. All the following conditions can be
extra rapidly effectively treated by beta-blockers
D. The patient may suffer benign EXCEPT:
prostatic hypertrophy and is being A. Angina pectoris
treated with doxazosin B. Essential hypertension
E. In this patient lidocaine is an C. Raynaud’s disease
ineffective local anesthetic D. Open angle glaucoma
E. Supraventricular tachycardia

82
29. The therapeutic action of beta- 34. The following beta-blocker is
blockers in angina pectoris is believed preferred to control tachycardia when
to be primarily due to: peripheral vascular disease is also
A. Reduced production of associated:
catecholamines A. Propranolol
B. Dilatation of the coronary vessels B. Dilevalol
C. Decreased myocardial oxygen C. Timolol
requirement D. Pindolol
D. Increased peripheral resistance E. Sotalol
E. Increased sensitivity to
catecholamines 35. One of the following drugs is best
chosen for the control of hypertension
30. Beta-blockers are contraindicated during pregnancy:
in bronchial asthma because: A. Captopril
A. They produce bradycardia and fall in B. Propranolol
COP C. Reserpine
B. They increase bronchial secretions D. Phenoxybenzamine
C. They decrease pulmonary blood flow E. Alpha methyldopa
D. They increase airway resistance and
narrowing 36. Positive Coomb’s test and hemolytic
E. They inhibit the respiratory center and anemia may follow the administration of:
impair ventilation A. Prazosin
B. Alpha methyldopa
31. Myocardial depression caused by
overdose of beta blockers can be
C. Guanithidine
reversed by parenteral administration D. Reserpine
of: E. Clonidine
A. Adrenaline
B. Dopamine 37. One of the following drugs should
be avoided in the control of chronic
C. Isoprenaline hypertension associated with peptic
D. Glucagon ulcer:
E. Insulin A. Reserpine
B. Prazosin
32. Excessive bradycardia induced by
beta-blockers is best treated by:
C. Propranolol
A. Dopamine D. Clonidine
E. Alpha methyldopa
B. Epinephrine
C. Isoprenaline 38. The following statements about
D. Neostigmine pilocarpine are correct EXCEPT:
E. Atropine A. It is a natural plant alkaloid
B. It acts selectively on muscarinic
33. Essential tremors can be best receptors
decreased by which of the following beta
C. It can block the hypotensive effect of
blockers?
neostigmine
A. Atenolol D. It is not metabolized by AChE enzyme
B. Propranolol E. It has a clinically useful miotic action
C. Betaxolol
D. Nebivolol 39. The following statements about
E. Bisoprolol anti-ChE drugs are correct EXCEPT:

83
A. Physostigmine lowers IOP D. Postpartum urine retention
B. Neostigmine may be used with E. Acute cystitis
atropine to treat myasthenia gravis
C. Pyridostigmine have fewer visceral 45. Relatively selective muscarinic
side effects than neostigmine. blocker that is used to treat urine
D. Rivastigmine can be used to treat incontinence in adults is:
paralytic ileus A. Pyrenzepine
E. Edrophonium has short duration of B. Benztropine
action C. Ipratropium
D. Tolterodine
40. A central AChE inhibitor that is used E. Oxybutinin
to improve symptoms of Alzheimer’s
disease is:
46. The metabolites of which of the
A. Pyridostigmine following neuromuscular blockers can
B. Edrophonium lead to seizures?
C. Donepezil A. d-tubocurarine
D. Neostigmine B. Atracurium
E. Echothiophate C. Mivacurium
41. A short acting AChE inhibitor used D. Vecuronium
in the diagnosis of myasthenia gravis E. Succinylcholine
is:
A. Edrophonium 47. When succinylcholine is used to
B. Neostigmine provide muscle relaxation during
C. Pyridostigmine delivery by cesarean section, the
D. Rivastigmine following is true:
E. Donepezil A. It can cause fetal hypotonia and even
fetal paralysis
42. Which of the following drugs has B. It can relax the uterus and aggravate
the longest duration of AChE inhibition: postpartum hemorrhage
A. Echothiophate C. It can cause acute hyperkalemia and
B. Neostigmine arrest the heart of the fetus
C. Physostigmine D. It can cause maternal tachycardia
D. Pyridostigmine E. It can decrease the effect of general
E. Donepezil anesthetics

43. The cause of death in 48. The following statements are true
organophosphate toxicity is: for neuromuscular blockers EXCEPT:
A. Bradycardia A. Succinylcholine can cause
postoperative muscle pain.
B. Increased bronchial secretions
B. Atracurium undergoes spontaneous
C. Paralysis of the respiratory muscles plasma hydrolysis
D. Depression of the respiratory center C. Vecuronium breakdown products may
E. All of the above cause seizures.
D. Neostigmine can reverse muscle
44. All the following are known block caused by competitive blockers
contraindications for the use of
E. Synthetic derivatives are generally
atropine EXCEPT: preferred than d-tubocurarine
A. Closed angle glaucoma
B. Senile prostatic enlargement
C. Paralytic ileus

84
49. A muscarinic blockers that is used B. Pralidoxime is completely ineffective
as a standard treatment of motion for enzyme regeneration after aging of
sickness is: the enzyme.
A. Pirenzepine C. Pralidoxime is effective regardless
B. Oxybutinine AChE is phosphorylated (e.g. by
C. Atropine organophosphates) or carbamylated
(e.g. by neostigmine).
D. Scopolamine
E. Tolterodine D. Atropine should not be stopped
before systolic blood pressure rises
above 110 mmHg and pulse rate
50. Zolmitriptan produce above 100 bpm.
vasoconstriction of cerebral vessels
E. Diazepam should be given to reduce
and decrease pain mediators during
bronchospasm
acute migraine by acting on the
following receptor subtypes:
54. Regarding the management of a
A. 5HT 1B/1D patient with iridocyclitis, the following
B. 5HT 1E/1F is true:
C. 5HT 2A/2C A. Mydriatics are used to help drainage
D. 5HT 3 of exudative fluids from the anterior
E. 5HT 7 chamber of the eye.
B. Short acting mydriatics such as
51. Bethanechol, a direct acting phenylephrine are preferred to avoid
muscarinic agonist used for relieving prolonged blurring of vision
post-operative urine retention in C. Atropine is preferred because it
absence of organic obstruction, could produces complete cycloplegia and
not be given parenterally because: mydriasis
A. It can cause annoying salivation D. If the patient was a child below 12
B. It can cause cardiac arrest years old, atropine eye drops would
C. It can cause histamine release and be contraindicated.
severe anaphylaxis E. Physostigmine eye drops should be
D. It can cause urine leak out of control used to help drainage of aqueous
E. It can cause undesirable nausea and humor
vomiting
Answers
52. A muscarinic agonist given orally to
increase salivary secretion and
1D 12 E 23 D 34 B 45 D
decrease symptoms of dry mouth
2B 13 A 24 D 35 E 46 B
associated with Sjögren syndrome is:
3B 14 C 25 B 36 B 47 C
A. Cevimeline 4A 15 D 26 E 37 A 48 C
B. Carbachol 5C 16 D 27 D 38 C 49 D
C. Bethanechol 6E 17 B 28 C 39 D 50 A
D. Pyridostigmine 7A 18 D 29 C 40 C 51 B
E. Rivastigmine 8B 19 B 30 D 41 A 52 A
9A 20 A 31 D 42 A 53 B
53. Regarding the management of a 10 D 21 B 32 E 43 E 54 C
patient with organophosphate toxicity, 11 D 22 A 33 B 44 E
the following is true:
A. With most types of
organophosphates, 90% of the
enzyme undergoes aging within the
first 3 hrs.

85
86
Part 1
1: Basiic inform
mation

▌TUBU
ULAR FUN
NCTION AN
ND URINE
E FORMAT
TION

 Thee renal bloo


od flow (RB BF) is 1.1 L /min (~ 22%
2 of CO
OP).
 Thee glomerulaar filtration
n rate (GFRR) is 125 ml/min.
m
 Thee capillary tuft filtrate
es ~ 180 L of fluid per day. 99% of thhe filtered fluid is
reab
bsorbed ag gain during
g passage e in the renal tubules..
 Watter reabso
orption is usually
u 2ryy to Na+ re eabsorptioon (exceptt in the co
ollecting
ug that ↓ Na reabs
tubules; ‘CT’)). Any dru +
= ↑ Na lo
sorption (= +
oss), also ↓ water
bsorption (= ↑ water loss or ‘di uresis’).
reab

Tubular reabsorrption and sites of ac


ction of 3 ty
ypes of diuuretics

87
■ Proximal convoluted tubules (PCT):
 Reabsorption: (75% of the glomerular filtrate).
– Active reabsorption of Na+ (~65%).
– Passive (2ry to Na+) reabsorption of equiosmotic amount of water.
– Reabsorption of all filtered K+, glucose, amino acids, and drugs.
 Secretion: active secretion and reabsorption of organic acids and bases into
tubular fluid.

■ Loop of Henle (LOH):


 Descending limb: passive reabsorption of water due to hypertonicity of the
medullary interstitium.

 Ascending limb: active reabsorption of Na+ (~25%) (this causes hypertonicity


of the medullary interstitium), Ca2+ and Mg2+.

■ Distal convoluted tubules (DCT):


 Proximal part:
– Active reabsorption of Na+ (5-7%).
– Passive (2ry to Na+) reabsorption of equiosmotic amount of water.
– Active reabsorption of Ca2+ (under the influence of parathormone ‘PTH’).
 Distal part:
– Active reabsorption of Na+ (2–5%) in exchange with K+ (under the influence
of aldosterone).
– Passive (2ry to Na+) reabsorption of equiosmotic amount of water.

■ Collecting tubules (CT): Reabsorption of water under the influence of ADH.

▌EDEMA AND EDEMATOUS CONDITIONS

 Edema is defined as the accumulation of fluid in the interstitial space due to


either:
– Increased capillary hydrostatic pressure
– Decreased plasma oncotic pressure.
– Increased capillary permeability.
 Edema can be either exudative (having high protein content) or transudative
(having low protein content).
 Exudative edema results from increased capillary permeability as part of the
acute inflammatory response. It is usually localized to the site of inflammation
and will not be considered in this chapter.

88
 Transudative edema is usually generalized and is associated with renal Na+
retention. The three most common clinical causes are:
– Congestive heart failure (CHF): the decreased COP causes renal ischemia
which stimulates the renin-angiotensin-aldosterone system (RAAS) → Na+ and
water retention → edema.
– Liver cirrhosis: the cirrhotic liver cannot synthesize sufficient albumin and
other plasma proteins → ↓ plasma oncotic pressure. Hypoalbuminemia
together with portal hypertension and 2ry stimulation of RAAS cause fluid
retention (edema) and accumulation of fluid in the peritoneal cavity (ascites).
– Nephrotic syndrome: glomerular dysfunction causes excessive loss of plasma
proteins in urine → ↓ plasma oncotic pressure → edema.

Part 2: Diuretic classes and agents

Diuretics are drugs that increase urine volume and Na+ excretion.
Natriuretic: a drug that increase Na+ excretion by the kidney.

Classification of diuretics:

Renal diuretics E x t r a - r e n a l di u r e t i c s
They act directly on the kidney: They act indirectly on the kidney:
■ Thiazide diuretics: act on the ■ Water diuresis: ↑ water intake → ↓
proximal part of the DCT e.g. ADH release → diuresis.
hydrochlorothiazide. ■ Digitalis in CHF: ↑ the COP
■ Loop diuretics: act on the leading to ↑ RBF → diuresis.
ascending limb of loop of Henle e.g.
■ i.v. albumin in ascites or
furosemide.
nephrotic edema: to increase
■ K+ sparing diuretics: act on the plasma osmotic pressure →
distal part of the DCT e.g. mobilization of edema fluid toward
spironolactone. the vascular compartment → ↑ RBF
■ Osmotic diuretics: substances that → diuresis.
↑ the osmotic pressure of tubular
fluid → ↓ water reabsorption by renal
tubules e.g. mannitol.

N.B. Carbonic anhydrase inhibitors e.g acetazolamide: they are weak diuretics
that ↓ NaHCO3 reabsorption from the PCT and may cause metabolic acidosis. They
also ↓ aqueous humor secretion and can be used in the treatment of glaucoma (see
pharmacology of the eye).

89
█ Loop diuretics
(Furosemide, torsemide, bumetanide , and ethacrynic acid)

Pharmacokinetics
 They are absorbed from the GIT and secreted into the lumen of the PCT by an
organic acid excretory system.
 The absorption of furosemide is erratic but bumetanide is complete.
 Diuresis occurs within 5 minutes after i.v. administration and within 30 minutes of
oral administration.

Mechanism and pharmacological effects


 Loop diuretics inhibit Na+/K+/2Cl― co-transport system in the thick ascending
limb of LOH leading to inhibition of the active reabsorption Na+, Cl―, and K+.
These ions are excreted with equiosmotic amount of water.
– They also increase excretion of Ca2+, Mg2+, halides and H+.
– Na+ and water loss at this segment is high, so they are potent (or high
ceiling) diuretics (i.e., up to 25% of the filtered Na+ load).
 They ↑ renal PGE2 and PGI2 production leading to VD and ↑ RBF and GFR.
– Non-steroidal anti-inflammatory drugs (NSAIDs) inhibit PG synthesis and
antagonize this effect of loop diuretics.
– VD of pulmonary vascular bed also occurs due to ↑ PG formation.

Therapeutic uses
■ Edematous conditions: e.g. CHF, nephrotic syndrome, etc.
– Many patients require fluid and sodium restriction to have the best results.
– Diuretics are not used to treat edema due to lymphatic obstruction
(lymphedema) or inflammatory edema (localized edema with high protein
content is difficult to be resolved by diuretics).

■ Acute pulmonary edema: loop diuretic ↓ pulmonary congestion by:


– They cause venodilatation → ↓ venous return.
– They cause VD of pulmonary vascular bed even before diuresis occurs.
■ Acute renal failure: to maintain adequate GFR and enhance K+ excretion.
■ Acute hypercalcemia and acute hyperkalemia: saline should be given to
compensate for Na+ and water loss.
■ Hypertensive emergencies: i.v. furosemide is usually given in emergencies:
– Loop diuretics ↓ plasma volume.
– They cause peripheral VD due to ↑ PGs production in many vascular beds.
– Hyponatremia ↓↓ sensitivity of the vascular smooth muscles to circulating
catecholamines.

90
Adverse effects
– Hypovolemia and hypotension.
– Electrolyte disturbances: Hyponatremia, hypokalemia, hypomagnesemia, and
hypocalcemia (all need to be properly replaced).
– Hypokalemic metabolic alkalosis: due to ↑ tubular secretion of K+ and H+.
– Hyperuricemia and precipitation of acute gout: This is caused by:
– Increased uric acid reabsorption in the PCT as a result of hypovolemia (It may
be prevented by using lower doses to avoid hypovolemia).
– Competition with uric acid excretion at the organic acid excretory system in
the PCT.
– Ototoxicity:
– It is reversible hearing loss. It occurs with very high doses.
– It may be due impairment of ion transport in the stria vascularis (inner ear).
– Occurs more frequent with:
 Patients with impaired renal function.
 Ethacrynic acid.
 Concomitant use of other ototoxic drugs e.g. aminoglycosides.
– Allergic reactions: all loop diuretics (except ethacrynic acid) are derivatives of
sulfonamides; they cause occasional skin rash, eosinophilia, and less often,
interstitial nephritis.

█ Thiazide diuretics

Classification
 True thiazides (they are derivatives of sulfonamides): hydrochlorothiazide,
bendroflumethiazide.
 Thiazide-like diuretics: metalozone, indapamide, chlorthalidone.

Pharmacokinetics
 Thiazide diuretics are absorbed from the GIT. They are secreted into the lumen of
the PCT by an organic acid excretory system.
 They produce diuresis within 1–2 hours.

Mechanism and pharmacological effects


 Thiazides inhibit Na+/Cl― co-transport system in the proximal part of DCT
leading to inhibition of the active reabsorption Na+, Cl―. These ions are excreted
with equiosmotic amount of water.
– Excess Na+ reaching the DCT is reabsorbed in exchange with K+ (→ K+ loss).

91
– They also increase excretion of halides and H+.
– They ↓ Ca2+ excretion and enhance its reabsorption.
– Thiazides have moderate efficacy (i.e., maximum excretion of filtered Na+
load is only 5-7%).
– Most thiazides are ineffective if the GFR is < 30-40 ml/min (so it is not useful,
or even harmful, in presence of renal failure).

 The action of thiazides also depends on renal PGs like loop diuretics but to
much less extent.

Therapeutic uses
■ Mild edematous states: cardiac, hepatic, or renal (same as loop diuretics).
■ Essential hypertension (mild to moderate):
– They have the same mechanisms like loop diuretics (mention them).
– They are often combined with other antihypertensive drugs to enhance their
blood pressure-lowering effects.
■ Hypercalcuria and renal Ca2+ stones: to ↓ urinary Ca2+ excretion.
■ Nephrogenic diabetes inspipidus (DI):
– Thiazides can reduce urine volume in some cases of DI. This is called
“paradoxical antidiuretic action” and it is not clearly understood. It may be due
to improvement of ADH receptor sensitivity in the renal collecting tubules.

Adverse effects
– Hypovolemia and hypotension.
– Electrolyte disturbances: Hyponatremia and hypokalemia.
– Hypokalemic metabolic alkalosis: due to ↑ tubular secretion of K+ and H+.
– Hyperuricemia the same as with loop diuretics.
– Hyperglycemia: due to both ↓ pancreatic release of insulin and ↓ tissue
utilization of glucose.
– Hyperlipidemia: due to ↑ cholesterol and LDL (by 5-15%).
– Allergic reactions: thiazides are derivatives of sulfonamides; they cause
occasional skin rash, dermatitis, and less often, thrombocytopenia.

█ Potassium-sparing diuretics
(Spironolactone – triameterine – amiloride)

 Spironolactone is a steroid congener of aldosterone.


 Triamterene and amiloride are synthetic drugs but not steroids.

Pharmacokinetics

92
 All a
are absorb
bed from thhe GIT.
 Spironolactonne and triamterene a re metaboolized by th
he liver
 Amiloride is excreted
e unnchanged in the urine
e.
 Theey have slo
ow onset (days).
(

anism and pharmac


Mecha cological e
effects
 Site n: the disttal part of the DCT where
e of action w Na+ is
reabbsorbed (2–5%)
( in exchang e with K+ under the t
influ
uence of aldosterone.
 Spironolacto one is a compe etitive an ntagonist of
aldo
osterone at
a its recep
ptor site att the distall part of DCT
ding to ↑ Na
lead +
N excretio on (with exxcretion of equiosmo otic
+
amoount of waater) and K retention .
 Tria
amterene and amilo hibitors off Na+ channnels in the distal
oride are direct inh
partt of DCT leading to ↑ Na+ exccretion (witth excretio
on of equio
osmotic am
mount of
+
water) and K retention.

 The
e net effec
ct is:
– Mild
d Na+ and water loss s (i.e., max imum excrretion of filtered Na+ is only 2-5
5%)
– Hypperkalemiaa: due to ↓ K excretio
+ +
on (K will be retained in blood)).
– Mettabolic acidosis: duee to ↓ H io n excretion
+
n (H+ will be retained in blood).

Therap
peutic use
es
■ All c
cases of edema
e due to hyperraldostero
onism:
– Primary hyyperaldoste
eronism: e
e.g. Conn’s s disease.
– S
Secondaryy hyperaldoosteronism
m: e.g. in liv
ver cirrhos
sis or nephhrotic syndrome.

■ Use
ed in comb
bination with
w loop d
diuretics or
o thiazide
es in orderr to:
– T
To minimizze the risk of electrollyte imbala
ance:
Loop diuretics cause hypo okalemia while K+ sparing diuretics cause
hyperkalem
mia. Their combinati on can minimize elec
ctrolyte dissturbance..
– T
To minimizze the risk of acid-ba
ase imbala
ance:
Loop diuretics caus se metabo sis while K+ sparing
olic alkalos g diuretics
s cause
metabolic acidosis. Their
T comb
bination caan minimiz
ze acid-basse imbalan
nce.
– T
To make synergism
s in cases o f refractory
y (resistantt) edema.

■ Treatment off female pattern haiir loss:


Spirronolacton
ne is a weak compe etitive inhib ndrogens aat their receptors
bitor of an
d ↓ synthesis of testtosterone (antianderrogenic efffect). Som
and me dermatoologists
use
e this feature to stop androgen--related fro
ontal hair lo
oss in wom
men.

93
Advers
se effects
– Hyp a due to ↓ K+ excreti on.
perkalemia
– Hyp c metabolic acidos
perkalemic sis: due to ↓ K+
andd ↓ H+ excreetion.
– Spirronolactonee has antiandrog
a genic efffects
(gyn
necomastia a and impottence in ma
ales).

Contra
aindication
ns
■ All c mia: espec
cases of hyperkalem
h cially in the
e following
g conditionns:
– Patients with
w chronic c renal failu
ure.
– W
With drugss that caus
se hyperka alemia e.g. ACEIs.
■ Spironolacto one shou uld not be give en with carbenoxxolone because
b
carbbenoxolonne has ald
dosterone--like action
n and can
n antagonnize the effect of
spirronolactone.

Spironolac
S ctone Triamtere
ene - amiloride
Structu
ure Synthetic
S ssteroid Synthetic non-stero
oids
Metabo
olism Extensive
E m
metabolism
m in the Amiloride is excrete
ed
liiver unchangeed in urine
Mecha
anism of action
a Competitiv
C ve antagonism N +
Direct inh ibition of Na
with
w aldostterone at itts channels at the distal part
receptor sitte in the DCT
D of DCT
Antiand
derogenic efcts Gynecoma
G astia & impotence Not preseent

█ Osm
motic diuretics: Mannitol,, Glycerol

They arre chemica ubstances given by i.v.


ally inert su on in emerrgency con
i infusio nditions

Mecha
anism of action
a
 Firsst, they ↑ osmotic
o pressure off plasma leading to withdrawaal of transscellular
fluid
d (e.g. aqueous humo or, excesssive CSF, etc).
e
 Sec cond, they are freely filtered byy the glom merulus and ↑ osmottic pressure of the
tubu eading to ↓ water reab
ular fluid le bsorption by
b renal tub
bules.

Therap
peutic use
es
Acute congestiv ve glauco oma and a acute rise cranial prressure: th
e in intrac hey are
given b apid ↑ drain
by i.v. infussion for ra nage of off aqueous humor or CSF respectively
by increasing thee osmotic pressure
p o f the plasm
ma before diuresis beegins.

se effects:: dehydration with hyypernatrem


Advers mia is the main
m adverrse effect.

94
95
Advantages and disadvantages of diuretics in some
Part 3:
edematous conditions

█ CONGESTIVE HEART FAILURE (CHF)

Patients with CHF have ↓COP due to weak cardiac muscle, fluid retention, and lung
congestion. Many patients have also high blood pressure.

Advantages of diuretics:
– Correction of fluid retention.
– Lowering of blood pressure.
– Decrease preload (venodilatation and ↓ venous return) and afterload (due to
arterial VD) leads to improvement of cardiac contraction.
– Decrease lung congestion causes improvement of tissue oxygenation.
– Recent evidence showed that spironolactone reduces morbidity and
mortality rates in patients with advanced heart failure.

Disadvantages of diuretics: Recommendation:


– Excessive hypovolemia can ↓ COP. Combination
– Diuretic-induced acid-base and electrolyte between a K+
sparing diuretic and
imbalance may impair cardiac function.
loop diuretic is the
– Diuretic-induced hypokalemia can predispose to
best choice.
digitalis toxicity and cardiac arrhythmia.

█ CHRONIC KIDNEY DISEASES

The majority of patients with chronic renal diseases (e.g. chronic renal failure,
diabetic nephropathy, etc.) have fluid retention, hypertension, hyperkalemia, and
acidosis.

Advantages of diuretics:
Recommendation:
– Correction of fluid retention. Loop diuretics are
– Reduction of hyperkalemia. best choice.
– Reduction of hypertension.

Disadvantages of diuretics:
– Thiazides are ineffective when GFR is <30 ml/min, moreover it may be harmful.
– K+ sparing diuretics are contraindicated because they can exacerbate
hyperkalemia and acidosis.
– Carbonic anhydrase inhibitors (acetazolamide) are contraindicated because
they can exacerbate acidosis.

96
█ LIVER
R CIRRHOSIS

Patientts with chronic


c liv
ver diseasse have fluid
retentio
on, ascite
es, hyperrammonem mia and 2ry
hyperaldosteronissm.

Adva
antages of
o diuretics s:
– C
Correction of fluid rettention.
– S
Spironolacttone antag gonizes ald
dosterone.

Disa
advantagees of diure
etics:
– AAggressive
e use of diuretics
d ccan precipitate
h
hepatorena
al syndrom
me.
– AAggressive
e use of diuretics
d ccan precipitate
h
hyperammo onemia and heppatic
e
encephaloopathy. Hoow?

– N
Normally, the urine pH is ac
cidic (~5.6
6). In
a
acidic uriine, most of the e absorb bable
ammonia (NH3) is converted
a c into the non-
a
absorbablee ammoniu um ions (N
NH4+) and so it
iss removed
d out by th
he kidney (this is kn
nown
a
as ammoniia trapping
g).

– D
Diuretics cause
c hypokalemia and metabolic
a
alkalosis. This
T leads to:
t

 Systemic alkalosiis causes the pH off the


urine to becom me less acidic. This
decreasses conveersion of the urina ary absorb bable NH33 into the non-
absorbaable NH4. The excreeted NH3 is s thus reab
bsorbed ag gain from urine to
blood → hyperammmonemia a.
 Systemic alkalosis
s increasess the entry
y of NH3 in
nto the braain cells.

█ LOW
WER LIMB EDEMA
E DURING PRE GNANCY

– L
Lower limb
b edema during late pregnancy mon conditiion and is usually
y is comm
b
benign (physiologic). It occurs due to ho mbalance, and comp
ormonal im pression
o
of pelvic ve
eins by the
e enlarged uterus.
– U
Unilateral leg edema, redness, warmth, and
a tenderness requuire evalua ation for
d
deep venou us thrombosis (DVT) .
– P
Physiologicc edema canc educed by elevating the lowerr extremities and
be re
w
wear elastic stockinggs.

97
– Diuretics are better avoided during pregnancy because they effectively
reduce maternal plasma volume and consequently may reduce amniotic
fluid and/or placental blood flow.

Part 4: Volume depletion and fluid replacement

 Volume depletion can be caused by loss of blood or other body fluids e.g.
vomiting, diarrhea, etc.
 In cases of mild volume depletion, resuscitation can be adequately achieved with
oral fluid alone. Sodium chloride tablets and electrolyte-containing solutions are
often used.
 In cases of severe dehydration, i.v. fluid therapy is preferred and may be life-saving.
 Water alone is not an appropriate fluid for volume resuscitation since it enters the
cells by osmotic effect. Only one third of each administered liter remains in the
extracellular space, and only one twelfth of each administered liter remains in the
intravascular space.
 When electrolyte disturbances are present, the fluid used for resuscitation should
be chosen to correct both volume depletion and electrolyte disturbances.

█ Crystalloid solutions

■ Sodium chloride solutions:


 Normal saline (0.9% NaCl): It is the most commonly used solution. It contains
154 mEq sodium per litre, a concentration similar to the sodium concentration of
plasma. Due to the relatively high chloride content, normal saline carries a risk of
inducing hyperchloraemic metabolic acidosis when given in large amounts.

 Hypotonic saline (0.45% NaCl): contains 77 mEq sodium per liter, and can be
used when there is dehydration with hypernatraemia. In these patients, 5%
dextrose in water can be given simultaneously with normal saline.
 Hypertonic saline (3% NaCl): contains 513 mEq sodium per liter, and can be
used for management of acute hyponatremia.

■ Lactated Ringer’s solution:


 It is an isotonic solution containing sodium, potassium, chloride, calcium and
lactate. The lactate is metabolized by the liver into bicarbonate, which can help
correct metabolic acidosis. In lactic acidosis and liver disease this conversion is
impaired, so lactate-containing fluids should be avoided.
 It is not suitable for maintenance therapy because the Na+ and K+ contents are
too low to compensate for daily electrolyte requirement.

98
■ Glu
ucose (Dex
xtrose) solutions:
 Variious conceentrations are availab
ble e.g. 5%
%, 10% and 25%. Thhe 5% dex xtrose in
watter (also kn
nown as D55W) is isottonic and is
s the mostt commonlly used.
 Hyp
pertonic glucose solutions (ab bove 5%) should be b infused very slow wly and
cau
utiously to avoid
a hype
erosmolarr syndrom me and life--threateninng dehydra
ation.

█ Collo
oid solutions

 Colloids are classified


c as either n
natural (albumin and fresh frrozen plassma) or
artifficial (starc
ch and dexxtran).
 Theey preserve a high colloid
c osm
motic pre essure in the
t blood and theoretically
dessigned to increase
i thhe intravasscular volu
ume with much lesss effect on
n tissue
watter. Howe ever, collo
oid solut ions are a less-p preferred choice for
f the
man nagement of volume e depletionn because they are very expenssive and have not
shoown a morttality beneffit over iso
otonic salin
ne.

█ Orall rehydrattion thera


apy (ORT
T)

 Oraal electrolyyte solution


ns are useed in childdren, particularly witth gastroe
enteritis.
Thiss product contains sodium,
s po
otassium, chloride,
c citrate,
c and
d dextrose e, and is
dessigned to replace th he electrolyytes and water thatt are lost with vom miting or
diarrrhea.
 Glucose is typ pically add
ded to thesse oral repllacement solutions
s too promotee uptake
of ssodium via the intestiinal sodium
m/glucose co-transporter mechhanism.

Part 5: Dis
sorders of serum
m sodiu
um and potassiu
p um

█ Hypo
onatremiia and SIA
ADH

 Hypponatremia a is defineed as seru um Na+ <135 mEq/L L. It can bbe caused by any
med dical illnesses, such as CHF, livver failure, renal failu
ure, pneummonia, or SIADH.
 Sevvere hypon natremia (N +
Na <120 mEq/L) lea ads to fall of plasmaa osmolality, with
movvement off water fro om plasma a to brain and otherr cells cauusing neuro ological
man nifestations (altered mental sta atus, weakkness, neu uromusculaar irritabilitty, focal
neuurologic de eficits, com
ma or seizu res).
 ADH H (or vaso opressin) is released d from poosterior pittuitary in rresponse to high
plassma osmolality. It bin nds to threee receptors: V1a in the
t vascul ature (VC), V1b in
the brain, and d V2 in renaal collectin ↑ water abs
ng ducts (↑ sorption).
 Whe
en the AD
DH system is working y, ‘the urine should rreflect the blood’,
g properly
i.e. concentra
ated urine occurs
o whe
en plasma
a osmolality
y is high, aand vice ve
ersa.

99
 Man
ny factorrs (includding drug gs and other o non-
pha
armacologiical condittions) can
n stimulate e release of
ADH
H irrespecctive of plasma
p ossmolality, leading to
t
hyp a, a cond
ponatremia wn as “sy
dition know yndrome of
o

inap
ppropriate cretion” orr SIADH.
e ADH sec

Manag
gement of SIADH

 Fluiid restriction (1L/d) is the main


n line.
 Dem ay be “rarely” used; they
meclocyclline and lithium may t impai r the response of
the collecting ducts to ADH
A (by a non-recep
ptor mecha
anism).

 Vas sts: “Vapta


sopressin receptor antagonis ans”:

 AAntagonism of the V2 V recepto rs results in aquares sis, a uniquue electroly


yte-free
wwater excrretion by th he kidneyss.
 CConivapta an is a mixxed V1a an nd V2 antaagonist, an
nd tolvapta an, a selec
ctive V2
aantagonistt.
 TTolvaptan n is appro oved for treating hyponatrem
h mia assocciated with h heart
ffailure, cirrrhosis, and
d SIADH.
 Recent sttudies sho owed thatt vaptans may be more effeective in treating
hypervolem mia in hearrt failure th
han diuretic
cs.

█ Hype
ernatrem
mia

 Hyppernatremia is defineed as a pla sma Na+ >145


> mEq//L, and rep
presents a state of
hyp
perosmolality
 Hyppernatremia may be caused byy a primary y Na+ gainn or a watter loss, th
he latter
ng much more com
bein mmon. Re enal wate er loss results from
m either osmotic
o
diurresis or diiabetes ins
sipidus (D
DI).
 Hyppernatremia results in contracttion of brain cells
as water sh hifts to attenuate
a the risingg ECF
osm
molality. Th hus, the most
m seveere sympto oms of
hyp
pernatremia a are neuro
ological maanifestatio
ons.

Manag
gement of symptom
matic hype
ernatremia
a

 The
e mainstay of manage ement is th
he adminis
stration
of water, prreferably by b mouth h or nasoogastric
tube
e. Alternattively, 5% dextrose e in water (D5W)
can
n be given intravenou usly.
 Speecific thera
apy of the underlying
u cause.

100
█ Hypo
okalemia
a

 Potassium is the major intracellula ar cation. 98% of K+ in the bo dy is found in the


intra
acellular co
ompartment, leaving g 2% in exttracellular fluid spacees.
 Ren +
nal K excrretion occu urs from thhe DCT and d is mediated by ald a Na+
dosterone and
delivery to thee distal nep
phron.
 Hyppokalemia is defined as serum K+ <3.5 mEq/L.m It ca
an result fro
rom diminisshed K+
ake, transcellular shift of K+, or increased K+ loss.
inta
 Thee most com mmon ma anifestatio ons are mu uscle weakness, craamps, flatttened T
wavve and pro olonged QT T interval in
n the ECG..
 Bec +
cause K iss usually ex xchanged with H+ att the DCT, hypokalem mia is often
n linked
to m
metabolic alkalosis.
a Anyone
A of these two conditions s can lead to the othher.

Druugs cause transcellu of K+ (from


ular shift o m plasma to
t tissue)::
Insu eta-2 agonists: they ↑ transmem
ulin and be a+/K+-ATPaase activity
mbrane Na y.

ugs cause renal loss


Dru s of K+:
Min
neralocortic
coids, gluc
cocorticoid
ds, diuretic
cs, amphottericin-B.

Dru o K+: laxa


ugs cause GIT loss of atives

Manag
gement

 Patients with a K+ level of


o 2.5-3.5 mEq/L ma nly oral K+ rreplaceme
ay need on ent
 If th +
he K level < 2.5 mE +
Eq/L, intravvenous K should be e given. Thhe rate of infusion
i
shoould not ex
xceed 20 mEq/hr,
m Raapid IV infu
usion may cause
c seri ous arrhythmia or
eveen cardiac arrest.
a
 Hyp pomagnese emia is frequently
f associate ed with hypokalem
h mia. Concomitant
mag gnesium deficiency
d aggravattes hypok kalemia and rendeers it diffiicult to
+
treaatment by K alone.

█ Hype
erkalemia

 Hypperkalemia a is defined
d as serumm K+ >5 mEq/L.
m It can
c result ffrom trans scellular
+ +
shifft of K , or decreased d renal exc
cretion of K (as in chhronic renaal failure).
 Thee most co ommon manifestati
m ions are muscle paralysis, p palpitations, high
peaaked T wavve and sho ort QT interrval in the ECG.
 Beccause K is +
i usually exchange ed with H+ at the DCT,D hypeerkalemia is i often
linkeed to meta abolic acid
dosis.

Druugs cause transcellu of K+ (from


ular shift o m tissue to
o plasma)::
Insu ency and β-blockers: they ↓ transmembra
ulin deficie ane Na+/K++-ATPase activity.
a

ugs that ↓ renal excrretion of K +:


Dru
K+ sparing diu
uretics, AC
CEIs, NSAIIDs, cyclos
sporins.

101
Management

 Mild hyperkalemia: could be corrected by diuretics and oral cation exchange


resins (Polystyrene sulfonate) to promote the exchange of Na+ for K+ in the GIT.

 Severe hyperkalemia with ECG changes:


 Intravenous calcium gluconate to reduce cardiac toxicity (↓ membrane
excitability). The usual dose is 10 mL of a 10% solution infused over 2 to 3
minutes.
 Intravenous insulin with glucose: 20 U regular insulin mixed with 500 ml D5W.
 Correct metabolic acidosis with i.v. NaHCO3 solution.
 Hemodialysis is reserved for patients with renal failure or with life-threatening
hyperkalemia resistant to other treatment.

Part 6: Pharmacological manipulation of the urine pH

Normal urine pH is 5.2-6.5. It is possible, by the use of pharmacological agents, to


produce urinary pH values ranging from ~ 5 to 8.5.

█ Alkalinization of the urine

 Indications:
 To enhance excretion of acidic drugs and organic compounds e.g. aspirin,
sulfonamides, and uric acid.
 To enhance dissolution of uric acid and cystine stones.
 To relieve dysuria (burning micturition) in some cases of bladder infection.

 Alkalinizing agents:
 Oral: sodium and potassium citrate salts: citrate is metabolized into
bicarbonate which is excreted in urine.
 Intravenous bicarbonate solution: contains 5% NaHCO3.

█ Acidification of the urine

 Indications:
 It is rarely used clinically except in a specialized test to discriminate between
different kinds of renal tubular acidosis.
 It can be very dangerous in cases of renal or hepatic impairment.

 Acidifying agents:
 Oral: ascorbic acid > 2 g/d.
 Intravenous ammonium chloride (NH4Cl) solution.

102
103
104
Review Questions

Mention the pharmacodynamic principles underlying the use of:

 Furosemide in arterial hypertension.


 Furosemide in acute pulmonary edema.
 Thiazides in diabetes insipidus.

Mention the pharmacodynamic principles underlying the contraindication of:

 Furosemide in acute hyperuricemia.


 Thiazides in uncontrolled diabetes mellitus.
 Spironolactone in chronic renal failure.
 Amiloride with ACEIs.
 Ethacrynic acid with aminoglycosides.
 Furosemide with NSAIDs.

Mention the advantages and disadvantages of diuretics in the following


conditions:

 Congestive heart failure.


 Chronic kidney disease.
 Chronic liver disease.

Mention the rational of the following combination:

 Furosemide with spironolactone.

Mention 3 differences between:

 Furosemide and spironolactone.

105
Of each of the following questions, 6. Which of the following diuretics
select THE ONE BEST answer: can enhance the parathormone-
mediated calcium reabsorption from
1. The ascending part of the loop of the distal renal tubules:
Henle is the principal site of action of A. Hydrochlorothiazide
the following diuretics: B. Triamterine
A. Hydrochlorothiazide C. Amiloride
B. Triamterine D. Bumetanide
C. Amiloride E. Spironolactone
D. Bumetanide
E. Spironolactone 7. Idiopathic calcium urolithiasis
(hypercalciuria) can be treated by:
2. Hyperkalemia is a contraindication A. Hydrochlorothiazide
of the following diuretics: B. Ethacrynic acid
A. Furosemide C. Furosemide
B. Bumetanide D. Triamterine
C. Ethacrynic acid E. Bumetanide
D. Chlorothiazide
E. Spironolactone 8. Spironolactone is characterized by:
A. It interferes with aldosterone synthesis
3. Loop diuretics are clinically useful B. It competitively inhibit aldosterone
in the treatment of all the following action in the distal part of the distal
edematous states EXCEPT: renal tubules
A. Edema caused by congestive heart C. It inhibits sodium reabsorption in the
failure proximal renal tubules
B. Edema caused by chronic liver failure D. It is more potent diuretic than
C. Lymphedema hydrochlorothiazide
D. Nephrotic syndrome E. It has rapid onset and short duration
E. Ankle edema due to chronic
hydralazine treatment 9. Hydrochlorothiazide is clinically
useful in the treatment of all the
4. Intravenous albumin is the ideal following conditions EXCEPT:
choice for treatment of the following A. Edema caused by congestive heart
conditions: failure
A. Ascites due to chronic liver disease B. Edema caused by chronic liver failure
B. Edema due to chronic kidney disease C. Edema caused by chronic renal failure
C. Edema due to congestive heart failure D. Hypertension with or without edema
D. Lymphedema E. Recurrent calcium urolithiasis
E. Inflammatory edema
10. Adverse reactions associated with
5. Vigorous diuretics are thiazide therapy include all the
contraindicated in resistant ascites due following EXCEPT:
advanced liver disease because: A. Hyperglycemia
A. It can lower blood pressure to a B. Hyperuricemia
critical level C. Metabolic acidosis
B. It can precipitate hepatorenal D. Fluid and electrolyte imbalance
syndrome E. Hypotension
C. It can lead to severe dehydration
D. It can decrease ascetic fluid suddenly 11. All the following diuretics can
and drastically aggravate digitalis toxicity EXCEPT:
E. It aggravate hypoalbuminemia A. Hydrochlorothiazide

106
B. Furosemide B. Hyperuricemia
C. Bumetanide C. Metabolic alkalosis
D. Amiloride D. Fluid and electrolyte imbalance
E. Ethacrynic acid E. Hypercalcemia

12. Adverse effects of loop diuretics 17. Acute pulmonary edema is best
include all the following EXCEPT: treated by i.v. administration of:
A. Magnesium deficiency A. Hydrochlorthiazide
B. Sodium deficiency B. Furosemide
C. Hypoglycemia C. Mannitol
D. Hypovolemia D. Amiloride
E. Hyperuricemia E. Metalozone

13. Hypokalemia can be caused by all 18. Which of the following diuretics has
the following drugs EXCEPT: the highest potential to cause
A. Captopril ototoxicity:
B. Salbutamol A. Chlorothiazide
C. Thiazides B. Furosemide
D. Corticosteroids C. Ethacrynic acid
E. Insulin D. Acetazolamide
E. Spironolactone
14. The following statements are true
concerning the precautions during the 19. All the following are uses of loop
use of diuretics in different metabolic diuretics EXCEPT:
disorders EXCEPT: A. Acute pulmonary edema
A. Furosemide may enhance digitalis B. Severe hypertension
toxicity in congestive heart failure C. Acute hypercalcemia
B. Furosemide may aggravate D. Acute oliguria
hyperammonemia in chronic liver E. Calcium urolithiasis
failure
C. Chlorothiazides may aggravate renal 20. In an addisonian patient, all of the
impairment in chronic renal failure following agents would have diuretic
D. Thiazides may aggravate action EXCEPT:
hyperglycemia in diabetes mellitus A. Mannitol
E. Thiazides may increase formation of B. Chlorothiazide.
urinary uric acid crystals in chronic C. Bumetanide
gout
D. Furosemide.
E. Spironolactone
15. All the following drugs can produce
salt and water retention after their
prolonged use EXCEPT:
21. Gynecomastia may occur with the
use of the following diuretics:
A. Nifedipine
A. Chlorothiazide
B. Minoxidil
B. Furosemide
C. Amiloride
C. Amiloride
D. Prazosin
D. Acetazolamide
E. Hydralazine
E. Spironolactone
16. Adverse reactions associated with
furosemide therapy include all the
22. The most dangerous complication
of injudicious use of diuretics in
following EXCEPT:
patients with advanced liver diseases
A. Hearing loss is:

107
A. Aggravation of hypotension and 27. A 64-year-old woman with
fatigue congestive heart failure. She complains
B. Electrolyte imbalance of swelling in her legs and ankles. The
C. Acid-base imbalance doctor decides to increase her level of
D. Precipitation of hepatorenal syndrome diuretics. What complication should the
E. Marked dehydration doctor be most aware of for this
patient?
23. Acute congestive glaucoma is best A. Diuretic-induced metabolic acidosis
treated by i.v. administration of: B. Hepatic encephalopathy
A. Bumetanide C. Hypercalcemia
B. Furosemide D. Hyperkalemia
C. Mannitol E. Hypokalemia
D. Amiloride
E. Metalozone 28. One of your clinic patients is being
treated with spironolactone. Which of
24. The following statements the following statements best
concerning hypokalemia are true describes a property of this drug?
EXCEPT: A. Contraindicated in heart failure,
A. It is a side effect predicted with all especially if severe
diuretics B. Inhibits Na+ reabsorption in the
B. It is commonly seen in patients with proximal renal tubule of the nephron
hyperaldosteronism C. Interferes with aldosterone synthesis
C. It can be manifested by ECG changes D. Is a rational choice for a patient with
D. It could be prevented by the use of K+ an adrenal cortical tumor
sparing diuretics E. Is more efficacious than
E. It is a risk factor for digitalis toxicity hydrochlorothiazide in all patients who
receive the drug
25. The best intravenous agent given
to patients with advanced liver disease 29. A patient taking an oral diuretic for
to correct ascites and edema is: about 6 months presents with elevated
A. Human albumin fasting and postprandial blood glucose
B. Mannitol levels. You suspect the glycemic
C. Furosemide problems are diuretic-induced. Which
of the following was the most likely
D. Chlorothiazide
cause?
E. Spironolactone
A. Acetazolamide
B. Amiloride
26. A 63-year-old man presents to the
emergency department with worsening
C. Chlorothiazide
heart failure. Physical exam reveals D. Spironolactone
pitting edema in his ankles. Past E. Triamterene
medical history is significant for an
allergic reaction following exposure to 30. Chlorthalidone and torsemide are
trimethoprim–sulfamethoxazole. Which members of different diuretic classes,
drug should the physician prescribe to in terms of mechanisms of action, but
him? they share the ability to cause
A. Acetazolamide hypokalemia. Which of the following
B. Ethacrynic acid statements best describes the general
C. Hydrochlorothiazide mechanism by which these drugs
cause their effects that lead to net
D. Mannitol
renal loss of potassium?
E. furosemide
A. Act as aldosterone receptor agonists,
thereby favoring K+ loss

108
B. Block proximal tubular ATP- C. Lupus
dependent secretory pumps for K+ D. Ototoxicity
C. Increase delivery of Na+ to principal E. Hyperuricemia
cells in the distal nephron, where
tubular Na+ is transported into the
cells via a sodium channel in
exchange for K+, which gets Answers
eliminated in the urine
D. Inhibit a proximal tubular Na,K- 1D 11 D 21 E 31 B
ATPase such that K+ is actively 2E 12 C 22 D 32 C
pumped into the urine 3C 13 A 23 C 33 B
E. Lower distal tubular urine osmolality, 4A 14 E 24 A 34 A
thereby favoring passive diffusion of 5B 15 C 25 A
K+ into the urine 6A 16 E 26 B
7A 17 B 27 E
31. Which of the following is a clinical 8B 18 C 28 D
indication for use of Mannitol? 9C 19 E 29 C
A. Chronic simple glaucoma 10 C 20 E 30 C
B. Cerebral edema
C. Pulmonary edema
D. Acute heart failure
E. chronic renal failure

32. All the diuretics act from the


luminal side of the renal tubule
EXCEPT:
A. Torsemide
B. Hydrochlorothiazide
C. Spironolactone
D. Chlorthalidone
E. Furosemide

33. A 55-year-old male with kidney


stones has been placed on a diuretic to
decrease calcium excretion. However,
after a few weeks, he develops an
attack of gout. Which diuretic was he
taking?
A. Furosemide.
B. Hydrochlorothiazide.
C. Spironolactone.
D. Triamterene
E. Acetazolamide

34. Your 60 year old male hypertensive


patient who had a myocardial infarction
a year ago is now showing signs of
CHF. You therefore add spironolactone
to his drug regimen. What side effect
should you warn him about?
A. Gynecomastia
B. Hypokalemia

109
Copyrighhts © 2016 by
b the Deparrtment of Clin
nical Pharma
acology at Fa
aculty of Meedicine, Mnas
soura
Universitty, Egypt.

Previouss editions copyright © 20


015, 2014, 20
013, 2012, 2011, 2010, 2009,
2 2008, 22000 by the
Departm
ment of Cliniccal Pharmacoology at Facu cine, Mnasou
ulty of Medic ura Universitty, Egypt.

No part of this book may be reprroduced or d distributed in


n any form orr by any meaans, or stored
d in a
databasee or retrievall system, witthout the prio
or written pe
ermission of the
t copyrighhts owner,
Departm
ment of Cliniccal Pharmaco ology at Facuulty of Mediccine, Mansouura Universitty.

This is a copyrightedd work and is s protected b


by the Egypttian Intellectu
ual Property Law 82 of 2002. Use
of this w
work is subjec
ct to this law w. The Departrtment of Clin
nical Pharmaacology at MMansoura Fac culty of
Medicine e reserves alll rights in an
nd to the worrk.

2000 ‫سنة‬
‫ لس‬1456 :‫رقم اإليداع بدار الككتب‬
‫م‬
2000/9/6 ‫بتاريخ‬
Preface

C
linical training for undergraduate students often focuses on diagnostic rather than
therapeutic skills. Sometimes students are only expected to copy the prescribing
behavior of their clinical teachers, or existing standard treatment guidelines, without
explanation as to why certain treatment is chosen. Books may not be much help either.
Pharmacology reference works and formularies are drug-centered, and although clinical
textbooks and treatment guidelines are disease-centered and provide treatment
recommendations, they rarely discuss why these therapies are chosen. Different sources
may give contradictory advice.
This book in primarily intended for under graduate medical students who are about to
enter the clinical phase of their studies. It will provide step by step guidance to the process
of rational prescribing together with many illustrative examples. It teaches also skills that are
necessary throughout a clinical career. Postgraduate students and practicing doctors may
also find it a source of straightforward information.
I wish to acknowledge the ongoing efforts of my contributing authors, and we are deeply
grateful to all those who have with such good grace given us their time and energy to supply
valuable facts and opinions, they principally include:

 Prof. Hussein El-Beltagi who took over the preparation of all books since the 1st edition
in 1995 including the revision process, printing control, distribution and selling control.
 Assist. Prof. Mohamed-Hesham Daba who took over the revision process and
amendments of the last two editions.
 Assist. Prof. Abdel-Motaal Fouda who prepared the last two editions in a readable up-
to-date text to provide essential information necessary throughout the clinical career.
 Dr. Sameh Abdel-Ghany who assisted in the revision process.

Much of any merit this book may have is due to the


generosity of those named above.

Gamal M. Dahab (MD, PhD)

Professor Emeritus in Clinical Pharmacology


Mansoura Faculty of Medicine

iii
Miss
sion and
a Vis
sion

Our mission
The Clinnical Pharm macology Department
D is seeking excellence
e and leadeership in fou
ur major
core acctivities: edu
ucation, ressearch, com
mmunity serrvice, and faculty
f and staff development.
We are e connectin ng basic medical
m sc iences with clinical care
c througgh innovattive and
disciplin
ned teachin ng of clinica
al pharmaco
ology in an integrative
i manner
m

Our v
vision
The dep partment off Clinical Ph
harmacolog gy is aiming
g to be a premier acad demic model in the
field of pharmacoloogy and the erapeutics in Egypt annd Middle East
E throug h promoting use of
the bestt therapeutics and dev veloping new
wer experimmental and clinical reseearch proje
ects.

Value
es

The gu
uiding prin
nciples and beliefs ffor the dep
partment

 Excellence, creativity, innovation, fairness, honesty, transparenncy, collab


boration,
teammwork and lifelong learning
 Reccognition tha
at our stude
ent comes ffirst
 All m
members ofo our department musst see them mselves as integral to the successs of our
misssion and ouur departmeent as integ ral to their personal
p su
uccess.
 As we subscrribe to the ese values,, we shall be profes ssionals in the profes
ssion of
education.

iv
Contributers

Effat A. Haroun MD, PhD Somaya A. Mokbel MD, PhD


Prof. of Clin Pharmacology Prof. of Clin Pharmacology
Mansoura Faculty of Medicine Mansoura Faculty of Medicine

Elhamy M. El-Kholy MD, PhD Amany A. Shalaby MD, PhD


Prof. of Clin Pharmacology Prof. of Clin Pharmacology
Mansoura Faculty of Medicine Mansoura Faculty of Medicine

Gamal M. Dahab MD, PhD, MSc (Int.Med) Amal Abdel-Hamid MD, PhD
Prof. of Clin Pharmacology Prof. of Clin Pharmacology
Mansoura Faculty of Medicine Mansoura Faculty of Medicine

Farida M. El-Banna MD, PhD Essam A. Ghyati MD, PhD


Prof. of Clin Pharmacology Assist. Prof. of Clin Pharmacology
Mansoura Faculty of Medicine Mansoura Faculty of Medicine

Aly M. Gaballah MD, PhD, MSc (int.Med) Mohamed-Hesham Y. Daba MD, PhD
Prof. of Clin Pharmacology Assist. Prof. of Clin Pharmacology
Mansoura Faculty of Medicine Mansoura Faculty of Medicine

Layla T. Hanna MD, PhD Abdel-Motaal M. Fouda MD, PhD


Prof. of Clin Pharmacology Assist. Prof. of Clin Pharmacology
Mansoura Faculty of Medicine Mansoura Faculty of Medicine

Mohamed Kheriza MD, PhD, MSc (Int.Med) Vivian Boshra MD, PhD
Prof. of Clin Pharmacology Assist. Prof. of Clin Pharmacology
Mansoura Faculty of Medicine Mansoura Faculty of Medicine

Abdel-Rahman A. Yassin MD, PhD Hala A. Al-Ashri MD, PhD


Prof. of Clin Pharmacology Assist. Prof. of Clin Pharmacology
Mansoura Faculty of Medicine Mansoura Faculty of Medicine

Mohmmad A. Attia MD, PhD Nageh Rizk MD, PhD


Prof. of Clin Pharmacology Lecturer in pharmacology
Mansoura Faculty of Medicine Mansoura Faculty of Medicine

Mohamed Abdel-Ghani MD, PhD Elsayed A. Hassan MD, PhD


Prof. of Clin Pharmacology Lecturer in Clin Pharmacology
Mansoura Faculty of Medicine Mansoura Faculty of Medicine

Hussien M. El-Beltagi MD, PhD Mohamed Abdel-Monem MD, PhD


Prof. of Clin Pharmacology Lecturer in Clin Pharmacology
Mansoura Faculty of Medicine Mansoura Faculty of Medicine

Karawan M. Abdel-Rahman MD, PhD Mahmoud A. Naga MD, PhD


Prof. of Clin Pharmacology Lecturer in Clin Pharmacology
Mansoura Faculty of Medicine Mansoura Faculty of Medicine

v
Ahmad Hassan MD, PhD Mohamed Abou El-khair MD, PhD
Lecturer in Clin Pharmacology Lecturer in Clin Pharmacology
Mansoura Faculty of Medicine Mansoura Faculty of Medicine

Ahlam El-masry MD, PhD Sameh A. Abdel-Ghani MSc.


Lecturer in Clin Pharmacology Assist. Lecturer in Clin Pharmacology
Mansoura Faculty of Medicine Mansoura Faculty of Medicine

Rehab Hamdy MD, PhD


Lecturer in Clin Pharmacology
Mansoura Faculty of Medicine

vi
Table of Contents

CHAPTER 4: AUTACOIDS AND ANTI-INFLAMMATORY DRUGS

Part 1: Autacoids 111


Histamine 111
Serotonin 114
Kinins 117
Endothelins 117
Purines 118
Fatty acid derivatives 119
Part 2: Non-steroidal anti-inflammatory drugs (NSAIDs) 122
Acetylsalicylic acid 122
Other NSAIDs 127
Part 3: Acute rheumatic fever 130
Part 4: Drug therapy of rheumatoid arthritis 133
Part 5: Drug therapy of gout 136
Review questions 142

CHAPTER 5: CARDIOVASCULAR PHARMACOLOGY

Part 1: Systemic hypertension and antihypertensive drugs 149


Basic information 149
Management of hypertension 150
Angiotensin-converting enzyme inhibitors 152
Calcium channel blockers 157
Vasodilators 160
Endothelin receptor blockers 161
Specialized vasodilators 162
Management of hypertensive emergencies 163
Part 2: Therapy of peripheral vascular disease 164
Part 3: Therapy of ischemic heart disease 165
Basic information 165
Management of stable angina 166
Management of acute myocardial infarction 172
Part 4: Therapy of congestive heart failure 173
Basic information 173
Therapy of heart failure 174
Management of acute pulmonary edema 180
Part 5: Management of shock 182
Part 6: Cardiac arrhythmia and antiarrhythmic drugs 184

vii
Basic information 184
Antiarrhythmic drugs 185
Review questions 194

CHAPTER 6: PHARMACOLOGY OF THE BLOOD

Part 1: Hyperlipidemia and drugs that lower plasma lipids 203


Basic information 203
Lipid lowering drugs 204
Part 2: Drugs affecting hemostasis 207
Anticoagulant drugs 210
Fibrinolytic drugs 212
Antiplatelet drugs 213
Hemostatic agents 215
Drugs affecting blood viscosity 217
Part 3: Drugs used in the treatment of anemia 218
Iron deficiency anemia 218
Red cell deficiency anemia 220
Megaloblastic anemia 221
Hemopoietic growth factors 223
Drug induced aplastic anemia 223
Review questions 226

CHAPTER 7: PHARMACOLOGY OF THE RESPIRATORY SYSTEM

Part 1: Agents used to treat cough 231


Antitussives 232
Mucolytics 233
Expectorants
Part 2: Therapy of bronchial asthma 234
Bronchodilators 235
Reduction of bronchial inflammation 238
Prophylactic treatment 239
Other drugs 240
Treatment of acute severe asthma 240
Part 3: Oxygen therapy 241
Basic information 241
Oxygen therapy 242
Review questions 245

viii
Part 1
1: Auttacoids

Inflamm mation is the comp plex biolo gical resp


ponse of vascular
v tisssues to harmful
oduces locally-acting
stimuli.. Injury pro g substancces that inittiate inflam
mmatory re esponse
and acttivate the repair
r proc
cess. Thesse substances are terrmed 'auta acoids'.
Examp
ples of auttacoids:
 Amino acid derivatives
d s: e.g. hista amine and d 5-HT (serrotonin).
 Vas
soactive peptides:
p e.g.
e angiote ensin, kininns and enddothelins.
 erivatives: e.g. prost aglandins, thrombox
Fattty acid de xanes, leukkotrienes, etc.
e
 e family of cytokines:: e.g. interle
The eukins, inte
erferones, TNF,
T growthh factors, etc.
e

█ Histtamine

 Hisstamine is formed frrom the a amino acid d histidine by the aaction of histidine
h
dec carboxylase and storred mainly in mast ce ells. Non-m
mast cell hhistamine is found
in sseveral tisssues includ
ding the brrain.
 No clinically useful dru ugs affect the synthe esis or meetabolism of histamiine, but
cerrtain drugs can cause e release o of histamin
ne from maast cells ass a side efffect.

Histam se: can oc


mine releas ccur throug
gh 2 proce
esses:
 Ca22+ dependent mech hanism: byy fixation of
o IgE to th
he surfacee of mast cell
c with
2+
sub
bsequent activation of the complem ment, this causes Ca influ
ux and
deg
granulation
n of mast cells.
c Manyy drugs can induce th
his type e. g. penicillin.

 Ca22+ indepen
ndent mec chanism:
– Displacem ment of hisstamine fro
om storag
ge granules s by drug
gs: e.g. mo
orphine,
tubocurarrine, vanco
omycin, an
nd amine antibiotics.
– Mast cell damage: by b venomss, or mechanical trauma.

111
Pharmacological effects of histamine
Histamine acts on at least 4 types of receptors:

R Postreceptor Site Effect


mechanism
H1 Gq CNS Functions related to appetite and satiety
↑ IP3 & DAG Nonvascular Spasm (→ bronchoconstriction, GIT spasm).
smooth ms
Blood vessels – VD of microcirculation (arterioles and
precapillary sphincters) → ↑ capillary
permeability (edema) and fall of BP.
– This VD is due to release of NO from
vascular endothelium.
Exocrine glds ↑ exocrine secretions (bronchial, lacrimal, etc)
Sensory nerves Pain and itching
H2 Gs Gastric parietal ↑ HCl secretion.
↑ cAMP cells
Heart ↑ cardiac contractility and HR.
Mast cells ↓ histamine release (-ve feed back effect).

H3 Gi CNS and Wakefulness and modulation of other


↓ cAMP presynaptic neurotransmitters release.
nerve endings
H4 Gi Inflammatory Regulation of the inflammatory response
↓ cAMP cells

Clinical uses of histamine


 Histamine itself has no clinical applications.
 Some selective agonists are available for diagnostic purposes only e.g. test for
gastric secretion.

Histamine antagonists

■ Physiological antagonists (adrenaline): adrenaline reverses all the effects of


histamine by action on different receptors (see ANS).
■ Histamine receptor antagonists:
– H1-blockers: e.g. diphenhydramine, loratidine, etc. (see below)
– H2-blockers: e.g. cimetidine, ranitidine, famotidine, etc. (see GIT)
– H3 and H4 -blockers: are not yet available for clinical use.
■ Histamine release inhibitors (mast cell stabilizers): ketotifen and cromoglycate
They inhibit Ca2+ influx into mast cells and prevent mast cell degranulation.

112
■ Desensitization therapy (immunotherapy):
Immunotherapy is a process in which an allergic patient can become
desensitized to antigens that trigger allergic responses. Small doses of the
allergic substance are injected weekly. Each week the dose is increased.
Gradually a protective antibody (IgG) is formed to block the allergic reaction.
Many patients notice an improvement within 6 months.

█ H1-receptor antagonists

1st generation H1 blockers 2nd generation H1 blockers


Examples Diphenhydramine Loratidine
Dimenhydrinate Azelastine
Clemastine Cetirizine
Antazoline Fexofenadine
Cyclizine - Meclizine
Chlorpheniramine Ketotifen is a 2nd gen H1 blocker
Promethazine and mast cell stabilizer (see
Cyproheptadine respiratory pharmacology).
CNS They can cross BBB (more Cannot cross BBB (less lipophilic)
effects lipophilic) and exert significant and have little or no CNS actions
CNS actions
Potency Less potent H1 blockers More potent H1 blockers
Effects – Relief of itching, pain and allergic response.
related to – ↓ capillary permeability and inflammatory edema induced by
H1 histamine
blocking – ↓ bronchoconstriction, bronchial secretions, lacrimation, etc.
Other Sedation: but excitation or even
effects convulsions may occur in some
cases (especially in children or
with high doses).

Atropine-like actions: most of


the first generation drugs can
block peripheral muscarinic Not present
receptors leading to urine
retention and blurred vision.

Antiemetic action: useful in


preventing motion sickness.

α-receptor blocking action:

113
leading to orthostatic (postural)
hypotension in susceptible
individuals.

5-HT receptor blocking action


(especially cyproheptadine): so it
is useful drug for treating
carcinoid syndrome.
Duration of 3-8 h 3-24 h
action
Therapeutic ■ Allergic conditions:
uses – The use of H1-blockers is effective in allergic conditions in which
histamine is the primary mediator (e.g. allergic rhinitis and
urticaria).
– N.B. In bronchial asthma, which involves several mediators, the
use of H1-blockers is generally ineffective.

■ Motion sickness and vestibular disturbances:


– The first-generation H1-blockers (especially diphenhydramine and
promethazine) are effective in preventing motion sickness, but their
efficacy in Ménièrs's disease is not established.
– They act by blocking H1 and muscarinic receptors in the
vestibulocerebellar pathway.
– N.B. Scopolamine remains the most effective treatment for motion
sickness.

■ Carcinoid syndrome:
– Carcinoid syndrome is caused by a serotonin-secreting neoplasm
of the enterochromaffin cells of the GIT.
– When the tumor is not operable, cyproheptadine and other 5-HT
blockers can be used to block 5-HT receptors.
Adverse – Sedation with impaired The second-generation drugs
effects concentration, but rarely are metabolized by the hepatic
excitation and convulsions CYP450 enzymes. Some of the
may occur in children after early drugs in this group
toxic doses. prolonged the QT interval and
caused serious arrhythmia
– Anticholinergic (atropine- (torsade de pointes) when given
like) actions: this may cause with other drugs that inhibit
urine retention, dry mouth and CYP450 system (e.g.
blurred vision.
ketoconazole), and in patients

114
– Orthostatic hypotension due with liver disease. These drugs are
to their α-receptor blocking now withdrawn from the market.
action.
Interactions These drugs can potentiate the Dangerous arrhythmia when
sedative effect of hypnotics, given with other drugs that inhibit
alcohol, etc. CYP450 system.

█ Serotonin (5-hydroxytryptamine; 5-HT)

5-HT is formed from the amino acid tryptophane and stored mainly in the
enterochromaffin tissue of the GIT (90%) and platelets (10%). Serotonin is found
also as a transmitter in many areas within the CNS. Like histamine, no clinically
useful drugs affect the synthesis or metabolism of 5-HT, but many drugs can act on
5-HT receptors as agonists or antagonists.

Pharmacological actions of 5-HT


5-HT acts on at least 7 types of receptors:

Distribution Postreceptor Effect


mechanism
5-HT1A CNS, cranial Gi Role in anxiety, cognitive function,
5-HT1B blood vessels ↓cAMP appetite, thermoregulation, memory,
5-HT1D nociception
5-HT1E VC of cranial BV.

5-HT2A Platelets Gq Platelet aggregation (5HT2A)


5-HT2B smooth muscle, ↑ IP3 VC and smooth muscle contraction
5-HT2C CNS Hallucinations

5-HT3 Area postrema, Na+ ion Nausea and vomiting


enteric nervous channel Nociception (pain)
system
5-HT4 CNS and enteric Gs ↑ GIT motility and secretions
nervous system ↑ cAMP

5-HT5A CNS Gi ?
↓cAMP

5-HT6,7 CNS Gs Anxiety, cognitive function,


↑ cAMP thermoregulation, memory, nociception

115
█ Som
me 5-HT agonists
s and an tagonistts in clinical use

▌5-HT
T agonists
s

■ Bus
spirone: (ssee also CN
NS)
– It acctivates ceentral 5-HT
T1A recepttors. It
is used forr treatment of an nxiety
diso
orders esppecially in elderly
e patiients.
– Theerapeutic effect
e may take as lo ong as
2wweeks to apppear.

■ Trip
ptans: sumatriptan n, zolmitrriptan,
narratriptan (ssee also AN
NS)
– Theey activate 5-HT1B/1D receptors..
– Thee major usse of tripta
ans is the treat-
men nt of acutte migrainne. Activat ion of
5-HHT1D recepptors inhibiits inflammmation
of meninges,, pain transmission , and
rele
ease of VD D substannces e.g. calci-
toniin gene-related peptide in trigeeminal
neuurons. Activation of 5-HT
5 1B rec
ceptors ca
auses VC of o the dilaated cerebral ves-
selss. About 500%–80% of o patients report reliief from pa
ain within 2 hours.
– 5-HHT1B activitty can cause corona ary spasmm so; these e drugs arre contraindicated
in p
patients witth ischemic
c heart dissease (IHD).

■ Teg
gaserod:
– It iss a specificc 5-HT4 ag
gonist. It in
ncreases GIT
G motility
y so it is uused for tre
eatment
of irrritable bow
wel syndro
ome with c
constipation.

▌5-HT
T antagon
nists

■ Cyp
proheptad
dine:
– It bllocks 5-HTT2, H1, & muscarinic
m c receptors
s.
– It is used for treatment
t of
o symptomms of carc
cinoid tumo
or and as aantiallergic
c.

■ Aty
ypical antip
psychotics: Olanzap eridone (see also CN
pine, rispe NS)
– Theey block 5-HT2A and
a dopam
mine D2 receptors in the CNS lead
ding to
imp
provement of the neg
gative symp
ptoms of schizophre
s enia.

■ Ond n, granisettron: (see a


dansetron also GIT)
– Theey selectiveely block 5-HT
5 3 recep pherally (GIT).
ptors centrally (CTZ) and perip
– It is used for trreatment off nausea & vomiting asssociated with
w cancerr chemothe erapy.

116
█ Kiniins (brad
dykinin)

 Kiniins are potent


p va
asodilator peptides
form
med from
m protein substrate
es called
kininogen byy the effect
e of kallikrein
enzyme.
 Kiniins act on
o at lea
ast 2 sub
btypes of
rece
eptors: B1 1 and B2 2. Activatio
on of B2
rece
eptors ca auses VD D and ↑ capillary
permmeability, smooth ms co ontraction
(bro
onchoconsstriction, GIT
G colic, etc.), and
stim
mulation of sensory nerves
n caussing pain.
 Icattibant is a 2nd gene
eration dec
capeptide
which acts as a a selecctive anta agonist at
braddykinin B22 receptorrs. It is give
en by s.c.
injection for the
t symptoomatic tre atment of
acuute attackss of hereditary angioedema.
It mmay also be usefu ul in drug g-induced
anggioedema, ascites, annd pancrea atitis.
 Reccently, a 3rd generation of oraally active
B2--receptor antagonists was d
developed
and
d currently under stud
dy.

█ End
dothelins
s

 End
dothelin is a vasocon
nstricting p
peptide pro
oduced by the endotthelium. Th
here are
3 issoforms. Endotheliin-1 (ET-1
1) is one of the strongest vasocons
strictors
currrently stud
died.
 End
dothelins act
a on at least 2 subtyypes of rec
ceptors: ET
TA and ET
TB.
 Mosst of ET-1 effects are
a mediatted throug
gh ETA rec
ceptors pre
resent in vascular
v
smo
ooth ms an nd other tissues. Its activation leads to potent
p VC,, vascular smooth
ms proliferatio
on, cardiac
c hypertropphy, and elevation
e off blood preessure.
 Incrreased pro plicated in a variety of CVS diseases,
oduction of ET-1 hass been imp
including prim
mary pulmmonary hyp pertension phy, heart failure,
n, cardiac hypertrop
athe
erosclerosis, and corronary arte
ery disease
e.
 Bossentan is orally activve nonseleective bloc
cker of ET
TA and ETBB receptors
s, while
ambrisentan is a selec
ctive ETA b
blocker. Bo
oth drugs are approv
oved for tre
eatment
of p
pulmonary
y hypertennsion.

117
█ Purines as mediator
m rs

Nucleoosides (adeenosine) an
nd nucleottides (ADPP and ATP
P) are extraacellular chemical
mediators. ATP isi stored in vesicless and relea
ased by ex
xocytosis or through h tissue
damage. Released ATP is s rapidly converted to ADP and adennosine. The three
puriness act on th
hree main families oof purine receptors
r and exertt a wide ra ange of
functions.

▌Drug
gs acting on purine recepto
ors:

Adeno
osine

It is a sshort actin
ng purine A recepto r agonist. It is given
n by i.v. ro
oute to inh
hibit AV
conduc ction and converts
c supraventr
s chycardia to the sinuus rhythm in non-
ricular tac
asthma atic patientts (see CVS
S).

Methy
ylxanthine
es

Caffein
ne, aminop
phylline and
d theophyylline are purine
p A1 receptor
r antagonistts. They
a
are use
ed as bron
nchodilatoors and CN
NS stimula ants (see re
espiratory)).

118
█ Fattty acid derivative
d es (eicos
sanoids)

They are group ofo fatty acid derivativves derived


d from ara
achidonic aacid (20 ca
arbons).
The fam
mily of eico
osanoids in
ncludes:

Prosta
aglandins
s (PGs)

PGs arre memberrs of fatty acid deriva


atives. All are derive
ed from araachidonic acid by
the action of cycllooxygenase (COX) eenzyme wh hich has 3 isoforms:

 COX X-1 (physiiological; constitutiv


c ve): is invoolved in sy
ynthesis o
of protective PGs
(e.g
g. PGE2, PGI2) respo onsible for protection n of stomaach from H
HCl, regulaation of
RBF F, inhibition of platele
et aggregaation, etc.

 COX X-2 (pathhological; induciblle): is


invoolved in synthesis of undessirable
PGss (e.g. PGF2α, PGD2) involvved in
infla
ammatory reacctions,
bronchoconsttriction, etc
c.

 COX X-3 (centrral): it is a variant off COX-


2 th
hat found only
o in the
e brain an d may
be included in syntthesis of PGs
respponsible for fev ver and pain
sensation. It is selectiv vely inhibitted by
anaalgesic/antipyretic drugs e.g.
aceetaminoph hen (para acetamol)) and
dipyyrone.

Throm
mboxanes
s (TXs)

 The
ey are also products of cyclooxxygenase enzyme.
e
 The
ey are synthesized in high conccentration in platelets
s.
 The
ey are of 2 forms: TX
XA2 (the acttive form) and
a TXB2 (the
( inactivve form).
 A2 is the most
TXA m potentt endogenoous stimulator of platelet aggreegation.

Leuko
otrienes (L
LTs)

 The
ey are derivved from arachidonic
a c acid by the action of
o lipooxyg genase enzzyme.
 The
ey are of 4 main types: LTB4, LT TC4, LTD4, and LTE4.
 A m
mixture off LTC4, LTDL 4, LTE E4 is terme ed (the slow-reactinng substa ance of
ana
aphylaxis = SRS-A) th hat is relea
ased during g inflammaatory respo onse.
 B4 is powerful bronchoconstri ctor and chemotact
LTB c tic for leukkocytes.

119
Pharmacological actions of eicosanoids

PGs and other eicosanoids act on cell surface G-protein coupled receptors. These
receptors are termed DP1-2, EP1-4, FP, etc. The following table summarizes the
important effects of different eicosanoids:

PGE1, PGE2 PGI2 PGF2α TXA2 LTB4


BV VD VC VD
Edema + – +++
Severe
Bronchi Relaxation Constriction
constriction
Pregnant: contraction
Uterus Contraction Contraction
Non-pregnant: relaxation
Kidney ↑ RBF ↓ RBF ?
Platelets ↓aggregation ↑aggregation
- ↓ gastric HCl and
Other
↑ mucus secretion Chemotaxis
effects
- Hyperpyrexia

Therapeutic uses of prostaglandins

■ PGE1:
– PGE1 (Alprostadil) is still available as urethral inserts for treatment of erectile
dysfunction in men: it produces VD of the vascular smooth muscle of the
corpus cavernosum and improves erection.
– To maintain patency of the ductus arteriosus in infants with congenital
cyanotic heart diseases until surgical correction is undertaken. The ductus
arteriosus in neonates and infants is highly sensitive to VD by PGE1. Patency
of the ductus is necessary in those patients to ensure additional oxygenation
of the blood until surgical correction of the problem is undertaken.
– Treatment of peptic ulcer: Mesoprostol is approved for use in patients taking
high doses of NSAIDs to prevent gastric ulceration.

■ PGE2:
– Induction of labor near full term (Dinoprostone).
– Treatment of peripheral vascular disease.

■ PGI2 (Prostacycline):
– To inhibit platelet aggregation and prevent thrombosis during
cardiopulmonary bypass surgery, hemodialysis, and retinal artery occlusion.
– Treatment of pulmonary hypertension and peripheral vascular disease.

120
■ PGF
F2α:
– Induction of
o labor ne
ear full terrm (Enzaprost).
– T
Therapeutic abortion: as va aginal supppositories. It may b
be combine
ed with
PGE2 and d mifepristo
one (anti-p
progesterone) to prod duce moree powerful uterine
c
contraction.
– T
Treatment of open n-angle g glaucoma: to dilate e uveoscleeral vesseels and
e
enhance uveoscleral
u l outflow (↑ drainage)) of aqueou
us humor ((Latanopro
ost).

■ Thrromboxanes and leu s: they hav


ukotrienes ve NO currrent clinicaal uses.

█ INHIBITORS OF
O EICOSA
ANOIDS

■ Corrticosteroids: they inhibit pphos-


phoolipase A2 enzyme and co onse-
queently inhibit synthesis of ALL
L the
osanoids family (s
eico see endoc crine
phaarmacologyy).

■ Nonn-steroida
al anti--inflamma
atory
drugs (NSA AIDs): they inhibit cy-
cloo
oxygenasee enzyme:
– Cla
assical N pirin)
NSAIDs (e.g. asp
inhiibit all COX enzyme
e non-
sele
ectively.
– Newwer NSAIIDs (e.g. celecoxib)) in-
hibit COX-2 selectively.
s

■ Leu
ukotriene inhibitors:
i :

Zaffirlukast an
nd Montelukast:
– They blocck leukotrie
ene recepto ors.
– They are absorbed
a orally.
o Zafiirlukast it is given tw
wice daily b
but Montelukast is
given once daily.
– Uses: LTss receptor antagonissts are new
w drugs to be used inn the treatment of
bronchiall asthma and
a other i nflammato ory conditio
ons.

Zile
euton:
– It inhibits 5-lipooxyg
genase enzzyme leading to decrrease LTs ssynthesis.
– Uses: treaatment of bronchial
b asthma and other in
nflammato ry conditioons.

121
Part 2: Non-steroidal anti-inflammatory drugs (NSAIDs)

Classification

■ Non-selective COX inhibitors (inhibit COX-1 and COX-2):

– Salicylic acid derivatives: aspirin, aloxiprine, aminosalicylic acid, diflunisal,


methyl salicylate, etc.
– Acetic acid derivatives:
– Carboxylic acetic acid: indomethacin, sulindac, etodolac.
– Phenyl acetic acid: diclofenac
– Propionic acid derivatives: iboprufen, ketoprofen, fenoprufen, naproxen.
– Fenamic acid derivatives: mefenamic acid, fulfenamic acid.
– Pyrazolone derivatives: phenylbutazone, azapropazone
– Oxicams: piroxicam, tinoxicam.

■ Selective COX-2 inhibitors: celecoxib, etoricoxib, meloxicam.

▌Acetylsalicylic acid (Aspirin)

Chemistry and pharmacokinetics

 Oral absorption is complete; most of absorption occurs from the stomach and
upper GIT.
 Widely distributed to all tissues including CNS.
 Metabolism of salicylates occurs by the hepatic microsomal enzymes.
– At low doses, elimination is done by the first-order process.
– At high doses, elimination is done by the zero-order process.
 Excretion is increased by alkalinization of urine (pH 8) because in alkaline urine,
most of aspirin is ionized and less re-absorbable.

Mechanism and pharmacological effects

Aspirin is non-selective and irreversible COX inhibitor leading to inhibition of both


PGs and TXs. This distinguishes it from other NSAIDs, which reversibly inhibit COX
enzyme.

Analgesic action: for mild to moderate intensity pain (not severe pain).

Mechanism:
– Peripheral effect: decrease PGs synthesis in the peripheral inflamed tissues.
– Central effect: decrease PGs synthesis in the subcortical sites (thalamus and
hypothalamus).

122
Antipyretic effect:

Aspirin is antipyretic but NOT hypothermic agent i.e. it can lower elevated body
temperature but not normal body temperature.
Mechanism:
– Decrease PGE2 synthesis in the hypothalamus.
– Decrease the hypothalamic response to interleukin-1 (endogenous pyrogen).
– Cutaneous VD and increase sweating.

Anti-inflammatory, anti-immunological and anti-rheumatic effects:

Mechanism:
By inhibition of COX enzyme, it ↓ synthesis of PGs and TXs, and possibly other
inflammatory mediators leading to:
– Decrease inflammatory cell activation and chemotaxis.
– Decrease capillary permeability.
– Decrease hyaluronidase enzyme and inhibits spread of inflammation.
– Stabilize lysosomal membranes of inflammatory cells.

Respiratory effects:

– Low toxic doses: produce metabolic acidosis → compensatory


hyperventilation to wash excess CO2 → prolonged respiratory alkalosis.
– High toxic doses: produce metabolic acidosis together with inhibition of RC →
death from severe acidosis.

CVS effects:

– Therapeutic doses: no significant effect on the CVS.


– Toxic doses: inhibit VMC leading to circulatory failure.

GIT effects: salicylates can produce 2 types of gastric ulcer:

– Acute gastric ulcer: occurs as a result of acute ingestion of large doses of


salicylates. The pathogenesis is related to trapping of salicylate ions inside the
gastric mucosal cells leading to acute, painful gastric bleeding.

– Chronic gastric ulcer: occurs as a result of chronic ingestion of salicylates. The


pathogenesis is related to chronic inhibition of the protective PGE1, PGE2 and
PGI2 synthesis leading to chronic ulceration and chronic blood loss.

Hepatic effects: salicylates can produce 2 types of hepatic injury:

– Mild hepatic injury: it is dose-dependent, reversible and asymptomatic. There


may be mild increase in serum transaminases (SGOT and SGPT).

123
– Sevvere hepatic injury: “Reye’s
“ syyndrome”:: it is a
rare
e and fatal condition occurs if a aspirin is used
u to
conntrol fever of some viral infectio ons (e.g. chicken
c
poxx, influenzaa, etc.) in children below 12 2 years
old.. There is severe fa atty infiltra
ation of the liver,
panncreas, and kid
dney asssociated with
enccephalopatthy. The etiologyy is unk known.
Man nagement is supporttive.

Hemattologic efffects:

– Antiplatelet acction: aspirrin inhibit p


platelet aggregation by:
– Irreverssible inhibition of COOX enzyme e → ↓ TXA2 → ↓ plateleet aggregattion.
– Irreverssible acetylation of pla
atelet cell membrane
m s → ↓ platellet adhesio
ons.
– Decrea ase plateleet ADP syn nthesis → decrease platelet accuumulation.

– Asp gh doses (> 6 gm /day) inhiibits hepatic prothrrombin (fa


pirin in hig actor II)
synthesis → prrolong blee
eding time
e.

124
Renal e
effects:
– Anaalgesic neephropathyy: it is cchronic re
enal
Aspirin in a dose of 75-
failu
ure due to chronic abuse o of analgessics,
150mg is not thought to
which produc ce chronicc renal ischhemia duee to have a signnificant effe
ect on
deccrease syntthesis of reenal PGE2 and PGI2. plasma uraate levels - the
– Saltt and wateer retentionn: due to d
decrease RBF
R British Socciety for Rheu-
R
matology recommen nd it
andd increase aldosteron ne should be continued if re-
– Antaagonize diuretic
d efffect of d
diuretics and quired for cardiovas scular
antiihypertensive effects s of β-blocckers: due
e to prophylaxiss.
deccrease syntthesis of PGE
P 2.
– Uricc acid excrretion: smaall-to-mode
erate doses of
asppirin can ↓ uric
u acid ex xcretion andd thus con
ntraindicate
ed in patiennts with gouut.

Metabo
olic effectts:
– High doses cause
c oupling of oxidative phosphorrylation → tachycard
unco dia and
hyp
perpyrexia.

Uterus
s:
– Prolongation of pregnancy and
d delay of
labo
or due to inhibition of
o PGs nec cessary for
uterrine contra
action during labor. T
The use of
NSAAIDs after 20 weeks of pregna ancy is not
recoommended d.

Therap
peutic use
es of salicy
ylates

 As analgesicc and anttipyretic: in mild-to--


mooderate paiin e.g. hea
adache, art
rthritis, etc.
It should notn be used rou utinely as s
antipyretic be
ecause fevver may bee a norma al
pro
otective meechanism.
 As anti-inflam
mmatory and
a anti-rrheumatic
c:
in rheumatic c fever, rheumatoid
r d arthritis,
osteoarthritis, etc.
 As antithrom
mbotic: see box.

125
 As keratolytic and counter-irritant:
– Salicylic acid has keratolytic action and is used for treatment of warts.
– Methylsalicylic acid is used as a counter-irritant for local rheumatic pain.

Adverse effects of salicylates

GIT: – Epigastric pain, nausea and vomiting.


– Acute and chronic gastric ulcers (why?).
Hepatic: – Mild reversible hepatic injury in adults.
– Severe hepatic injury in children: “Reye’s syndrome”.
Kidney: – Analgesic nephropathy (why?).
– Salt and water retention.
– ↓ the diuretic effect of loop diuretics and antihypertensive effects
of β-blockers (why?).
– Precipitation of acute gout in hyperurecemic patients.
Blood: – Increase bleeding tendency: (why?).
– Displacement of other drugs from plasma proteins.
Uterus: – Prolongation of pregnancy and delay of labor (why?).
Hypersensitivity – Bronchospasm (aspirin asthma) in patients with history of
reactions: asthma or allergic rhinitis due to accumulation of LTs.

Precautions and contraindications

 GIT disorders: peptic ulcer, gastritis, hemorrhagic pancreatitis, etc.


 Hemorrhagic disorders: hemophilia, thrombocytopenia, etc.
 Chronic renal diseases: aspirin may aggravate renal failure
 Chronic liver diseases: (those patients have bleeding tendency).
 Uncontrolled hypertension: risk of fatal bleeding.
 Gout: small-to moderate doses may inhibit uric acid excretion.
 Before surgery: aspirin must be stopped at least 7 days before surgery.
 Children <12 years old: fear of Reye’s syndrome.

Drug interactions

 Aspirin antagonizes the uricosuric effect of probenecid (see gout).


 Aspirin antagonizes the diuretic effect of diuretics and the antihypertensive effect
of antihypertensives (e.g. β-blockers, ACEIs) by inhibition of PGs synthesis.
 Aspirin can displace anticoagulants (heparin and warfarin), and other drugs from
plasma proteins leading to increase their plasma concentrations.
 Antacids decrease absorption of aspirin.

126
▌Acute aspirin toxicity

Aspirin toxicity is dose-dependent and effects are determined by plasma drug levels

Manifestations:
– Tachypnea (hyperventilation) is the earliest sign of aspirin toxicity.
– Tinnitus, vomiting, hematemesis.
– Metabolic acidosis.
– Hyperthermia, hypoglycemia, dehydration, and coma.

Treatment:
– Repeated gastric lavage with activated charcoal.
– i.v. fluids and sodium bicarbonate to correct dehydration and acidosis.
– Vit K 10 mg i.m. or slowly i.v. to control hemorrhage.
– Alkalinization of urine: to enhance salicylate excretion.
– Hemodialysis in severe cases (when blood levels > 100 mg/dl).

█ OTHER NSAIDS

Diclofenac
– One of the most widely used NSAIDs.
– Generally, it is less gastric irritant than other NSAIDs but more nephrotoxic. A
dose of 150 mg/day can impair renal blood flow and GFR.
– It is often combined with PGE analogue misoprostol to decrease gastric
ulceration.
– Topical gel is available for local rheumatic pain and osteoarthritis of the knee and
hands.

Sulindac and ketorolac


– Both have minimal effect on platelet aggregation.
– Sulindac is a prodrug and has long t½.
– Ketorolac is a potent analgesic but it can cause severe GIT bleeding.

Ibuprofen, fenoprofen, ketoprofen


– They have no reported interactions with oral anticoagulants
– Ibuprofen is a good alternative to aspirin in children with flu fevers.
– Fenoprofen has been reported to induce nephrotoxic syndrome
– Long-term use of ibuprofen is associated with an increased incidence of
hypertension in women.

127
Indom
methacin
– It iss a very po
otent COX inhibitor b
but
relaatively moore toxic than oth her
NSA AIDs.
– It iss approved
d to speed the closu ure
of patent ductus
d arrteriosus in
prem mature infants (othherwise it is
not used in children);
c itt inhibits tthe
prod duction of PGs wh hich preve ent
clossure of the
e ductus.

Piroxic
cam
– It haas long t½
½ (~ 45 hours).
– Likee aspirin and indomethaccin,
bleeeding and d ulceratio
on are mo ore
likely with piro
oxicam thaan with oth
her
NSA AIDs.

▌Sele
ective CO
OX-2 inhibitors:
Celeco
oxib, etoric
coxib, me
eloxicam

Selectivve COX-2 inhibitors


s are newe er
forms of NSAIDss that dire ectly targe
et
COX-2 enzyme e respon nsible fo or
mation and
inflamm d pain. Celecoxib iis
approxximately 30
0 times more
m potennt
at inhhibiting COX-2 tha an COX-1 1.
Meloxiccam is slightly
s se
elective foor
COX-2 than COX X-1 enzyme e.
Seleectivity forr COX-2 re educes th he
risk of peptic ulc ceration bu ut does no ot
seem tto affect other
o adveerse-effectts
of NSA AIDs (espec cially the risk
r of renaal
failure).. Selectivve COX-2 2 inhibitorrs
may also incrrease the e risk o of
cardiov vascular accidents
a s (myocard d-
ial infa
arction, thrrombosis and a strokee)
due to relative in ncrease in TXA2 and platelet ag ggregation
n (TXA2 is fformed by COX-1
where PGI2 is formed by b COX-2 ). Rofeco oxib (Vioxxx®) and valdecoxib b were
withdraawn from the t market in 2004 an nd 2005 re
espectively
y because of these potential
p
adverse e effects.

128
Non-selective COX inhibitors Selective COX-2 inhibitors

Mechanism: They inhibit COX-1 (non- They inhibit COX-2


inducible) and COX-2 (inducible) enzyme only
(inducible) enzymes
Pharmacological
Both classes have equal analgesic and antipyretic effects
effects:
Gastric side effects: Frequent Less frequent
Renal side effects: Frequent Frequent
Thrombotic Decreased Increased
complications: (due to decreased platelet (due to increased platelet
aggregation) aggregation)

Hypersensitivity Frequent Less frequent


reactions:

129
Part 3
3: Acu
ute Rheu
umatic F
Fever (A
ARF)

Definittion:
It is ann inflamma
atory disea
ase that m
may develo op as a co
omplicationn of untre
eated or
poorly treated Grroup A β-hemolytic c streptococcal (GA ABHS) infeection of th
he URT.
It is cau
used by an
ntibody cro
oss-reactivvity (i.e. autoimmune disease).

nce
Inciden
 Agee: 5-15 yea
ars (rare 2--5 years - n
never below 2 years).
 Envvironmentaal factors: poor
p living conditions
s, overcrow
wding.
 Gennetic factors may plaay a role.
 No sex differe
ence in thee incidence e or pathog
genesis.

Etiolog
gy and patthogenesiis

The au utoimmun ne theory (molecula ar mimicrry): the disease usuually follow


ws URT
infectio
on with GABHS.
G This rheu c strain contain ssurface antigens
umatogenic a
(hyaluro
onic acid, cell wall polysaccha
p arides, M protein,
p c.) that aree immunologically
etc
similar to some host's
h tissu
ue. Antiboddies forme
ed against these ant igens will later on
cross-react with h connecttive tissuee antigens in the heart, syno ovial membranes,
caudate nucleus, skin and subcutan neous tissu
ue. Repea ated strepttococcal in
nfection
is required for se n of the im
ensitization mmune system.

Clinica
al manifes
stations: “J
Jones critteria first published
p 1944”:

Major criteria:
■ Polyrthritis (7
70%): the joints are e swollen and
a tendeer. It is fleeeting (mig
gratory),
affe
ects large joints,
j and subsides without deeformity.
■ Carrditis (50%
%): murmurrs, arrhythm mia, cardio
omegaly, pericardial
p rub.
■ Rhe eumatic ch horea (10% %): jerky li mb movemments and d emotionaal instability occur
moore in girls. It is revers
sible.
■ Sub bcutaneou us nodule es (5%): painless
noddules besid de muscle tendons.
■ Eryythema ma arginatum (1%): red d patches
oveer the limbss and trunk.

Minor criteria:
 Fevver.
 Arth
hralgia: pa
ain in the jooint.
 ↑ PR interval in the ECG G.
 ↑ an
ntistreptolyysin O titre
e (ASO).
Ac
cute carditis oof a child 8 years
y
 ↑CC-reactive protein
p andd ESR.

130
Diagno
osis: Revis
sed Jones
s criteria ( 1992
update
e):

At leasst 2 major criteria must


m be pre
esent OR 1
major + 2 mino or criteria PLUS evvidence of o
previouus streptoc
coccal infeection:
– Hisstory of ph haryngitis confirmed
c by positiv
ve
thro
oat culture
e for GABH HS.
– Elevvated AS SO titer > 400 U or othe er
stre
eptococcal antibodie es.

gement
Manag

▌TREAT
TMENT OF
F THE ACUTE EPISOD
DE

■ Bed
d rest:

– 2 weeks in
n absence of carditiss.
– 4 weeks in
n presence
e of carditis.
– 8 weeks in
n presence
e of heart ffailure or
cardiomeggaly.

■ Diet: Salt andd fluid resttriction in presence of


card
ditis or hea
art failure to
t avoid vo olume overrload.

■ Anttibiotics: to
o eradicate
e streptoco
occal infec
ction

– First choic
ce: benzylp
penicillin (1
1 million IUU/6h i.v. or i.m.).
– Second ch hoice (in penicillin alllergy): Co--trimoxazolle or erythrromycin.

■ Antti-inflamm gs: to sup press acutte inflammation


matory drug

A. RF withou ut carditis: salicylattes (aspirin


n)
Dose: 1000 mg/kg/d for 2 wee ks (till clinical manife
estations ssubside) th
hen give
75 mg/kg/d for 2 weeks
w theen 50 mg//kg/d for another 2 weeks (M Max 16
ttabs/d).
– Rapid breaathing (tacchypnea) i s the earliiest sign of
o aspirin tooxicity.
– Naproxen is an alternative for p patients who
w are alle ergic to asp
pirin.

B. RF with se evere card


ditis: cortiicosteroid ds (prednissolone)
Dose: 2 mg/kg/d
m forr 2 weeks (till clinica
al manifesta
ations sub
bside) then reduce
tthe dose to
o 1mg/kg//d for one mmonth.

131
Why corticosteroids, but not aspirin, in presence of carditis?

 Aspirin produces excellent symptomatic relief of arthritis and fever but exerts
non-specific effect on carditis.
 Aspirin cannot prevent pericardial rub or valve deformity, but steroids can
prevent them.
 High doses of aspirin increase myocardial O2 consumption and heart work and
so precipitate valvular lesions and CHF.

▌TREATMENT AFTER THE ACUTE EPISODE (PREVENTION OF RECURRENCE)

■ Long-acting penicillin: benzathine penicillin.


Dose: 1,200,000 IU / 3-4 weeks by deep i.m. injection

Duration of prophylaxis?
– For mild or no carditis → for 3 years from last episode.
– For moderate carditis → prophylaxis till 21 years.
– In severe carditis or recurrent episodes of RF → lifelong prophylaxis.

Precaution during treatment of ARF with other disease

ARF with – Avoid using corticosteroids because they are immunosuppressant


TB and cause flaring of TB infection.
– If it is necessary to use corticosteroids, it must be used under
umbrella of antituberculos drugs.

ARF with – Fluid and salt restriction


CHF – If digoxin is needed, use with caution to avoid arrhythmia (the
heart is very sensitive to digoxin during Rh activity).

ARF with – If corticosteroids are mandatory, consider adjusting the insulin


diabetes dose to avoid hyperglycemia.
– Consider possible interaction of salicylates with other antidiabetic
drugs.

ARF with – Give salicylates through multiple routes or as enteric coated


peptic preparations to avoid gastric irritation.
ulcer – Give salicylates after meals with plenty of fluids.
– Use H2 blockers or proton pump inhibitors.

132
Part 4: Drug
g therap
py of rhe
eumatoid arthrritis (RA))

Definittion
RA is a chronic inflammato ory diseasse characteerized by symmetricc small join nt infla-
mmatioon, swelling
g, and deformity. Ext
xtra-articula
ar manifesttations aree also com
mmon.

Pathop
physiology
y
The ex xact aetiollogy is un nclear howwever; gen netic facto
ors play a n importa ant role.
Activated T cells,, B cells, polymorpho
p onuclear le
eukocytes (PMLs), m macrophag ges, and
comple ement systtem lead to productiion of solu uble mediaators (lysossymes, cyttokines,
e.g. TNNF-α, IL-1, etc.) tha at cause jjoint inflam
mmation, cartilage
c d
destruction
n, bone
erosionn, and defformity. Why
W the im
mmune sys stem attac cks the jooint structu
ures as
"foreign
n" antigens, is unkno own.

Clinica
al manifes
stations

■ Artticular manifestation
ns:
Thee disease affects
a mainly small jjoints of
the hands anda toes in a sym metrical
fash
hion. In se
evere case es, large jo
oints are
also
o affected.. Affected joints
j are sswollen,
painful and d with limited m mobility.
Symmptoms area especia ally worsee in the
moorning. In late case es, chara acteristic
ms of joint deformity are prese nt.
form

■ Exttra-articular manifestations:
The
e spectrumm of clinical manifesstations off RA can extend
e to include sy
ystemic
gans e.g. pleural effus
org sion, perica mia, vasculitis, etc.
arditis, anem

133
Diagnosis

 Serological:
– Abnormal blood antibodies "rheumatoid factor" can be found in 80% of RA
patients.
– Detection of Cyclic Citrullinated Peptide antibodies (anti-CCP antibodies):
more specific than RF and can be detected up to 10 years before the
development of RA.
 Joint X-ray can show joint swelling and bony erosions typical of RA.

█ Management of RA

▌Symptomatic treatment

■ NSAIDs: large doses of NSAIDs are usually required. They offer symptomatic
relief of pain and inflammation but don't prevent joint destruction.

■ Corticosteroids: they are given as short-term therapy till DMARDs give their
effect.

▌Disease-Modifying Antirheumatoid Drugs (DMARDs)

 They are the most important and should be started as early as possible because
their effect may take 3 weeks to 3 months to be evident.
 They prevent progression of the disease and slow joint destruction by modifying
the immune reactions.
 Combinations of two or more DMARDs are more effective than a single drug.

■ Biologic DMARDs: TNF-α inhibitors


TNF-α plays a crucial role in the pathogenesis of RA, and these drugs became
the most important DMARDs.

– Adalimumab and infliximab: they are monoclonal antibodies that complex with
TNF-α and prevent its interaction with T cells and macrophages.
– Etanercept: it is a recombinant protein that interferes with TNF-α and prevents it
from binding to its receptors.
– Acute and chronic infections, live virus vaccination, demyelinating disorders,
class III or IV heart failure, and recent malignancies are contraindications to the
use of TNF inhibitors.
– Because these drugs are expensive, recent guidelines do not recommend their
use until at least one nonbiologic DMARD, usually methotrexate, has been
administered without sufficient success.

134
■ Methotrexate
– It is a folic acid antagonist with cytotoxic and immunosuppressant properties. It
is one of the first line drugs being used in more than 60% of RA cases.
– It inhibits multiple intracellular enzymes needed for activation of PMLs, T cells,
and macrophages.
– It is given once weekly orally. Folic acid 5mg must be given 24h after
methotrexate dose to compensate for folic acid deficiency.
– Common adverse effects: hepatotoxicity is common (monitoring liver functions
is essential), myelosuppression (bone marrow), teratogenicity.

■ Hydroxychloroquine (antimalarial drug)


– It decreases synthesis of DNA and RNA in the inflammatory cells and decrease
response of T cells to antigens.
– It also stabilizes lysosomal membranes.

■ Sulfasalazine
It is metabolized into sulfapyridine and 5-aminosalicylic acid. 5-ASA causes
inhibition of IgA and IgM production, and suppresses T and B cell functions.

■ Immunosuppressant drugs

– Cyclophosphamide and azathioprine: suppress immune function by their


cytotoxic action through a variety of mechanisms, particularly inhibition of DNA
synthesis.
– Cyclosporine: It is a potent and specific inhibitor of T cells. It inhibits
phosphorylation of calcineurin, a protein that regulates nuclear transcription
factors in the T cells.

■ Gold salts
Gold salts could be given i.m. or orally. Gold inhibits the functions of human
macrophages and decrease the release of inflammatory cytokines and growth
factors from inflammatory cells.

■ Leflunomide
– It suppresses pyrimidine synthesis and suppresses T cell and B cell functions.
– It is effective as methotrexate in inhibition of bone damage.

■ Anakinra
– It is a competitive IL-1 receptor antagonist.
– It has a relatively short half-life and must be administered subcutaneously daily.

135
Part 5
5: Drug
g therap
py of go
out

█ Basiic information

Gout iss a form of inflammatoory arthritiss due to


deposittion of mo onosodiumm urate cryystals in
the joiint tissue.. It is caaused by chronic
hyperuricemia (u uric acid > 7 mg/dll = 0.42
mmol/L L).

emistry
Bioche
 Uric
c acid is the end product
p o f purine
mettabolism byb xanthene e oxidase e
enzyme.
 Thee majorityy of body’s uric acid is
exccreted by the
t renal PCT.
P Som
me drugs
(e.g
g. diuretics) may y interfer e with
exccretion of uric acid at this ssegment
lead
ding to its retention.
 Thee renal excretion
e of uric acid is
enhhanced by alkalization of urine.

Causes
s of hyperruricemia
 Deccreased uric
u acid excretion:
e account
for 9
90% of ca
ases of prim
mary gout:
– Drugs: e.g. diurettics, aspirin, pyrazina
amide, ethambutol.
– Kidneyy diseases.
 Incrreased uric acid pro
oduction:
– High protein diet.
– During treatmentt of some hhematologic maligna ancies (e.g.. leukemia)) due to
tissue breakdown
b n (tumor lyysis syndro
ome).
– Sex-linnked uricac
ciduria (Lessch-Nyhann syndrome).
 xed causes: alcohol..
Mix

Pathog
genesis off gouty artthritis
 Whe en serum uric acid d increase es, monosodium ura ate crystalls are selectively
preccipitated in
n, and arouund the joiint tissue causing
c irritation and
d inflammattion.
 PNLLs and ma acrophages come an nd phagoc cytose the "foreign" uric acid crystals
cau
using swelling and infflammation n of the artticular tissue.
 Leu
ukocytes eventually
e die
d with th he release of proteolytic enzym mes and innflamm-
atorry mediatoors (e.g. LT
TB4, IL1, PGGs, etc) caausing seve ere inflammmation.

136
█ HYP
POURICEM
MIC DRUG
GS

▌Uric
cosuric drugs:
d Prrobeneciid

anism of action:
Mecha a it ha
as biphasic
c action:
 In SSmall doses: it inhib bits tubula
ar Secretiion of org ganic acidds (e.g. uric acid,
pennicillin, rifam
mpin) by in
nhibiting orrganic acid d transportters in tubuular cells.
 In la
aRge dose es (≥500 mg
m twice/da ay): it inhib
bits tubular Reabsorrption of uricu acid
by iinhibiting the urate trransporter (URAT1), leading to ↑ excretio on of uric acid.
a

Therap
peutic use
es
 As a uricosuriic agent in chronic goout.
 To p
prolong the t½ of some acidic drugs e.g. penicillin
and
d rifampicin
n by inhibittion of theiir renal exc
cretion.

Advers
se effects
– Gasstric irritatio
on, skin ra
ash, and ra
arely intersttitial nephrritis and ap
plastic ane
emia.

137
– If given during the acute attack, it can aggravate the inflammation because rapid
lowering of serum uric acid during the acute attack causes mobilization of uric
acid crystals from the tissue stores and aggravates the acute inflammation.

Precautions during the use of uricosuric drugs


– They should not be used during the acute attack but 2-3 weeks after the acute
attack has been subsided.
– Excess fluids and alkalinization of urine help uric acid excretion and prevent
stone formation.
– Aspirin may decrease uric acid excretion and antagonize probenecid.

▌Inhibitors of uric acid synthesis: Allopurinol

Mechanism of action
 Allopurinol is a synthetic purine analogue. It inhibits xanthine oxidase enzyme
leading to inhibition of uric acid synthesis.
 It has long duration of action because both the parent compound and its
metabolite are potent inhibitors of xanthine oxidase.

Therapeutic uses
 Recurrent attacks of gout (more than 2 attacks per year): the target uric acid
level is 5 mg/dl (0.3 mmol/L). Start with 100 mg once daily then gradually
increase the dose every 1-2 weeks to reach 300 mg/d.
 As an adjuvant therapy during treatment of hematologic malignancies (e.g.
leukemia) to prevent hyperuricemia resulting from tissue destruction.
 Patients with Lesch-Nyhan syndrome should receive allopurinol for life.

Adverse effects
– Gastric irritation and hypersensitivity reactions (skin rash).
– Precipitation of acute gout at the start of therapy due to mobilization of the
deposited uric acid crystals from tissue stores leading to transient hyperuricemia
Precautions:
– Do not give allopurinol during the acute attack of gout.
– Give prophylactic cover with NSAIDs or colchicine to avoid exacerbation of
gout at the start of therapy.

Drug interactions
Mercaptopurine, azathioprine, and theophylline are metabolized by xanthine oxidase,
and coadministration with allopurinol will dramatically increase levels of these drugs.

138
▌Increase uric acid metabolism: Pegloticase

 Pegloticase is a recent recombinant form of porcine uricase, the enzyme that


converts uric acid into the water-soluble allantoin.
 It is given as 8 mg i.v. /2 weeks. It can lower serum uric acid within 24 hours. .
 It is indicated in severe gout not responding to other agents.

█ ANTI-INFLAMMATORY DRUGS

Colchicine

Mechanism of action
 Colchicine is a plant alkaloid. It binds to intracellular microtubular system
leading to inhibition of leukocyte motility, phagocytosis, and cell division (mitotic
blocker).
 It inhibits the release of LTB4 and other mediators by leukocytes.

Therapeutic uses
 Acute attacks of gout:
– It is specific for gouty arthritis. Other
arthritic pains are not relieved by Familial Mediterranean
colchicine. Fever (FMF)
– It has slower onset than NSAIDs and Also known as (recurrent
corticosteroids. Reduction of polyserositis) is an
inflammation and relief from pain occur autosomal recessive disorder
12–24 hours after oral administration. which typically presents by
– Because of its slow onset and high the second decade. It is
toxicity, it is not a first-choice drug in characterized by recurrent
acute gouty arthritis unless the patient episodes of abdominal pain
has contraindication for NSAIDs or (due to peritonitis), pleurisy,
corticosteroids. pericarditis, arthritis. Attacks
typically last 1-3 days.
– The recommended dose is 1.2 mg orally
(2 tablets) initially followed by one tablet
(0.6 mg)/ 12 hour until the attack resolves.

 Familial Mediterranean fever (FMF): by an unclear mechanism.

Adverse effects

– The most common: diarrhea. Why?


Because the GIT epithelium has rapid rate of turnover. High oral doses of
colchicine will inhibit this continuous renewal of GIT epithelium (by inhibiting cell

139
division) leading to accumulation of toxins and bacterial products with diarrhea.
– The most rare: alopecia and myopathy.
– The most serious: aplastic anemia and agranulocytosis (bone marrow
depression).

NSAIDs
Indomethacin and naproxen

– Indomethacin and naproxen are FDA approved NSAIDs for acute gouty arthritis
although other NSAIDs can also work (but not aspirin).
– They provide symptomatic relief in acute attacks faster than colchicine.
– Their dose must be reduced if taken with probenecid because probenecid
inhibits their renal excretion.

Corticosteroids

– They are used as anti-inflammatory agents when colchicine and NSAIDs are not
sufficient to control the acute attacks or when they are contraindicated.
– For more details about the mechanism, see endocrine pharmacology.

140
141
Review Questions

 Mention uses of H1 blockers.


 Mention the main differences between 1st and 2nd generation antihistamines
 Mention the main differences between aspirin and celecoxib.
 Mention the main differences between aspirin and colchicine.
 Mention 2 uses of PGE1, PGE2, PGI2, PGF2α.
 Mention side effects of aspirin.
 Mention side effects of colchicine.
 Mention mechanism of aspirin as antiplatelet.
 Mention precautions during the use of antigout drugs.

Discuss on pharmacological basis each of the following:


 Aspirin is contraindicated as antipyretic in children with viral infection.
 Aspirin is contraindicated in chronic liver disease.
 Allopurinol is better to be avoided during acute attack of gout.
 Aspirin (low dose) is contraindicated with uricosuric drugs.
 Diphenhydramine is contraindicated in old males with benign prostatic
hypertrophy.

Discuss the pharmacodynamic principles underlying the use of:


 PGI2 during extracorporeal shunt operations (e.g. hemodialysis).
 PGI1 during surgical correction of patent ductus arteriosus.
 Colchicine in acute gouty arthritis.
 Methotrexate in rheumatoid arthritis.

Mention the rationale of the following combinations:


 Mesoprostol is advised with diclofenac (NSAID).
 Colchicine is given before allopurinol in acute gout.

142
Of each of the following questions, B. To produce uterine relaxation during
select THE BEST SINGLE answer: pregnancy
C. Temporary maintenance of the
1. Which of the following patency of ductus arteriosus in
antihistamines has the LEAST sedation preterm neonates
at therapeutic dose? D. Prevention of gastric ulceration
A. Diphenhydramine caused by NSAIDs
B. Loratidine E. Treatment of open angle glaucoma
C. Promethazine
D. Antazoline 7. All the following are effects of
E. Cyproheptadine serotonin EXCEPT:
A. Increased force of cardiac contraction
2. Cyproheptadine is an antagonist at: B. Vasoconstriction of the arterioles of
A. D1 and M1 receptors the pulmonary and renal beds
B. M1 and Nn receptors C. Stimulation of pain response
C. H1 and 5-HT receptors D. Contraction of bronchial smooth
D. H1 and H2 receptors muscles
E. β1 and α1 receptors E. Relaxation of the GIT smooth muscles

3. Sumatriptan is effective for the 8. The following statements about


treatment of acute migraine by acting NSAIDs are correct EXCEPT:
as: A. Diclofenac may cause permanent
A. An antagonist at β adrenergic receptors platelet dysfunction
B. A selective antagonist at H1 receptors B. Indomethacin may cause headache
C. An inhibitor of prostacycline synthesis C. Sulindac is a prodrug
D. An agonist at nicotinic receptors D. They may impair renal function
E. A selective agonist at 5-HT1D receptors E. Aspirin may displace coumarin
anticoagulants from plasma proteins
4. The pharmacologic effects of
acetylsalicylic acid include: 9. The following statements about
A. Reduction of high body temperature aspirin are correct EXCEPT:
B. Promotion of platelet aggregation A. May cause GIT hemorrhage after a
single dose
C. Reduction of pain by stimulation of
PGs synthesis B. Enteric-coated tablets cause less
gastric bleeding
D. Efficacy equals to that of diflunisal as
analgesic agent C. May cause metabolic alkalosis in high
doses
E. Less gastric irritation than other
NSAIDs D. May cause Rye’s syndrome in children
E. Its toxicity may require treatment with
hemodialysis
5. Aspirin reduces the synthesis of
the following eicosanoids EXCEPT:
A. TXA2 10. In the treatment of acute rheumatic
fever:
B. PGE2
A. Aspirin is used to suppress rheumatic
C. PGF2α
carditis
D. LTB4
B. Polyarthritis never resolve without
E. PGI2 residual deformity
C. Rheumatic chorea requires intensive
6. All the following are therapeutic therapy with corticosteroids
uses of prostaglandins EXCEPT:
D. Benzathine penicillin is used to
A. Medical abortion prevent recurrent streptococcal
infections

143
E. Erythromycin is considered the first E. Aspirin
line antibiotic to eradicate
streptococcal infection 16. In the treatment of gout:
A. Aspirin should be combined with a
11. Which one of the following uricosuric
statements concerning Cox-2 inhibitors B. Therapy with sulphinpyrazone should
is correct: be accompanied by high fluid intake
A. They show greater analgesic activity and alkalinization of the urine
than traditional NSAIDs C. Allopurinol will provide useful relief
B. They show anti-inflammatory activity from an acute attack
greater than traditional NSAIDs D. Colchicine can be used prophylacti-
C. They increase platelet aggregation caly to prevent recurrence
D. They harm the kidney as do non- E. Loop diuretics are indicated to wash
selective cox inhibitors the excess uric acid from blood
E. They are cardio protective
17. The following is true about
12. Manifestations of acute salicylate allopurinol:
intoxication include all of the following A. It has short duration of action
EXCEPT: B. It can act as a uricosuric drug
A. Hyperpyrexia C. It may worsen acute gout
B. Hyperpnoea D. The parent drug but not its
C. Hypertension metabolites are xanthine oxidase
D. Convulsions inhibitors
E. Metabolic acidosis E. Is not recommended for patients
treated by anticancer drugs
13. Which of the following NSAID is
used to treat patent ductus arteriosus 18. The following is true about PGF2α:
in neonates: A. It causes bronchodilatation
A. Ketoprofen B. It can be injected in the amniotic sac
B. Mefenamic acid to induce abortion
C. Celecoxib C. It increases renal blood flow
D. Phenylbutazone D. It increases platelet aggregation
E. Indomethacin E. It causes hypotension

14. Leukotrienes: 19. All the following drugs can


A. Are used for inducing labor in decrease platelet aggregation EXCEPT:
pregnant women A. Ketanserine
B. May be inhibited in patients treated B. Celecoxib
with aspirin and similar drugs C. Sulphynpyrazone
C. Are produced from amino acids and D. Aspirin
stored in granules E. Epoprostenol (prostacyclin)
D. May participate as SRS-A in asthmatic
or anaphylactic reactions 20. All the following drugs can be used
E. LTB4 is powerful platelet aggregator in the treatment of rheumatoid arthritis
EXCEPT:
15. Drugs used in the treatment of gout A. Methotrexate
include all of the following EXCEPT: B. Gold
A. Indomethacin C. Leflunomide
B. Allopurinol D. Prednisolone
C. Colchicine E. Colchicine
D. Probenecid

144
21. The most serious side effect of
colchicine therapy is: 27. Inflammation is a complex tissue
A. Agranulocytosis reaction that includes the release of
B. Diarrhea cytokines, leukotrienes,
C. Alopecia prostaglandins, and peptides.
D. Amyloidosis Prostaglandins involved in
E. Anorexia inflammatory processes are produced
from arachidonic acid by:
22. Which one of the following drugs A. Cyclooxygenase 1
could significantly impair the ability to B. Cyclooxygenase 2
drive an automobile? C. Glutathione – S – transferase
A. Diphenhydramine. D. Lipoxygenase
B. Ergotamine. E. Phospholipase A2
C. Fexofenadine.
D. Ranitidine. 28. A 60-year-old woman has
E. Sumatriptan. glaucoma following cataract surgery.
Which of the following can be used to
23. Which one of the following is an reduce intraocular pressure?
action of colchicine? A. Leukotriene LTD4 or its analogs
A. It produce uricosuria B. Prostaglandin E2 or its analogs
B. It produce constipation C. Prostaglandin F2α or its analogs
C. It produce polycythemia D. Slow-reacting substance of
D. It stimulates cell division anaplylaxis (SRS-A)
E. It inhibits leukocyte migration and E. Thromboxane A2 or its analogs
phagocytosis
29. A 45-year-old surgeon has
24. Which of the following is a developed symmetric early morning
uricosuric agent used in chronic gout? stiffness in her hands. She wishes to
take a nonsteroidal anti-inflammatory
A. Sulfasalazine
drug to relieve these symptoms and
B. Sulfinpyrazone
wants to avoid gastrointestinal side
C. Allopurinol
effects. Which one of the following
D. Colchicine drugs is most appropriate?
E. Chloroquine A. Aspirin
B. Celecoxib
25. Which one of the following drugs C. Ibuprofen
acts by blocking interleukin-1
D. Indomethacin
receptors?
E. Piroxicam
A. Anakinra
B. Sulfasalazine
30. The following drug is considered a
C. Gold saults
mast cell stabilizer:
D. Methotrexate
A. Adrenaline
E. Etanercept
B. Salbutamol
C. Cromoglycate
26. Which one of the following drugs D. Loratidine
acts by blocking soluble receptors of
E. Ondansetron
TNF-α?
A. Anakinra
B. Sulfasalazine
31. The following drug blocks 5HT2A
receptors in the CNS and is useful in
C. Gold saults
relieving the negative symptoms of
D. Methotrexate schizophrenia:
E. Etanercept

145
A. Buspirone A. It is an acute exacerbation of joint pain
B. Ondansetron treated with short-term anti-
C. Cyproheptadine inflammatory therapy
D. Olanzapine B. It is a chronic disease characterized
E. Alosetron by acute changes within nonsynovial
joints
32. The toxicity spectrum of aspirin C. It is an acute disease that is
does not include: characterized by rapid synovial
A. Increased risk of encephalopathy in changes due to inflammation
children with viral infections D. It is a chronic disease characterized
B. Increased risk of peptic ulcers by acute exacerbations followed by
remissions with consequences
C. Hyperprothrombinemia
associated with chronic inflammatory
D. Metabolic acidosis changes
E. Respiratory alkalosis E. It is a joint disease characterized by a
marked loss of calcium from the
33. Accidental poisonings are common bones and a resultant thinning of the
with both aspirin and ibuprofen, the bones
two drugs are available in tasty
chewable tablets. In cases of overdose, 37. All of the following statements
aspirin is more likely than ibuprofen to concerning an acute gouty arthritis
cause: attack are correct EXCEPT
A. Autonomic Instability A. The diagonosis of gout is assured by a
B. Hepatic necrosis good therapeutic response to
C. Metabolic acidosis colchicine because no other form of
D. Thrombocytopenia arthritis responds to this drug
E. Ventricular arrhythmias B. To be assured of the diagnosis,
monosodium urate crystals must be
34. Which of the following compounds identified in the synovial fluid of the
is most likely to lower circulating levels affected joint
of leukotrienes? C. Attacks frequently occur in the middle
A. Zileuton of the night
B. Montelukast D. An untreated acute attack may last up
C. Ibuprofen to 2 weeks
D. Aspirin E. The first attack usually involves only
E. Allopurinol one joint, most frequently the big toe
35. The action of aspirin that results in (first metatarsophalangeal joint)
its greater efficacy as an
antithrombotic (anti-platelet) drug is its 38. TNF- α is an example of:
ability to A. Eicosanoids
A. Inhibit lipoxygenase as well as B. Interleukins
cyclooxygenase C. Cytotoxic factors
B. Selectively inhibit cyclooxygenase I D. Interferons
C. Inhibit leukocyte migration E. Colony stimulating factors
D. Promote uric acid excretion
E. Acetylate cyclooxygenase 39. Potential adverse effects
associated with aspirin include all of
36. Which of the following statements the following EXCEPT
best describes the usual course of A. Gastrointestinal ulceration
rheumatoid arthritis? B. Renal dysfunction
C. Enhanced methotrexate toxicity
D. Cardiac arrhythmias

146
E. Hypersensitivity asthma

40. The cyclooxygenase isoenzymes


COX-1 and COX-2 differ from each
other in that:
A. They catalyse different pathways in
prostanoid biosynthesis
B. COX–1 is inhibited by aspirin but not
COX–2
C. COX–2 is inhibited by ibuprofen but
not COX–1
D. COX–1 is constitutive while COX–2 is
inducible
E. COX–2 is essential for renal function

41. Which one of the following drugs is


a selective inhibitor of central COX-3 in
the brain?
A. Paracetamol
B. Ibuprofen
C. Aspirin
D. Celecoxib
E. Colchicine

42. Irritable bowel syndrome with


constipation can be treated by:
A. Alosetron
B. Tegaserod
C. Ondansetron
D. Olanzapine
E. Buspirone

Answers

1B 11 C 21 A 31 D 41 A
2C 12 C 22 A 32 C 42 B
3E 13 E 23 E 33 C
4A 14 D 24 B 34 A
5D 15 E 25 A 35 E
6B 16 B 26 E 36 D
7E 17 C 27 B 37 D
8A 18 B 28 C 38 C
9C 19 B 29 B 39 D
10 D 20 E 30 C 40 D

147
148
Part 1
1: Systtemic hy
ypertens
sion and
d antihy
ypertens
sive dru
ugs

█ Bas
sic inform
mation

▌Vasc
cular smo
ooth musc
cle contrraction an
nd relaxation

Vasoco n is initiated by ope


onstriction ening of vooltage gateed L-type Ca2+ chan nnels in
2+
the meembrane off vascular smooth m muscle cellss. Increase
ed intracel lular Ca triggers
osphorylation of myo
the pho osin light ch e which iniitiates conttraction.
hain kinase

Vasodiilation is in
nitiated by
y either:
 Activation of guanylyl cyclase → ↑cGMP → dephosp phorylationn of myos
sin light
ain kinase → smooth ms
cha m relaxatiion.
 Ope +
ening of K chann nels in thee vasculaar smooth muscle ccell memb brane →
smoooth ms sttabilization (hyperpolarization) → relaxation.

▌The v
vascular endothellium and local vas
somotor control
c

Besidee the neu ural contrrol of blo ood vesssels through the ssympathettic and
parasympathetic systems, the vascu helium pro
ular endoth oduces vaarious compounds
that he
elp controlling the vas
scular tone
e:

Nitric o
oxide It is a diffusible gaas with very
v short half-life. It causes VD by
(NO; orr EDRF) increasing
g intracellu
ular cGMP P. It protec
cts againsst atheroscclerosis,
high blood
d pressure e, heart failure and th
hrombosis..
PGI2 It is synergistic to NO. It cause
es VD via specific
s reeceptors
Endoth
helin Is a 21-a
amino-acid
d peptide. By its action on ETA recepptors, it
causes se
evere VC a
and vascula
ar smooth muscle hyypertrophy
y.

149
Angiotensin- Located on the endothelial cell membrane and converts circulating
converting angiotensin-I (synthesized by the action of renin on
enzyme (ACE) angiotensinogen) to angiotensin-II. By its action on AT1 receptors,
Ang-II causes VC and stimulates aldosterone release.
Other factors Histamine causes VD, bradykinin (synthesized from kininogen)
causes VD, and serotonin (released by platelets) causes VC.

█ Management of systemic hypertension

Systemic hypertension is persistent elevation of BP above 140/90.

Classification of hypertension:
■ According to etiology: primary (= essential, 90%) or secondary.
■ According to type: systolic, diastolic, or mixed.
■ According to degree: see table.

Classification of blood pressure levels of the British Hypertension Society

Category Systolic BP (mmHg) Diastolic BP (mmHg)


Normal blood pressure <140 <90
Hypertension
Stage 1 (mild) 140–159 90–99
Stage 2 (moderate) 160–179 100–109
Stage 3 (severe) ≥180 ≥110
Isolated systolic hypertension >140 <90
N.B. If systolic and diastolic BP fall into different categories, the higher value should be
taken for classification.

Target BP:

 For most patients, the goal of therapy is to maintain BP < 140/90 mm Hg.
 In patients with DM or chronic kidney disease, BP should be < 130/80 mm Hg.

▌Non-drug therapy = life style modification:


 Dietary sodium restriction and potassium and magnesium supplementation.
 Stop smoking and avoid stress.
 Weight reduction alone can correct hypertension in 75% of obese patients.
 Daily physical exercise (~30 min/day): it lowers serum cholesterol, improves
endothelial functions, and enhances tissue oxygenation.

150
 Control of risk factors: e.g. diabetes mellitus, hyperlipidemia, and obesity.
 Avoid drugs that ↑ BP e.g. sympathomimetics, sodium-containing drugs, oral
contraceptive pills, corticosteroids.

N.B. Patients failing to normalize BP after 2 weeks of non-pharmacological therapy


should be considered for drug therapy.

▌Drug therapy (antihypertensive drugs):

First choice groups Second choice groups


(commonly used drugs) (used in special cases):

■ A ngiotensin-converting ■ α1- blockers: prazosin and doxasosin.


enzyme inhibitors (ACEIs) ■ Combined α and β-blockers: labetalol.
■ B eta-blockers ■ Adrenergic neuron blockers: α-
methyldopa and reserpine.
■ C alcium channel blockers
■ Vasodilators: e.g. hydralazine.
■ D iuretics
■ Central α2 stimulants: clonidine.
■ Endothelin blockers: Bosentan.
■ Dopamine agonists: fenoldopam.

█ Beta-blockers

For detailed mechanism and pharmacology: see chapter 2.

Indications of beta-blockers in hypertension:


 Hypertension associated with cardiac problems e.g. IHD, cardiac arrhythmia,
aortic dissection, etc.
 Hypertension associated with thyrotoxicosis.
 Hypertenstion associated with increased sympathetic overactivity.

█ Diuretics

For detailed mechanism and pharmacology: see chapter 3.

Indications of diuretics in hypertension:


 Thiazides are given as initial therapy in most cases of mild, non-complicated
essential hypertension.
 Hypertension associated with volume overload (salt and water retention): e.g.
CHF, liver cirrhosis, primary hyperaldosteronism, pulmonary edema, etc.

151
█ Angiotensin--convertin
ng enzym
me inhibittors (ACE
EIs)

▌Back
kground

▌The rrenin-angiotensin-aldosteron
ne (RAAS) system

 Whe ccurs, therre is ↓ RBF and ↓ GFR. This m


en renal isschemia oc may lead to acute
olig
guria that may
m develo op to acutee tubular necrosis.
 As a compensatory me echanism, rrenin is re
eleased fro
om the juxttaglomerular cells
of the kidney as a rescu
ue messag e to initiate
e stimulation of RAA
AS as follow
ws:

 Ang
g-II acts on
n AT1 rece
eptors in th
he efferent arterioles
s of the kid
dney causing their
VC and thus maintains
m adequate
a GFR in spiite of the ↓ RBF.
G
 Unffortunatelyy, Ang-II ac 1 receptorrs in other vascular beds and tissues
cts on AT1
cau
using systemic VC, cell hyp pertrophy, and apoptosis ((i.e. degen nerative
cha
anges).

152
Should we inhibit the RAAS?!
Inhibitors of RAAS
 Inhibition of the RAAS will  Inhibitors of renin release: β-blockers,
correct the hypertension but α-methyldopa, and clonidine.
also will ↓ GFR and aggravate  Inhibitors of plasma renin activity:
RF if renal ischemia was grave. aliskiren.
 Normal S. creatinine is 0.3-1.2  Inhibitors of Ang-2 formation: ACEIs
mg/dl. If S. creatinine is up to 3  AT1 receptor blockers (ARBs) e.g.
mg/dl (i.e. mild renal impairm- losartan, valsartan
ent) → you can block RAAS.
 If S. creatinine >3 mg/dl (i.e. severe renal impairment) → blocking the RAAS is
dangerous and will aggravate RF.

█ Angiotensin converting enzyme inhibitors

Classification
 SH-containing drugs: Captopril, zofenopril, alacepril
 Non SH-containing drugs: Enalarpril, fosinopril, lisinopril, benazepril, ramipril

Pharmacological properties of some ACEIs

Captopril Fosinopril Enalapril Lisinopril Benazepril


SH group + – – – –
Immune S/E + – – – –
Prodrug – + + – +
Metabolism Liver, Liver, Liver No Liver,
kidney kidney metabolism kidney
Onset 1–4h
Frequency of
/8 h /12 h /12 h /24 h /24 h
administration
SL dose 25 mg None None None None

Mechanism of action
They inhibit Ang-converting enzyme (ACE) in the vascular endothelium leading to:
– Inhibition of both Ang-II formation and aldosterone release (→ ↓ both VC
and salt & water retention).
– Prevent degradation of bradykinin which is a potent VD.
– Most ACEIs have direct VD action to both arteries (i.e. ↓ afterload) and
veins (i.e. ↓ preload).

153
Pharm
macologica
al effects

CVS
S:  They ↓ BP mainly by decreasin
T ng peripheral resistannce but no
o reflex
t
tachycard ia or channges in th
he COP ca an occur. This may be due
t either (1) resettting of barorecepto
to b ors; and/o or (2) ennhanced
p
parasympaathetic actiivity.
 They ↑ CO
T OP (only inn presence of CHF)) due to rreduction of both
v
venous retu
urn (preloa
ad), and sy
ystemic BP
P (afterload
d).
 They ↓ myocardia
T m l changes complic cating accute myoocardial
infarction (ventricu lar hyperttrophy, and densee collagen
n scar)
b
because they
t preve
ent myoc cyte cell hypertrophhy and collagen
c
s
synthesis (i.e. preven
nt cardiac
c remodeliing).
Otheer  They ↓ ap
T poptosis, ↓ cell hypertrophy, & ↓
tissu
ue c
collagen synthesiss (i.e. they
t reduce
d
degenerative change es caused by the action
o Ang-II on
of n AT1 receeptors).

peutic use
Therap es

■ Sys
stemic hyp
pertension
n:
– Hyperrenin
nemic hype
ertension.
– Normoreninemic hyppertension :
because th
hey are dirrect VDs.

■ Con
ngestive heart
h failurre (CHF):
– T
To ↓ afte
erload and
d preload through
reduction of both systemic vascular
resistancee, and aldo osterone rrelease (↓
Na and H2O retention n).
– T
To ↓ myyocardial hypertrop phy and
d
dilatation (i.e.
( ↓ remo
odeling).

■ To prevent LV
L hypertrrophy (rem
modeling)
er acute MI
afte M through
h:
– ↓ arterial BP (↓ myoca
ardial strain
n).
– ↓ myocytee cell hyperrtrophy, ap poptosis, and
a collage
en synthessis.

■ Diabetic neph
hropathy & microalb
buminuria
a:
– T
They ↓ re
enal chang
ges compl icating dia ephropathyy (mesang
abetic ne gial cell
a
apoptosis,, proliferration, aand colla agen syn nthesis), thus re
educing
m
microalbuuminuria (pprovided th
hat renal im
mpairmentt is not gra
ave).

154
Advers
se effects::
C : Dry Coug gh (the moost commo on): inhibittion of ACE leads too accumula
ation of
bradykinin
n and PGs, which caause bronchial irritatioon and spaasm.
Treatmentt: stop ACE
EIs. Cough
h will resolv
ve after a fe
ew days.
A : Angioede ema (edem ma of the e face, to ongue and
d
throat): du
ue to accuumulation of bradykinin or duee
to immune e reaction. It may be
e life threatening.
P : Aggravatio on of Proteinuria
P a in patients with
h
significantt renal failu
ure.
T : anges: tem
Taste cha mporary lo
oss of tastte (ageusia
a
and dysge
eusia).
O : Orthostattic (First dose)
d hyppotension:: especially
y
in sodium depleted (hypovolem mic) patien
nts.
Prevention
n: start by small at b edtime the
en increase
e graduallyy.
P : cy: teratoge
Pregnanc enesis (feta
al pulmonarry hypoplas
sia and rennal dysfunc
ction)
R : skin Rash
h
I : Increased erkalemia)) due to ↓ aldosteron
d K+ (hype a e release.
L : nia (neutro
Leukopen openia): esp
pecially in patients with
w impaireed renal fu
unction.

N.B. Th
he sulfhydryl group (-SH)
( pres ent in cap
ptopril may be responnsible parttially for
the im
mmunologic cal side effects
e e.g
g. angioe edema, ta aste chan nges, skin n rash,
leukop
penia.

Contra
aindication
ns
■ Hyp
potension: when sys
stolic BP iss less than 95 mm Hg
g.
■ Sev
vere renal failure or bilateral rrenal artery stenosiis (SCr > 3 mg/dl). Why?
W
– In these conditions, the use off ACEIs is dangerous becausee they ↓ An
ng-II → ↓
VC of the efferent arterioles
a → ↓ glomerular filtrattion pressuure and ↓ GFR →
on of renal failure in b
aggravatio both kidne
eys.
– Dangerouss hyperkalemia may occur.
– Dangerouss neutropeenia may o occur.
■ Pre
egnancy and
a lactattion: theyy may cau
use fetal pulmonary
p y hypoplas
sia and
grow
wth retardation.
■ Hyp
perkalemia
a.
■ Neu
utropenia,, thrombo a, or seve
ocytopenia ere anemia (ACEIs may caus
se bone
marrrow depre
ession).
■ Imm
mune prooblems: whether due to autoimmune diseaases or due
d to
imm
munosupprressive dru
ugs.

155
Precautions
– Initial dose should be small and at bedtime to avoid 1st dose hypotension.
– Frequent monitoring of kidney functions (S. creatinine) and potassium levels
one week after treatment and then every 3 months.
– Avoid use of K+ sparing diuretics to avoid severe hyperkalemia.
– Remember all other contraindications…

█ Angiotensin II receptor blockers (ARBs)


(Losartan- Valsartan- Candesartan- Telmisartan)

 They selectively block AT1 receptors and they exert most of the pharmacological
effects seen with ACEIs.
 They have no effect on bradykinin metabolism.
 They have the potential for more complete inhibition of Ang-II action compared
with ACE inhibitors because there are non-ACE enzymes (cathepsin and
chymase) that can convert Ang-I into Ang-II.
 The adverse effects and contraindications are similar to those of ACE
inhibitors but cough and angioedema are less common than with ACE
inhibitors.

ACE inhibitors ARBs


Class Angiotensin converting enzyme Angiotensin receptor blocker
inhibitor (ACEI) (ARB)
Mechanism Inhibition of ACE leading to: ↓ Blocking of AT-1 receptors
VC effect of Ang-II leading to ↓ VC effect of Ang-II
↑ VD bradykinins

Efficacy in Less effective because other More effective because it blocks


inhibition of enzymes rather than ACE can AT-1 receptor, the final station
RAAS convert Ang-I into Ang-II responsible for Ang-II effects.
Cough and Frequent due to ↑ bradykinins Less frequent (they do not ↑
angioedema bradykinins)

█ Direct renin inhibitors: aliskiren

 Activation of angiotensinogen into Ang-I by renin is the rate limiting step in


formation of RAAS.
 Aliskiren is a recently approved drug for treatment of hyperreninemic
hypertension. It inhibits renin activity and consequently the RAAS.
 The efficacy and side effects of aliskiren are comparable to ACEIs and ARBs.

156
█ Calc
cium chan
nnel bloc
ckers (CC
CBs)

These a
are drugs that
t block voltage-g
gated
2+
Ca ch hannels.

Classiffication off CCBs ac


ccording to
o
tissue selectivityy
■ CCBB with mainly cardiaac effects:
vera
apamil, diiltiazem.
■ CCB B with mainly vascular effectss
(dih
hydropyridiines): nifed
dipine,
amlodipine, nimodipin
n pine
e, nicardip
■ CCBB with main effect on
n other tis
ssue:
flun
narizine, cinnarizine
c e

Pharm
macologica
al effects

CVS Nifedip
pine Diltiazem
D Vera
apamil
Heart:
- Neggative inotrropic effec
ct — ++ +++
- A-VV conductioon — ↓↓ ↓↓↓
- HR ↑ (refle
ex) ↓ ↓↓

Blood vessels:
- Corronary VD +++
+ ++ ++
+
- Peripheral VDD +++
+ ++ ++
+
- Bloood pressurre ↓↓↓
↓ ↓ ↓

Other e
effects
 Theey relax all smooth muscles (v (vascular, bronchial,
b GIT, uterin ne, etc.).
 Theey ↓ Ca - mediated
2+
m cell
c necro osis and ap poptosis.
 Verrapamil ↓ in ase from pa
nsulin relea ancreatic beta
b cells bu
ut of little cllinical significance.

Therap
peutic use
es

▌Cardiio-selectiv
ve CCBs (v
verapamill & diltiaze
em):

■ Isch
hemic hea
art disease
e (IHD):
– They ↓↓ myocardial contractilit
c ty and myoocardial O2 demand.
– They prod
duce coronnary VD and d ↑ corona
ary blood flow.
– They ↓ Ca - mediate
2
2+
ed myocytte cell necrrosis.

157
B. vasculosselective CCBs
N.B C (dihyydropyridinnes) e.g niifedipine aare also be
eneficial
in IIHD but they cause e conside erable peripheral VD D, so the dose sho ould be
adju
usted to avvoid hypotension andd reflex tac
chycardia.

■ Carrdiac arrhy
ythmias: supravent
s ricular tac
chycardia (SVT):
– SVT includ
de atrial flu
utter, fibrilla
ation, paro
oxysmal atrial tachyccardia, etc.
– Verapamil ↓ AV cond duction, so o it protec
cts the ven
ntricles fro
om the rappid atrial
rate (also β-blockers
β s have the same effect).

B. Nifedipine is contraindicated a
N.B as it causes
s hypotens
sion and refflex tachyc
cardia.

■ Hyp
pertrophic
c obstructive cardio
omyopathy
y (HOCM)::
– In hypertrophic obsstructive caardiomyop
pathy, the wall of
the left ventricle
v and interve entricular septum is s much
thickened leading to o narrowin ng of the aortic outtlet and
on of blood
obstructio d flow.
– Increasing
g contracttility worse ens the obstruction
o n while
decreasingg contracttility reduc es resistan
nce to blood flow
through the aortic ou
utlet and im
mprove exerrcise tolera
ance.

B. Nifedipin
N.B ne is contra
aindicatedd because it produce es reflex
hycardia → worsening
tach g of the ou
utflow obsttruction.

■ Arte
erial hypertension:
– They ↓ myyocardial contractilityy and COP.
– They causse periphe eral VD d ue to ↓ CaC 2+ influx
x in the
vascular smooth
s ms
s.

▌Vascu
ulo-selecttive CCBs (Nifedipin
ne and am
mlodipine)::

 Arteerial hyperttension.
 Perripheral vascular
v disease (e.g. Ray ynaud’s disease
d aand intermittent
udication): to improve peripherral microcirrculation.
clau
 Nife
edipine is sometimes used to rrelax the uterus
u and delay pretterm labor..
 Nim
modipine has
h high afffinity for c
cerebral BV
V. It is use
ed for preveention of cerebral
c
vasospasm an
nd ischemia complic
cating sub
barachnoid hemorrhaage.

Advers
se effects
Verrapamil & diltiazem::
– Bradycard dia and hea
art block.
– Worsening g of CHF (d
due to theiir –ve inotrropic effectt).
– Constipatiion due to ↓ GIT mottility.

158
Nife
edipine:
– Hypotension and refflex tachyccardia.
– Gingival (g gum) hyperrplasia.
– Salt & watter retentio
on (=ankle edema;
more com mmon than with verappamil due
to significa
ant VD and
d hypotenssion).

Contra
aindication
ns & preca
autions

Verapa
amil & diltiazem:
– Con
ngestive he
eart failure
e.
– Braadycardia and
a heart block.
b
– Wolff-Pakinso
on-White syndrome:
s
 In WPW syndrom me, there e is
accessoryy conducting path hway
between atria
a and ventricles
v o
other
tthan the e normal AV sysstem
causing a uniqu ue type of
atrioventric
cular re-en
ntry tachyc
cardia.
 A
As a rule, most druggs that ↓ A
AV nodal co onduction (Adenosinne; Beta-bllockers;
CCBs; Dig goxin) can paradoxic cally ↑ the conduction in the ab
bnormal pathway
p
leading to worseningg of the tac
chycardia.
 W
WPW syndrome can be man naged by amiodaro
a ne but deefinitive tre
eatment
aser ablatio
includes la on of the a
accessory pathway (s
see later).
– Veraapamil sh
hould not be comb h digitalis or β-blo
bined with ockers as it may
agg
gravate bra
adycardia caused
c byy these dru
ugs (nifedip
pine is the drug of ch
hoice to
be u
used with β-blockers
β s because it doesn't cause braddycardia).

Nifedip
pine:
– Hyp
potension.
– Hyp
pertrophic obstructiv ve cardiom yopathy (H HOCM) Wh
hy?
– Unsstable angina (risk off reflex tachhycardia).
– Sup
praventricuular tachyccardia (SVT T) Why?

159
█ Vaso
odilators

Classiffication

■ Arte
erio-dilato
ors: Nifedipine – Hyddralazine – Minoxid
dil – Diazoxxide
– TThey dilate a ↓↓ BP (→ ↓ afterlo
e arteries and oad)
– TThey are used
u in sevvere system
mic hypertension.

■ Ven
nodilators: Nitrates
– TThey dilate eins →↓ ven
e mainly ve n (→ ↓ preload)
nous return
– TThey are used
u in acuute pulmonnary edema.

■ Mix
xed dilatorrs: Sodium
m nitropru sside – Prrazosin – ACEIs
A – Trrimetapha
an
– TThey ↓ pre
eload & afte
erload so tthey are us
sed in CHF
F.

Genera
al conside
erations
 Vassodilators relax
r ular smootth ms and ↓ peripheral resistannce.
vascu
 Theey usually cause reflex sympaathetic stiimulation (e.g. reflexex tachycardia) so
theyy can be combined
c with
w beta-b blockers.
 Theey usually cause sa alt and wwater rete ention (du
ue to refleex stimula ation of
aldo
osterone reelease) so they shou
uld be combined withh diureticss.
 Thee use of vasodilators
v s is declin
ning as a result of newer mo odalities, such
s as
ACEEIs and CCCBs, which h are more
e effective with
w fewerr adverse eeffects.

Hydralazine

 It is a direcct arterio
olodilator by uncle
ear
mec chanism.
 It iis used in severe e hyperte ension an nd
rd
hyppertension in pregna ancy (3 c choice aftter
α-mmethyldopa a and nifed
dipine).
 It is usually combine ed with d diuretics to
couunteract sa alt and waater retenttion, and β-
blocckers to counteract
c reflex tachhycardia.
 It m
may cause systemic lupus eryythematos sis
(SLEE)-like syyndrome especiallyy in slo ow
aceetylators. Th
he patient develops mild form of arthritis, renal imp
pairment, and
a skin
rashh that usua
ally disappe
ear upon sttopping of the drug.

Minox
xidil

 It iss a direct arteriolodilator byy opening


g K+ channels → hyyperpolarizzation →
rela
axation of the
t vascula
ar smooth ms.

160
 It is given orally for chronic hypertension but its use as antihypertensive is
declining; however, it should be combined with diuretics and β-blockers.
 It was found to stimulate hair growth (hypertrichosis) (by unclear mechanism),
so it is now used topically to prevent hair loss in both males and females.

Diazoxide

 Structurally related to thiazides but it is not diuretic


 It is a direct arteriolodilator by opening K+ channels → hyperpolarization →
relaxation of the vascular smooth ms.
 It is given parenterally in hypertensive emergencies but its use is declining.

Sodium nitroprusside

 It liberates nitric oxide (NO) → ↑ cGMP → dilatation of both arteries and veins →
↓ both preload and afterload.
 It is given by i.v. infusion in hypertensive emergencies and acute heart failure
because it has rapid action
 It can be converted to cyanide and thiocyanate. The accumulation of cyanide
and risk of toxicity are minimized by concomitant administration of sodium
thiosulfate (see angina) or hydroxocobalamin (vitamin B12).
 Sodium nitroprusside in aqueous solution is sensitive to light and must be
made up fresh before each administration and covered with opaque foil.

Fenoldopam

 It stimulates peripheral dopamine (D1) receptors in renal and mesenteric arteries,


leading to VD and decrease peripheral resistance.
 It is used parenterally as a rapid-acting vasodilator to treat emergency
hypertension in hospitalized patients.

█ Endothelin-1 receptor antagonists

 Endothelin-1 is 21 amino acid peptide. It is the predominant endothelin secreted


by the vascular endothelium. It is elevated in patients with pulmonary
hypertension and coronary artery disease.
 It acts on two types of receptors, ETA and ETB. ETA receptors is the main
subtype in smooth ms and mediates VC and vascular smooth ms hypertrophy.
 Bosentan is orally active nonselective blocker of ETA and ETB receptors, while
ambrisentan is a selective ETA blocker.
 Both drugs are approved for treatment of primary pulmonary hypertension.

161
█ Specialized vasodilators
Drugs used for erectile dysfunction: Sildenafil, tadalafil

 These drugs inhibit phosphodiesterase (PDE) type 5 the enzyme responsible for
breakdown of cGMP in erectile tissue (and lung) → ↑ cGMP → VD of the corpus
cavernosum.
 They are very effective for treatment of erectile dysfunction in men. The effect
of the drug appears after 30 min of oral administration and lasts for 4-5 hours.
 Sildenafil is also approved for treatment of pulmonary hypertension.
 Side effects include blue discloration of vision, headache, and optic neuropathy.
 These drugs are contraindicated in patients taking nitrates or nicorandil for
treatment of angina because nitrates also act by ↑ cGMP leading to severe VD,
hypotension, and reflex tachycardia → aggravation of angina and development of
arrhythmia.

▌Choice of the antihypertensive drugs:

Clinical condition Best choice


Starting therapy for non-complicated Patient <55 years old: ACE inhibitors
essential hypertension Patient >55 years old: CCBs
If not adequate, add thiazide or beta
blockers.
Hypertension of pregnancy α-methyldopa
Labetalol – Nifedipine - Hydralazine
Hypertension in a diabetic patient with ACE inhibitors
diabetic nephropathy (proteinuria)
Hypertension with chronic kidney disease S. creatinine <3 mg/dl: ACEIs
S. creatinine >3 mg/dl: CCBs
Hypertension with CHF ACE inhibitors - diuretics
Hypertension with bronchial asthma CCBs
Hypertension with angina CCBs - beta-blockers
Hypertension with thyrotoxicosis, Beta-blockers
sympathetic overactivity, arrhythmia,
or, HOCM
Hypertension with BPH α1 blockers
Primary pulmonary hypertension Endothelin receptor antagonists

162
▌Management of hypertensive emergencies:

Hypertensive emergencies are clinical Hypertensive


situations associated with one or more of the encephalopathy
following:
Is a clinical presentation
 BP > 180/120mmHg. consists of severe headache,
 Target organ damage e.g. cerebral mental confusion, blurred
stroke, encephalopathy, heart failure, vision, and focal neurologic
aortic dissection, edema of the optic disc signs. If untreated it may
(papilledema). progress over a period of 12–
48 hours to convulsions,
Management: coma, and even death.

– Hospitalization and start parenteral


therapy to lower BP rapidly (within a few hours not minutes).
– Chronic hypertension is associated with autoregulatory changes in cerebral,
myocardial, and renal blood flow; so if sudden lowering of BP is done,
cerebral, renal, and myocardial ischemic events can develop.
– The initial target in the first 1-2 hrs is to lower systolic BP by no more than
25%, maintaining diastolic BP at no less than 100 mmHg.
– Drugs commonly used: sodium nitroprusside, labetalol, fenoldopam, all
are given by slow i.v. infusion.
– Recent guidelines state that the following drugs are not recommended:
 Nifedipine, nitroglycerin, and hydralazine: because these agents can
cause sudden, uncontrolled, and severe reductions in BP that may
precipitate cerebral, renal, and myocardial ischemic events with fatal
outcomes.
 Furosemide can lead to significant volume depletion and should be used
only if there is associated volume overload as in case of pulmonary edema
and acute heart failure.

163
Part 2
2: The
erapy off periph
heral vas
scular disease
d ((PVD)

It is a narrowing of the arteries o other thann coronary y and cerrebral ves ssels. It
commo only affectss arteries of
o lower lim
mbs, but also renal and mesentteric arteries.

Risk fa
actors
 Diabbetes and smoking area the mo ost importa
ant risk factors.
 Age e > 65 yearrs, hyperch
holesterole
emia, hype
ertension and obesityy.

Manife
estations
 Inte
ermittent cllaudication
n: pain in mmuscles wh hen walkin
ng
 Resst pain in th
he soles off the feet, p
particularly
y when the
e feet are eelevated.
 Skinn: cool, blu
uish, ischemic ulcerss.

gement
Manag

▌Life-s
style:
 Stop smoking: the most important factor
 Conntrol of risk
k factors:
 S
Statins for hypercho
olesterolem
mia.
 T
Treatment of hyperte
ension andd DM.

▌Drug therapy:

Pentox
xifylline

 It in
ncreases RBC
R deform
mability byy inhibition
n of PDE enzyme;
e thhis effect reduces
r
bloo od viscositty and facilitates passsage of RBCs through narrow wed capillarries and
isch hemic sitess.
 It iss the 1st drrug approv
ved to imp prove micrrovascular circulationn in patien
nts with
inte ermittent claudicati
c ion, diabettic angioppathy, and chronic leeg ulcers. Typical
dosse is 400 mgm twice a day.

azol
Cilosta

 It is the 2nd drug recently


y approvedd for treatm
ment of inttermittentt claudicattion.
 It in
nhibits PDEE enzyme (type 3) le V and ↓ platelet agg
eading to VD gregation.
 Hea adache is the most common
c sside effect.
 Clioostazol is contraind dicated in patients with CHF F because recent ev vidence
sho owed that inhibitors of
o PDE enzzyme type 3 increase e mortality in those patients.
p

Clopid
dogrel
Antipla
atelet agennts such clopidogr el (75 mg g/d) have additionaal benefits
s when
compa ared with aspirin in diabetic pattients with PVD.

164
Part 3
3: Isc
chemic heart
h dis
sease and
a antia
anginal drugs

█ Bas
sic inform
mation

▌Ische
emic hea
art diseas
se include
es:

■ Chrronic sta able ang gina (Cl assic;


exe
ertional anngina):
- It is due to atheromatous naarrow-
ing of the coronary artery.
- Pain is induced by effortt and
disappearrs with rest.

■ Acu
ute corona
ary syndro
omes (ACS
S):
- Unstable
e angina: It is du
ue to
rupture ofo atheromatous p plaque
and form mation of thrombuss. The
patient exxperiences
s accelerattion in
the frequency or severity
s of chest
pain, or ne
ew-onset angina
a pain
n.
- Myocardiial infarcttion: An inntraluminall thrombuss complettely occlud
des the
epicardial coronary artery at the site of plaque rup
pture leadiing to irrev
versible
coagulativve necrosis
s.

■ Prin
nzmetal’s angina (V
Variant ang
gina; angin
na of rest; α-mediatted angina):
The
e coronary artery und
dergoes se evere spas
sm due to overactivitty of α1 rec
ceptors.
The
e patient de
evelops pa
ain at rest.

▌Chro
onic stablle angina
a

Definittion: retrosternal pa
ain due too ischemia
a of the myocardium
m m as a reesult of
imbalan nce betweeen heart work
w emand) and
(O2 de d coronary
y blood flow
w (O2 supp
ply).

165
Clinica
al picture:
Centra ain is the cardinal
al chest pa c syymptom:
 Site
e and radiiation: retrrosternal, radiating to
t
the left should
der and thee left arm.
 Chaaracter: an ny character (usuallly sense of
cheest tightnesss).
 Precipitated by
b 3E: exertion, emo otion, eating,
andd relieved by
b rest andd nitrates.
 Durration: usuually < 10-15 min. If longer tha an
15 mmin → susppect ACS.

Diagno
osis:
 ECG
G:
- RResting 12
2-lead ECG G: this is o
often normmal
a
and doess not exc clude isch hemic hea art
d
disease.
- DDuring atttack: the ere is S ST segme ent
d
depression
n and T-wa ave inversiion.
- IIn myocardial infarcttion: ST ellevation an
nd
d
deep Q-waave.

 ercise ECG: reco


Exe ording E ECG und der
con
ntrolled physical effo
ort to reco
ord ischem
mic
cha
anges.

 Nuc
clear isoto s imaging..
ope stress
 Corronary ang
giography
y.

█ Man
nagemen
nt of stab
ble angin
na

■ Non
n-drug the
erapy = life
e style mo
odification
n:
 T
The same as hyperte
ension (see
e before).

■ Pha
armacolog
gical thera
apy:
 Immediate treatme ent of acutte chest pain:
p
– Glyceryyl trinitrate ublingual or spray.
e (GTN): su
– Aspirin
n 300 mg lo oading dosse as soonn as possib
ble. It
reducees the risk of progresssion to MI.
– Refer the patient to hospitaal if an ACS
S is suspec
cted.

 Long-term
m therapy:

166
– Beta-blockers: the first-line agents for chronic stable (exertional) angina.
– CCBs: the second-line agents for chronic stable angina
– Long and intermediate acting nitrates.
– pFOX inhibitors: trimetazidine
– Newer antianginal drugs: ranolazine and nicorandil
– Lipid lowering drugs: statins (see chapter 6).
– Antiplatelet drugs: e.g. aspirin, clopidogrel (see pharmacology of blood).

■ Surgical treatment (myocardial revascularization).

Organic nitrates and nitrites

Classification
Dose Onset Duration
Short-acting nitrates:
Amyl nitrite crushable ampoules 0.3 ml inhalation 1-2 min 5-10 min
Glyceryl trinitrate tablets or spray 0.5 mg SL 1-5 min 10-20 min
Isosorbide dinitrate 5 mg SL 3-5 min 60 min
Glyceryl trinitrate (Tridil®) 5 μg/min i.v.i.
Intermediate-acting nitrates:
Isosorbide dinitrate 10 mg oral 15 min 3-6 hrs
40 mg oral SR 30 min 6-10 hrs
Long-acting nitrates:
Isosorbide mononitrate 20 mg oral 30 min 6-8 hrs
60 mg oral SR 30 min 6-10 hrs
Transdermal patches 30 min 12-18 hrs

Pharmacokinetics
Absorption: nitrates are rapidly absorbed from all sites of administration.
Metabolism: in the liver:
– If given oral → extensive first-pass metabolism (oral bioavailability <10%)
– If given sublingual → no first-pass metabolism → high bioavailability.
– Mononitrate: has no hepatic metabolism → long duration of action.
Excretion: via the kidney.

Mechanism of action
 Nitrates cause formation of the free radical nitric oxide (NO) which is identical to
the endothelial derived relaxing factor (EDRF) → ↑ cGMP → VD (more on veins
than arteries).
 They also ↑ formation of vasodilator PGE2 and PGI2.

167
Pharmacological effects

CVS: Blood vessels:


– VD of the venous (and to lesser extent the arterial) side leading
to ↓ preload and ↓ afterload → ↓ cardiac work.
– VD of coronary arteries leading to increased coronary blood
flow.
– VD of arteries in the face and neck leading to flushing of the
face.
– VD of meningeal arteries leading to throbbing headache.
Heart: Reflex tachycardia (in high dose) 2ry to ↓ BP.
BP: High doses cause ↓↓ in both systolic and diastolic BP.
Smooth ms: Relaxation of all smooth ms (bronchial, GIT, uterine, and biliary).
Respiration: Reflex tachypnea due to hypotension in high doses.
Blood: Methemoglobinemia in high doses due to oxidation of Hb into met-
Hb.

Therapeutic uses

■ Angina pectoris
Nitrates are used for treatment of all types of angina both for relieving the acute
attack and for prophylaxis. The mechanism is due to:

 Nitrates cause formation of the free radical nitric oxide (NO) which is
identical to the endothelial derived relaxing factor (EDRF) → ↑ cGMP → VD
(more on veins).
 They also ↑ formation of vasodilator PGE2 and PGI2.

These effects lead to:


 Decrease cardiac work & myocardial O2 demand through:
 Venodilatation → ↓ venous return (preload = ↓ end-diastolic pressure).
 Arteriolodilatation → ↓ peripheral resistance (afterload).

 Enhancement of coronary blood flow (perfusion) through:


 Coronary VD.
 Redistribution of blood from large epicordial vessels to ischemic
subendocardial vessels.

■ Myocardial infarction: to ↓ the area of myocardial damage.


■ Acute heart failure: to ↓ preload and afterload.
■ Treatment of cyanide poisoning: see box

168
Adverse effects
Treatment of cyanide
– Hypotension and reflex tachycardia:
may aggravate angina. poisoning

– Throbbing headache: due to VD of Principle: cyanide has high affinity


meningeal arteries. for metHb more than normal Hb.

– Flushing of the face.  Sodium nitrite (300 mg i.v.) is


– Nitrate tolerance: means diminished given to convert part of Hb to
metHb to attract cyanide ions
response to nitrates with continuous
and form cyan-metHb.
administration which cannot be
corrected by increasing the dose. The  Sodium thiosulphate (25 gm
exact mechanism is unclear but there i.v.) is given to convert cyan-
are 2 theories to explain this: metHb to thiocyanate (non-
toxic) → renal excretion.
 Recent studies showed that
continuous administration of nitrates
leads to formation of free radicals of the reactive oxygen species (ROS)
leading to oxidation and inhibition of the enzyme MALDH2 responsible for
bioactivation of nitrites into the vasoactive NO.
 Prolonged VD by nitrates leads to reflex sympathetic stimulation and
activation of renin-angiotensin system → VC and salt & water retention.
 Prevention of nitrate tolerance: make a daily nitrate-free interval (10–12 h) to
give chance for bioactivating enzymes to regenerate. During this period, give
another anti-anginal drug e.g. beta-blocker or CCBs.

– Methemoglobinemia: rare and require high doses.

Precautions during nitrate therapy


– Use the smallest effective dose to avoid hypotension and reflex tachycardia.
– The patient should consult his doctor if anginal pain does not improve after
taking 3 SL tablets of GTN during 15 min (the pain may be due to MI).
– Nitroglycerine tablets should not be put in direct sunlight (light sensitive) or with
cotton (to avoid formation of the explosive nitrocellulose).
– The expiry date should be checked (active tablets have burning taste).
– Nitrates should not be used with sildenafil. Why?

Beta-blockers

 Beta-blockers are considered first-line in chronic exertional (classic) angina


(note that short acting nitrates are the first line during the acute attack).
 Treatment objectives include lowering the resting HR to 50-60 beats/min and
limiting maximal exercise HR to ~ 100 beats/min or less.

169
 There is little evidence to suggest superiority of any particular β-blocker, but β-
blockers with ISA should be avoided because the reduction in HR and O2
consumption would be minimal.
 They are contraindicated in Prinzmetal’s (variant) angina because they block
the β2-mediated coronary dilatation leaving the α1 receptors unopposed → ↑
coronary spasm.

Mechanism of β-blockers in exertional angina


 They ↓ contractility, HR, and systolic BP → ↓ myocardial work and O2 demand.
 They ↑ diastolic (coronary) filling time.
 Cause redistribution of blood from normal to ischemic (subendocardial) regions
 Cytoprotective effect: they produce metabolic switch from myocardial fat
utilization to carbohydrates utilization (i.e. improves myocardial metabolism).

Combination of BBs and nitrates ↑ their efficiency & ↓ their side effects:

β-blockers Nitrates Combination


– HR ↓ ↑ (Reflex) ↓ or no effect
– Contractility ↓ ↑ (Reflex) ↓ or no effect
– Diastolic filling time ↑ ↓ ↑ or no effect
– Blood pressure ↓ ↓ ↓↓

Calcium channel blockers (CCBs)

 They are considered first-line treatment for Prinzmetal’s (variant) angina.


 They are considered second-line alternative after beta-blockers in chronic stable
angina in whom beta-blockers are contraindicated.
 Short acting dihydropyridines are associated with increased risk of ACS and
should be avoided. Long acting dihydropyridines (e.g. amlodipine) and non-
dihydropyridines (verapamil and diltiazem) are more preferred.
 Amlodipine is the CCB of best choice for symptomatic treatment of angina
and/or hypertension in patients with chronic heart failure.

█ Newer options for treatment of chronic angina

 pFOX inhibitors, potassium channel openers, and ranolazine are examples of


new anti-anginal drugs. These drugs alter the balance between myocardial work
and O2 supply by novel mechanism(s) of action.
 Their efficacy in treatment of angina is controversial; however they are approved
for treatment of chronic stable angina in combination with β-blockers, CCBs,
and nitrates.

170
pFOX inhibitors (metabolic mod ifiers): Trrimetazidin
ne

– Theey are teermed pFOX inhi bitors


beccause they partially y inhibit fatty
acidd oxidatioon in the myocardium
m m.
– Thiss “metabo olic switch
h” from faats to
carbbohydrate utilizationn requiress less
O2 cconsumptiion.
– By inhibition of fatty acid oxid ation,
theyy ↓ intra acellular lactic aciidosis
leadding to ↓ intracellular Ca2+ & Na+
acccumulation and ion disturbanc
d ce, so they y prevent cell necro
osis and preserve
p
conntractile fun
nction.
– It do
oes not afffect HR, blood presssure or corronary bloo
od flow.

Potass
sium cha
annel ope
eners: Nic
corandil

– Nico
orandil is a new antia
anginal dru
ug with 2 proposed
p mechanism
m ms of actio
on:
 It opens ATP-depen
A ndent K+ c channels in the vasccular wall leading too VD of
peripheral and coron nary arteriees.
 Nitrate-like e activity: it has a n nitrate com nd ↑ cGMP
mponent an P like nitra
ates but
tolerance tot its effeccts is less m
marked.
– Likee nitrates, it should not
n be used d with sild
denafil.

Ranola
azine

– It ↓ intracellular Ca2+ in g the late Na+ curreent that facilitates


ndirectly byy reducing
Ca22+ entry into myocard dial cells. T Na+ and Ca
The reducttion in intrracellular N a2+ load
reduces cardiiac contrac ctility and w
work.
– It do
oes not afffect HR, blood presssure or corronary bloo od flow.

Antiplatelets and
a wering drrugs: see pharmaco
cholesterol low ology of blo
ood.

Choice
e of antian
nginal drug
gs in patie
ents with another disease:

Angina
a with…. Most prreferred Least pre
eferred
Bronch
hial asthma
a Nitrates,, CCBs Beta-blocckers
Heart fa
ailure Amlodip
pine Beta-blocckers, Vera
apamil
Hyperte
ension Beta-blo
ockers, CC
CBs Nitrates
Diabete
es mellituss Nitrates,, Nifedipine
e Beta-blocckers, Vera
apamil

171
█ MAN
NAGEMENT
T OF ACUT
TE MYOCAR
RDIAL INFA
ARCTION (AMI)
(

Manife estations: persistentt central


crushinng chest pain
p + ST segment
s
elevatio
on or depression +
patholoogical Q wave + raised
biochemical mark kers of my
yocardial
cell dea
ath (tropon
nin enzymee).
All cases must beb hospita alized in
a speciialized coronary care
e unit.

■ Non
n-pharmacologic th
herapy:
Patientts presentiing within 12 hours of sympto om onset, the treatmment of ch hoice is
percutaaneous co oronary in ntervention
n (PCI, or coronarry angiopllasty). A balloon
cathete
er, guided by x-ray immaging, is introduced
d into the occluded
o aartery to open it.

■ Pha
armacolog
gic therap
py:
 Morphine sullfate (5 mg
g i.v.):
– T
To producce analges sia and ↓ stress
oof the patient
p → ↓ sympa athetic
ddischarge and heart work.
– Morphine causes ve on → ↓
enodilatatio
vvenous retturn and ca
ardiac worrk.

 Oxyygen: receent evidenc


ce suggessts that
routtine O2 addministration has do
oubtful
sign
nificance an
nd did not reduce mo
ortality.

 Nitrroglycerin
ne and be ers: to
eta-blocke
limitt the infarc
ct size.

 Antticoagulan
nt drugs: heparin 1
10,000
IU ii.v. then 5000
5 IU/8h
h s.c. esppecially
whe en the patient is obe ese or if th
here is
histtory of prevvious MI.

 Thrrombolytic
c (fibrino
olytic) the
erapy:
stre
eptokinase, urokinas se, or t-P
PA as
earlly as possiible (see blood).

 Sed
datives: diazepam 5 mg i.v.

 Treatment off Complica


ations:
– C nic shock → dobutam
Cardiogen mine i.v.i - Arrhythmia → lido
ocaine i.v.

172
Part 4
4: The
erapy of conges
stive hea
art failurre (CHF))

█ Bas
sic inform
mation

Definittion: Hearrt failure (HF)


( is a
progresssive clinical synd drome in
which e either stru
uctural or functional
f
abnorm
malities im mpair the ability of
the heeart to meet the metabolic
m
demands of the body.
b .

Classiffication, clinical
c pic
cture,
and theerapeutic aim:

■ Ana
atomical classificat
c ion

Leftt-sided he
eart failure e (LSHF):
– Usually du ue to syste emic hyperttension.
– T The cardin
nal manifes stations arre those off pulmona
ary congesstion e.g. tachyp-
t
nea, dysppnea (diffic culty in brreathing), orthopnea
a (dyspneaa on lyingg back),
paroxysma al nocturnnal dyspne ea, cough with expectorationns, bilatera
al basal
lung crepittation, etc..

Right-sided heart
h failu
ure (RSHF)):
– Usually duue to pulmo onary hype o lung disease.
ertension or
– TThe cardinal manifestationss are tho ose of sy ystemic congestio on e.g.
congestedd neck veins, conge ested liver, bilateral leg edemaa, right ven
ntricular
hypertroph
hy, etc.

Tottal or cong
gestive he e (CHF): co
eart failure ombination
n between the two.

■ Patthological classifica
ation

Sys stolic dysffunction (decreased


( d contracctility) is caused
c byy reduced muscle
pow wer e.g, MII, cardiomyyopathies, and ventrricular hypertrophy (sstrain). Ven
ntricular
hyppertrophy can
c be caaused by p pressure overload
o (e
e.g, system
mic or pulmonary
hyppertension and aortic or pulm monic valvee stenosis s) or volum
me overloa ad (e.g,
valvvular regurrgitation, high-outpu
h ut states e.g.
e thyrotoxicosis aand arterio ovenous
fistu
ula).

Diastolic dyssfunction (restrictio


( on in ventrricular filling) is cauused by inc creased
ventricular stiffness e.g. ventricul ar hypertrrophy, infilttrative myo
ocardial diseases

173
(e.g
g. amyloido osis), myoocardial isschemia an
nd MI, an
nd pericard
dial diseas
se (e.g,
periicarditis an
nd fibrosis)).

Thee decrease ed COP le eads to the e


follo
owing com mpensato ory
mec chanisms:
 Sympa athetic overactivitty
leadingg to tachyc cardia.
 Renal ischemia → ++ o of
RAAS → VC (A Ang-II) an
nd
fluid re
etention (a aldosterone e)
→ ↑ afterload
a and
a preloa
ad
→ morem HF and
a edemma
(viciouss circle).

■ Clin
nical class
sification

HF is classifie
ed by the New Yorrk Heart Association
A n (NYHA) iinto the fo
ollowing
classses:

NYHA Class S
Symptom
I (mild) No symptoms while performing ordinary
N y physical activity (w
walking,
climbing sta
airs, etc.).
II (mild)) Mild symptoms (mil d shortne
M ess of bre eath, palppitations, fatigue,
and/or angina) and sliight limitation during ordinary p
physical ac
ctivity.
III (mod
derate) Marked limitation in a
M activity due
e to sympttoms, evenn during le
ess than
ordinary acttivity. Com
mfortable only at rest.
IV (seve
ere) Severe limittations with
S h symptom
ms even while at restt. Mostly
bedbound patients.
p

174
█ Therrapy of heart
h failu
ure

■ Non
n-drug the
erapy = life
e style mo
odification
n:
– Rest.
– Dietary sodium (salt)) and fat re
estriction.
– AAvoid stre
ess, smokinng and alc ohol.
– WWeight redduction.
– Control of risk factors e.g. surgical correction
c of valvullar diseas
ses and
ttreatment of hyperthhyroidism, hypertension, etc.
– A gs that ↑ BP
Avoid drug P: e.g. symmpathomimetics, sodiu
um containning drugs, etc.

■ Dru
ug therapy
y:
– Positive in
notropic drugs:
 Carrdiac glyco osides (Diggitalis).
 Dop pamine & dobutamin
d ne (used for short terrm only).
 Pho osphodiestterase inhi bitors: inammrinone & milrinone..
– Diuretics.
– A
ACEIs.
– V
Vasodilatoors: nitrates
s and hydrralazine
– Other drug gs: e.g. betta-blockerrs and spironolactonee.

▌Carddiac glycoosides:
Digoxin, Digitox
xin, Ouaba
ain

Source
e and chemistry
– Nattural plant derivatives
s (Foxglovve plant).
– Carrdiac glycoosides conntain a lacttone ring and
a
a stteroid (aglycone) mo oiety attachhed to sug
gar
molecules.
– Dig
goxin is thee most widely used.

Pharm
macokinetics of digo
oxin
– Dig
goxin distriibutes to most
m bodyy tissues and
a accum mulates in n cardiac tissue.
Thee concentrration of th he drug in the heart is
i twice that in skeleetal muscle
e and at
leasst 15 timess that in plasma.
– Elim
mination iss renal. Do ose adjusttment should be done accord ding to creeatinine
clearance.

anism of action
Mecha a

■ Pos
sitive inotrropic effec
ct:

175
Digitalis ↑ ca ardiac con asing free intracelluular Ca2+ through
ntractility by increa t
+ +
inhiibition of membrane
m e-bound N Na /K ATPase enzy yme. This result in in
nhibition
+ +
of N
Na /K pum mp with sub
bsequent a acellular N a and Ca2+
accumulattion of intra + 2
via:

– Increase C
Ca2+ releas
se from the
e
ssarcoplasm
mic reticulu
um.
– Displacemment of intracellula ar
2+
CCa from its bindingg sites.
– Increased intracellular Naa+
prevents Ca2+ expu ulsion from
m
tthe cell by the Na /Ca22+ +

eexchangerr.

■ Auttonomic effects:
– ↑ vagal acttivity: By direct
d and iindirect meechanismss.
– ↓ sympatheetic activityy due to relieve of hypo
oxia & imprroved tissuue oxygenattion.

Pharm
macologica
al effects

 ↑ Contracttility and COP:


C due tto +ve inottropic effec
ct. This leaads to:
- Improvvement of RBF
R → diurresis and ↓ RAAS (i.e
e. ↓ fluid reetention).
- Relief of
o lung con
ngestion.

 ↓ HR: (B r a d y c a r d i a ) d
due to:
- ↑ vagall tone and ↓ sympathetic activityy on the hea
art.
- Direct inhibition
i of
o the AV c
conducting
g system.

 C
Conductioon velocityy:
ction: ↑ due
- Intra atrial conduc e to vagal stimulation.
s .
- A-V conduction: ↓↓ by direcct and vagal effects.

 ↑ Excitabillity and automatici


a ity:
le
eading to o ectopic
c foci a and
a
arrhythmia of any typ
pe (bigemin
ny,
trrigeminy, etc.).
e

 E
ECG changges:
- Prolongged PR intterval.
- Short QT
Q interval.
- ST seggment deprression & T
T-
wave in
nversion.

 N
Normalizattion of arterial
a a
and
v
venous pressures
s due to
im
mproved heemodynammics.

176
Therapeutic indications

 The use of digoxin is now limited to cases of chronic CHF associated with
atrial flutter or fibrillation (AF) because digoxin is the only drug that ↑
contractility and ↓ AV conduction in the same time.
 In cases of AF alone, other drugs like verapamil or beta-blockers are preferred.
 In cases of CHF without AF, other drugs like diuretics and ACEIs are preferred.

Contraindications

■ Absolute contraindications:

– Heart block: because digitalis worsens AV conduction by direct and vagal


effects.
– Hypertrophic obstructive cardiomyopathy (HOCM): because increasing
cardiac contractility will ↑ the outflow tract resistance and accelerates heart
failure (see before).
– Wolff-Parkinson-White (WPW) syndrome: although digitalis (and verapamil)
↓ conduction in the normal pathway, they can ↑ conduction in the abnormal
pathway leading to ↑ arrhythmia (see before).
– Paroxysmal ventricular tachycardia: digitalis ↑ excitability and automaticity.

■ Relative contraindications:

– All causes of bradycardia (e.g. sick sinus syndrome, hypersensitive carotid


sinus, with beta-blockers or verapamil) because addition of digoxin can lead
to severe bradyvardia or even heart block.
– Systemic hypertension: digitalis will ↑ the strain of the LV (you must correct
hypertension first by giving VDs or diuretics).
– Pulmonary hypertension due to chronic lung disease: digitalis will ↑ the
strain of the RV (you must correct pulmonary hypertension first by giving VDs).
– Cardiac diseases:
 Acute MI: digitalis ↑ the infarct size and aggravates arrhythmia.
 Cardiomyopathy…. digitalis is useless.
– Renal or hepatic diseases: digoxin must be avoided in renal patients while
digiToxin must be avoided in hepaTic patients.
– In patients likely to require cardioversion: because digitalis may lead to fatal
arrhythmia if given with cardioversion.

177
Drug in
nteraction
ns

Drug Me
echanism/effect
Antacids - Kaolin Bin
nd digoxin in the gut and decre
ease bioavaailability
Choles
styramine (ab
bsorption)
Metoclopramide
e Inc
crease gut motility lea
ads to dec
crease digo
oxin absorption.
Atropin
ne Decrease gutt motility le
eads to inc
crease digo
oxin absorrption
Quinidine Decrease ren
nal clearance of digoxin and dissplace digoxin
from plasma proteins. Serum
S digo
oxin is incrreased.
Loop ddiuretics and
a Thiiazides or loop diurettics may cause hypo okalemia and a
thiazid
des hyp
pomagnessemia whic ch can ↑ thee risk of diigitalis toxicity

Dosage and adm


ministratio
on

■ Initiial digitalization:
– It is done by giving one
o tablett 0.25 mg /day
((5 days/weeek) from the
t start.
– TThe Cpss will be achieved
a a
after 5 t½ (i.e.
aafter one week
w for digoxin).
– Rapid (lo
oading) method is done in
e
emergencyy conditio ons to a achieve raapid
C
Cpss. It is given as 2 tablets (0
0.5 mg) tw
wice
d
daily for 2 days (2x2
2x2).

■ Maiintenance
e dose: on
ne tablet (0
0.25 mg)/d
day,
5 da
ays/week.

Precau
utions durring digitalis therapyy
– Nevver give diigitalis i.v. before beeing sure that
t
the patient ha as not receeived digitaalis during the
lastt 14 days to avoid diggitalis toxic
city.
– Con ntinuous monitoring
m of plasma K+ level.
– Men ntion all the relative contraind dications.

▌Digittalis toxic
city

Predisposing fac
ctors
 Hyppokalemiaa: increases digoxin b
binding an
nd effect.
 Hyp mia (N.B. Hypocalcem
percalcem H mia renderrs digitalis less effecttive).
 Presence of reenal impairment.

178
Manife
estations
■ Carrdiac:
– Bradycard
dia and va
ariable de
egree
o
of heart block.
– Paroxysmaal atrial tac
chycardia
– A
Any type of arrhythm mia (premaature
beats, bige
emeny, trigemeny, etcc.)

■ Extrracardiac::
– T
The first manifestattion is fattigue
aand anorrexia, follo owed by GIT
ssymptomss e.g. diarrh hea, nause
ea & vomiting due to stimulatio
on of CTZ.
– CCNS: head dache, delirium, halluucination, and
a convulsions.
– VVision: yelllow visionn (chromat
atopsia), diplopia, am
mblyopia, sscotoma, etc.
e due
tto retrobullbar neuritiis.
– OOthers: sk
kin rash, gyynecomasttia, galactoorrhea.

Manag
gement
 Stop digitalis administra ation.
 Corrrect hypok kalemia by y giving K+ either i.v. or oral (2 gm/4
g h).
 Antiarrhythmic c drugs:
– Lidocaine (in ventric cular arrhyythmia): 1-2 2 mg/kg i.v. bolus tthen 1-2 mg m /min
i.v.i.
– Phenytoin (in ventricular arrhytthmia with heart bloc ck): 250 mgg i.v. over 10 min.
– A Atropine: if there is bradycardia
b a or heart block.
 Speecific digittalis antibo
odies (Fab b fragments; Digib bind) to bbind digitaalis and
promote its reenal clearance (the m most specific therapy y).

█ Othe
er positiv
ve inotrop
pic drugs
Phospphodiesteerase (PD DE) inhibittors:
Inamrinone – Miilrinone

 Theey inhibit PDE


P enzym me (type 3) → ↑ cAMP → ↑ Ca2+ in nflux in myyocardial cells
c →↑
myo ocardial coontractility..
 Theey have VD D properties.
 Theey are use ed only i.v. for sh hort term treatment of CHF F in patients not
respponding to o digitalis. They
T are n
not safe forr long term
m use.
 Advverse effe ects includ de: thromb bocytopen nia, cardiacc arrhythmmia, increa
ase liver
enzzymes (hep patotoxicityy).
 Milrrinone is le
ess toxic thhan inamrinnone

179
█ Diuretics

Mechanism in heart failure


 They ↓ fluid retention and pulmonary congestion leading to improvement of
tissue oxygenation
 They ↓ preload & afterload so improve myocardial function.
 Spironolactone antagonizes the effect of aldosterone that is increased in CHF
due to secondary stimulation of RAS. Recent evidence showed that it reduces
mortality rates in patients with advanced heart failure (NYHA class III and IV).

Disadvantages of diuretics
 Excessive use of diuretics will ↓ ECF volume → ↓ COP.
 Diuretic-induced acid-base imbalance may impair cardiac function.
 Diuretic-induced hypokalemia can ↑ digitalis toxicity and cardiac arrhythmia.

These adverse effects could be minimized by diuretic combination (loop diuretics


plus K+ sparing diuretics) to minimize hypokalemia and acid-base imbalance.

█ Vasodilators (nitrates and hydralazine)

 Nitrates and hydralazine have complementary hemodynamic actions:


 Nitrates are primarily venodilators, →↓ preload.
 Hydralazine is a direct arterial dilator →↓ systemic vascular resistance and
afterload.
 Recent evidence showed that combination of nitrates and hydralazine reduces
mortality and hospitalizations for patients with HF.
 A fixed-dose combination product is available (USA and Europe) that contains
isosorbide dintrate 20 mg and hydralazine 37.5 mg.
 Guidelines recommend addition of hydralazine and nitrates to moderate to
severe HF despite therapy with ACE inhibitors, diuretics, and β-blockers.
 The combination is also appropriate as first-line therapy in patients unable to
tolerate ACE inhibitors or ARBs due to any contraindication.

█ ACEIs and ARBs

Beneficial effects in heart failure:


 They ↓ arterial BP → ↓ afterload.
 They ↓ aldosterone → ↓ Na & H2O retention → ↓ preload.
 They prevent myocardial wall thickening and cardiac remodeling.

180
█ Beta
a-blockerrs

High ddoses of β-blockerrs are ge enerally not


recommmended in heart failuref be
ecause the ey
producce –ve inotropic effec ct and mayy precipita
ate
cardiac
c decompe ensation, but
b small doses hav ve
some bbenefits in heart failure:

Beneficial effectts in heartt failure:


 β-b
blockers reduce tachyca
ardia an
nd
mpathetic overactivity
sym o y.
 β-b
blockers redduce BP → ↓ ventricu ular strain associated
a d with HF.
 β-b
blockers inhhibit renin release → ↓ cardiac remodeling
r g caused bby RAAS.
 Carrvedilol is a new beta-blocker with additional VD and antioxid dant prope
erties.

Accord
ding to currrently ava
ailable eviddence, bis ol, and carrvedilol
soprolol, metoprolo
m
have sh
hown the most
m usefu
ul effects in
n patients with chron
nic HF.

█ MAN
NAGEMENT
T OF ACUT
TE CARDIO
OGENIC PU
ULMONARY
Y EDEMA

Pathop
physiology
y of APE
Acute c
cardiogenic pulmonary edema a (APE) is accumula
ation of fluuid (transudate) in
the lun
ng interstiitium and alveoli aas a result of incre
eased cap pillary hydrostatic
pressure seconda ary to LV dysfunction
d n.

Manife
estations
– Dysspnea, orth
hopnea and d wheezess.
– Cheest x-ray: patchy
p or diffuse
d alve
eolar filling (haziness).
(

181
Manag
gement
 Hosspitalization and sitting or sem
mi-sitting po
osition.
 High-flow oxyygen (hypo
oxia causess pulmona
ary VC and
incrreased card
diac load).
 Furrosemide (20-80
( mg IV): to ↓ ve
enous returrn and
pulm
monary con
ngestion. It is the mosst importantt
trea
atment.
 Morphine (2-4 mg IV):
– T
To ↓ stresss and anxieety.
– V
Venodilatattion → ↓ VR
R → ↓ lung co
ongestion.
– It ↓ pulmon ch reflex → ↓ tachypnea & work
nary stretc
o
of breathin
ng.
 Nitrroglycerin
ne (sublingual or i.v.).
 Hem
modynamic c support according to systolic
c BP:
– Maintain systolic
s BP
P >100 mmmHg.
– If the SBP is <100 mmHg,
m starrt IV dopam
mine or
dobutamine at 5-15
d 5 μg/min.

Part 5
5: Man
nagemen
nt of sh
hock

Shock is a circu ulatory dissorder. It i s complexx state of hypotens ion and im


mpaired
tissuess perfusion of vital organs (acutte circulato
ory failure)..

Diagno
osis
 S
Systolic BP < 90 mm mHg OR m ean arteria al BP (MAP P) < 60 mm
mHg.
 T
Tachycard dia and tac
chypnea
 O
Oliguria (urine < 20 ml/h),
m or an
nuria.
 C
Cool pale extremitiess with sloww capillary refilling.
 C
Confusion and restle essness.

█ Anap
phylactic
c shock

It is a m
medical em
mergency that require
es immedia
ate recogn
nition and iintervention.

Causes
s, manifes
stations, and
a time c
course
Acute hypersenssitivity reac
ction in re
esponse to
o allergic substancee (food, drugs or
venom)) → massivve release of
o histaminne from ma
ast cells:

182
– Skin changes (the first feature): erythema, urticaria, or angioedema.
– Hypotension (fainting).
– Bronchoconstriction (wheeze or hoarse voice).
 Fatal food reactions typically occur after 30–35 minutes.
 Insect stings cause shock after 10–15 minutes.
 Intravenous drug reactions occur within 5 minutes.

Treatment
 Apply tourniquet proximal to the site of injection if possible.
 Epinephrine 0.5 ml i.m. (s.c. injection is slow).
– It is the drug of choice and potentially lifesaving.
– i.m. injection in the thigh (vastus lateralis) gives more rapid effect.
– Repeat after 5 min if no response.
 Antihistamine (e.g. chlorpheniramine 10 mg i.v.).
 Hydrocortisone 200 mg i.v to ↓ antigen/antibody reaction.
 Intravenous fluids (0.5-1 L) and monitor BP.

█ Neurogenic Shock

 Neurogenic shock is caused by loss of sympathetic tone of blood vessels


resulting in the massive dilatation of arterioles and venules.
 It can be caused by spinal anesthesia, spinal cord injury, pain, and anxiety.

Treatment
 Recumbent position with head down.
 Vasopressor sympathomimetics and sedatives.

█ Hypovolemic shock

Causes
Blood loss (hemorrhage); Plasma loss (severe burn); Water loss (vomiting, diarrhea).

Grades of hypovolemic shock

Blood loss Response


Class I Up to 15% (750 ml) Compensatory mechanisms maintain BP.
Class II 15-30% (750-1500 ml) Hypoxia, hypotension, ↓ urine output
Class III 30-40% (1500-2000 ml) Profound shock with severe acidosis
Class IV > 40% (> 2000 ml) Coma …….. death

183
Treatment
 I.v. fluids (Ringer solution is more superior to normal saline for use in massive
hemorrhage).
 Blood transfusion
 Dopamine: to ↑ COP.

█ Cardiogenic shock

Causes
 Acute MI
 Blunt cardiac trauma
 Myocardial depression due to any cause (drugs, infection, etc.).

Treatment
 All measures of treatment of acute MI (see before).
 Dobutamine i.v.i. (usually at 5 - 20 μg/kg/min).
 Lidocaine i.v. for control of ventricular arrhythmia.

█ Septic shock

Cause: severe infection → release of inflammatory cytokines from the inflammatory


cells → venodilatation (↓ venous return) and progressive tissue hypoxia.

Treatment
 Antibiotics according to the type of pathogen.
 Dopamine or dobutamine to ↑ COP.
 Low dose steroids to minimize toxemia.
 Oxygen if O2 saturation is <94%.

184
Part 6
6: Card
diac arrrhythmia
a and an
ntiarrhytthmic d
drugs

█ Bas
sic inform
mation

▌Cardiiac action potential

 In the restin
ng state, K+ ions is
founnd mainlyy intracellular, whi le
+
Na and Ca2+ are main
nly
extrracellular making
m the
e interior o
of
the cell electrically negaative.
 Conntraction and
a relaxa
ation occu ur
wheen rapid redistribut
r ion of ionns
acrooss the ce ell membrrane occurrs
during 4 phasses known n as “actioon
pottential”.

Phases
s of action
n potentia
al:

 Pha
ase 0: rap
pid depolarization o
of
the cell due too rapid influx of Na+.
 Phaase 1: sho ort period of rapid re e-
+
polaarization due to outflow of K .
 Phaase 2: “p plateau”: delay
d in ree-
polaarization due
d to slo
ow influx o of
+
++
Ca .
 Phaase 3: seco ond period d of rapid rrepolarization due to rapid out--flow of K+.
 Phaase 4: the resting sta ate is restoored. Na+ ions are extruded
e o ut the cell and K+
ionss returns back
b by thee Na+/K+ pu ump and so
s on.
 Thee slope of phase 4 determines
d s when thee 2nd action
n potential starts. When the
slop
pe is incrreased, th he distanc ce betwee en 2 cardiac cyclees shorte ens (i.e.
tachhycardia) and
a vice ve ersa.

▌Impulse formattion (automaticity)

 Carrdiac auto omaticity refers to the abilitty of certtain cells to self-generate


elec
ctrical impu
ulses that spread thrroughout the heart.
 Undder normal condition ns, the SA
A node is the
t domina ant pacemmaker (i.e. has the
high
hest autommaticity).
 Norrmal myoca ardial cells
s don’t havve automatticity i.e. ca
annot geneerate impuulses.
 Undder certain patholo ogic cond ditions, soome myoc cardial ceells may acquire
spoontaneous repetitive firing, thiss is called
d abnorma al automa aticity or ectopy.
e
Theese ectopicc pacemak kers comp pete with th
he SA node for contrrol of the heart.
h

185
▌Impulse condu
uction

 Elecctrical actiivity sprea


ads from th he SA
nodde to the ventricles
v via
v the AV V node
andd the bund dle of His, and then down
thro
ough the rig ght and lefft bundless.
 In th
he ECG, the P wave e represen nts the
spreead of dep polarization wave th hrough
the atria (atrial contrac ction). Thee QRS
commplex rep presents the sprea ad of
deppolarizationn wave through the
ventricles (ventricular contraction
c n). The
ST segmentt and T wave w rep resent
ventricular reppolarizationn (relaxatio
on).

█ Card
diac arrh
hythmia

hmia means disturba


Arrhyth ance in the
e normal heart rhythm
m. It resultts from:
 AAbnormal impulse ge
eneration;
 AAbnormal impulse co
onduction;;
 Both.

▌Abno
ormal impu
ulse forma
ation:

■ Nod mality: e.g


dal abnorm g. sinus tac
chycardia and
a sinus bradycard ia.
■ Extranodal abnormal
a ity: e.g. premature e atrial or
o ventricuular contrractions
(ecttopic beatss).

▌Abno
ormal impu
ulse condu
uction

■ Re--entry:
– Thiss is a circcus movement of aan
imppulse thatt circulattes aroun nd
certtain area in a un nidirection
nal
fashhion and excites
e the conductin ng
systtem more than once.
– It iss the mosst commo on cause o of
atriial flutter and
a fibrilla
ation (AF).
– Wolff–Parkinsson–White
syndrome (WP PW) is ana examp ple of an y defined re-entry. WPW
natomically
syndrome is an atrio oventricula
ar re-entra ycardia, ssecondary to an
ant tachy
acccessory AV V conductin
ng pathwa ay (see befo
ore).

186
■ Hea
art block:
– AV conductio on is delayed (firsst degree)), intermitttent (seccond degrree), or
com
mpletely blocked
b (th
hird degree
e).

█ ANT
TIARRHYT
THMIC DRUGS

 Antiarrhythmic
c drugs produce efffects by altering
a one or moree of the fo
ollowing
facttors: 1) Automatici
A ty; 2) Coonduction velocity; 3) Refraactory perriod; 4)
Mem mbrane ressponsiveness.
 Alm
most all anttiarrhythmic drugs hhave more e than one mechanissm of actio
on. The
simplified Vau
ughan Williams clas n system assumes
ssification a thhat each drug has
onee main mechanism of o action:

Class II: Na+ chan


nnel block
kers:

■ Class IA: e.g. qu uinidine,


pro
ocainamide e, disopyrramide:
modderately bllock Na+ channels
c
d ↑ ERP (e
and effective re
efractory
periiod) and d APD (action
poteential dura
ation)

■ Class IB: e.g. lidocaine,


mexxiletine: weakly
w bloc annels and ↓ ERP and
ck Na+ cha d APD.

■ Class IC: e.g. flecainid ck Na+ and


enone: strrongly bloc
de, propafe d K+ channels with
no e
effect on ERP
E or APDD.

Class III: Beta-blockers: e..g. propra


anolol, biso
oprolol, metoprolol
m

They ↓ AV conductio
T on and inhibit phase 4 depolariz
zation.

Class IIII: K+ chan


nnel block
kers: e.g. a
amiodarone, dronedarone, ib
butilide, so
otalol

T bit mainly K+ channelss and ↑ ER


They inhib RP.

Class IIV: Ca+ channel bloc


ckers: e.g
g. verapam
mil and dilttiazem

T
They inhib
bit mainly Ca nels and ↑ ERP.
C 2+ chann E

Other u
unclassifie
ed drugs: digoxin, a
adenosine
e, Mg sulp
phate

187
Quinid
dine (subc
class 1A))

Mecha
anism and pharmac
cological e
effects
 It bblocks acttivated Na+ channe els leading g to decrrease the rate of phase-0
p
dep polarization
n, decrease e excitabil ity, and ↑ APD
A and ERP.
E
 It blocks mus scarinic and α rece eptors lead ding to atroopine-like action (va
agolytic)
andd hypotenssion.
 Quinidine hass complex x effect on n AV cond duction due
d to direect and vaagolytic
actions:
– A At low dosses: its vag
golytic acti on predom minates → ↑ AV conduuction.
– A At therapeeutic dosess: its directt action pre
edominate es → ↓ AV cconductionn
 It ha
as –ve inottropic effecct and antiimalarial efffect (again
nst P. falcip
iparum).

peutic use
Therap es
 Quinidine wass used for many ye ears to tre
eat suprav
ventricularr and ventricular
arrh
hythmias, anda to maintain sinu us rhythm after conversion from
m atrial fluttter and
fibriillation; ho
owever, it is rarelyy used todday becauuse of avaailability of
o more
ective and less toxic drugs.
effe

Advers
se effects and preca
autions
– Cin
nchonism: tinnitus (i.e. hearing
g of ringin
ng or hiss), headachhe, blurred
d vision,
vom
miting, and
d diarrhea.
– Hyp
potension: after rapid i.v. infussion due to
o α-recepto
ors blockad
de.
– Parradoxical tachycard
dia: quinid
dine has atropine-lik
a ay ↑ AV
ke action aand, it ma
connduction an
nd cause "paradoxic
" cal tachycaardia". Digitalis or verrapamil should be
give
en before quinidine
q to offer rate b ↓ AV con
e control by nduction.
– Quiinidine syn uinidine ↑ Q
ncope: qu QT intervall and may predisposse the patie
ent to a
seriious type of
o arrhythmmia (torsad
de de pointtes). Quinid
dine therefo
fore should
d not be
give
en to patie
ents with lo
ong QT synndrome or with otherr drugs thaat ↑ QT inte
erval.

Procainamide (subclass
( s 1A)

 Thiss drug is equiva alent to quinidine e as an n


antiiarrhythmic
c agent and has simiilar cardiac c and toxic
c
effe
ects. Like quinidine,
q its use now
w is very lim
mited.
 Add ditional adverse effect: procaina amide is s
mettabolized by
b hepatic c acetylatio
on; 30% of o patientss
(slow acetyla ators) dev velop drug g-induced systemic c
lupuus erythem
matosus (SLE) after loong term th herapy.

188
Lidocaine (subclass 1B)

 Lidocaine (lignocaine) is exclusively Na+ channel blocker; it is highly selective


for damaged tissues.
 It undergoes extensive first-pass metabolism so, it is not given orally.
 It is given only i.v. for acute suppression of ventricular arrhythmia associated
with acute MI (not for long-term treatment). The usual dose is 50-100 mg i.v. half
of this dose may be repeated after 5-10 min if necessary.
 It has no effect on AV conduction, so it is not used for supraventricular
arrhythmia.
 Most adverse effects are neurologic.

N.B.
 Mexiletine is very similar to lidocaine but can be given orally. It is used primarily
for long-term treatment of ventricular arrhythmias associated with previous MI.

 Phenytoin is antiepileptic drug with class 1B activity. It is used primarily in the


treatment of digitalis-induced tachyarrhythmia. It has a limited role in the
treatment of other ventricular arrhythmias. The IV loading dose is 250 mg given
over 10 minutes.

Flecainide (subclass 1C)

 It blocks both Na+ and K+ channels leading to decrease the rate of phase-0
depolarization and slows AV conduction. Due to its complex effects on cardiac
tissue, the APD is not altered.
 It is used for atrial and ventricular arrhythmia and for maintenance sinus rhythm
after conversion from atrial flutter and fibrillation.
 Flecainide increases the incidence of ventricular fibrillation and sudden death
after MI (proarrhythmic effect), so it is contraindicated for patients with ischemic
heart disease or structural heart disease (e.g. LV hypertrophy).

█ Class II: Beta blockers

Mechanism of action
They ↓ sympathetic stimulation, inhibit phase 4 depolarization, depress automaticity,
prolong AV conduction, ↑ heart rate and ↓ contractility.

Therapeutic uses
 All arrhythmia induced by sympathetic overactivity.
 Arrhythmia due to thyrotoxicosis.

189
 Arrh
hythmia asssociated with
w HOCM M.
 Suppraventricu
ular arrhyth
hmia (AF).
 Arrh
hythmia duue to mitral valve pro
olapse.

█ Clas
ss III: Amiodarone
e

 It iss structura
ally related to thyroxiine. It con
ntains ~ 40
0% iodine.. Droneda
arone is
che emically sim
milar to ammiodarone b but does not
n contain n iodine.
 Amiodarone has h long t½ t and la rge Vd so o, it can accumulate
a e in many tissues
lead ding to wid
de range off adverse eeffects.

Mecha
anism of action
a
 Bloccks mainlyy K+ chann
nels → slow
wing of pha
ase 3
→ ↑ ERP.
 Bloccks Na+ chhannels → ↓ excitabiliity.
 Βloc 2+
cks Ca channels → – ve inotropic and
chro
onotropic effects.
e

Therap es: (most ty


peutic use ypes of arrrhythmia)
 Suppraventricu
ular and ve
entricular a
arrhythmia.
 WPW syndrom me.
 hythmia resistant to other
Arrh o druggs.

Advers
se effects
– Dosse-related pulmonarry toxicity (fibrosis)
is th
he most im
mportant ad dverse effeect.
– Hep patic toxic
city.
– Thy yroid dysfuunction: hypo-
h or hyyperthy-
roid
dism becau use of its io
odine conttent.
– Corrneal micrrodeposits s: reversib le, does
not affect vision.
– Braadycardia and heart block.
– Pho otosensitivvity leading to gray-b blue skin
disccoloration in sun-expposed area as.

█ Clas
ss IV: CCB
Bs (verap
pamil and
d diltiazem
m)

Mecha
anism of action:
a ey ↓ SAN a
The activity and
d AV conduction

Therap
peutic use
es

190
 Non n-dihydrop
pyridines (v
verapamil and diltiazzem) are primarily
p ussed to reduce HR
in s chycardia (SVT) and arrhythmia
supraventrricular tac a associateed with HO
OCM.
 CCB Bs have no role in the chronic
c managem ment of ve
entricular ttachycarddia (VT).
IV vverapamil should neever be ussed in the acute ma anagementt of VT, as s it may
cauuse hemodynamic co ollapse.

█ Othe
er antiarrrhythmic agents: A
Adenosin
ne

 It iss a purinergic A1 rec ceptor ago onist; this leads to opening


o off K+ channnels and
2+
inhibition of CaC chann nels (i.e. h
hyperpolariization) in the AV co onducting system
and d directly in
nhibits AV V nodal con nduction.
 It ha as very shoort half-life
e of 8-10 se econds.
 It iss the drug of choice for immed diate terminnation of paroxysma
p al supravenntricular
tach hycardia (including WPWW synd
drome). It is given as s a bolus dose of 6 mg i.v.
folloowed, if neecessary, by b a dose o of 12 mg.
 The e drug is le
ess effectiv ve in the prresence off adenosine e receptorr blockers such
s as
theophylline or caffeine e.
 It is contraindicated in pa atients with
h asthma because
b it can
c cause bronchos spasm.

█ Non-pharmac
cological methods
s

DC carrdioversio
on
 It iss applicatio
on of directt current (e
electric sho
ock) to
the chest wall for emerrgency co ntrol of an ny type
of a
arrhythmia a especiallyy rapid AFF in an un nstable
patiient (i.e. hyypotensivee).
 Thee patient should be e hepariniized befo ore the
procedure.
 Folllowing elec ctrical card
dioversion , patients should
s
be aanticoagulated for att least 4 we eeks.

Laser a
ablation
 It iss used for many
m types of arrhyt hmias.
 A ccatheter is inserted into
i a speecific area of the
heaart. A spec
cial machin
ne directss energy th
hrough
the catheter to small areas
a of thhe heart muscle
m
thatt causes the abno ormal hea art rhythm
m. This
eneergy "disc connects" the pa athway of o the
abnnormal rhytthm.
 Lasser radiofrrequency ablation is the definite
d
trea
atment of WPW
W synd
drome.

191
Artificiial pacemakers and d implanta able cardio
overter
defibrillators
 Theey are batttery-powered electroonic devic ces that
are implanted d under the
e skin or in
n the ches
st cavity
to mmonitor and pace thee heart.

█ Man
nagementt of cardiac arrestt

Cardiac
c arrest innvolves ce essation off cardiac mechanic
m al activity as confirm
med by
absencce of signss of circulattion (absen
nt pulse an
nd apnea).

s
Causes
 Corronary heart disease (~80%).
 Carrdiac disea
ase: e.g. HO
OCM, Brug gada synd drome.
 Carrdiac arrhytthmia espe
ecially ven
ntricular.
 Oth
hers: traumma, electrolyte imbala
ance, electrical shock
k, drugs, eetc.

Pattern
ns of arres
st
 Com mplete asyystole: the ECG is flatt
line.
 Venntricular fib
brillation: ECG showss
fibriillation wavves.
 Pulsseless elec ctrical activ
vity (PEA):
Theere is some e electrical activity (o
other
thann VF) witho out detecta able pulse .

Manag
gement
 The
e ultimate goal of treatment is to
presserve life by early CPR 30:2 2 i.e.
cycles of 30 chest
c comp pressions ffollowed by b 2 rescuee breaths.
 Adm ectrical deffibrillation at 360J then repeat CPR for 2 min.
minister ele
→N No responsse → Epine ephrine 1 mg i.v. /3-5 min with h CPR.
→NNo responsse → DC shock
s with CPR for 2 min.
→NNo responsse → Amio odarone 3 00 mg i.v. with CPR..
→NNo responsse → DC shock
s with CPR.

ock – Drug
Sho g – Shock – Drug – S
Shock

192
193
Review Questions

Mention the pharmacodynamic principles underlying the use of:


 Verapamil in angina pectoris
 Verapamil in hypertrophic obstructive cardiomyopathy
 Nifedipine in arterial hypertension
 Lisinopril in arterial hypertension
 Captopril in diabetic nephropathy
 Nitroglycerine in angina pectoris
 Digoxin in congestive heart failure

Mention the pharmacodynamic principles underlying the contraindication of:


 Nifedipine in supraventricular tachycardia
 Verapamil in Wolff-Parkinson-White syndrome
 Captopril in hypertension associated with severe chronic renal failure
 Digoxin in heart failure associated with heart block
 Beta-blockers in Prinzmetal’s (variant) angina

Mention the precautions underlying the use of:


 Nitrates in angina pectoris
 ACEIs in hypertension
 Quinidine in atrial arrhythmia

Mention the essential lines of treatment of the following emergency conditions:


 Acute severe arterial hypertension  Acute myocardial infarction
 Acute pulmonary edema  Acute anaphylactic shock

Mention 3 side effects of each of the following drugs:


 Verapamil  Isosorbide dinitrate
 Captopril  Amiodarone

Mention the rational of the following combinations:


 Nitrates with beta-blockers in angina pectoris
 Hydralazine with diuretics in systemic hypertension
 Captopril with furosemide in arterial hypertension

Mention the main differences between:


 Verapamil and nifedipine
 Digitalis and quinidine
 Captopril and lisinopril

194
Of each of the following questions, E. Verapamil
select THE BEST SINGLE answer:
6. Which of the following mechanisms
1. The followings drugs given alone best explains the antihypertensive
are useful for treatment of hypertension actions of clonidine:
EXCEPT: A. Blockade of β1 adrenergic receptors
A. Amiodarone. B. Blockade of α1 adrenergic receptors
B. Propranolol. C. Blockade of central α2 receptors
C. Captopril D. Stimulation of presynaptic α2
D. Furosemide receptors
E. Verapamil E. Stimulation of presynaptic β2 receptors

2. Significant relaxation of arteriolar 7. When glyceryl trinitrate


and venular smooth muscles can be (nitroglycerine) tablets are prescribed
pro-duced by which of the following for angina pectoris, the patient should
drugs: be told:
A. Hydralazine A. To take it to prevent pain
B. Minoxidil B. To take it at the onset of pain
C. Diazoxide C. That if throbbing headache and
D. Sodium nitroprusside palpita-tion occur the patient should
E. Nifedipine take another
D. That if he feels faint he should stand
3. One of the following drugs is up perfectly
effective in treatment of ischemic E. To keep the tablet in a warm humid
episodes in patients with variant place e.g. a shelf over the bath
angina:
A. Propranolol 8. Which of the following
B. Nitroglycerine antihypertensive drugs may cause fetal
C. Sodium nitroprusside pulmonary hypoplasia and growth
D. Diltiazem retardation if used during pregnancy:
E. Hydralazine A. Hydralazine
B. Alpha-methyldopa
4. The following is true about calcium C. Atenolol
channel blockers: D. Clonidine
A. Verapamil has a positive inotropic E. Captopril
effect
B. Verapamil dilates coronary arteries 9. All the following statements are
more than nifedipine true for nitroglycerine EXCEPT:
C. Verapamil effectively relieves A-V A. Can cause adverse reactions of
block headache and tachycardia
D. Diltiazem may cause ankle edema B. Undergoes significant first-pass
E. Nifedipine is contraindicated in biotransformation
Raynaud's disease C. Can be used i.v. for the treatment of
acute heart failure
5. Negative inotropy and chronotropy D. Has duration of action of several hours
are most prominent with the use of the after sublingual administration
following drug: E. Can increase exercise tolerance if
A. Nifedipine taken immediately before exercise
B. Nicardipine
C. Nimodipine 10. A 57-year-old man is being treated
D. Diltiazem for an atrial arrhythmia. He complains

195
of headache, dizziness, and tinnitus. A. Amyl nitrite.
Which one of the following B. Nitroglycerin (sublingual).
antiarrhythmic drugs is the most likely C. Nitroglycerin (transdermal).
cause? D. Esmolol.
A. Quinidine. E. Hydralazine.
B. Amiodarone.
C. Procainamide. 14. Compensatory increases in heart
D. Propranolol. rate and renin release that occur in
E. Verapamil heart failure may be alleviated by which
of the following drugs?
11. A 58-year-old woman is being A. Milrinone.
treated for chronic suppression of a B. Digoxin.
ventricular arrhythmia. After 2 months C. Dobutamine.
of therapy, she complains about feeling D. Enalapril.
tired all the time. Examination reveals a E. Metoprolol.
resting heart rate of 10 beats per
minute lower than her previous rate. 15. A 58-year-old man is admitted to
Her skin is cool and clammy. the hospital with acute heart failure and
Laboratory test results indicate low pulmonary edema. Which one of the
thyroxin and elevated thyroid- following drugs would be most useful in
stimulating hormone levels. Which of treating the pulmonary edema?
the following antiarrhythmic drugs is A. Digoxin.
the likely cause of these signs and
B. Dobutamine.
symptoms?
C. Furosemide.
A. Amiodarone.
D. Minoxidil.
B. Procainamide.
E. Spironolactone.
C. Propranolol.
D. Quinidine.
16. A 45-year-old man has recently
E. Verapamil. been diagnosed with hypertension and
started on monotherapy designed to
12. A 56-year-old patient complains of reduce peripheral resistance and
chest pain following any sustained prevent NaCl and water retention. He
exercise. He is diagnosed with has developed a persistent cough.
atherosclerotic angina. He is Which of the following drugs would
prescribed sublingual nitroglycerin for have the same benefits but would not
treatment of acute chest pain. Which of cause cough?
the following adverse effects is likely to A. Losartan.
be experienced by this patient?
B. Nifedipine.
A. Hypertension. C. Prazosin.
B. Throbbing headache. D. Propranolol.
C. Bradycardia. E. Verapamil.
D. Sexual dysfunction.
E. Anemia. 17. Which one of the following drugs
may cause a precipitous fall in blood
13. A 68-year-old man has been pressure and fainting on initial
successfully treated for exercise- administration?
induced angina for several years. He A. Atenolol.
recently has been complaining about
B. Hydrochlorothiazide.
being awakened at night with chest
C. Nifedipine.
pain. Which of the following drugs
would be useful in preventing this
D. Prazosin.
patient's nocturnal angina? E. Verapamil.

196
18. Which one of the following stable (“effort-induced”) angina, and
antihypertensive drugs can precipitate the incidence and severity of anginal
a hypertensive crisis following abrupt attacks are reduced. Which of the
cessation of therapy? following best explains the
A. Clonidine. pharmacologic action by which the
B. Diltiazem. beta blocker does this?
C. Enalapril. A. Decreases myocardial oxygen
D. Losartan. demand
E. Hydrochlorothiazide. B. Dilates the coronary vasculature
C. Exerts antiplatelet/antithrombotic
19. Which of the following statements effects
is most correct regarding the use of D. Reduces total peripheral resistance
drugs in hypertension? E. Slows AV nodal conduction velocity
A. beta-blockers are contraindicated if
hypertension is associated with heart 23. Your patient is a 50-year-old man
failure with essential hypertension and no
B. calcium channel blockers are other complications. Which of the
contraindicated if the patient also has following drugs would be the most
obstructive pulmonary disease rational first choice for starting his
C. diuretics are contraindicated in antihypertensive therapy?
hypertension associated with diabetes A. Angiotensin-converting enzyme (ACE)
D. vasodilators are contraindicated in a inhibitor or angiotensin receptor
patient taking a diuretic blocker
E. ACE inhibitors are contraindicated if B. Beta-adrenergic blocker
the patient is pregnant C. Nifedipine
D. Thiazide diuretic
20. β-Blockers have been effective in E. Verapamil or diltiazem
the treatment of heart failure. They
primarily exert their effect by: 24. We have a 50-year-old man with
A. Binding to the receptor that binds asymptomatic hyperuricemia, and we
norepinephrine are about to start therapy for newly
B. Inducing a prominent diuretic effect diagnosed essential hypertension (BP
C. Increasing contractility 136/90 mm Hg, based on repeated
D. Improving asthma control measurements with the patient supine
E. Increasing heart rate to meet the and at rest). Which of the following
additional demands placed upon the antihypertensive drugs is most likely to
heart in CHF increase his serum uric acid levels
further, and possibly precipitate a gout
21. A patient has periodic episodes of attack?
paroxysmal supraventricular A. Captopril
tachycardia (PSVT). Which of the B. Hydrochlorothiazide
following drugs would be most suitable C. Labetalol
for outpatient prophylaxis of these D. Losartan
worrisome electrophysiologic events? E. Verapamil
A. Adenosine
B. Lidocaine 25. We’ve just diagnosed essential
C. Nifedipine hypertension in a 58-year-old female
D. Nitroglycerin patient. She tends to be tachycardic.
E. Verapamil Notes written by her ophthalmologist
indicate that she has chronic open-
22. We prescribe a beta-adrenergic angle glaucoma. Which of the following
blocker for a patient with chronic- drugs would be the most rational

197
choice for this woman, given only the Which of the following drugs is the
information presented in this question? most likely cause?
A. Captopril A. Captopril
B. Diltiazem B. Clonidine
C. Hydrochlorothiazide C. Labetalol
D. Timolol D. Methyldopa
E. Verapamil E. Prazosin

26. Our newly diagnosed hypertensive 29. A patient has Stage III essential
patient has a history of vasospastic hypertension. After evaluating the
angina. Which of the following drugs or responses to several other
drug classes would be the most antihypertensive drugs, alone and in
rational for starting antihypertensive combination, the physician places the
therapy because it exerts patient on oral hydralazine. Which of
antihypertensive effects, directly the following adjunct(s) is/are likely to
lowers myocardial oxygen demand and be needed to manage the expected and
consumption, and also tends to inhibit unwanted cardiovascular side effects
cellular processes that otherwise favor of the hydralazine?
coronary vasospasm? A. Captopril plus nifedipine
A. Angiotensin-converting enzyme (ACE) B. Digoxin plus spironolactone
inhibitor or angiotensin receptor C. Digoxin plus vitamin K
blocker D. Hydrochlorothiazide and a beta
B. Beta-Adrenergic blocker blocker
C. Nifedipine E. Triamterene plus amiloride
D. Thiazide diuretic
E. Verapamil (or diltiazem) 30. At high (but not necessarily toxic)
blood levels, a cardiovascular drug
27. Quinidine is ordered for a patient causes lightheadedness, tinnitus, and
with recurrent atrial fibrillation and who visual disturbances such as diplopia.
refuses any interventions other than What is the most likely drug that
drugs in an attempt to terminate and caused these responses?
control the arrhythmia. Which of the A. Atropine
following is the most likely effect of B. Captopril
quinidine? C. Dobutamine
A. Is likely to increase blood pressure via D. Propranolol
a direct vasoconstrictor effect E. Quinidine
B. Is contraindicated if the patient also
requires anticoagulant therapy
31. We have a patient who is
C. Slows spontaneous SA nodal diagnosed with variant (vasospastic)
depolarization as its predominant
angina. Which of the following drugs
effect
would be most appropriate, and
D. Tends to slow electrical impulse generally regarded as most effective,
conduction velocity through the AV
for long-term therapy aimed at
node
reducing the incidence or severity of
E. Will increase cardiac contractility the coronary vasospasm?
(positive inotropic effect) independent
A. Aspirin
of its antiarrhythmic effects
B. Atorvastatin
C. Diltiazem
28. A patient who has been taking an
oral antihypertensive drug for about a
D. Nitroglycerin
year develops a positive Coombs’ test. E. Propranolol

198
32. The following treatments can be 37. An ideal drug for treatment of
applied for rapid relief of acute anginal ventricular arrhythmia due to acute
pain EXCEPT: myocardial infarction is:
A. Sublingual nitroglycerine A. Propranolol
B. Sublingual isosorbide dinitrate B. Lidocaine
C. Oral aspirin C. Quinidine
D. Inhaled amylnitrite D. Procainamide
E. Intravenous nitroglycerine E. Verapamil

33. Which of the following statements 38. Digoxin is not indicated to treat
is true concerning the action of heart failure as long systemic
enalapril: hypertension is present because:
A. It is prodrug for an angiotensin II A. The heart may develop more strain
blocker and failure
B. Stimulates angiotensin II receptors B. Absorption of digoxin will be reduced
C. Increases endogenous bradykinin C. Excretion of digoxin will be reduced
levels D. Liability for digoxin toxicity is high with
D. Does not lower blood pressure in hypertension
“normal renin” hypertension E. Systemic hypertension will be reduced
E. Best indicated to patients with hyper- suddenly
tension associated with liver disease
39. Wolff-Parkinson-White syndrome
34. Beta-blockers are considered the can be successfully treated by:
first choice in the control hypertension A. Quinidine
in the following cases EXCEPT: B. Verapamil
A. Hypertension with thyrotoxicosis C. Atenolol
B. Hypertension with ventricular D. Digoxin
premature beats E. Amiodarone
C. Hypertension with variant angina
D. Hypertension with high plasma renin 40. All the following are positive
E. Young patient with increased inotropic drugs EXCEPT:
sympathetic activity A. Digitalis
B. Adrenaline
35. Severe myocardial pain due to C. Aminophylline
acute inferior MI with excessive vagal D. Dopamine
stimulation can be best managed by: E. Amiodarone
A. Morphine
B. Meperidine 41. All the following are class I
C. Butorphanol antiarrhythmic drugs EXCEPT:
D. Diclofenac A. Lidocaine
E. Indomethacin B. Quinidine
C. Propranolol
36. The enhancement of myocardial D. Phenytoin
contractility after digitalis is due to: E. Procainamide
A. Increased cyclic AMP
B. Stimulation of calmodulin 42. Which of the following cardiac
C. Beta-adrenergic stimulation glycosides has the clearly higher
D. Increased production of adenosine therapeutic index:
E. Inhibition of the sodium pump A. Ouabain.
B. Digoxin.
C. Digitoxin.

199
D. Deslanoside D. Decreased extracellular K+
E. None of the above E. Decreased extracellular Na+

43. The ECG of a patient who is 48. The most specific line for treatment
receiving digitalis in the therapeutic of digitalis toxicity is:
range would be most likely to show: A. Stop digitalis administration
A. Prolongation of the QT interval. B. Administration of potassium
B. Prolongation of the PR interval. C. Administration of Fab fragments
C. Symmetrical peaking of the T-wave D. Administration of phenytoin
D. Widening of the QRS complex E. Administration of calcium
E. Deep Q wave
49. The usefulness of digitalis in
44. If both quinidine and digoxin are management of atrial fibrillation
administered concurrently, which of the depends on its ability to:
following effects does quinidine have A. Decrease atrial impulse formation
on digoxin? B. Decrease vagal control over the heart
A. The absorption of digoxin from the GIT C. Decrease conduction time through the
is decreased A-V node
B. The metabolism of digoxin is D. Increase the effective refractory period
prevented of the A-V node
C. The concentration of digoxin in the E. Increase conduction time in the atria
plasma is increased
D. The excretion of digoxin is increased 50. Quinidine is either contraindicated
E. The ability of digoxin to inhibit N+/K+ or should be used with caution in all of
ATPase is reduced the following EXCEPT:
A. Complete atrioventricular block.
45. All of the following are considered B. Digitalis intoxication
mechanisms of antiarrhythmic drugs C. Severe congestive heart failure
EXCEPT: D. Atrial fibrillation of recent origin
A. Calcium channel blockade E. History of idiosyncrasy due to
B. Inhibition of catecholamine action on quinidine
the heart
C. Sodium channel blockade 51. A 45-year-old man asks his
D. Increasing the slope of phase 4 physician for a prescription for
depolarization sildenafil to improve his sexual
E. Prolongation of effective refractory performance. Because of risks from a
period (action potential duration) serious drug interaction, this drug
should not be prescribed, and the
46. All the following are side effects of patient should be urged not to try to
amiodarone EXCEPT: obtain it from other sources, if he is
A. Corneal microdeposits also taking which of the following
B. Tachycardia drugs?
C. Thyroid dysfunction A. An angiotensin-converting enzyme
D. Pulmonary infiltration inhibitor
E. Hepatotoxicity B. A beta-adrenergic blocker
C. A nitrovasodilator (e.g., nitroglycerin)
47. Digitalis toxicity is enhanced by D. A statin-type antihypercholesterolemic
which one of the following: drug
A. Decreased extracellular Ca++ E. A thiazide or loop diuretic
B. Decreased stimulation rate
C. Increased extracellular Mg++

200
52. A 70-year-old woman is treated A. Prinzmetal’s angina
with sublingual nitroglycerin for B. Unstable angina
occasional bouts of effort-induced C. Classic angina
angina. Which of the following best D. Variant angina
describes the mechanism by which E. Preinfarction angina
nitroglycerin causes its desired
antianginal effects, or a mediator 57. Which of the following drugs is a
involved in it? class IV antiarrhythmic that is primarily
A. Blocks beta adrenergic receptors indicated for the treatment of
B. Forms cyanide, much like the supraventricular tachyarrhythmias?
metabolism of nitroprusside does A. Diltiazem
C. Increases local synthesis and release B. Digoxin
of adenosine C. Mexiletine
D. Raises intracellular cGMP levels D. Quinidine
E. Stimulates phosphodiesterase E. Propranolol

53. All of the following are


recommended at the initial stages of Answers
treating patients with heart failure
EXCEPT: 1A 13 C 25 D 37 B 49 D
A. Reduced salt intake 2D 14 E 26 E 38 A 50 D
B. Verapamil 3D 15 C 27 C 39 E 51 C
C. ACE inhibitors 4D 16 A 28 D 40 E 52 D
D. Diuretics 5E 17 D 29 D 41 C 53 B
E. Vasodilators 6D 18 A 30 E 42 E 54 D
7B 19 E 31 C 43 B 55 B
54. Which one of the following drugs 8E 20 A 32 C 44 C 56 C
increases digoxin plasma 9D 21 E 33 C 45 D 57 A
concentration by a pharmacokinetic 10 A 22 A 34 C 46 B
mechanism? 11 A 23 D 35 B 47 D
A. Captopril 12 B 24 B 36 E 48 C
B. Hydrochorothiazide
C. Lidocaine
D. Quinidine
E. Sulfasalazine

55. Regarding verapamil, which one of


the following statements is false?
A. Angina pectoris is an important
indication for the use of verapamil
B. Contraindicated in the asthmatic
patient
C. Relaxes vascular smooth muscle
D. Slows the depolarization phase of the
action potential in AV nodal cells
E. Used in management of
supraventricular tachycardias

56. Exertion–induced angina, which is


relieved by rest, nitroglycerin, or both,
is referred to as:

201
202
Part 1
1: Hyp
perlipide
emia an
nd drugs
s that lo
ower pla
asma lip
pids

█ Bas
sic inform
mation

 Lipooproteins consist off a hydroph hobic lipid


d core (TGs
s or cholessterol) surrounded
by a hydrophilic coat of phospho olipids and d proteins (apoprotei ns), whichh render
themm miscible e in aqueouus plasma .
 Theere are 5 classes
c of lipoproteinns depend ding on the
eir relative proportionn of the
coree lipids, type of apop
protein, sizze, and den nsity:

 The
e intestine e is the maain source of lipid pre
ecursors. The
T liver i s the mainn site of
syntthesis of liipoproteins
s. The adi pose tissuue is the main
m site o
of storage of TGs.
Fat cells don’tt synthesizze any lipo
oproteins.

▌Lipop
protein metabolis
m sm

■ In tthe exoge enous pathway, abssorbed cholesterol and a TGs aare transpoorted in
plassma as chylomicro
c ons. On the vascu ular endothelium, tthe core TGs is
hyddrolyzed byy a surface e-bound lippoprotein lipase
l into FFA whicch enter thee tissue
andd utilized. The
T chylom microns reemnants (c containing mainly chholesterol) pass to
the liver wherre cholesterol is sto ored, oxidized to bilee acids, o
or secretedd in the
bile
e. Alternativvely, it may
y enter the
e synthesiss of VLDL.
■ In the endo ogenous pathway,
p cholesterool and ne ewly synthhesized TGs
T are
assembled ass VLDL and delivered d to the blood wheree TGs coree is hydrolyyzed by

203
lipo
oprotein lippase into FFA as d described above. The T smalleer VLDL particles
p
havving less TGs
T and more
m choleesterol are now termmed LDL. C Cholestero ol in the
LDLL may be: (1) utilized by the tis sues; (2) re
eturns agaain to the liiver; (3) de
eposited
subbintimal in blood
b vess
sels and ca ause atherrosclerosiis.
■ Whe en cells diie, cholesterol in the
eir plasma membrane es is returnned to the liver as
plassma HDL particles.
p HDL
H functi ons as scaavenger lip
poproteins..

▌Class
sification
n of hyperlipidemiia

■ Prim
mary (fam
milial; hered
ditary) hyp mia: is genetically dettermined.
perlipidem

Class
s In
ncreased lipoprotein
n Synonym
S
Type I ↑ chylomicro
ons Familial chy
ylomicroneemia
Type IIa ↑ LDL Familial hyp
percholesteerolemia
IIb ↑ LDL and VLDL
V Familial com
mbined hyp perlipidem
mia
Type III ↑ IDL Familial dys
sbetalipoprroteinemiaa
Type IV ↑ VLDL Familial hyp
pertriglycerridemia
Type V ↑ VLDL and chylomicrrons Familial mix
xed hyperliipedemia

■ Sec
condary (a
acquired) hyperlipid
demia:
– Hyperchollesterolemia: hypothyyroidism, nephrotic
n syndrome,
s , and drugs
s.
– Hypertriglyyceridemia
a: DM, alco
ohol, gout,, chronic re
enal failuree.

204
█ LIPIID LOWER
RING DRU
UGS

Classiification of
o drugs Drug theerapy
is indica
ated in:
■ Inhiibitors of intestinal cholesterrol absorp ption:
– Bile acid binding
b res
sins: cholesstyramine, colestipol Failure of non-
– Ezetimibe drug therapy.
■ Acttivators off plasma liipoproteinn lipase: fib
bric acid Primary
deriivatives (heredita
ary)
■ HMMG-CoA re eductase in nhibitors: statins. hyperlipidemia.
■ Inhiibitors of hepatic lip
pid produc ction: nico
otinic acid,
acip
pimox
■ Oth
her drugs: d-thyroxin n, neomyciin, and proobucol

Chole
estyramin
ne and colestipo
c ol

Mecha
anism of action
a
They fo orm comp plexes withh bile acidds in the
ne and ↓ en
intestin nterohepatic absorptioon of bile
nd ↓ absorp
salts an ption of cho
olesterol.

Therappeutic use es
 Hyp percholeste erolemia (ttype IIa): B
Bile acid
seq
questrants are effec ctive in rreducing
plassma choolesterol (10%–20 0%) in
patiients with some
s normmal LDL re eceptors.
 Diarrrhea due tot bile acid
d malabso rption.
 Pruritus due to o obstructiive jaundic
ce.

Adverrse effectss
– GITT upset (the most co ommon): n ausea, vom miting and hea (due to ↓ fat
d steatorrh
abssorption).
– ↓ abbsorption of
o fat-solub
ble vitamin
ns.
– ↓ abbsorption of
o anionic drugs e.g . digitalis and
a warfarrin.

Ezetim
mibe

ansism of action
Mecha
Ezetimibe is a se elective inhibitor of intestinal cholestero
ol absorptiion. It is effective
e
even inn the abssence of dietary ch holesterol because it inhibitss reabsorp ption of
cholestterol excre
eted in the bile.

205
Therap
peutic use
es
Hyperc
cholesterolemia: ezettimibe is ssynergistic with HMG
G-CoA red
ductase inh
hibitors,
produc
cing decrea
ase of 25%
% in LDL ch holesterol..

Advers
se effects
Reversiible hepatic
c dysfunctio
on: liver fun
nction tests
s should be
e done at reegular interv
vals.

HMG--CoA red
ductase inhibitor
i rs (Statin
ns)
(Lovas
statin, Prav
vastatin, Mevastatin
M n, Atorvas
statin)

Mecha
anism of action
a
Compeetitive inhibition off hydroxyy-methyl-g
glutaryl
coenzyyme-A (HM MG-CoA) reductase
r e → ↓ cholesterol
esis and ↑ hepatic up
synthe ptake of LD
DL.

Therap
peutic use
es
 Hyp
percholesteerolemia (ttype II).
 With
h other dru
ugs for com
mbined hyyperlipidem
mia.

Advers
se effects
H : Heppatic dysffunction lleading to o elevationn of serum m transam minases.
The
erapy shou uld be stop pped if liv
ver enzyme es rise > 33-folds the
e upper
norm
mal value.
M : Myoopathy, my yositis andd rhabdom myolysis in n both skeeletal and cardiac
g to ↑ of crreatine pho
musscle leading osphokinas se (CPK) ennzyme.
G T upsets: nausea,
: GIT n vo miting, anoorexia (the
e most commmon).
Co-A : Cattaract (lentticular Opaacity) in miiddle-Agedd individuaals.
Reductasse nal dysfunc
: Ren ction (espeecially withh lovastatin
n).

Fibric
c acid de
erivatives
s (Fibrate
es)
(Clofib
brate, Feno
ofibrate, Bezafibrate
B e, Gemfib
brozil)

Mecha
anism of action
a
Fibrate
es act on n nuclearr recepto ors calledd peroxisoome proliiferator acctivated
receptoors-α (PPA ding to ↑ ssynthesis of lipopro
AR-α) lead otein lipasse → ↑ pe
eripheral
cataboolism of VLDL and chhylomicron ns (TGs).

Therap
peutic use
es:
 Hyppertriglycerridemia (ty
ypes IIb, II I, IV and V).
V
 Fennofibrate haas antidiuretic actioon in individuals with
h mild to m
moderate diabetes
d
insipidus.

206
Adverse effects
– GIT upsets: nausea, vomiting (the most common).
– Increase formation of cholesterol gallstones.
– Hepatic dysfunction and elevation of serum transaminases.
– Fibrates increase the risk of myopathy if used in combination with statins.
– Skin rash and dermatologic reactions.

Nicotinic acid (Niacin; vitamin B3)

Mechanism of action
 Niacin (but not nicotinamide) inhibits lipolysis in adipose tissue and inhibits fatty
acid synthesis by the liver → ↓ hepatic VLDL and LDL synthesis.
 This is distinct from the role of niacin as a vitamin, in which it is converted to
nicotinamide and is used for the biosynthesis of the cofactors NAD and NADP.

Therapeutic uses
In combination with other drugs for all types of hyperlipidemia (except type I which
is mainly treated by diet control).

Adverse effects
– Skin flushing and burning sensation (the most common). It is harmless effect
mediated by PGs and histamine release and can be diminished by taking aspirin
30 minutes before taking nicotinic acid.
– Gastric irritation (the drug should be avoided in peptic ulcer).
– Hyperglycemia, hyperuricemia, and reversible increase in serum transaminases.

█ Summary
Effect on LDL Effect on HDL Effect on TGs
Bile acid-binding resins ↓↓↓ ↑ ----
Reductase inhibitors ↓↓↓ ↑ ↓
Fibrates ↓ ↑ ↓↓↓
Niacin ↓ ↑↑↑ ↓↓

█ Treatment with drug combinations


Hypercholesterolemia Cholestyramine + Reductase inhibitors
Hypertriglyceridemia Niacin + Fibrates
Familial combined Cholestyramine + Fibrates.
hyperlipidemia Cholestyramine + Niacin.
Statins + Fibrates (this combination may ↑ risk of myopathy).

207
Part 2
2: Dru
ugs affe
ecting he
emostas
sis

█ Basiic inform
mation

stasis is th
Hemos he spontan
neous arresst of bleed
ding from damaged
d b
blood vess
sel.

▌The c
coagulatiion syste
em:

It conssists of a number of plasma a


proteinns (facto ors). Eac ch factorr
activatees anothe er till acttivation off
fibrinog
gen to formm fibrin.
Activation cascad de is done
e through 2
systemms: the inttrinsic sys stem (can n
be mon nitored by the activaated partia
al
throbop plastin tim
me “APTT””) and the e
extrinssic system m (can be monitored d
by protthrombin time).

Preven
ntion of clo
otting:
 Plassma conttains a number
n off
protteins thatt normally y preventt
clottting thro ough inhibition off
clottting factors (e.g .
antithrombin n III).
 Hepparin activvates antith hrombin III
thuss inhibits ac
ctivation of factors IX, X, XI, and XII.
 Oraal anticoag gulants inh hibit synthhesis of clo
otting facto
ors by the l iver.

Dissolu
ution of bllood clots
s:
 Theere are oth a proteinss that can dissolve blood clotts (i.e. fibrrinolytic
her plasma
systtem e.g. plasmin).
 The
ere are som me drugs that
t activa
ate plasmin nogen to form the acctive plasm
min and
thuss enhancee fibrinolysis (e.g. stre
eptokinas se, urokina
ase, etc.).
 Amminocaproiic acid is ana inhibitorr of fibrinolysis.

▌Drug
gs can afffect hemo
ostasis a
and throm
mbosis through:

 By modifying the integriity of the vvessel wall.


 By modifying blood coa agulation (clot forma ation).
 By modifying platelet adhesion an nd activation.
 By modifying fibrinolysis.

208
█ DRU
UGS THAT
T PREVEN
NT COAGU
ULATION (ANTICOA
( AGULANTS
S)

Heparin Warfarin (Ora l anticoag


gulant)
Source
e& Naatural sulfate
ed polysac ccharide Synthetic cuomaarin compound
chemis
stry preesent in maast cells & c
carries –ve
chaarge
Coommercial preparation
p ns are de-
rive
ed from bovine lung o or
porcine intesttinal extractts
Absorp
ption  No becaus se it precip
pitates by  Good (bioavaailability is 100%).
gastric HC
Cl.  Can
n cross BB
BB and pla acenta.
Distribttion  Cannot crross BBB o or placenta
a.
 Its action depends
d o
on the pres s-  Wa arfarin inhib
bits vitamin K
ence of a natural clo otting inhibb- epooxide redu uctase enz zyme
itor called heparin co ofactor in the liver leaading to in
nhibition
(antithrom mbin III) in plasma. of formation
f o
of the activve form
 Small quantities of h heparin can n of vitamin
v K → ↓ synthes sis of
Mechanism of action

activate anntithrombin n III leading vita


amin K-dep pendent cllotting
to inhibitio
on of severral clotting factors (II, VIII, IX, and X).
X
factors especially fac ctor X &  Thee action off warfarin could
c
thrombin (factor
( II). be antagonizeed by vitam min K.

Onset and Immediate and


a short ((2-4 hrs). Delayyed for 8-1 2 hrs (time e
duratio
on needeed for deple letion of clo
otting
factorrs & vit K) aand long (33-7 d)
Pharma acol  Anticoagullant in vivo
o & in vitro Antico
oagulant in
n vivo only
y.
effects
s  Stimulates
s lipoproteiin lipase →
↓↓ serum triglyceride
t es (TGs).
Therappeu-  Treatmennt of estabblished Warfa
arin is givenn oral 2-10
0
tic use
es thrombos sis: heparin
n is given mg/daay for prevvention an nd
parenteral 5000-10,0000 U i.v. treatm
ment of:
then 5000 U s.c./8h to maintaiin – Deeep vein thrrombosis (DVT)
(
blood coagulation 2--3 times ass – Posstoperativee thrombossis.
normal and preventt further – Cerrebral venoous thromb bosis.

209
extension of the thrombus. – Coronary thrombosis:
treatment continued for
 Prevention of thrombosis: several years.
5000 U s.c./8-12 hrs. – Acute arterial & pulmonary
embolism: anticoagulation is
initiated by heparin and
maintained by warfarin.
– AF and artificial heart valves
Monitor- By activated partial thrombo- By prothrombin time (PT) or
ing of plastin time (APTT). International Normalized Ratio
therapy It must be kept 2-3 times as the (INR). It is the ratio of the PT in
normal value. the patient to that of normal
person. It must be kept 2-3 times
as the normal value.
Adverse  Bleeding is the most common and dangerous SE (e.g. hematuria
effects & major organ bleeding). It could be treated by the following:
(a) Immediate stopping of the drug.
(b) Fresh frozen plasma (FFP): to provide fresh clotting factors.
(c) Protamine sulfate (Protam): (c) Vitamin K1: 10 mg slowly
- Protamine carries +ve charge i.v. or i.m. to enhance
that combines with heparin synthesis of clotting factors.
(carries -ve charge) to form
stable complex.
- 1 mg of protamine can bind to
100 U of heparin.
 Hematoma if given IM (so, con-  Hemorrhagic skin necrosis:
traindicated to give it IM). when starting warfarin, biosyn-
 Thrombocytopenia: immune- thesis of protein C is reduced
mediated reaction due to for- leading to temporary procoag-
mation of antibodies that can ulant state. This can lead to
bind to platelets. Platelet count hemorrhagic infarction of skin,
should be performed regularly breast, intestine and fatty tiss-
 Osteoporosis and spontaneous ue. normally avoided by conc-
fractures on long-term therapy urent heparin administration.
 Alopecia and dermatitis: rare  Teratogenicity: abnormal bone
and transient. formation in early pregnancy
(fetal warfarin syndrome).
 CNS Hemorrhage in the fetus
if given in late pregnancy.
 Sudden withdrawal may lead
to thrombotic catastrophes.

210
Low-m
moleculaar-weigh ht hepari ns
(LMW
WH) (Enoxa
aparin – Dalteparin)
D )

 Sta nfractionatted) heparrin is a


andard (un
mixxture of different
d molecular
m weight
frac
ctions (MW W 3000-30,000 Da) thhat can
affe
ect more th han one cooagulation
n factor
andd produce thromboc cytopenia (↑ risk
of b
bleeding).
 LMW WH has a MW less than 80 000 Da
thatt makes it specific for
f factor X with
minnimal effec cts on platelets and
d other
clottting factorrs.

Unfra
actionated
d heparin LMWH
H
Molecu
ular weightt range Wide (ranges fro
om 3000 to
o Less thhan 8000 Da
D
30,00
00 Da)
Anti-fac
ctor Xa acttivity Less sspecific More sspecific
Non-sppecific bind
ding to High Low
vascula
ar endothe elium and
plasma
a proteins
Bioavailability after s.c. Low High
injectio
on (due tto binding to s.c.
tissue)
e)
Half-life
e Short (given 3 tiimes/d) Long (g
given once
e/d)
Thromb
bocytopen
nia Comm
mon (10%)) Less ccommon (<
<2%)
Risk off bleeding High Low
Lab mo
onitoring APTTT Anti-faactor Xa lev
vels
(Essen
ntial) (May b be unneces ssary)

Synthe
etic facto
or Xa inhibitors

Fondap
parinux - Synthe etic polysaaccharide that
t have the
t same m mechanism
m like
LMWH H (i.e. selecctive inhibitor of facttor Xa).
- It is given by s.c c. injection once dailly (has long
g t½).

Directt thrombin inhibito


ors (DTIs)
s)

Bivaliru
udin - It is a synthetic h nalogue (hirudin is a direct thro
hirudin an ombin
inhibittor peptidee present in
n the saliva
a of mediccinal leech).
- Bivalirrudin is a re
eversible and
a short-a acting DTI . It is given
n i.v.

211
- It is approved as alternative to heparin to treat patients with
heparin-induced thrombocytopenia.
- Bleeding is the major side effect.
Argatroban - Synthetic compound that binds reversibly to thrombin.
- It can be used as alternative to heparin to treat patients with
heparin-induced thrombocytopenia.
- It is given i.v. and has immediate onset of action.
Dabigatran - It is the first oral DTI approved by the FDA in 2010.
- It was found superior to warfarin in preventing the risk of stroke
and systemic embolism in susceptible patients.

Use of anticoagulants during pregnancy

– Pregnancy is a hypercoagulable state due to increase levels of coagulation


factors and venous stasis. Pegnancy increases the risk of venous thrombosis
and pulmonary embolism.
– The use of warfarin in the first trimester is associated with birth defects (5%),
while its use near fullterm increases the risk of fetal hemorrhage.
– Neither UFH nor LMWH cross the placenta; therefore, do not cause fetal
bleeding or teratogenicity, but they can reduce bone mass density and cause
osteoporosis if used throught pregnancy.
– Anticoagulation is recommended in most pregnant patients with a mechanical
heart valve. The American College of Chest Physicians (ACCP) recommends the
use LMWH until 13 weeks' gestation, THEN change to warfarin until the patient is
close to delivery (34 weeks), and THEN restart LMWH. Long-term anticoagulants
should be resumed postpartum.

Contraindications of anticoagulant therapy

Hematological: – Hemorrhagic blood diseases e.g. hemophilia &


thrombocytopenia.
Neurological: – Recent hemorrhagic stroke within 3 weeks.
– Recent brain or eye surgery.
CVS: – Subacute bacterial endocarditis (SBE).
– Uncontrolled hypertension (→ risk of cerebral bleeding).
GIT: – Active PU, esophageal varices, and hemorrhagic pancreatitis
– Active inflammatory bowel disease (ulcerative colitis).
Liver: – Liver failure (this patient has bleeding tendency)
Renal: – Renal failure.
Gynecological: – Threatened abortion.
– Warfarin is not given in the first timester.

212
Drug in
nteraction
ns of oral anticoagu
a ulants (warfarin)

Dru
ugs that potentiate
p warfarin Drugs
s that inhib
bit warfariin
 Mic
crosomal enzyme
e in
nhibitors  Microsomal enzym ers (e.g.
me induce
(e.g
g. cimetidinne, chloram mphenicol)) ↓ phenoba mpin) ↑
arbital, rifam
mettabolism of o warfarin. metabolism of warffarin.
 Oraal antibiotiics: ↓ vit K synthesiss  Oral con
ntraceptive es and vit K. ↑
by kkilling the gut
g flora synthesis
s of clottin g factors
 Liquuid paraffiin: ↓ vit K absorption
a  Aluminum hydroxxide: ↓ warffarin
 NSA AIDs: disp place warfa arin from p
pp. absorptio
on

█ FIBR
RINOLYTIC (THROM
MBOLYTIC
C) DRUGS
S

 Norrmally, whe en a fibrin clot is form


med, plasm minogen gets
g in conntact with this
t clot
andd becomess activated d into plas smin by th he help off naturally occurring g tissue
plassminogen n activatorrs (t-PAs). Plasmin causes
c lysis of the clo
ot.
 Theese endoge enous activators pre eferentially
y activate plasminog gen that is
s bound
to ffibrin, which (in the eory) confiines fibrinolysis to the formeed thromb bus and
avooids systemmic activatiion.
 Fibrrinolytic drrugs cause rapid ac ctivation of
o plasminogen to fo orm plasmmin, but
unfoortunately,, they may y activate both fibrin n-bound plasminoge
p en and circculating
plassminogen thus, bo oth prote ective hemostatic thrombi and path hogenic
thro
omboembo oli are brok
ken down ((→ risk of bleeding).
b

Strepttokinase
 Streeptokinasee is a prote
ein (not an
n enzyme) that is isoolated from
m streptoc
cocci; it
activates plassminogen into plasmiin non-enzzymaticallyy.
 It m
may be antigenic (cauuses allergyy) in some people.

213
Urokin
nase
 Uro okinase is a proteas se originallly isolated
d from
urinne; the druug is now prepared in recomb binant
formm from culttured kidney cells.
 It is less antigenic than streptokina
s ase.

Recommbinant tissue
t pla
asminoge en activa
ators (t-
PAs): ((Alteplase
e, reteplas
se, and ten
necteplase
e)
 Theey are recoombinant human protteins produ uced in cultured cellss.
 Theey are mos c to fibrin-b
st specific bound pla asminogen; local actiivation of plasmin
p
at th
he thrombus site red duces the incidence of o systemic bleeding g.
 Retteplase an nd tenectteplase h have long half-life. The long half-life permits
admministrationn as a bolus i.v. injeection rath
her than byy continuoous infusio on. (two
injections i.v. separatedd by 30 minutes for reteplase,
r , one singlle i.v. injec
ction for
teneecteplase e).

Therap
peutic use
es of throm
mbolytic d
drugs
 Theey are giveen by i.v. route
r in ca
ases of ac cute myoc arction, is
cardial infa schemic
stro
oke, pulmo onary embo olism, and arterial thrombosis.
 In c
cases of ac cute MI, th hey should d be givenn within 12
2 h of onseet. The ma aximum
bennefit is obta
ained if treatment is ggiven within 90 minuutes of the onset of pain.
p
 Befoore throm mbolysis, care
c shouldd be take en to ensu
ure there is no liab bility for
blee
eding in a critical
c site
e e.g. retina
a, CNS, ettc.

Advers
se effects of thromb
bolytic dru
ugs
– Sysstemic ble
eeding is the major adverse effect. The
e risk is hhigh with strepto-
s
kina
ase and loww with the recent rec
combinant tissue pla
asminogen activators s.
– Streeptokinasee can cause allergy, fever, and
d hypotenssion during
g i.v. infusion.

█ ANT
TIPLATELE
ET (ANTIT
THROMBO
OTIC) DRU
UGS

Aspirin
 Asp
pirin inhibit platelet ag
ggregation
n by:
– Irreverssible inhibition of COOX enzyme e → ↓ TXA2 → ↓ plateleet aggregattion.
– Irreverssible acetylation of plaatelet cell membrane
m s → ↓ platellet adhesio
ons.
– Decreaase platele et ADP synnthesis → decrease platelet accuumulation.
 At h
higher dosses (> 325 5 mg/day), aspirin maym decrea ase endothhelial synth hesis of
PGII2, which inhibits pla
atelet activvity. Low doses
d (75-150 mg/daay) ↓ synth
hesis of
plattelet TXA2 more thann PGI2 in eendothelial cells and avoid this effect.
 Othher NSAIDss do not haave compa arable antithrombotic activity.

214
Blockeers of plaatelet ADP recepto ors:
(Ticlop
pedine, clo
opidogrel, prasugre
el)

 Theese drugss irreverrsibly


inhibit the binnding of AD DP to
its receptorss on pla atelets
andd, therebyy, inhibit the
activation of the
glyccoprotein IIb/IIIa
receeptors required for
plattelets to bind to fibrin
nogen andd to each other.
o
 Cloopidogrel is i approve ed for propphylaxis off thrombossis in bothh cerebrov vascular
andd cardiovasscular dise ease (e.g. coronary artery dise ease, coro onary angio oplasty,
periipheral vasscular diseease, etc.). The mainttenance do ose is 75 mmg/d orally y.
 Clopidogrel iss a prodru ug, and its therapeuttic efficacyy relies enttirely on its
s active
mettabolite. So ome people have ge enetic defic
ciency in th
he enzymees that mettabolize
pidogrel; they are calle
clop ed “poor m
metabolizerrs”, and caannot beneffit from thiss drug.
 Ticlopedine has h been largely rreplaced by b cloped dogrel beccause of serious
advverse effec cts (neutroppenia and bleeding)). Clopidog grel has feewer incidence of
thesse effects.

Blockeers of pla
atelet gly ycoprotein n IIb/IIIa receptors:
(Abcixiimab, eptiifibatide, tirofiban)
t

 Activation of this recep


ptor compllex is the “final com
mmon pathhway” for platelet
agggregation and
a bindinng with fib
brinogen. Persons
P la
acking thiss receptor have a
blee
eding disorder called
d Glanzman nn’s throm
mbasthenia
a.
 Abc
ciximab is the Fab fragment o
of a monoc
clonal antib binds to gpIIb/IIIa
body that b
andd blocks binding of platelets to fibrinogenn.
 Epttifibatide is a small syntheticc peptide that com mpetitively blocks gpIIb/IIIa
receeptor.
 Tiro
ofiban is a peptide ofo low MW that binds s to the gpIIIb/IIIa receeptor.
 Theese drugs have been n approvedd for use in patientss undergo ing percuttaneous
coroonary interrvention, fo
or unstable
e angina, and
a for pos st-MI.

Dipyridamole

 Dipyyridamole inhibits phosphodi


p zyme → ↑ cGMP → VD and
iesterase (PDE) enz
inhibition of platelet
p acttivity. It alsso inhibits o adenossine into platelets
s uptake of p
andd RBCs lea ading to ex
xtracellular accumula ation and prolongatio
p on of its action.
 Thee use of dipyridamo
d ole as an antithromb botic agennt is limiteed to prop phylaxis
(com
mbined witth warfarinn) in patien nts with proosthetic (m
mechanicaal) heart va alves.

215
█ DRUGS USED IN BLEEDING DISORDERS (HEMOSTATIC AGENTS)

▌Systemic agents

– Vitamin K: essential for synthesis of factors II, VII, IX, X by the liver.
– Vitamin C and rutin: preserve the integrity of the vascular wall.
– Fresh blood or plasma transfusion: as sources of coagulation factors.
– Plasma fractions:
– Thromboplastin (factor III): prepared from mammalian tissues.
– Antihemophilic globulin (factor VIII): given in hemophilia A.
– Calcium (factor IV): as a coagulation factor.
– Aminocaproic acid and tranexamic acid: inhibitors of fibrinolytic system.
– Ethamsylate (Dicynone): given i.m. to reduce capillary bleeding.

▌Local agents

– Physical methods: application of pressure, cooling or heat coagulation.


– Vasoconstrictor drugs: e.g. adrenaline nasal pack in epistaxis.
– Astringents: drugs which precipitate surface proteins e.g. alum sulphate.
– Thrombin and thromboplastin: applied on the bleeding surface as powders.
– Fibrin and fibrinogen: available as dried sheets and used in surgery.
– Oxidized cellulose: it forms an adhesive mass on the bleeding surface.
– Sclerosing agents: chemicals that cause thrombosis in veins and permanent
obliteration, e.g. ethanolamine oleate (given i.v. for varicose veins).

Vitamin K

 Vitamin K1 (phytomenadione) is a naturally occurring fat-soluble vitamin


present in green vegetables. It is available clinically in oral and parenteral forms.
 Vitamin K2 is synthesized by intestinal bacteria.
 Both vitamins K1 and K2 require bile salts for absorption from the intestine.

Mechanism of action: Vitamin K is required for posttranslational modification of


clotting factors II, VII, IX, and X by liver cells.

Therapeutic uses
 To reverse bleeding episodes caused by overdose of warfarin, salicylates, and
oral hypoglycemic drugs.
 To correct vitamin deficiency caused by dietary deficiency, or in patients
receiving oral antibiotics.
 To prevent hypothrombinemia of the newborn: all newborns should routinely
receive 1–2 mg of vitamin K directly after birth (especially in premature infants).

216
Advers s: parenteral vitamin K1 is diss
se effects solved in oil;
o rapid ii.v. administration
can cau
use dyspnnea, chest pain, or evven death.

N.B.. severe hepatic failure


f is associate ed with bleeding
b teendency (due
( to
redu
uced synthesis of clo
otting factoors) that do
oes not res
spond to vvitamin K.

Plasm
ma fractions

 Deficiencies in plasma
a coagulat ion
facttors cann cause
e bleedinng.
Spoontaneous bleeding occurs wh hen
facttor activity is less than 5–10% % of
norrmal. Factor VIII deficien ncy
(classic hemo ophilia, or hemophilia
h a A)
andd factor IX X deficienccy (Christmmas
diseease, or hemophilia
h a B) accou unt
for the mosst commo on herita able
coaagulation defects.
 Plassma prottein prepa arations a are
avaailable for treatmen nt of the ese
disoorders. The ey are admministered i.v.
 Plassma fractions are frrequently prepared from bloo od or plassma pooleed from
mulltiple indivviduals; thus, they a are assoc h high riskk of exposure to
ciated with
ection (he
infe epatitis andd HIV). Fo or this reason, recoombinant preparatioons are
recoommended d wheneve er possible
e.

Inhibittors of fib
brinolysis
s:
(Amino
ocaproic acid
a and trranexamic
c acid)

Aminocaproic acida is a synthetic agent tha at competitively inhi bits plasm


minogen
activatiion. Trane
examic aciid is more potent ana
alogue of aminocapr
a roic acid.

Therap
peutic use
es
 To prevent bleeding frrom tissuees rich in
plassminogen activators e.g. lung, prostatic
surggery, meno orrhagia, and
a ocular trauma.
 Prophylaxis for rebleeding
r from
intrracranial aneurysm.
a .
 As a adjunctive therapy in
n hemophiilia.
 To stop blee eding caus sed by tox xicity of
fibrrinolytic drrugs.

217
█ DRU
UGS AFFE
ECTING BL
LOOD VIS
SCOSITY

Pentox
xifylline

 Pen
ntoxifyllinee is a synth
hetic meth
hylxanthine
e structurallly similar tto caffeine.
 Pen
ntoxifylline appears to inhibit RBCs phosphodie esterase (PDE) → ↑ RBC
mem mbrane fle nd ↓ blood viscosity (rheologic
exibility an ( modifier).
 It iss approveed for use e to improove microv vascular circulation
c in patien
nts with
ermittent claudication, diab
inte betic ang giopathy, chronic leg ulcers, and
periipheral vasscular dise
ease.

Part 3
3: Dru
ugs used
d in the
e treatment of anemias
a s

Anemiaa is definned as a low hem moglobin (H


Hb) conceentration due to reeduced
producction or in
ncreased loss of RB
BC. The WH s anemia aas Hb <13 g/dL in
HO defines
men orr <12 g/dL in women.

█ Iron deficien
ncy anemia (hypoc
chromic microcyti
m c anemia
a)

 Iron
n deficiency is the mo ost commo on cause of
o anemia.
 Thee total am mount of transferrin
t determines the tootal iron-bbinding caapacity
(TIB
BC) of plasma.
 Transferrin saturation
s (i.e. plasm
ma iron/TIB
BC) is norm
mally 20–550% and provides
p
a usseful indexx of iron sta
atus.
 In irron-deficie
ency states s, there iss ↓ in plasm nd ↑ in TIB
ma iron an BC, so tra
ansferrin
satuuration is < 20%. The e opposite e occurs in hemochroomatosis.

Iron

■ Abssorption:
 Iron is abssorbed fro om upper ssmall intes stine in the
e ferrous sstate (Fe2++) more
tthan in the
e ferric statte (Fe3+). A
About 10% % of dietary
y iron is abbsorbed.
 Factors tha at ↑ absorpption: pregn nancy & gro owing infan
nts, gastricc HCl and vit
v C.
 Factors that ↓ absorption:
 Malabssorption sy yndromes: e e.g. celiac disease.
d
 Antacid ds: form inssoluble commplexes with iron.
 Tannic c acid (stro
ong tea): prrecipitates iron
 Tetracyyclines: ch helate iron..
■ Transport: in n plasma viav transfe errin and stored in
the reticulo-en ndothelial system an nd bone marrow
m as
ferrritin.

218
■ Excretion: There is no active mechanism for excretion of iron. Small amounts are
excreted through stool, urine, and sweat. The main regulation of iron level in the body
is done at the level of absorption.

Causes of iron deficiency anemia (= Indications of iron therapy)


 Deficient intake: e.g. poor diet or malabsorption syndrome.
 Increased demand: e.g. in pregnancy and premature babies (iron & folic acid
should be given to pregnant women from the 4th month).
 Excessive blood loss.

Preparations of iron

■ Oral iron:
 Many oral iron preparations are available (Ferrous sulfate, ferrous gluconate,
& ferrous fumarate) which are are effective and recommended for most
patients. Vitamin C may be given with iron to improve absorption.
 The Hb usually reaches normal levels within 1-2 months but treatment should
be continued for 3-6 months to replenish iron stores.

■ Parenteral iron:
Indications:
 Inability to tolerate or absorb oral iron.
 Severe anemia or extensive blood loss cannot be corrected by oral iron.
Administration:
 Iron dextran or iron carboxymaltose is given by slow i.v. infusion as a 1000
mg diluted in 500 ml saline over 1-2 hours. The initial 25 ml should be
infused slowly (as a test dose) and the patient should be observed for
allergic reactions.
 Other parenteral preparations should be administered according to
product labeling information.

Adverse effects of iron

 Oral iron: GIT upset (most common): nausea, epigastric pain, abdominal
cramps, constipation or diarrhea. These effects can be minimized by taking iron
after meals.

 Parenteral iron:
 Local pain and tissue staining with i.m. injection.
 Anaphylactoid (allergic) reaction in 3% of patients. True anaphylaxis is very
rare.
 Lymphadenopathy.

219
▌Iron toxicity

■ Acute iron toxicity:

Causes: more frequent in children due to accidental ingestion of iron tablets.

Manifestations:
 Vomiting, hematemesis and bloody diarrhea.
 Metabolic acidosis, shock, coma & death.

Treatment:
 Gastric lavage by phosphate or carbonate solutions to form insoluble
complexes.
 Desferrioxamine (Desferal; iron chelating compound): 1-2 gm i.m. to
chelate systemically absorbed iron. It can cause hypersensitivity reaction.
 Supportive treatment for GIT bleeding, metabolic acidosis, and shock.

■ Chronic iron toxicity:

Causes:
 Excessive iron absorption: e.g. in hemochromatosis (autosomal recessive
disorder characterized by ↑ iron absorption; transferrin saturation is >50%)
 Excessive blood transfusion: e.g. in thalassemia major.

Manifestations: excess iron is deposited in the liver, pancreas and skin.

Treatment:
 Intermittent phlebotomy is the treatment of choice: one unit of blood is
removed every one week until the excess iron is removed.
 Periodic desferrioxamine i.m. when phlebotomy cannot be performed e.g.
in patients with thalassemia major.

█ Red cell deficiency anemia (Anemia of chronic renal failure)

 Erythropoietin (EPO) is a glycoprotein secreted by interstitial cells of the renal


cortex (90%) and 10% by the liver. It is essential for normal reticulocyte
production. Patients with chronic renal failure develop red cell deficiency
anemia due to deficiency of EPO.
 Recombinant human EPO is available (epoetin alfa, and darbepoetin) for
treatment of the following anemias:
– Anemia of chronic renal failure.
– Anemia due to bone marrow suppression e.g. cancer chemotherapy or
zidoviodin (AZT) therapy.
– Anemia of multiple myeloma and rheumatoid arthritis.
 EPO is given s.c. injection once weekly to target Hb level between 10-12 gm/dl.

220
Advers
se effects
– Hyp n and incrreased blo
pertension sity (due to ↑ RBC mass) lea
ood viscos ading to
thro
ombotic coomplicationns especia ally when HbH is > 12 gm/dl.
– Seizzures and headache e probably caused by y rapid exppansion off blood volume.
– Iron
n deficienccy due to rapid ↑ in RBC mass s, so parenteral ironn should be
b given
with
h EPO (N.BB. in renal failure, ora
al iron is not effective due to p
poor absorrption in
succh case).

█ Vitamin defic
ciency (m
megalobla
astic) ane
emias

 Meg
galoblastic
c (macrocytic) anem
mia results
s
from
m failure ofo DNA syn nthesis du uring RBCs s
devvelopment. Failure of DNA synthesis s
ds to continuing cell
lead c growtth withouut
division, whic ch presentts as nonffunctioning g
mac crocytosis.
 Thee defect in red cell DN
NA synthe esis is most
ofte
en due to deficiency y of vitamin B12 and d
/or ffolic acid. Vitamin B12
B is nee
eded as a
co-ffactor for conversion
c n of dihydrrofolic acid
d
into
o the activ ve form (ffolinic acid
d) which iss
impportant for DNA synth hesis.

Vitamiin B12 (cya


anocobollamine)

 Vita
amin B12 is essentia
al for norm
mal
DNA A synthe esis, whic
ch is mo ost
app ely
parent in tissues thatt are active
dividing, such h as the GI
G tract a and
erytthroid preccursors.
 amin B12 is absorb
Vita bed from tthe
disttal ileum after bind ding to tthe
intriinsic factor.

Deficie
ency of vittamin B12
– Blo
ood: megalloblastic an
nemia.
– GIT
T: sore tong
gue and abbnormal G
GIT epithelium.
– CNSS (Subac cute Com mbined D Degenera ation):
deg
generation of the brain, sp pinal cord and
dem
myelination
n of nerve sheath.
s

221
Therap
peutic use
es

 Meg
galoblastiic and perrnicious an
nemia:
– If theree is no neurologicaal manifesstations, it is given as 1 mg i.m. 3
times/w
week for 2 weeks, th en once ev very 3 monnths.
– Improvvement in Hb
H concen ntration is apparent in 7 days aand norma
alizes in
1–2 moonths.
– Pernicious anemiia requiress life-long treatment.
t
 Perripheral neu
uropathy especially
e ddiabetic neuropathy
n y.
 Hyddroxocoba alamin (varriant of vitt B12) can
n be used
d in cyaniide poisoning. It
acq
quires the CN
C ions an
nd is conve
erted into cyanocobo
c olamine.

Folic a
acid (vita
amin B9; leucovori
l in)

Folic ac
cid is composed of three
t subu
units: pterid
dine, para--aminobennzoic acid (PABA),
and on ne to five glutamic acid resi dues. Abs sorbed folic acid frrom diet requires
r
reduction by dihyydrofolate reductase
r to the active form te
etrahydroffolate.

Deficie
ency of follic acid
– Meg a is more c
galoblastiic anemia common with
w folic acid
a deficieency than vitamin
B12
2 deficiecnncy. It occu
urs after 4 months off folic acid deficiencyy.
– Folic acid deficiency during pregnnancy leadss to neura al tube deffect.

Therap
peutic use
es
 Meg
galoblastiic anemia:
In m
megaloblasstic anemia a there is d
deficiency of vit B122 or folic a
acid. If the patient
hass combined d deficiency of both h agents, it is nece
essary to ttreat vitam min B12
defiiciency firsst because e administ ration of folic
f
acid
d alone lea ads to utilizzation of a
all vitamin B12
B
storred in tisssues for de eveloping red cells; so,
aneemia wou uld be partially co orrected but
neuurogenic symptomms are ated
aggrava
(sub
bacute com mbined degeneration n)

 Pre quirement is increas


egnancy: folate req sed
from
m 400g to 800g g/day to p prevent fe
etal
neuural tube defect
d (spiina bifida)).
 To prevent to
oxicity of antifolate
e drugs: see
box
x.
 Mallabsorption
n syndrom
me.

222
█ Hemopoietic growth factors

■ Erythropoietin (EPO): see before.

■ Granulocyte/macrophage colony stimulating factor (GM-CSF)

– GM-CSF is a recombinant protein expressed in yeasts. IIts principal action is to


stimulate myelopoiesis in granulocyte–macrophage pathways as well as
megakaryocytic and erythroid progenitor cells.
– Uses:
– It is given i.v. in cases of neutropenia and aplastic anemia.
– To stimulate stem cells after bone marrow transplantation.
– Adverse effects:
– Fever & Fatigue
– Arthralgia (bone pain) & myalgia (muscle pain).
– Capillary leak syndrome: edema, pleural and pericardial effusion.

■ Granulocyte colony stimulating factor (G-CSF; Filgrastim)

– G-CSF stimulates maturation of immature neutrophils.


– Uses: Neutropenia due to cancer chemotherapy.
– Adverse effects: similar to GM-CSF

■ Megakaryocyte (platelet) growth factor

– Interleukin-11 (IL-11) is a protein produced by fibroblasts and stromal cells in


the bone marrow. It acts on specific cell receptors to stimulate growth of
primitive megakaryocytes to form mature platelets.
– Oprelvekin is a recombinant IL-11 approved for treatment of thrombocytopenia.

█ Drug-induced aplastic anemia (bone marrow depression)

 It is considered the most serious drug-induced hematologic effect because of


the high mortality rate, (> 50% of cases). It is diagnosed by presence of two of
the following criteria:
– WBC count ≤ 3500/mm3,
– Platelet count ≤ 55,000/mm3
– Hb ≤ 10 g/dL
 There is fever and pharyngitis not responding to antibiotics, pallor, petechiae,
and bleeding appear on the average about 5-6 weeks after the initiation of the
offending agent.

223
Mechanism of drug toxicity
Damage to the hematopoietic stem cells due to:
– Direct, dose-dependent drug toxicity (the most common mechanism)
– Formation of toxic metabolites (idiosyncrasy).
– Immune reaction.

Management
 Stop the offending drug.
 Give fresh blood transfusion.
 Corticosteroids to inhibit the immune reaction.
 C-CSF or GM-CSF.
 Penicillin to combat infection.
 Stem cell transplantation in severe or resistant cases.

Drugs associated with drug-induced aplastic anemia


– Chloramphenicol – Carbamazepine
– Anti-cancer drugs – Sulfonamides
– Anti-thyroid drugs – Interferone-alfa

224
225
Review Questions

Classify hypolipidemic drugs. Mention the mechanism of action of each class.


Mention the mechanism of action and adverse effects of each of the following:
– HMG CoA reductase inhibitors (statins).
– Clopidogrel
– Erythropoietin
– Niacin
Mention the mechanism of action and therapeutic uses of:
– Vitamin K1
– Tranexamic acid
– Warfarin
Mention the essential lines of the treatment of:
– Iron toxicity
– Heparin toxicity
– Warfarin toxicity
Give the reason:
– vit B12 must be combined with folic acid during treatment of megaloblastic
anemia.
– Parenteral iron should be given during erythropoietin therapy.
Mention 3 differences between:
– Streptokinase and alteplase (t-PA).
– Standard heparin and low molecular weight heparin.
Mention the mechanism and management of drug-induced bone marrow
depression.
Mention the drug-drug interactions of warfarin.

226
Of each of the following questions, A. Therapy with iron should last for 6
select ONE BEST answer: months to replenish iron stores
B. Parentral iron is usually preferred than
1. All of the following lipid-lowering oral iron
drugs are correctly matched with their C. The patient is advised to take oral iron
mechanism of action EXCEPT: on an empty stomach.
A. Cholestyramine – bile acid binding D. Black stool is a strong indication for
resin discontinuation of oral iron.
B. Niacin – inhibits lipolysis E. Metabolic alkalosis is the major
C. Clofibrate – HMG-CoA reductase feature of acute iron toxicity
inhibitor
D. Gemfibrozil –activation of lipoprotein 6. Vitamin B12:
lipase A. Is normally absorbed in the upper
E. Lovastatin – inhibits cholesterol small intestine
biosynthesis B. Therapy by mouth is the first choice in
pernicious anemia
2. Niacin or nicotinic acid: C. Is good trial therapy in undiagnosed
A. Inhibits HMG-CoA reductase anemias
B. Increases plasma levels of VLDL D. Its deficiency can lead to anemia and
C. Lowers triglycerides due to its action neurological symptoms
as a vitamin. E. Is the first choice for the treatment of
D. Should not be combined with a bile aplastic anemia.
acid-binding resin
E. Inhibits the production of VLDL in the 7. Recombinant human erythropoietin
liver. has been used for the treatment of:
A. Aplastic anemia
3. Simvastatin, an HMG-CoA B. Anemia associated with renal failure
reductase inhibitor- All are true C. Megaloblastic anemia
EXCEPT: D. Sickle cell anemia
A. Lowers low density lipoprotein (LDL) E. Thalassemias
cholesterol
B. Is particularly useful in heterozygous 8. All of the following anticoagualnt
familial hypercholesterolemia drugs are correctly matched with their
C. Acts locally on HMG-CoA reductase in mechanism of action EXCEPT:
the intestine A. Warfaine – inhibits synthesis of vitamin
D. May be associated with K-dependent clotting factors in the
rhabdomyolysis liver
E. Is effective if prescribed with a bile B. Heparin – increases the activity of
acid binding resin. antithrombin III in plasma
C. Bivalirudin – selectively inhibits
4. Absorption of iron: thrombin action
A. Is greater in anemic man than in D. LMWH – selectively inhibit factor IX.
normal one E. Fondaparinux – selectively inhibits
B. Is decreased by ascorbic acid. factor X.
C. Is more efficient when it is in ferric
form 9. The anticoagulant action of heparin
D. Takes place mostly in the ileum. can be effectively antagonized by:
E. Is enhanced by co-administration of A. Vitamin C
desferroxamine B. Vitamin K
C. EDTA
5. In the treatment of iron deficiency D. Thromboplastin
anemia: E. Protamine

227
10. All the following statements about E. Inhibition of aldosterone synthesis
vitamin K are correct EXCEPT:
A. Is widely distributed in plants. 15. The lowering of plasma lipids by
B. Requires bile for its absorption lovastatin occurs because it:
C. Is necessary for the formation of factor A. inhibits HMG coenzyme A (CoA)
IX in the liver. reductase
D. Increases synthesis of prothrombin in B. binds bile acids
patients with severe liver disease. C. it inhibits VLDL secretion
E. Deficiency may occur as a result of D. stimulates HMG CoA excretion
oral antibiotics. E. increases the activity of lipoprotein
lipase
11. In thrombolytic therapy, one
statement is NOT true: 16. All of the following statements are
A. Streptokinase should be given within 6 true EXCEPT:
hrs of coronary thrombosis. A. Heparin acts both in vitro and in vivo
B. Recombinant tissue plasminogen B. Coumarin (warfarin) acts in vivo only
activator is superior to streptokinase C. Heparin can enter the placenta
C. Streptokinase is easier to use than causing fetal bleeding
urokinase since it is non-antigenic D. Coumarin (warfarin) is considered a
D. Bleeding remains the major side vitamin K antagonist
effects of all fibrinolytic drugs. E. Heparin must be administered
E. The complication of bleeding should parenterally
be treated by tranexamic acid.
17. Which of the following statements
12. The following statements about is true concerning cholestyramine?
platelet activity are correct EXCEPT: A. It inhibits free fatty-acid release from
A. Intact vascular endothelium does not adipose tissue
attract platelets because it synthesizes B. It releases lipoprotein lipase
PGI2. C. It is an anion-exchange resin that
B. TXA2 is synthesized mainly by binds bile add in the intestinal lumen
platelets. D. It blocks the final step in the formation
C. Dipyridamole reduces platelet activity of cholesterol in the body
by increasing cAMP concentration. E. When used in large doses, it
D. Clopidogrel blocks platelet ADP decreases serum cholesterol,
receptors. triglycerides, and phospholipids,
E. Aspirin is a reversible inhibitor of possibly via an effect on synthesis.
TXA2.
18. Which statement regarding heparin
13. Which of the following is NOT true:
pharmacological agents activates A. It inhibits the action of thrombin.
plasminogen after binding to fibrin? B. Is less effective in-patients with
A. Streptokinase inherited or acquired deficiency of
B. Urokinase antithrombin III.
C. Alteplase (tPA) C. Is monitored in the laboratory by
D. Antiplasmin measurement of activated partial
E. Aminocaproic acid thromboplastin time (APTT).
D. It inhibits antithrombin III.
14. The major untoward effect of E. Is reversed by protamine sulfate.
heparin is:
A. Bleeding 19. All of the following are recognized
B. Crossing the placenta adverse effects of heparin EXCEPT:
C. Elevation of hepatic transaminases A. Osteoporosis.
D. Allergic reactions B. Alopecia.

228
C. Thrombocytopenia. 25. Select the most appropriate
D. Allergy hypolipidemic drug for a patient with
E. Fetal cleft palate. raised LDLcholesterol level but normal
triglyceride level:
20. All of the following inhibit platelet A. Simvastatin
activation and/or aggregation EXCEPT: B. Clofibrate
A. Warfarin C. Probucol
B. Eptifibatide D. Nicotinic acid
C. Ticlopedine E. Thyroxin
D. Prostacyclin (PGI2).
E. Dipyridamole. 26. Combined therapy with
dipyridamole and warfarin is
21. A 22-year-old woman with deep recommended in subjects with the
vein thrombosis in her second following:
trimester of pregnancy, was treated for A. Risk factors for coronary artery
7 days with intravenous unfractionated disease
heparin. Which one of the following B. Prosthetic heart valves
drugs would be most appropriate C. Chronic arteriovenous shunts for
outpatient follow-up therapy for this repeated haemodialysis
patient? D. Pulmonary embolism
A. Warfarin. E. Deep vein thrombosis
B. Aspirin.
C. Alteplase.
D. Unfractionated heparin.
E. Low-molecular-weight heparin Answers
(LMWH).
1C 7B 13 C 19 E 25 A
22. Protamine sulfate administered 2E 8D 14 A 20 A 26 B
intravenously to stop heparin-induced 3C 9E 15 A 21 E
bleeding. The mechanism of protamine 4A 10 D 16 C 22 E
is: 5A 11 C 17 C 23 B
A. Degrades the heparin. 6D 12 E 18 D 24 C
B. Inactivates antithrombin.
C. Activates the coagulation cascade.
D. Activates tissue-plasminogen
activator.
E. Ionically combines with heparin.

23. The most important complication


of streptokinase therapy is:
A. Hypotension
B. Bleeding
C. Fever
D. Anaphylaxis
E. Thrombosis

24. Severe cases of bleeding due to


fibrinolytic agents are treated with:
A. Aspirin
B. Heparin
C. Aminocaproic acid
D. Vitamin K
E. Thromboplastin

229
230
Part 1
1: Age
ents use
ed to tre
eat coug
gh

█ Bas
sic inform
mation

 Couugh is one of the mos st commo n symptom ms seen in clinical prractice.


 Thee initial stimulus arisses in thee bronchia
al mucosa,, where irrritation results in
stim
mulation of
o “Cough” recepto ors, which are specialized sstretch rec ceptors.
Affeerent impulses reachh the coughh center in
n the medu
ulla via thee vagus nerve and
trigg
ger a coug
gh reflex.
 Cou ugh may be acute (<3 3 weeks), ssubacute (3-8
( weeks
s) or chronnic (>8 wee
eks).

Causes
s of cough
h:
 Acu
ute cough: respiratory infection
n is the mo
ost commo
on cause.
 Chrronic coug
gh:
 Upper airwway coug gh syndrom me (post--nasal drip
p): due to
o allergic rhinitis,
c
chronic sin
nusitis or to
onsillitis. Itt is the mo
ost commo
on cause off chronic cough.
c
 Bronchial asthma: th nd
he 2 mostt common cause.
 G
Gastoesop phageal reflux diseasse (GERD)..
 O
Other causses: ACEIs s, lung tum mors, CHF.

█ MAN
NAGEMEN
NT OF COU
UGH

■ Speecific trea atment: directed


d tto the ca
ause of cough
c e.g
g. antibiottics for
resp
piratory inffections.
■ Non
n-specific
c treatmen
nt:
 A ves: are us
Antitussiv sed to stopp dry cough h.
 M
Mucolyticcs and ex xpectoran nts: are used in prroductive cough to liquefy
b
bronchial secretions
s and facilittate their re
emoval.

231
█ COU
UGH SUPP
PRESSAN
NTS (ANTIT
TUSSIVES
S)

Definittion: they are


a drugs that
t reduce
e the frequ
uency or in
ntensity of coughing.

▌Perip
pheral an es: they ↓ afferent im
ntitussive mpulses of the coughh reflex.

■ Ste
eam inhala
ation with menthol o
or tincture
e benzoin compoun
nd
 It iss one of the best and
d easy wayys to relievve acute cough.
c Inhaaling water steam
with h or withouut medications (e.g. m
menthol) helps
h flush out mucu s and moisturizes
dry,, irritated air
a passagees. The effficacy of ad
dded mediication is nnot proved.

■ Ben
nzonatate
 It iss a glycero
ol derivativ
ve chemica
ally related
d to the lo
ocal anesthhetic proc caine. It
dep presses peeripheral coough recep
ptors at the
e lung by lo
ocal anestthetic effec
ct.

▌Centtral antitu
ussives: they inhibiit cough ce
enter in the
e medulla.

■ Opiioids: Cod
deine and hydrocod
done
 Theey are natu
ural derivattives of mo
orphine. Th
hey directly
y inhibit thhe cough center
c in
the medulla at
a doses that are lowe er than tho
ose needed d for analg gesia.
 Theey are geneerally not recommend ded becauuse of adveerse effectts.

Advers
se effects
 Droowsiness and
a constip pation.
 Druug depende ence if use
ed for long duration.
 Resspiratory depression
d in large d
doses. Chilldren
lesss than 5 years old d are morre sensitivve to
resppiratory depression
d n so, co odeine is not
recoommended d to childre
en < 5 yeaars.

■ Opiioid isome
ers: Dextro
omethorp
phan
 It iss synthetic L-isomer of o opioids..
 It has selectivve central antitussive e action with
w very few f opioidd effects (i.e. less
add diction liability, analgesic action
n, respirato
ory depres ssion, or co
onstipationn).
 High doses ca an cause neuropsych
n hiatric effe
ects e.g. se
edation and d hallucina
ations.

COLYTICS
█ MUC S

Definittion: they are agents that red


duce viscosity of res
spiratory ssecretions without
increassing their amount.
a

232
■ Bromhexine
 Bromhexine acts on the mucus at the formative stages within the mucus-
secreting cells. It disrupts the structure of acid mucopolysaccharide fibers in
mucoid sputum and produces a less viscous mucus, which is easy to expel.
 Because bromhexine can disrupt the gastric mucosal barrier, it should be
avoided in patients with peptic ulcer.

■ Ambroxol
 It stimulates synthesis and release of surfactant by type II pneumocytes.
Surfactant acts as an anti-glue factor by reducing the adhesion of mucus to the
bronchial wall.

■ N-Acetylcysteine and carbocyseine


 Acetylcysteine has free sulfhydryl (-SH) groups that break disulfide bonds in
mucus and reduces its viscosity.
 Unlike acetylcysteine, carbocysteine does not act directly on mucus but rather, it
affects the structural components of mucus.

Therapeutic uses of mucolytics


 Chronic respiratory diseases: chronic bronchitis, emphysema, bronchiectasis
and cystic fibrosis.
 Post-operative and post-traumatic pulmonary complications.
 Chronic sinusitis and chronic otitis media.
 N.B. Intravenous N-acetylcysteine is used as an antidote for acetaminophen
(paracetamol) toxicity (quite apart from its mucolytic activity).

█ EXPECTORANTS

Definition: drugs that increase water content and amount of the respiratory
secretions. This action facilitates the removal of respiratory secretions.

Adequate hydration is the single most important measure to encourage


expectoration. Using an expectorant in addition may produce the desired result.

■ Potassium iodide
 Iodides accumulates in the bronchial glands and stimulate secretion of low
viscosity watery mucous.
 The use of iodides is accompanied by wide range of side effects including
unpleasant (metallic) taste, increase lacrimal and salivary secretions, painful
salivary swelling, hypothyroidism, and skin eruptions (rash).

233
■ Gua
aifenesin
 It iss one of the mostt widely u used overr-the-countter (OTC) expectorants. It
incrreases bro
onchial fluid
d secretion ear mechanism.
ns by uncle

■ Oth
her expecttorants
 Man ny other traditiona al expecttorants (e e.g., ammmonium cchloride, tincture
ipec
cacuanha, herbal re emedies) a are found in numerrous OTC cough mixtures.
m
Theeir efficacy is doubtfu
ul, particula
arly in the dosages of
o most preeparations.

Part 2
2: The
erapy off bronch
hial asth
hma

ma is a chrronic inflam
Definittion: Asthm mmatory disorder
d of the airwayys causing
g airflow
obstrucction and recurrent episodes of wheezing, breath hlessness,, chest tig
ghtness,
and coughing.

Pathog
genesis
Frequeent exposure to allergic stim muli
causess infiltration of the bronchial
b w
wall
by acu ute and chronic inflammat
i tory
cells. These cells re elease m many
inflammmatory cy ytokines e.g.
e histam mine,
adenossine, PGs, LTs, PAF F, etc. lead
ding
to:
 Hypertroph hy of airway
y smooth mms
 Increased mucus se ecretion tha
at is difficu
ult to expel.
 Congestio on and ede ema of the respiratory y mucosa.

Predisposing fac
ctors
 Rec
current re
espiratory n: the mo
y infection ost
impportant facttor.
 Gennetic facttors: asthm ma occurss in familiies
with
h positive history
h of allergy.
a
 Psyychologica al factors: are pressent in 40 %
of a
asthmatics.

Clinica
al presenta
ation
■ Chrronic asth hma
 Theere is dysp pnea, ches st tightnesss, coughing
(parrticularly at night), and
d expiratoryy wheezing
g.

234
 Patients can present with mild intermittent symptoms that require no
medications or only occasional short-acting inhaled β2-agonists to severe
symptoms despite multiple medications.

■ Acute severe asthma


 Uncontrolled asthma can progress to an acute state in which inflammation,
airway edema, mucus accumulation, and severe bronchospasm that is poorly
responsive to bronchodilator therapy.
 There are severe dyspnea, inspiratory wheezing, cyanosis; the patient may be
able to say only a few words with each breath.

█ MANAGEMENT OF ASTHMA

█ BRONCHODILATORS

■ β-adrenergic agonists.
3 groups of
■ Muscarinic receptor blockers: ipratropium.
bronchodilator drugs:
■ Xanthines: theophylline.

▌β-adrenergic agonists

Classification
■ Non-selective β-receptor agonists: e.g. adrenaline, isoprenaline and
ephedrine. They are rarely used as bronchodilators.

■ Selective β2 agonists:
Short acting: salbutamol, terbutaline, fenoterol (duration 3-4 hrs).
Long acting: salmeterol and formoterol (duration 12 hrs).

Mechanism of action
 Stimulation of bronchial β2 receptors → ↑ cAMP → bronchodilatation.
 Stimulation of β2 receptors in the mast cells → ↓ histamine release
 They ↓ bronchial inflammation and wall edema.

Administration
 In acute asthma: short acting β2 agonists are given by inhalation or i.v infusion.
 In chronic asthma: long acting β2 agonists are given orally or by inhalation.

Adverse effects
 Tachycardia and arrhythmia due to:
 Reflex from hypotension (caused by VD of sk ms BV).

235
 Directt activation
n of cardiac
c β1 (due to
o loss of se
electivity inn high dos
ses).
 Trem
mors of sk
keletal ms and nervou usness.
 Tole
erance with prolonge ed use (reqquiring tem
mporary ces ssation of the drug).
 Hyp +
pokalemia (due to shift of K froom blood tot cells).

▌Musc
carinic an
ntagonistts: Ipratro
opium brromide

 Atro
opine bloc
cks M3 rec
ceptors in a
airway ms
s leading to
o bronchod
dilatation through
t
uno
opposed β2 action, bu
ut it is not used for treatment
t ma because
t of asthm e:

 It is non-selective M3 blocker lleading to many sidee effects e..g. dry mouth and
urine reten
ntion.
 It can crosss BBB andd causes CCNS side effects
e e.g. sedation..
 It causes excessive
e dryness off bronchiall secretions making iit difficult to
t expel

 Ipra
atropium is
i more prreferred th
han atropiine because:
 It is more selective
s muscarinic
m blocker th
han atropinne.
 It is quaterrnary ammmonium com mpound th hat can't cross BBB.
 Does not cause
c exce
essive dryn
ness of bro
onchial seccretions.

 Ipra
atropium is not sufficient
s alone fo
or bronch on. It is usually
hodilatatio
com
mbined with β2 agonists to gett synergistiic effect.

▌Meth
hylxanthin
nes

Classiffication
 Nattural: e.g. caffeine, th
heophylline
e, and theo obromine.
 Sem
misynthetiic: e.g. am minophyllinee (salt of th
heophylline
e).

anism and pharmac


Mecha cological e
effects
 Xan
nthines are
e adenosine
e A recepttor antago
onists lead
ding to:
 Bronchodiilatation (block broncchoconstrictor effect of adenossine).
 C
CNS stimuulation (blo
ock the inh ibitory effe
ect of aden
nosine on tthe CNS)
 ↓ mediatorr release frrom mast c
cells.
 ↑ AV conduction.

 Theey inhibit phosphodiestrase ennzyme (PD DE4) → ↑ cA


DE3 and PD AMP leading
g to:
 Bronchodilatation (PD DE4).
 ↑ cardiac contractility
c y and arrhyythmogeniic action (PPDE3).
 V
VC of cereebral and VD
V of perippheral vesssels.
 Relaxationn of most smooth
s mu T, biliary, ureteric, etcc).
uscles (GIT
 Diuresis.

236
Therap
peutic use
es

■ Res
spiratory uses:
u

 Acuute severe asthma: aminophyllline may be given


by slow i.v. infusion 250
2 mg i.vv. (at least over 15
minnutes to avoid sy yncope orr cardiac arrest),
follo
owed by maintenanc
m ce i.v infusiion of 0.7 mg/kg/h.
m
 Chrronic broncchial asthm
ma: sustain ned releas se tablets
of theophylline 100 0-300 mg g/day orr rectal
supppository of
o aminoph hylline can be given.

■ CNS
S uses:
 To rreverse CN
NS depression.
 To ddelay physsical and mental
m fatig
gue (caffein
ne).
 Treaatment of migraine (caffeine
( + ergotamine): to increase VC of cerebra
al blood
vessels and inncrease abbsorption oof ergotam
mine from GIT.
G
 Neoonatal apnea syndrom me: caffeinne is the ag
gent of cho
oice.

■ CVS
S uses:
 Acu
ute pulmon
nary edema
a due to ac
cute left sided HF.

Advers
se effects
 CNSS: irritabilitty, headac
che, insom nia, nervou
usness and d convulsioons.
 CVSS: palpitations, tachy ycardia, an
nd arrhyth
hmias. Rap ection can
pid i.v. inje n cause
hyp
potension, syncope and a cardia ac arrest.
 GIT
T: nausea, anorexia, hyperacid
h dity and rea
activation of peptic uulcer.

Precau
utions
 Aminophyllinee must be
e given byy slow i.v.ii. (at least over 15 minutes to
o avoid
sud
dden synco ope or card
diac arrestt).
 Use
ed with caaution in se
evere card diac diseasse, severe hypoxem
mia, and renal and
hep
patic disease and in elderly
e andd neonates s.
 The
ey should not
n be give en to patiennts with pe
eptic ulcer.

Drug in
nteraction
ns
 Enzzyme inhib
bitors (cim
metidine an omycin) →
nd erythro
↑ s hines and ↑ their
serum levvels of methylxanth
m
toxiicity (arrhyythmia).
 Enzzyme inducers (sm moking, riffampin) → ↓ their
seru
um levels and a reducee their effe
ect.

237
█ REDU
UCTION OF BRONCH
HIAL INFLA
AMMATION
N

▌Cortiicosteroids

Mecha
anism of action
a
Corticoosteroids can efffectively ↓ bron nchial
inflammmation and d hyperrea activity thrrough:
 The ey inhibit B cell function → ↓ an ntigen-antibbody
reacction.
 The ey inhibit T cell functtions → ↓ mediators s and
cytookine releaase.
 The ey inhibit macrophag
m ge activityy and stab bilize
lyso
osomal me embranes.
 The ey inhibit mast
m s → ↓ histtamine release
cells
andd capillary permeabili
p ty.
 The ey inhibit ph
hospholipa ase A2 enzzyme →↓ synthesis
s of
o PGs & LT
Ts.
 Cortticosteroid ds cause up-regulatio on of β2 re
eceptors.

Use in asthma
 Acuute severe asthma: hydrocorti
h isone 2000 mg givenn by i.v. injjection. It may be
repeeated everry 4 h if necessary.
 Chrronic bronchial asthm ma: oral p
prednisoloone 20 mg g/d or dexxamethazo one 4-8
mg//d, or by in
nhalation (bbeclomethhasone).
 Inhaaled cortiicosteroid ds (e.g. be
eclometha azone): shhould be cconsidered d the 1st
cho
oice in newwly diagnos sed asthma a. They ha
ave the follo
owing advvantages:

 Their efficacy
e is equal
e to in
nhaled β2 agonists.
a
 Minimaal systemic c side effe
ects.

Advers
se effects
 If used systemically: …..see
… enddocrine cha
apter.
 If us halation: → oropharyyngeal candidiasis. It could bee avoided by:
sed by inh
 Mouth wa ash and gaargle after e
each inhala
ation.
 Candida infection ca an be treaated by nys uthwash o
statin mou or amphottricin-B
lozenges.

Precau
utions
 Theey must be e withdraw wn gradua ally to avoid
d acute Ad
ddisonian ccrisis.
 Dieet should be +
b rich in K and prot eins and lo ow in NaCl and carbbohydrates s.
 Con ntinuous check
c for any
a increaase in weight, edema, sugar in uurine or BP
P.
 If the patient develop ps acute infection, he mustt be treatted by ad dequate
antiibiotics witth decreassed dose o
of steroid.

238
█ PROPHYLACTIIC TREATM
MENT

The following classes of drugs aree not bron


nchodilatorrs but aree used to reduce
frequen
ncy of asth
hma exace
erbations.

▌Leuk
kotriene inhibitors
i s:

Zafirlukast and monteluka


m ast
 Theey block le eukotrienee (LTD4) re eceptors.
 Zafiirlukast is given
g twice
e daily butt monteluk
kast is given once da ily.

Zileuto
on
 It in
nhibits 5-lip
pooxygena
ase enzymee →↓ leuko otriene syn
nthesis.
 Zileeuton is microsomal
m P450 inh ibitor and can inhib
bit the mettabolism of
o many
drugs e.g. wa
arfarin and theophyllin
ne.

 In clinical trials, leu


ukotriene inhibitors s reduced d frequenncy of asthma
exa
acerbationss as equal to corticossteroids.
 Monntelukast is approved to controol asthma in children
n.

▌Mastt cell stab


bilizers:

Cromo
olyn and nedocromiil
 Cro
omolyn sod
dium (diso
odium crom e) and ned
moglycate docromil aare poorly soluble
drugs and sho ould be giv ven as miccronized po owder thro
ough spinhhaler.
 Theey inhibit mast
m cell de
egranulationn by uncleear mechannism probaably by alte
ering the
funcction of chloride cha annels in th
he mast celll membranne.
 The ey were ussed to reduce freque ency of atttacks
(i.e.. prophyla actic trea atment) in n some alllergic
connditions e.g g. bronchia al asthma, allergic rh
hinitis,
andd allergic conjunctivittis.
 The eir use now w become es very li mited and d has
bee en largely replaced
r by leukotrie
ene inhibito
ors.
 Adv verse effe ects occurr at site off administration
e.g.. local irrita
ation of thhe throat, cchest tighttness,
andd bronchosspasm.

Ketotiffen
 It iss a 2nd geneeration anttihistaminee and a ma
ast cell stab
bilizer.
 It iss used as a eye drops to trreat allerg gic conjunctivitis, and orally
y as a
prophylactic treatment
t in
i bronchia
al asthma and other seasonal aallergies.

239
█ OTHER DRUGS USED IN TREATMENT OF BRONCHIAL ASTHMA

■ Expectorants and mucolytics: to reduce mucus viscosity.

■ Mixture of oxygen (20%) and helium (80%): Heliox


 Helium is an inert gas. Its low density facilitates O2 diffusion through
obstructed airways → ↓ work of breathing.

■ Anti-IgE monoclonal antibodies: Omalizumab


 Omalizumab is a new drug that inhibits the binding of IgE to mast cells and
prevents mast cell degranulation. It may also inhibit IgE synthesis by B cells.
 It reduces frequency and severity of asthma even when the dose of
corticosteroids is reduced.

█ Stepwise approach for treatment of chronic asthma

The management of stable asthma is now well established with a stepwise


approach:

Step 1: Inhaled short-acting B2 agonist (SABA) as required


Step 2: Add inhaled steroid at 400 mcg/day
Step 3:  Add inhaled long-acting B2 agonist (LABA)
 Still inadequate: continue LABA and increase inhaled steroid dose to
800 mcg/day
Step 4:  increase inhaled steroid up to 2000 mcg/day
 Add a 4th drug e.g. Leukotriene antagonist, SR theophylline, oral B2
agonist tablets.
Step 5:  Use oral steroid tablet in lowest dose in addition to the high dose
inhaled steroid (2000 mcg/day).
 Refer the patient for specialist care.

█ Treatment of acute severe asthma (status asthmatics)

Definition: acute severe asthma (status asthmatics) is a condition in which


bronchodilators are poorly effective in relieving the attack.

Management
 Hospital admission.
 Oxygen: to maintain peripheral capillary O2 saturation (SpO2) between 94-98%.

240
 β2 a
agonists: use high dose
d by inh
halation.
 Add
d ipratropiium bromide by inh halation to the
β2 a
agonists.
 Hyd one: 200 mg
drocortiso m i.v. / 6hss.
 Con
nsider a single dose of IV magnesiium
sulpphate (1.2-2 g over 20
2 min) to patients who
w
faile
ed to respond to ini tial inha
aled
bronchodilatoor therapy.
 Corrrection of acidosis and dehyd
dration by i.v.
fluid
ds.

Part 3
3: Oxy
ygen therapy

█ Bas
sic inform
mation

▌Arteriial blood gases


g (AB
BG): norma
al values and
a interp
pretation

Param
meter Normal range
r IInterpreta
ation
pH 7.34 – 7.4
44 s ↓ binding of O2 to H
 Acidosis Hb (tissue hypoxia)
h
s ↑ binding
 Alkalosis g of O2 to H
Hb.
Arterial O2 94 –100 %  At open air, SaO2 must be > 94% and PaO2
P
saturatiion at >80 mmm Hg (or 10..6 kPa).
ambien nt air
 Hypoxeemia means s SaO2 < 990% and/or PaO2
SaO2
<60 mmm Hg (or 8 kPa).
k At thiss level,
Arterial O2 80 – 100 mmHg supplem
mental oxyggen should be administered.
partial p
pressure (10.6 –13 kPa)  At a PaO
O2 < 30 mm
mHg, the p
patient is at risk of
PaO2 death an
nd must be
e oxygenateed immediately.
Arterial CO2 35 – 45 mmHg
m  High vallue (respira
atory acidoosis) denottes
partial p
pressure (4.5 – 6 kPa) hypovenntilation pro
oblem (e.g.. COPD).
PaCO2 2
 Low valu
ue (respira osis) denottes
atory alkalo
hyperve
entilation prroblem.

HCO3 22 – 26 mmol/L
m  Low valuue denotess metabolic
c acidosiss.
 High vallue denotes
s metaboliic alkalosis.

N.B.
 Hyp
poxia: low oxygen level at the ttissues.
 Hyp
poxemia: low
l oxygen he arterial blood.
n level in th

241
█ OXY
YGEN THE
ERAPY

oints
Key po
 Oxyygen is stored
s eith
her in liq
quid or
gasseous form m in metal cylinders (green
tankks). Oxyge en concenntrators arre also
avaailable for long term home us e; they
extrract oxyge en from air
a using e electric
currrent.
 Oxyygen is co ommonly delivered to the
patiient through nasal cannula o or face
massk.
 Diaggnosis of hypoxemiia is done e either
by pulse oxim meter (eas
sy) or by arterial
bloood gas analysis (see tab ble for
refe
erence rang ges).
 Dysspnea doe es not neccessarily in
ndicate
hyppoxemia. Severe
S hypoxemia c can be
pressent withoout dyspnea and vicee versa.

Indicattions of ox
xygen therapy
Oxygen
n should not be used
d routinely unless the
ere is evide
ence of hy poxia.
 Acu ons: acute
ute hypoxiic conditio e MI, acute
e pulmonarry edema, carbon mo
onoxide
poissoning, ca
ardiac and
d respirato
ory arrest, low COP and m etabolic acidosis
a
(bic
carbonate <18
< mmol/L).
 Chrronic hypo
oxic condiitions (long
g-term oxy
ygen thera COPD and CHF.
apy): e.g. C

Monito
oring and stopping
 ABG
G or oxime
etry should
d be done
e within 2 hours
h of starting
s oxyygen thera
apy and
oxyygen delive
ery is adjus
sted accorddingly.
 Oxyygen should be stop pped when n SaO2 > 92% while e the patieent is breatthing at
roomm air.
 For patients with
w COPD D, the oxyg
gen saturattion target should noot exceed 92% to
avo
oid CO2 rettention (see below).

Dange
ers, advers
se effects and preca
autions
 Fire
e and explo afety meas ures are re
osions: (sa equired).
 Irrita
ation of no
ose, pharyn
nx and trac
chea.
 Inhiibition of mucociliary
m y function → ↓ trachea
al outflow of
o mucouss.
 CO2
2 toxicity:

242
 In patients with COPD who receive supplementary O2, CO2 retention can
occur through 2 mechanisms:
 Increase O2 saturation of blood reduces deoxygenated Hb which carries
CO2 in blood (in the form of bicarbonate).
 In patients with COPD, hypoxia is the main stimulant of respiration
(hypoxic drive). If 100% O2 was administered, hypoxia is corrected and
the hypoxic drive is lost leading to CO2 retention.
 CO2 retention can lead to headache, drowsiness (narcosis) or even death.

 Prevention: In people with COPD, CO2 toxicity can be prevented by careful


control of the supplemental oxygen. Just enough oxygen is given to maintain
an oxygen saturation of 88 - 92%.

 Retrolental fibroplasias: high concentration of O2 in neonates (<32 weeks)


stimulates fibrous tissue formation posterior to the eye lens and permanent
blindness. So, O2 should be used only when needed and its concentration must
not exceed 35-40% in premature infants.

 Pulmonary oxygen toxicity: high O2 concentrations (> 60%) given for >48 hours
can damage alveolar membrane and cause alveolar edema.

243
244
Review Questions

 Discuss different drugs used in the treatment of cough.


 Discuss the different lines of treatment of bronchial asthma.
 Discuss pharmacology of xanthenes: classification, mechanism of action,
pharmacologic effects, therapeutic uses, and side effects.
 Mention uses of mucolytics.
 Mention precautions during the use of methylxanthines.
 Mention precautions during the use of corticosteroids in BA.
 Mention the main side effects of oxygen therapy.
 Mention the main side effects of selective beta-2 agonists.
 Mention the main lines of treatment of status asthmaticus.
 Mention 3 drugs causing bronchoconstriction and mention the mechanisms.

Mention the pharmacodynamic principles underlying the use of:


 Beta-2 agonists in bronchial asthma.
 Methylxanthines in bronchial asthma.
 Montelukast for prophylaxis of bronchial asthma.
 Acetylcystiene in chronic bronchitis.

Mention the rationale of the following combinations:


 Oxygen with helium in chronic obstructive lung disease.
 Corticosteroids with beta-2 agonists in bronchial asthma.

245
Of each of the following questions, A. Sodium benzoate
select ONE BEST answer: B. Trypsin
C. Ambroxol
1. Which of the following mucolytics D. Tincture Ipecacucnha
act by breaking disulfide bonds of E. Iodides
proteoglycans, which causes
depolymerization and reduction of 7. Which one is considered the major
viscosity of sputum? side effect of Theophylline:
A. Acetylcysteine A. Bradycardia
B. Iodides B. Arrhythmia
C. Trypsin C. Depression of respiratory center
D. Water vapor D. Elevation of the arterial blood pressure
E. Licorice E. Depression of mood

2. Which of the following drugs is a 8. Choose the drug belonging to class


methylxanthine used in the treatment of mast cell stabilizers:
bronchial asthma? A. Zileuton
A. Salbutamol B. Zafirlucast
B. Ipratropium C. Epinephrine
C. Theophylline D. Salbutamol
D. Pentoxifylline E. Sodium cromoglycate
E. Dipyridamole
9. Which of the following drugs can
3. The mechanism of methylxanthines disrupt mucus synthesis within
action in bronchial asthma is: bronchial glandular cells?
A. Inhibition of the enzyme A. Potassium iodide
phosphodiesterase B. Acetylcystiene
B. Beta2 -adrenoreceptor stimulation C. Bromhexine
C. Inhibition of the production of D. Trypsin
inflammatory cytokines E. Ambroxol
D. Inhibition of M-cholinoreceptors
E. Inhibition of leukotriene receptors 10. Which of the following is not a
recognized action of terbutaline?
4. Which of the following drugs can A. Diuretic effect.
decrease the effect of leukotrienes in B. Cardiac arrhythmia.
the inflamed tissue? C. Skeletal muscle tremors.
A. Zileuton D. Smooth muscle relaxation.
B. Montelukast E. Tachycardia.
C. Theophylline
D. Loratidine 11. Disodium cromoglycate has as its
E. Cromoglycate major action:
A. Smooth muscle relaxation in the
5. Which of the following drugs can bronchi.
decrease the concentration of B. Stimulation of cortisol release by the
leukotrienes in the inflamed tissue? adrenals.
A. Zileuton C. Block of calcium channels in the
B. Montelukast lymphocytes.
C. Theophylline D. Block of mediator release from mast
D. Loratidine cells.
E. Cromoglycate E. Block of cAMP synthesis in the
basophils.
6. Indicate the expectorant which
increase lung surfactant:

246
12. The following may induce C. Noscapine
bronchospasm EXCEPT: D. Dextromethrphane
A. Propranolol. E. Ticture benzoin Co.
B. Morphine.
C. Aspirin 18. In designing a dosage regimen for
D. Captopril aminophylline all are true EXCEPT:
E. Theophylline. A. Children (ages 3-12 y) have faster
clearance of this drug than adults or
13. Which of the following drugs given infants
for cardiovascular indications might B. Patients with history of cardiac
complicate the management of asthma disease should be carefully monitored,
in the same patient? because aminophylline may induce
A. Hydralazine. ventricular extrasystoles and other
B. Verapamil. arrhythmias
C. Nitroglycerine C. Excessive doses result in skeletal
D. Quinidine. muscle tremors and CNS excitation.
E. Propranolol. D. The use of tobacco by the patient
inhibits the metabolism of
14. Methylxanthines: aminophylline.
A. Cause decreased gastric acid E. Erythromycin increases the serum
secretion. levels of theophylline.
B. Are potent diuretics in most patients.
C. Cause increase in mental alertness 19. Ipratropium bromide is:
and even insomnia in some patients. A. Derivative of phenylephrine.
D. Are potent vasoconstrictors. B. Less effect on sputum viscosity.
E. Cause fatigue of respiratory muscles C. Tertiary amine
D. Given by inhalation, oral and
15. Theophylline is therapeutically parenteral routes
useful in asthma because: E. More effective bronchodilator than B2
A. It has no significant effects on the agonist
heart.
B. It stimulates the respiratory center. 20. Disodium cromoglycate, all are true
C. It increases mediator release from EXCEPT:
mast cells. A. Is well absorbed from the GIT.
D. It is a potent stimulant of beta- B. Stabilizes mast cells and prevent their
receptors. degranulation.
E. It increases cAMP without C. Is inhaled as a powder for prophylaxis
desensitizing receptors in bronchioles of bronchial asthma.
D. Can cause bronchconstriction during
16. In status asthmaticus, all are true administration.
EXCEPT: E. Can be used in the treatment of
A. Salbutamol and ipratropium may be allergic rhinitis.
given by nebulized aerosol
B. IV aminophylline is useful. 21. All of the following drugs are
C. Hydrocortisone is preferred to mucolytics EXCEPT:
prednisolone for initial therapy. A. Bromhexine
D. Humidified oxygen should be given. B. Iodides
E. Morphine provides useful sedation C. Acetylcysteine
D. Ambroxol
17. All of the following drugs are E. Ketotifen
central antitussives EXCEPT:
A. Codeine
B. Pholcodeine

247
22. Oropharyngeal candidiasis is a 27. Drugs that can dilate bronchi
common side effect of which of the during an acute asthmatic attack
following: include all of the following except
A. Inhaled beta-2 agonists A. Epinephrine
B. Inhaled ipratropium B. Terbutaline
C. Inhaled corticosteroids C. Nedocromil
D. Inhaled disodium cromoglycate D. Theophyline
E. Oxygen E. Ipratropium

23. Oropharyngeal candidiasis could 28. Which of the following is a


be treated by: prophylactic agent that appears to
A. Liquorice lozenge stabilize mast cells?
B. Amphotericin-B lozenge A. Aminophyline
C. Ampicillin B. Cromolyn
D. Aminoglycosides C. Epinephrine
E. Steam inhalation D. Ipratropium
E. Metaproterenol
24. Which of the following statements
is NOT true: 29. Which of the following has
A. Antitussives decrease the amount and overdose toxicity that includes
liquefy bronchial secretions. insomnia, arrhythmias, and
B. Peripheral antitussives are used when convulsions?
the cough arises above the larynx A. Aminophylline
C. Mucolytics used in the treatment of B. Cromolyn
productive cough to liquefy copious C. Epinephrine
bronchial secretion. D. Ipratropium
D. Iodides can be used as expectorant E. Metaproterenol
and mucolytic
E. Inhalation of water vapor or adequate 30. Which of the following is a very
intake of water is an excellent long acting β2 – selective agonist that
mucolytic & expectorant. is used for asthma prophylaxis?
A. Aminophyline
25. Combination of B2 agonists and B. Cromolyn
ipratropium can lead to: C. Epinephrine
A. Antagonistic effect. D. Ipratropium
B. Additive effect. E. Salmeterol
C. Potentiating effect.
D. Synergistic effect. 31. In the emergency department, the
E. No effect preferred first-line therapy for asthma
exacerbation is
26. The Symptoms of allergen- A. Theophyline
mediated asthma result from which of B. A beta2-agonist
the following? C. A corticosteroid
A. Increased release of mediators from D. Cromolyn sodium
mast cells E. An antihistamine
B. Increased adrenergic responsiveness
of the airways 32. A drug administered by inhalation
C. Increased vascular permeability of of powder as a prophylactic for asthma
bronchial Tissue A. Ephedrine
D. Decreased calcium influx into the mast B. Disodium cromolyn
cells C. Isoproterenol
E. Decreased prostaglandin production D. Ocytriphylline
E. Epinephrine

248
33. When administering oxygen to a
hypoxemic patient with COPD, the
target oxygen saturation level would
be:
A. 94 – 98%
B. 92 – 94%
C. 88 – 92%
D. 60%
E. 100%

34. Regarding oxygen therapy, one


statement is true:
A. Severe dyspnea does not necessarily
indicate hypoxia.
B. Oxygen should be started immediately
to all acutely ill patients.
C. Oxygen should be stopped as soon as
oxygen saturation reaches 88% in
room air.
D. Acidosis increases binding of oxygen
to hemoglobin.
E. Oxygen therapy reduces mortality in
acute myocardial infarction.

35. Which of the following drugs given


for cardiovascular indications might
precipitate bronchospasm in asthmatic
patients?
A. Hydralazine.
B. Verapamil.
C. Nitroglycerine
D. Quinidine.
E. Adenosine.

Answers

1A 7B 13 E 19 B 25 D 31 B
2C 8E 14 C 20 A 26 A 32 B
3A 9C 15 E 21 E 27 C 33 C
4B 10 A 16 E 22 C 28 B 34 A
5A 11 D 17 E 23 B 29 A 35 E
6C 12 E 18 D 24 A 30 E

249
Copyrighhts © 2016 by
b the Deparrtment of Clin
nical Pharma
acology at Fa
aculty of Meedicine, Mnas
soura
Universitty, Egypt.

Previouss editions copyright © 20


015, 2014, 20
013, 2012, 2011, 2010, 2009,
2 2008, 22000 by the
Departm
ment of Cliniccal Pharmacoology at Facu cine, Mnasou
ulty of Medic ura Universitty, Egypt.

No part of this book may be reprroduced or d distributed in


n any form orr by any meaans, or stored
d in a
databasee or retrievall system, witthout the prio
or written pe
ermission of the
t copyrighhts owner,
Departm
ment of Cliniccal Pharmaco ology at Facuulty of Mediccine, Mansouura Universitty.

This is a copyrightedd work and is s protected b


by the Egypttian Intellectu
ual Property Law 82 of 2002. Use
of this w
work is subjec
ct to this law w. The Departrtment of Clin
nical Pharmaacology at MMansoura Fac culty of
Medicine e reserves alll rights in an
nd to the worrk.

2000 ‫سنة‬
‫ لس‬1456 :‫رقم اإليداع بدار الككتب‬
‫م‬
2000/9/6 ‫بتاريخ‬
Preface

C
linical training for undergraduate students often focuses on diagnostic rather than
therapeutic skills. Sometimes students are only expected to copy the prescribing
behavior of their clinical teachers, or existing standard treatment guidelines, without
explanation as to why certain treatment is chosen. Books may not be much help either.
Pharmacology reference works and formularies are drug-centered, and although clinical
textbooks and treatment guidelines are disease-centered and provide treatment
recommendations, they rarely discuss why these therapies are chosen. Different sources
may give contradictory advice.
This book in primarily intended for under graduate medical students who are about to
enter the clinical phase of their studies. It will provide step by step guidance to the process
of rational prescribing together with many illustrative examples. It teaches also skills that are
necessary throughout a clinical career. Postgraduate students and practicing doctors may
also find it a source of straightforward information.
I wish to acknowledge the ongoing efforts of my contributing authors, and we are deeply
grateful to all those who have with such good grace given us their time and energy to supply
valuable facts and opinions, they principally include:

 Prof. Hussein El-Beltagi who took over the preparation of all books since the 1st edition
in 1995 including the revision process, printing control, distribution and selling control.
 Assist. Prof. Mohamed-Hesham Daba who took over the revision process and
amendments of the last two editions.
 Assist. Prof. Abdel-Motaal Fouda who prepared the last two editions in a readable up-
to-date text to provide essential information necessary throughout the clinical career.
 Dr. Sameh Abdel-Ghany who assisted in the revision process.

Much of any merit this book may have is due to the


generosity of those named above.

Gamal M. Dahab (MD, PhD)

Professor Emeritus in Clinical Pharmacology


Mansoura Faculty of Medicine

iii
Miss
sion and
a Vis
sion

Our mission
The Clinnical Pharm macology Department
D is seeking excellence
e and leadeership in fou
ur major
core acctivities: edu
ucation, ressearch, com
mmunity serrvice, and faculty
f and staff development.
We are e connectin ng basic medical
m sc iences with clinical care
c througgh innovattive and
disciplin
ned teachin ng of clinica
al pharmaco
ology in an integrative
i manner
m

Our v
vision
The dep partment off Clinical Ph
harmacolog gy is aiming
g to be a premier acad demic model in the
field of pharmacoloogy and the erapeutics in Egypt annd Middle East
E throug h promoting use of
the bestt therapeutics and dev veloping new
wer experimmental and clinical reseearch proje
ects.

Value
es

The gu
uiding prin
nciples and beliefs ffor the dep
partment

 Excellence, creativity, innovation, fairness, honesty, transparenncy, collab


boration,
teammwork and lifelong learning
 Reccognition tha
at our stude
ent comes ffirst
 All m
members ofo our department musst see them mselves as integral to the successs of our
misssion and ouur departmeent as integ ral to their personal
p su
uccess.
 As we subscrribe to the ese values,, we shall be profes ssionals in the profes
ssion of
education.

iv
Contributers

Effat A. Haroun MD, PhD Somaya A. Mokbel MD, PhD


Prof. of Clin Pharmacology Prof. of Clin Pharmacology
Mansoura Faculty of Medicine Mansoura Faculty of Medicine

Elhamy M. El-Kholy MD, PhD Amany A. Shalaby MD, PhD


Prof. of Clin Pharmacology Prof. of Clin Pharmacology
Mansoura Faculty of Medicine Mansoura Faculty of Medicine

Gamal M. Dahab MD, PhD, MSc (Int.Med) Amal Abdel-Hamid MD, PhD
Prof. of Clin Pharmacology Prof. of Clin Pharmacology
Mansoura Faculty of Medicine Mansoura Faculty of Medicine

Farida M. El-Banna MD, PhD Essam A. Ghyati MD, PhD


Prof. of Clin Pharmacology Assist. Prof. of Clin Pharmacology
Mansoura Faculty of Medicine Mansoura Faculty of Medicine

Aly M. Gaballah MD, PhD, MSc (int.Med) Mohamed-Hesham Y. Daba MD, PhD
Prof. of Clin Pharmacology Assist. Prof. of Clin Pharmacology
Mansoura Faculty of Medicine Mansoura Faculty of Medicine

Layla T. Hanna MD, PhD Abdel-Motaal M. Fouda MD, PhD


Prof. of Clin Pharmacology Assist. Prof. of Clin Pharmacology
Mansoura Faculty of Medicine Mansoura Faculty of Medicine

Mohamed Kheriza MD, PhD, MSc (Int.Med) Vivian Boshra MD, PhD
Prof. of Clin Pharmacology Assist. Prof. of Clin Pharmacology
Mansoura Faculty of Medicine Mansoura Faculty of Medicine

Abdel-Rahman A. Yassin MD, PhD Hala A. Al-Ashri MD, PhD


Prof. of Clin Pharmacology Assist. Prof. of Clin Pharmacology
Mansoura Faculty of Medicine Mansoura Faculty of Medicine

Mohmmad A. Attia MD, PhD Nageh Rizk MD, PhD


Prof. of Clin Pharmacology Lecturer in pharmacology
Mansoura Faculty of Medicine Mansoura Faculty of Medicine

Mohamed Abdel-Ghani MD, PhD Elsayed A. Hassan MD, PhD


Prof. of Clin Pharmacology Lecturer in Clin Pharmacology
Mansoura Faculty of Medicine Mansoura Faculty of Medicine

Hussien M. El-Beltagi MD, PhD Mohamed Abdel-Monem MD, PhD


Prof. of Clin Pharmacology Lecturer in Clin Pharmacology
Mansoura Faculty of Medicine Mansoura Faculty of Medicine

Karawan M. Abdel-Rahman MD, PhD Mahmoud A. Naga MD, PhD


Prof. of Clin Pharmacology Lecturer in Clin Pharmacology
Mansoura Faculty of Medicine Mansoura Faculty of Medicine

v
Ahmad Hassan MD, PhD Mohamed Abou El-khair MD, PhD
Lecturer in Clin Pharmacology Lecturer in Clin Pharmacology
Mansoura Faculty of Medicine Mansoura Faculty of Medicine

Ahlam El-masry MD, PhD Sameh A. Abdel-Ghani MSc.


Lecturer in Clin Pharmacology Assist. Lecturer in Clin Pharmacology
Mansoura Faculty of Medicine Mansoura Faculty of Medicine

Rehab Hamdy MD, PhD


Lecturer in Clin Pharmacology
Mansoura Faculty of Medicine

vi iii
Table of Contents

CHAPTER 8: GASTROINTESTINAL PHARMACOLOGY

Part 1: Peptic ulcer and reflux esophagitis 251


Peptic ulcer 251
Gastroesophageal reflux disease 259
Part 2: Management of liver disease complications 262
Hepatic encephalopathy 262
Bleeding esophageal varices 263
Part 3: Antiemetic drugs 264
Part 4: Antispasmodic drugs 266
Part 5: Therapy of constipation 266
Part 6: Therapy of diarrhea 269
Part 7: Drug therapy of gall stones 274
Review questions 276

CHAPTER 9: ENDOCRINE PHARMACOLOGY

Part 1: Diabetes mellitus and antidiabetic drugs 281


Insulin 283
Oral antidiabetic drugs 288
Treatment of diabetic complications 292
Part 2: Medical treatment of obesity 295
Part 3: Thyroid gland and antithyroid gland 297
Part 4: Adrenocortical steroids 303
Part 5: Regulation of calcium metabolism 307
Part 6: Sex hormones 311
Part 7: Hypothalamic and pituitary hormones 316
Review questions 320

CHAPTER 10: CNS PHARMACOLOGY

Part 1: CNS stimulants 327


Part 2: Analgesics 329
Opioid analgesics 329
Non-opioid analgesics 337
Part 3: Sedative hypnotic drugs 339
Part 4: Skeletal muscle relaxants 343
Part 5: Antiepileptic drugs 345
Part 6: Antidepressant drugs 350

vii
Part 7: Mood-stabilizing drugs 354
Part 8: Antipsychotic drugs 355
Part 9: Antiparkinsonian drugs 358
Part 10: Drugs used for Alzheimer disease 362
Part 11: General anesthetic drugs 363
Part 12: Local anesthetic drugs 369
Review questions 373

CHAPTER 11: ANTIMICROBIAL DRUGS

Part 1: Basic principles of antimicrobial drugs 379


Part 2: Individual classes of antibiotics 385
Cell wall inhibitors 385
Inhibitors of bacterial protein synthesis 394
Inhibitors of bacterial nucleic acid synthesis 402
Inhibitors of bacterial metabolism 404
Miscellaneous antibacterial drugs 406
Part 3: Urinary tract infection 408
Part 4: Chemotherapy of TB and leprosy 412
Part 5: Antiviral drugs 418
Part 6: Chemotherapy of fungal infections 421
Part 7: Antiamoebic drugs 425
Part 8: Antimalarial drugs 428
Part 9: Anthelmintic drugs 433
Review questions 437

viii
Part 1
1: Pep
ptic Ulce
er Disea
ase and Reflux Esophag
E gitis

█ PEP
PTIC ULCE
ER

Definittion: ulcera
ation of the
e duodenuum or stom
mach due tot imbalannce betwee
en local
invasive force (e.g
g. HCl andd pepsin) a
and protecttive mecha
anisms.

Invasiv
ve factors:
 ess: ↑ HCl and pepsin secretion
Stre n by pariettal cells.
 Diett: coffee, alcohol
a and
d spices.
 Druugs: NSAID Ds, corticosteroids, m
morphine, methylxan nthines, etcc.
 ection with
Infe h Helicoba acter pylorri.
H. ppylori is sppiral gram –ve flageellates foun
nd in the antrum
a of human sttomach.
Cerrtain enzym mes and toxins
t prooduced by y the bacteria causee tissue damage.
d
Infe
ection with h H. pylori can be d diagnosed by endosc copic biop psy or serological
marrkers.

Defens
sive mech
hanisms:
 Muccus prod duction by gasttric
muccosa.
 Pan
ncreatic bic
carbonate secretion.
 Goo
od mucosa al blood flo
ow.
 Loc
cal PGE2 annd PGI2 production.

Regula
ation of HC
Cl secretio
on
 Ach
h: ↑ HCl seecretion thrrough M1 rreceptors → ↑ intrace
ellular Ca2+.
 Gasstrin: ↑ HC n through G receptorrs → ↑ intra
Cl secretion acellular Caa2+.
 Histamine: ↑ HCl secretion througgh H2 rece eptors → ↑ intracellulaar cAMP.

251
Both C Ca2+ and cAMP activate H+ /K+
ATPase e at the membrane ofo the pari etal
+
cell to secrete H into the gastric lum
men
“proton pump”.
 PGE
E2 and PGI2: act on PG
P recepto
ors
→ ↓ cAMP → ↓ HCl secre
etion.

Clinica
al picture
 Epig
gastric paiin: charactterized by:
– DDiffuse annd worsen ns by food d in
GGU.
– L Localized (point ten nderness) a and
rrelieved byy food in DU.
D
 Signns of comp plications e.g. bleediing,
ane
emia, etc.

Diagno
osis
– End
doscopy: visualizatio
v cer.
n of the ulc
– Rad
diologic: byy barium meal.
m

█ The
erapy of peptic
p ulcer

▌Non-d
drug thera
apy = life style
s mod
dification

 Resst and Sedation: theyy improve hhealing and relief


pain of DU.
 Stopp Smoking g, Spices, alcohol, coffee, an nd tea:
beccause theyy ↑ HCl.
 Avo
oid Stress: because stress
s ↑ HC
Cl.
 Avo
oid ulcerogenic drugs s: e.g. NSAAIDs.
 Diett:
– Frequeent small meals
m in DU
U in order to buffer high acidityy.
– Encourage milk anda fats.
– Avoid Spices
S and
d fried foodd

▌Pharm
macologic
cal therapy
y

■ Dru
ugs that neutralize HCl:
H antac
cids

■ ugs that ↓ HCl secre


Dru etion:

252
– Selective M1 blockers: pirenzepine, telenzepine.
– H2 blockers: cimetidine, ranitidine, famotidine.
– Proton pump inhibitors: omeprazole, lanzoprazole, etc.

■ Drugs that ↑ mucosal defense mechanisms:


– Sucralfate
– Colloid bismuth compounds: e.g. bismuth subcitrate.
– Carbenoxolone
– PGE1 analogues: misoprostol.

■ Antimicrobial drugs for H. pylori: see later.

■ Adjuvant therapy: Sedatives and multivitamins to ↓ stress to enhance healing.

█ ANTACIDS

 Antacids are weak bases that are taken orally and partially neutralize gastric acid
and reduce pepsin activity.
 They are used as symptomatic relief of hyperacidity and should not be used as
long-term treatment.

Sodium bicarbonate Calcium carbonate Magnesium and


aluminum salts
(Mg hydroxide and
Aluminium hydroxide)
 It can be absorbed  Partially absorbed ant-  They are poorly ab-
systemically leading to acid. sorbed from GIT and
salt & water retention,  Ca2+ may act directly to have no systemic
and metabolic stimulate gastrin secre- effects.
alkalosis. tion leading to acid  The unabsorbed Mg
 It is contraindicated in rebound. salts cause osmotic di-
hypertension and heart  It is contraindicated in arrhea; the unabsorbed
failure. hypercalcemia and Al salts cause
 It has rapid onset and renal stones. constipation.
short duration.  They have slow onset.

Adverse effects
■ Change in bowel habits: Al3+ hydroxide causes constipation, while Mg2+
hydroxide cause diarrhea. For this reason, both salts are combined together to
manage this problem.

253
■ Reb
bound hyp
peracidity a2+ and
y: with Ca
NaH
HCO3 containing anta
acids.

■ Cattion overlo
oad:
– Na+ saltss → hyp pertension
n and
ssystemic alkalosis.
a
– C 2+
Ca saltss → hyperrcalcemia, renal
sstones and
d milk-alk kali syndro
ome.

■ Dec
crease ab bsorption of other d drugs: the e metal ion n in some preparatio
ons can
che
elate other drugs esp
pecially tetrracycline, digitalis
d an
nd iron.

N.B. wwhen iron or


o tetracyccline is pre
escribed with
w antacid ds, we sho
ould make 30 min
interval between both drugs to avoid chelationn with the antacid.
a

█ DEC
CREASE HCL
H SECR
RETION

1. Sellective M1 blocke
ers:
(Pirenz
zepine - Te
elenzepine)

Mecha anism of action: selectively


s astric M1 receptors → ↓ bas
block ga sal HCl
secretio
on.

Uses: TThe selecttive M1 blo


ockers are hibitors of HCl secreetion and are
e weak inh a now
replace
ed by morre effectivee drugs. T
They are sometimes
s s used as adjuvant therapy
with H2 blockers.

Advers s: high do
se effects oses prod pine-like effects: dryy mouth, blurred
duce atrop
vision, tachycardia, urine re
etention.

N.B. Attropine itse


elf is not used
u in the
e treatment of PU be
ecause it iss non-sele
ective M
blockerr and may y aggravatte esophag geal reflux
x.

2. H2 blockerrs:
(Cimettidine – Ra
anitidine – Famotidi ne - Nizattidine)

Mecha
anism and pharmac
cological e
effects

 Theey act as competitive


c e inhibitorss of histam
mine H2-receptors onn the parie
etal cell.
Thiss results in d ↓ in histam
n a marked mine-stimu
ulated HCl secretion .
 Alth
hough othe er agents such as gastrin an nd ACh maym inducee acid seecretion,
histtamine is thhe predom
minant final mediator that stimulates HCl ssecretion.

254
Therapeutic uses
 Duodenal and gastric ulcers.
 Prophylaxis & treatment of stress ulcers (e.g. after burn or major trauma).
 Prophylaxis against bleeding of esophageal varices.
 Reflux esophagitis.
 Zollinger-Ellison syndrome (gastrin-secreting tumor of the pancreas which ↑ HCl
secretion): usually larger doses are required.
 With ulcerogenic drugs (e.g. NSAIDs) to protect the gastric mucosa from injury.

Adverse effects (mostly with cimetidine):


– Cimetidine has anti-androgenic effects (due to block of androgen receptors)
leading to ↓ sperm count, impotence and gynecomastia.
– Cimetidine inhibits hepatic microsomal enzymes (P450) leading to ↓
metabolism of other drugs e.g. theophylline, warfarin, sulphonylureas, etc.
– Reversible hepatotoxicity and Reversible anemia.
– CNS symptoms: headache, slurred speech, delirium, coma, etc. occurs mainly in
elderly people with i.v. administration.

Precautions of H2 blockers
– Avoid sudden withdrawal to prevent rebound ulceration.
– Avoid their use in pregnancy and lactation (they cross the placental barrier and
secreted in breast milk).
– Avoid combination of cimetidine with drugs having narrow therapeutic index
(because cimetidine inhibits microsomal P450 and ↑ their toxicity).

Cimetidine Ranitidine Famotidine


H2 Blocking effect: Weak Potent More potent
Anti-anderogenic effect: Strong Minimal No
Liver enzyme inhibition Strong Minimal No
Adverse CNS effects: Frequent Less frequent Less frequent
Duration of action: 8h 12 h 24 h
Dose: 800 mg/day for 300 mg/day for 20 mg/day for
6-8 weeks then 6-8 weeks then 6-8 weeks then
400 mg/day for 150 mg/day for 10 mg/day for
6-8 months 6-8 months 6-8 months

255
3. Pro
oton pum
mp inhibiitors (PP
PIs):
(Omep
prazole – Lansopraz
L zole – Panttoprazole)

Chemistry: all arre imidazole derivativves.

Mecha
anism of action
a
 Theey are pro
odrugs. Th
hey are co
onverted
into
o the activee form in thhe gastric mucosa
andd produce e irreversiible inhib bition of
+ +
gasstric H /K ATPase
A ennzyme leadding to ↓
both basal an nd stimula ated HCl se ecretion to
o around th
he zero levvel for 1-2 days.
 Full restoratio
on of acid secretion n after stopping the PPI takess about 3--5 days
+ +
(tim
me of re-synnthesis of H /K ATP Pase).
 Theeir bioavaillability is decreased d significan
ntly by food and, id deally, shoould be
admministered 1 hour beffore a mea al.

Therap peutic usees: The saame as H2 blockers (dose of omeprazo


o ole: 20-40 mg/day
orally fo
or 4-6 wee
eks then 10
0-20 mg/d
day for 4-6 months to
o prevent re
recurrence)).

Advers
se effects
– Low
w incidenc
ce of diarrrhea, abdo
ominal colic, dizzine
ess, skin rrash, leuco
openia,
andd transient increase of o liver enzzymes.
– Deccrease vitt B12 abso orption aftter > 12 weeks
w of therapy
t duue to interrference
with
h intrinsic factor
f secrretion by th
he stomac ch.
– Inhiibition of gastric
g acid
dity leads to alteratiion of bioavialability ty of somee drugs
e.g.. ketocona azole, digox xin, and iro
on.
– Ommeprazole inhibits microsomall P450 enzymes and decreasses metabo olism of
pheenytoin, wa arfarin, and cyclosporrin. Newer PPIs
P do noot affect liveer enzymess.
– Ommeprazole in n high dos se induced d gastric ca
arcinoid tuumor in ratts.

█ ENH
HANCING MUCOSA
AL DEFENS
SE MECHANISMS

1. Suc
cralfate

 It iss an alumin
num salt off sulfated ssucrose.
 Slig m metal maay accumulate in
ghtly (3%) absorbed from the GIT. The aluminum
casses of renal failure, so
o it should be avoide
ed in renal failure.

Mecha
anism of action
a
 It n dic medium to be a
needs acid esence of aacidic med
activated. In the pre dium, it

256
forms a complex with protein debris at the ulcer base and forms a physical
barrier (so not taken with antacids, H2 blockers, or PPIs).
 It ↓ pepsin secretion and ↑ secretion of endogenous PGs.

Adverse effects
N.B. Both sucralfate and
– Constipation (due to presence of aluminum).
bismuth compound are not
– ↓↓ absorption of tetracycline, digoxin and given simultaneously with
phenytoin. antacids or H2 blockers (at
least 30 min must be elapsed
in-between). Why?
2. Bismuth compounds:
Bismuth subsalicylate and subcitrate

Mechanism of action
 In acidic pH, it forms a complex with protein debris at the ulcer base and forms a
physical barrier.
 It ↓ pepsin secretion and ↑ secretion of endogenous PGs.
 It has additional antimicrobial activity against H. pylori.

Adverse effects
– Stool and teeth discoloration.
– Encephalopathy in presence of renal failure

Contraindications: Chronic renal failure and CNS diseases.

3. Carbenoxolone

It is a liquorice derivative having steroid structure.

Mechanism of action
 It ↑ production and viscosity of gastric mucus and ↑ mucosal resistance.
 It ↓ pepsin secretion and ↑ secretion of endogenous PGs.

Adverse effects
Salt & water retention (aldosterone-like effects) → edema and hypertension
especially in cardiac and renal patients. This edema can be treated by thiazide
diuretics (not by spironolactone) because both spironolactone and carbenoxolone
have steroid structure and can compete with each other.

Contraindications: Hypertension and/or renal failure

257
4. Syn
nthetic PG
GE1 analo
ogue: Mis
soprostol

Mecha
anism of action
a
 It aacts on specific rece
eptors on gastric pa s to ↓ histtamine-stim
arietal cells mulated
HCll secretion
n.
 ↑m
mucus and bicarbonate secretio on (cytopro
otective acction).
 ↑m
mucosal blo ood flow an
nd stimulat
ates mucossal cellular regeneratiion.

Therap
peutic use
es
Preven
ntion of peeptic ulcerr in high risk patients e.g. th
hose on l ong term use of
NSAIDss for chronic inflamm
matory disseases. [mmisoprostool 200 g is combined with
xen or dicllofenac in single tab
naprox blet].

Advers
se effects
– Diarrhea and cramping pain: due tto ↑ GIT motility
m and water seccretion.
– Uteerine contraactions during pregnnancy → ab
bortion.

Contra ns: pregna


aindication ancy.

█ ERA
ADICATION THERAP
PY FOR H
H. pylori

 Infe
ection with H. pylori is
s a main c
cause of re e of PU.
ecurrence
 Thee following 10 days
“sequential proto-
p
col”” is highly effective
for e
eradicationn of H.
pylo
ori:

█ THE
ERAPY OF
F BLEEDIN
NG PEPTIC
C ULCER

 Hosspitalizatio
on and Fre
esh blood
d transfusion.
 Acid suppre ession witth high d
dose PPIs s by continuous i.vv. infusion is the
stan
ndard of ca
are e.g. om
meprazole 80 mg i.v. bolus follo
owed by 8 mg/h for 72h.
7
 Vita
amin K1: 10 mg i.m or
o s.c.
 Enddoscopic therapy:
t several
s type
es of endo
oscopic tre
eatments aare available.

258
█ TREATMENT OF GASTROESOPHAGEAL REFLUX DISEASE (GERD)

Definition: reflux of gastric contents into the esophagus due to incompetent lower
esophageal sphincter (LES). Heartburn and chest pain are the major complain
which may be misdiagnosed of angina pectoris.

▌Non-drug therapy = life style modification


Drugs ↑ LES pressure:
 Head of bed elevation: because most damage to  Metoclopramide
the esophagus occurs at night when HCl can  Domperidone
 Bethanechol
remain in contact with the mucosa for long period.
 Erythromycin
 Weight reduction.
 Avoid: Drugs ↓ LES pressure:
– Stress, Smoking, Spices, alcohol.  Anticholinergic
– Ulcerogenic drugs e.g. NSAIDs. drugs
 Nitrates

▌Drug therapy

 Decreasing HCl secretion: H2 blockers and proton pump inhibitors.


 Prokinetic drugs.
 Antacids and antacid combinations: (Gaviscon).

▌Surgical treatment: if medical therapy failed.

█ PROKINETIC DRUGS

Definition: they are drugs that increase upper GIT motility and enhance gastric
emptying. They include:
■ Dopamine antagonists: e.g. Metoclopramide and Domperidone.
■ Serotonin (5-HT4) agonists: e.g. Mosapride
■ Cholinomimetic agents: e.g. Bethanechol.
■ Macrolide antibiotics: e.g. Erythromycin

1. Metoclopramide

Mechanism and pharmacological effects


 Metoclopramide ↑ LES tone and enhances gastric emptying and upper GIT
motility through:
 Blocking of dopamine (D) receptors (central & peripheral) leading to decrease
the inhibitory action of dopamine on the GIT motility.
 Enhances cholinergic transmission in the upper GIT.

259
 N
N.B. Metoclopramide has no e
effect on small
s intesttinal or collonic motility.
 Anttiemetic action:
a du
ue to bloc
ckade of D2 recepttors in thee chemoreceptor
trigg
ger zone of
o the medu
ulla (CTZ)..

Therap
peutic use
es
 Gasstroesoph
hageal reflux: to enh
hance gastric emptyinng and ↑ LLES pressu
ure.
 Disorders off gastric emptying:
e e.g. diabetic gastro
oparesis aand postop
perative
gasstric retentiion.
 Beffore small bowel en ndoscopy (20 mg given by slo ow i.v.i.): to
o enhance
e gastric
eva
acuation an nd peristalttic movem
ment. Also to
t prevent vomiting.
 Beffore emerrgency su urgery and d labor to evacuate the stom mach and prevent
asp
piration of gastric
g con
ntents duri ng anestheesia.
 Tre
eatment off nausea anda vomitiing of vario ous causes.

Advers
se effects

– Sed
dation (the most commmon adve erse effect)).
– Extrapyramid s (e.g. dys
dal effects ystonia andd dyskines
sia): (especcially in old age)
due
e to blocka
ade of D2 in
n the basall ganglia.
– Hyp
perprolacttinemia du ue to blockkade of D2 in the pitu
uitary gland
d.

Drug in
nteraction
ns

– Antticholinerg pine) antag


gic drugs (e.g. atrop gonize its prokinetic
p action.
– Oth
her dopamine blo ockers (e e.g. antips
sychotic drugs) ad dministere
ed with
mettoclopramiide may prrecipitate a
acute extra
apyramidal effects.

2. Dom
mperidon
ne

Mecha
anism and pharmac
cological e
effects
 It blocks periipheral D2 receptorss leading to ↓ the inhibitory acction of do
opamine
on G
GIT motilityy. It does NOT
N crosss BBB so itt has no CNS
C side efffects.
 Anttiemetic efffect less than
t meto clopramide.

Therap
peutic use
es
 Thee same usees as meto
oclopramid
de.
 To counteracct nausea a and vo omiting caused by
y
levoodopa an nd bromocriptine d during tre
eatment off
Parrkinson’s disease because
b it blocks D2
2 receptors s
in th
he CTZ ressponsible for
f vomiting g but does
s not blockk
D2 receptors in
i the basaal ganglia re
esponsible
e for parkins
sonism.

260
Adversse effectss: there is growing
g at domperidone may ↑ QT interrval and
evvidence tha
predisp
pose to seriious arrhyth
hmia and sudden dea ath.

M
Metoclopra
amide D
Domperido
one
Dopam
mine recepttor blockad
de C
Central and
d periphera
al P
Peripheral only
o
Choline
ergic transsmission In
ncrease N
No effect
Antiem
metic effect S
Strong W
Weaker
Extra-p
pyramidal side
s effects P
Present N
No
Hyperp
prolactinem
mia S
Significant M
Minimal

3. Betthanechol

Bethannechol stim mulates mu uscarinic MM3 in the smooth


s ms of the GGIT and my yenteric
plexus.. It was ussed in the past for tthe treatmment of GE ERD and g gastroparesis, but
now, it is rarely used for this indicatio
on due to multiple
m cholinergic sside effects
s.

4. Mac
crolide an
ntibiotics
s: erythro
omycin

Macrolide antibiootics such as erythro omycin dirrectly stimulate mottilin receptors on


GIT sm mooth mu uscle and promote e the onset of a migrating motor co omplex.
Intravenous erythhromycin (33 mg/kg) iss beneficia
al in some patients w
with gastrop
paresis;
howeve er, tolerance rapidly
y developss. It may beb used in n patients with acutee upper
GIT hemmorrhage to promote e gastric e
emptying of
o blood be efore endosscopy.

█ ANT
TACIDS AN
ND ANTAC
CID-ALGIN
NIC ACID PRODUC
CTS

Gavisc
con: (algin
nic acid + Mg-trisilica
M ate + Al-hy
ydroxide +
NaHCOO3):

Alginic acid in prresence off saliva an


nd NaHCO O3 forms a
highly vviscous fo
oamy soluttion of Na--alginate that
t floats
on the gastric co ontents as
s a raft an nd prevents gastric
reflux.

█ H2 B
BLOCKER
RS AND PP
PIS

 PPIs are moree effectivee than H2 blockers foor the treattment of G


GERD.
 Oncce-daily do
osing of PPPIs provide
es effective
e symptom m relief and
d tissue he
ealing in
85––90% of paatients; up to 15% off patients require
r twice-daily doosing.

261
Part 2
2: Ma
anageme
ent of L iver Dis
sease Co
omplica
ations

█ MAN
NAGEMEN
NT OF HEP
PATIC EN
NCEPHALO
OPATHY

Definittion and pathogenesis


Hepatic c encephalopathy is the
syndrome of disordered
d con-
sciousn ness and neuromu uscular
activityy seen in n patients s with
acute o or chronic liver failure
e.
The failing liver cannot metabo-
m
mmonia and
lize am a benzoodiaze-
pines-llike med diators (G GABA-
ansmitters) generatted by
like tra
the inntestinal bacteria. These
toxins a
are shunte S causing encephalo
ed directly to the CNS opathy.

gement: Treatment
Manag T is aimed a
at reductio
on of hype
erammone
emia:

■ Diet:
– Protein reestriction to
t decreasse formatio on of ammo onia by inttestinal bac
cteria.
– VVegetablee protein is
s better tole
erated tha
an animal protein.
p
– TThe rationa
ale and beenefit of die
etary prote
ein restriction is conttroversial.

■ Ene
emas: clea
ansing of the colon n is a rap
pid and efffective meethod to remove
moniagenic substrates. It can b
amm be done with
w lactulosse or tape water.

■ Lac
ctulose:
– It is syntheetic non-ab bsorbable disaccharride. In the
e colon, it iis transformed by
bacteria intto lactic an acids → ↓ pH
nd acetic a p of the co
olonic med dium leadin ng to:
– Inhibitio
on of intes eria → ↓ pro
stinal bacte oduction of
o ammoniaa.
– ↑ transsport of ammonia from blood to intestinal lum men wherre it is
converrted to the poorly abssorbed am
mmonium io
on.
– Osmotic laxation → ↑ excrettion of ammonium io
on.

– It is admin
nistered ora
ally or as eenema (for patients in
n coma).
– A
Adverse effects:
e rellatively saffe drug.

■ Ora
al antibiotiics:
Neo
omycin:
– It is non-absorbable aminoglyccoside antibiotic.

262
– It ↓ blood ammonia by killing intestinal bacteria that generate ammonia.
– It is used in a dose of 1-2 g 4 times daily orally or as retention enema.
– Small amounts of neomycin may be absorbed (~1%) and result in ototoxicity
and nephrotoxicity especially in patients with renal impairment.

Other antibiotics:
– Metronidazole acts on anaerobic bacteria. It is the preferred option if there is
fear from adverse effects of neomycin (but given for short term).
– Rifaximin: is non-absorbable and better tolerated antibiotic.

█ MANAGEMENT OF VARICEAL BLEEDING DUE TO PORTAL HYPERTENSION

▌Management of acute bleeding

■ Fresh blood transfusion.


■ Acid suppression with omeprazole (80 mg) to minimize HCl irritation.
■ i.v. vasopressin or its analogues:

 Vasopressin:
– It produces mesenteric VC leading to  portal venous flow and pressure.
– It can produce systemic VC (coronary, cerebral, limb, etc), so it is better
combined with i.v. nitroglycerine to reduce systemic and coronary VC.
– The vasopressin/nitroglycerine combination is rarely used now.
 Terlipressin:
– It is synthetic analog of vasopressin that is released in a slow and sustained
manner allowing more sustained hemodynamic effects with fewer systemic
side effects than vasopressin.

■ Prophylactic antibiotics: to prevent infectious complications after GI


hemorrhage. The preferred antibiotic is i.v. ceftriaxone 1 gm/day for 7 days.

■ Endoscopic sclerotherapy: injection of the varices with a sclerosing agent to


induce fibrosis and obliteration.

▌Prevention of re-bleeding (prophylaxis):

 Beta-blockers (propranolol 40 mg twice daily). It ↓ portal BP through:


– They ↓ COP → ↓ portal blood flow.
– They cause unopposed α- action → VC of the splanchnic vascular bed.
 H2 blockers or PPIs: to prevent gastroduodenal erosions.
 Metoclopramide: to enhance gastric evacuation and  LES pressure.

263
264
Part 3: Antiemetic Drugs

Several classes of antiemetic drugs are available that antagonize the neurotransmitter receptors known to be involved in the
physiology of nausea and vomiting. The antiemetic drugs are classified according to their primary action; some agents affect
multiple receptors.

Drug Antiemetic mechanism Uses as antiemetic Adverse effects

1. Muscarinic blockers: They block M1 receptors in the Prevention and treatment of  Blurred vision
 Atropine vestibulocerebellar pathway, vomiting due to motion sickness  Dry mouth
 Hyoscine solitary tract nucleus, and (0.3-0.6 mg/8 hrs orally).  Urine retention
chemoreceptor trigger zone  Glaucoma
(CTZ).  Tachycardia

2. H1-blockers:  They block H1 (also M1)  Vomiting due to motion  Sedation (excitation may
 Diphenhydramine receptors in the vestibulo- sickness (diphenhydramine) occur in children).
 Cyproheptadine cerebellar pathway and CTZ.  Vomiting of pregnancy  Atropine-like actions (dry
 Cyclizine  They have sedative action. (cyclizine and meclizine) mouth, blurred vision, urine
 Meclizine  True vertigo: combined with retention).
VDs to improve labyrinthine  Hypotension
blood flow.

3. 5-HT3 blockers: They block 5HT3 receptors in  Vomiting due to cancer  Dizziness, headache, and
 Ondansetron the GIT, solitary tract nucleus chemotherapy or radio- constipation.
 Granisetron and CTZ. therapy.
 Tropisetron  Postoperative nausea and
vomiting.

4. Dopamine blockers  They block D2 receptors in  Vomiting due to drugs or  Sedation


 Metoclopramide the CTZ. fevers.  Extrapyramidal effects e.g.
 Domperidone  They inhibit peripheral trans-  Vomiting due to cancer dystonia and dyskinesia.
 Phenothiazines e.g. mission to VC. chemotherapy.  Hyperprolactinemia
chlorpromazine  Postoperative nausea and  Postural hypotension
vomiting.

5. Cannabinoid  It is a partial agonist at  Vomiting due to cancer  Sedation


derivatives: central and peripheral chemotherapy  Hallucinations
 Nabilone and cannabinoid receptors (CB1).  Patients refractory to other  Psychotropic effects
Dronabinol The exact mechanism is antiemetics.  Postural hypotension
unclear.  Drug abuse.

6. Vitamin B6 (pyridoxin) May be related to the balance  Vomiting in pregnancy (50 mg -


between GABA (CNS inhibitory at bedtime).
transmitter) and glutamate  Vomiting in children
(CNS excitatory transmitter).

7. Corticosteroids The exact mechanism is  Combined with Vit B6 to treat  See endocrine chapter
 Dexamethasone unclear. vomiting in pregnancy.
 Prednisolone  Vomiting due to cancer
chemotherapy.

8. Benzodiazepines Allosteric facilitation of central  Stress-related vomiting  See CNS chapter


 Lorazepam GABA inhibitory transmission  To controls symptoms in
 Diazepam Ménière disease

9. Neurokinin-1 receptor Substance-P induces vomiting  In combination with 5-HT3  Diarrhea and fatigue
blockers: through stimulation of NK-1 blockers to treat vomiting due
 Aprepitant receptors. Aprepitant blocks to cancer chemotherapy
this receptor.

265
Part 4: Antispasmodic Drugs (smooth ms relaxants)

Classification

■ Anticholinergic drugs: atropine, hyoscine, propantheline, oxyphenonium.


■ Direct smooth muscle relaxants: papaverine, mebeverine, alverine,
drotaverine.
■ Mixtures: Librax (clidinium + chlordiazepoxide), Donnatal (hyoscine + phenobarbital).

Mebeverine, Alverine,
Papaverine Librax
Drotaverine
It is opium alkaloid but They are synthetic It is a combination of:
Chemistry & mech-

 Chlordiazepoxide:
anism of action

chemically different drugs


from morphine benzodiazepine that
has antianxiety effect.
The exact mechanism is unclear but may be due
 Clidinium: anticho-
to inhibition of PDE enzyme  cAMP 
linergic drug which ↓
smooth muscle relaxation.
GIT motility and spasm

 Spasms of the GIT, bile duct and genitourinary tract.


Use

 Librax is used for treatment of irritable bowel syndrome (IBS).

– Cardiac arrhythmia. – Atropine-like actions


Side effects

– Abnormal liver functions in the form of elevated e.g, dry mouth, urine
serum transaminases and alkaline retention, etc.
phosphatase. – Sedation, drowsiness,
– Headache and dizziness confusion, etc.

– Paralytic ileus.
C/I

– Constipation for more than one week

Part 5: Therapy of Constipation

▌Non-drug therapy: It is the first line in all cases of constipation


 Diet rich in fibers e.g. fruits, vegetables, whole meal bread, etc. to be increased to 30
g/day.
 Increase fluid intake
 Minimize tea and coffee.
 Physical exercise to activate abdominal muscles and intestinal peristalsis. This
help food move more efficiently through the gut.

266
▌Drug therapy: LAXATIVES:
Drug causes of constipation:
1. Bulk-forming agents:
 Atropine and related drugs.
[Dietary fibers – Methylcellulose – Bran]
 Aluminum containing antacids
Mechanism of action  Adsorbents (kaolin & pectin).
 CCBs: e.g. Verapamil
They are non-digestible fibers; they retain  Opioids: morphine &
water in the gut and distend the large loperamide
intestine → activation of stretch receptors →
stimulation of peristalsis.

Adverse effects: they are safe laxatives but may cause:


– Bloating and abdominal distension.
– ↓ absorption of some drugs e.g. digoxin.
– They may form masses in the gut leading to intestinal obstruction.

2. Osmotic laxatives:
[Mg sulfate & Na salts – Lactulose – Polyethylene glycol]

Mechanism of action
They are retained in the gut lumen and retain water by their osmotic effect →
activation of stretch receptors → stimulation of peristalsis.

Adverse effects
– Mg & Na salts (saline laxatives) may be absorbed systemically and produce
hypermagnesemia and hypernatremia especially in patients with renal failure.
– Lactulose may produce abdominal discomfort.
– Polyethylene glycol may produce electrolyte disturbance (hypokalemia).

3. Irritant (or stimulant) laxatives:


[Castor oil – Senna – Bisacodyl]

Mechanism of action
They produce inflammation (irritation) of the intestinal mucosa and inhibit Na+/K+
ATPase enzyme leading to:
– Accumulation of water and electrolytes in the gut lumen.
– Direct stimulation of peristalsis by their irritant effect.

Adverse effects

267
Castorr oil – Bad taste
e.
– Stimulatio
on of uteri ne contrac
ction and abortion
a
Senna – It passess in urine annd cause urine
u discoloration
– It passess in breast milk and cause
c hartic effecct in the ba
cath aby.
– Prolonged d use → degeneration of gutt nervous plexus → atonic
(cathartic)) colon.
– Increase menstrual blood flow w and abo ortion in prregnancy.
– Laxative depend dence: Irrritant lax xatives ccause co omplete
evacuatio on of the c colon. The
e colon requires 2-55 days beffore the
normal fe ecal masss can be reestablish hed. The patient be ecomes
worry reg garding th he lack of bowel mo ovement d during thiss period
and may use the la axative agaain and a vicious cyycle is esta
ablished
leading too partial orr complete
e loss of no
ormal boweel functionn.
Bisaco
odyl – It is prepared as ennteric coated tablets to avoid ggastric irrittation. If
it is given
n with milkk or with otther drugs that chang ge gastric pH, the
enteric coating
c maay dissolve e in the stomach
s aand cause gastric
irritation and
a pain
– Prolonged d use → degeneratiion of gutt nervous plexus → atonic
(cathartic) colon (shoould not bee used morre than 10 ddays).

4. Sto
ool softeners: Do
ocusate so
odium

Mecha anism: theyy are anion


nic surfacttants that enable
e add
ditional waater and fatts to be
incorpo
orated in th
he stool, making
m it ea
asier to moove throug
gh the GIT.

5. Lub
bricant laxatives
s:
[Liquid
d paraffin – Glycerin
n supposittories – Ev
vacuant en
nema]

anism of action
Mecha a
– Parraffin oil it coats th
he fecal m
matter and
reta
ards water absorption by the co olon.
– Glyycerin has hygrosco opic effect . It draws
watter from rectal muc cosa and lubricates
the anal can nal. It also stimulattes reflex
recttal contra
actions an nd promo otes stool
evaacuation in 15-20 minn.

se effects:: paraffin oil


Advers o decreasses absorption of fat--soluble vittamins

268
6. Chloride channel activators: Lubiprostone

Mechanism of action
It acts by activating chloride channels to increase fluid secretion in the intestinal lumen.
This eases the passage of stool and causes little change in electrolyte balance.

General indications of laxatives


 Constipation: laxatives should not be used for prolonged duration to avoid
laxative dependence.
 To fasten excretion of toxic substances from the GIT.
 To prepare the bowel before X-ray or colonoscopy.
 Hepatic encephalopathy (lactulose): to kill ammonia producing bacteria.
 Painful anal conditions e.g. anal fissure or piles.
 Postoperative: e.g. after hemorrhoids (piles) to avoid strain.

Contraindications of laxatives
 Laxatives are dangerous in cases of undiagnosed abdominal pain or
inflammatory bowel disease. They may lead to intestinal perforation.
 Organic obstruction of the GIT.

Part 6: Therapy of Diarrhea

▌Causes of diarrhea

■ Infectious diarrhea: bacterial, viral, fungal, protozoal, etc. Infectious diarrhea is


the most common type.

■ Hormonal (secretory) diarrhea: e.g. serotonin in carcinoid syndrome; calcitonin in


medullary thyroid carcinoma, histamine in mastocytosis, thyroxine in hyperthyroidism.
Diarrhea is caused by: (1) increased water secretions into the intestinal lumen
and (2) increased intestinal motility.

■ Malabsorption syndromes: e.g. bile acid malabsorption

■ Inflammatory (exudative) diarrhea: caused by inflammatory bowel diseases


e.g. Crohn’s disease (transmural lesion in the small intestine) and ulcerative
colitis (ulceration of the colon with bloody diarrhea).

■ Iatrogenic (drug induced) diarrhea:


– Overuse of laxatives.
– Mg-containing antacids.

269
– Antibiotic--associated
d diarrhea (pseudome s colitis) seee chemoth
embranous herapy
– Cholinomimetic drug gs.

▌Patte
erns of dia
arrhea

■ Acu
ute self-limmited diarrrhea: acutte diarrhea
a disappeaars within 224 hrs.
■ Acu
ute diarrhea (<2 we eeks): passsage of watery
w stool more thhan 10 tim
mes/day
associated wiith dehydra ation and e
electrolyte imbalancee.
■ Chrronic diarrrhea: (>2 weeks): persistent diarrhea for 3 weeeks in adults or 4
weeeks in infan
nts. It caus
ses weightt loss and weakness.
w

▌Inves
stigations of diarrhe
ea
 Sto nation: – Macrosc
ool examin copic: con
nsistency, color,
c bloo
od, etc.
– Microsccopic: RBC Cs, WBCs, parasites, ova, etc.
– Stool cuulture and sensitivity tests.
 End
doscopy and
a biopsy y in chronic
c cases.
 Rad
diologic ex
xamination
n: by bariu m enema.

▌Trea
atment of diarrhe
ea

Lines o
of therapy
y

 Maiintenance e of fluid and


a electro olyte balannce: is the
e first priorrity.
 Non n-specific
c antidiarrheal agen nts: should
d be applied after eexclusion of
o other
rele
evant causees of diarrh
hea.
 Speecific antid
diarrheal agents:
a tre
eatment off the causee e.g.
 Antimic crobials for infectiouss diarrhea..
 Antiinflammatory y drugs for inflammattory bowel diseases.
 Anttispasmod dic drugs: if there is colic or abbdominal cramps.
c

I. MAIN
NTENANCEE OF FLUID
D AND
ELECTR ROLYTE BALANCE

■ Oraal Rehydra ation Therrapy (ORT)):


 Balaanced salts solu
ution con ntaining
elecctrolytes and glucose (gluc cose is
impportant for sodium and
a conse
equently
watter absorpttion).
 90%% of acute cases of childhood
c d
diarrhea
can be correctted using ORT
O only.
■ Intrravenous solutions:: if dehydrration is
sevvere.

270
N.B. in
n infants with
w dehydrration, blo ood pH, se erum Na+ and K+ m must be meeasured
before giving anyy i.v. solutiion to avoiid electroly
yte and acid/base im
mbalance.

II. NON
N-SPECIFIC
C ANTIDIAR
RRHEAL TH
HERAPY

1. Ads
sorbents:: Kaolin, pectin, act ivated cha
arcoal

Mecha
anism
 The
ey adsorb water,
w microoganism
ms and toxiins.
 The
ey coat the
e mucosa and
a protec ct it.

Advers y ↓ absorpttion of othe


se effects:: they may er drugs.

2. Bism
muth sub
bsalicylatte

Mecha
anism
 Bismmuth: provvides a pro oat for the
otective co e mucosa anda bindss toxins prroduced
by ppathogenic c bacteria.
 Sub bsalicylate:: hydrolyze
ed by inte
estinal bactteria into salicylic
s acid →↓ in
a ntestinal
infla
ammation, hypermottility and se ecretions.

3. Antti-choline
ergic drug
gs: atropin
ne, hyosciine and prropantheliine

Mecha
anism
 Antidiarrheal action: ↓ colonic
c peeristalsis by
b blockingg the respponse of in
ntestinal
smo
ooth musc cle to cholin
nergic stim
mulation.
 Antispasmodiic action: relieve
r cram
mps assoc ciated with diarrhea.

4. Syn
nthetic op
pioid prep
parations
s: dipheno
oxylate and
d loperam
mide

Mecha
anism
 The
ey act on opioid
o mu) and δ (delta) rec
μ (m ceptors in the entericc nervous system
(both pre- and
d postsyna
aptic) lead ing to:
 ↑ segm menting (no
on-propuls ive) contra
actions of the
t small inntestine.
 ↑ waterr absorptio
on and ↓ wwater secreetion by intestinal muucosal cells
s.
 ↓ Ach release
r by cholinergi c neurons in the ENSS.

 Lop
peramide cannot
c crross BBBB while dipphenoxylatte can cro
oss BBB in very
small amoun nt (no CN NS effects in usual therapeutiic doses) but it can
n cause
add
diction if ussed in large doses an
nd for prollonged durration.
 The
ey are com
mmonly combined w omotil® is a combination of
with atropiine (e.g. Lo

271
diphenoxylate 2.5 mg + atropine 0.25 mg) to produce more ↓↓ in intestinal motility
and decrease liability for abuse.

Adverse effects
 Anti-cholinergic side effects e.g. dry mouth.
 Addiction: if used for prolonged duration.
 Precipitation of toxic megacolon if used in ulcerative colitis.

5. Cholestyramine

Mechanism: it binds bile acids in the intestine preventing their absorption and
decreasing their irritation.

Therapeutic uses: diarrhea due to bile salt malabsorption.

III. SPECIFIC ANTI-INFECTIVE AGENTS

 It is not necessary in simple gastroenteritis as most cases are viral in origin.


 Chemotherapy is necessary in specific types of enteritis e.g.

C. difficile colitis Metronidazole 400 mg/8h orally for 10 days (1st choice).
Clostridium difficile If no response to metronidazole: vancomycin 250
mg/6h.
Campylobacter jejuni Azithromycin 500 mg/day orally for 3 days.
E. coli (enterotoxigenic Ciprofloxacin 500 mg/12h for 1-3 days
and enteropathogenic)
Non-typhoid Salmonella Ciprofloxacin 500 mg/12h for 5-7 days or ceftriaxone
spp. 1g IV/24h.
Shigella spp. Ciprofloxacin 500 mg/12h for 3-5 days or co-
trimoxazole 2 tab (80/400)/12h.
V. cholerae Doxycycline 300 mg orally single dose or ciprofloxacin
500 mg/12h orally for 3 days.
V. parahemolyticus Ciprofloxacin 500 mg/12h for 1-3 days

IV. ANTI-INFLAMMATORY DRUGS

1. Sulfasalazine (5-aminosalicylic acid "5-ASA" + sulfapyridine)

Mechanism: In the colon, the “azo” bond is broken by intestinal bacteria to release 5-
ASA and sulfapyridine: 5-ASA has anti-inflammatory and immunosuppressive, while
sulfapyridine is poorly absorbed sulfonamide with antibacterial action.

272
Therapeutic uses:
 Active ulcerative colitis (3-4 gm/day) and to maintain remission (2 gm/day).
 Rheumatoid arthritis.

Adverse effects: mainly due to sulfapyridine (sulfonamide) → drug allergy, bone


marrow depression and megaloblastic anemia (avoided by giving folic acid).

Other aminosalicylates:
 Mesalazine: modified-release preparation.
 Olsalazine is a dimmer of 5-ASA. It is cleaved in the lower bowel to release 5-ASA.

2. Corticosteroids
Travellers’ diarrhea:
Mechanism
Diarrhea that occurs to
 Stimulation of Na+ absorption from the travellers and tourists in high
intestine. endemic areas. Transmission of
 Anti-inflammatory action (see endocrine). infection is done through
contaminated food or water.
Therapeutic uses: they are given orally or as Drug therapy:
enema in severe cases.  Prophylaxis: doxycycline
(100 mg /day orally)
 To control acute episodes of Inflammatory
 Treatment: doxycycline +
bowel diseases
bismuth subsalicylate +
 Refractory diarrhea unresponsive to other fluids
agents.

3. Immunosuppressive agents

 Cytotoxic drugs (azathioprine and 6-mercaptopurine): can be used in


ulcerative colitis and Crohn’s disease in small dose to avoid bone marrow
depression.
 Cyclosporine-A use is limited for severe cases due to potential toxicity.

N.B.
 Metronidazol may be used in Crohn’s disease to eradicate anaerobic bacteria.
 Infliximab (monoclonal antibodies): can be used in Crohn’s disease to ↓ TNFα.
 Aspirin and indomethacin may of value in acute diarrhea because they ↓ PGs
synthesis → ↑ absorption and ↓ secretions of intestinal fluids.
 Clonidine (α2 stimulant) can be used in diabetic diarrhea to ↑ intestinal water
absorption and ↓ electrolyte secretion.

273
Part 7: Drug Therapy of Gallstones (cholelithiasis)

Choleretics: are agents that increase bile production e.g. bile acids and bile salts.
Hydrocholeretics: are agents that increase volume of bile e.g. dehydrocholic acid.
Cholagogues: are agents that stimulate the evacuation of gall bladder e.g. olive oil.

The following drugs are used to dissolve non-calcified cholesterol stones:

Chenodeoxycholic acid (CDCA): It ↓ hepatic cholesterol synthesis and ↑ bile acid


and phospholipid synthesis.

Ursodeoxycholic acid (UDCA)


 It is a metabolite of chemodeoxycholic acid.
 It is more potent in reduction of hepatic cholesterol synthesis without affecting
the endogenous bile acid synthesis.
 Both CDCA and UDCA are given orally in a dose of 10-15 mg/kg/day for 6-12 month.
 Diarrhea is common side effect with CDCA but is unusual with UDCA.

274
275
Review Questions

1. Classify laxatives. Mention the mechanism of action of each class.


2. Mention the mechanism of action and adverse effects of each of the following:
– Metoclopramide.
– Cimetidine
– Omeprazole
– Senna
– Lomotil
3. Mention the mechanism of action and therapeutic uses of:
– Papaverine
– Sulfasalazine
– Terlipressin
4. Mention the drug-drug interactions of metoclopramide.
5. Mention 2 drugs used for the treatment of nausea and vomiting due to the
following conditions. For each drug, mention the mechanism of action and the
main adverse effects:
– Vomiting due to cancer chemotherapy
– Motion sickness
6. Give the reason for:
– Atropine is combined with diphenoxylate in Lomotil preparation.
– Domperidone, and not metoclopramide, is used for treatment of vomiting due
to levodopa.
– Irritant laxatives should not be given for long period.
– Folic acid should be supplemented with sulfasalazine during treatment of
ulcerative colitis and rheumatoid arthritis.
7. Mention 3 differences between:
– Metoclopramide and domperidone.
– Cimetidine and omeprazole.

8. Mention the mechanism and management of portosystemic encephalopathy.


9. Mention the essential lines of the treatment of bleeding esophageal varices.

276
Of each of the following questions, B. Sulfasalazine.
select ONE BEST answer: C. Sulfapyridine.
D. Sulfamethoxazole.
1. Toxicities of cimetidine include E. Salicylate sodium
which one of the following?
A. Blurred vision 7. An important drug in the therapy of
B. Diarrhea portal systemic encephalopathy is:
C. Orthostatic hypotension A. Lactulose.
D. P450 inhibition B. Lactate.
E. Sleepiness C. Loperamide.
D. Lorazepam.
2. Which of the following will result E. Doxapine.
from blockade of H2 receptors?
A. Decreased cAMP in cardiac muscle 8. Bismuth salts are thought to be
B. Increased cAMP in cardiac muscle effective in peptic ulcer disease
C. Decreased IP3 in gastric mucosa because they have bactericidal
D. Increased IP3 in gastric mucosa properties against:
E. Increased IP3 in smooth muscle A. Escherichia coli.
B. Bacteroides fragilis.
3. Which of the following is most C. Clostridium difficile.
effective in the treatment of peptic D. Helicobacter pylori.
ulcer disease? E. Staphylococcus aureus.
A. Cimetidine
B. Lanzoprazole 9. Misoprostol has a cytoprotective
C. Pirenzepine action on the gastrointestinal mucosa
D. Ondansetron because it:
E. LSD A. Enhances secretion of mucus and
bicarbonate ion.
4. A 55-year-old woman with insulin B. Neutralizes acid secretion.
dependent diabetes of 40 years’ C. Antagonizes nonsteroidal anti-
duration inflammatory drugs (NSAIDs)
complains of severe bloating and D. Relieves ulcer symptoms.
abdominal distress, especially after E. Coats the mucosa.
meals. Evaluation is consistent with
diabetic gastroparesis. The drug you 10. The primary pharmacologic action
would be most likely to recommend is: of omeprazole is the reduction of:
A. Docusate A. Volume of gastric juice.
B. Dopamine B. Gastric motility.
C. Loperamide C. Secretion of pepsin.
D. Metoclopramide D. Secretion of gastric acid.
E. Sucralfate E. Secretion of intrinsic factor.

5. The steatorrhea of pancreatic 11. Cimetidine slows the metabolism of


insufficiency can best be treated by: many drugs because it inhibits the
A. Cimetidine activity of:
B. Misoprostol. A. Monoamine oxidase (MAO)
C. Bile salts. B. Tyrosine kinase.
D. Pancreatic lipase. C. Hydrogen - potassium - adenosine
E. Secretin triphosphatase (H+, K+, ATPase).
D. Phase II glucuronidation reactions
6. A drug of choice in the therapy of E. Cytochrome P450.
inflammatory bowel disease is:
A. Sulfadiazine.

277
12. The absorption of iron is reduced D. A potent anti-anderogenic action.
when large and prolonged doses of E. None of the above.
which of the following drugs are given?
A. Vitamin C 18. Radiation-induced vomiting can be
B. Alum hydroxide. treated by drugs that act on:
C. Aspirin. A. 5-HT3 receptors.
D. Cimetidine. B. M1 receptors.
E. Lactuolse. C. H1 receptors.
D. Alpha receptors
13. Omeprazole, an agent for the E. Beta receptors.
promotion of healing of peptic ulcers,
has a mechanism of action that is 19. Cholesterol gallstones may be
based on: dissolved by oral treatment with:
A. Prostaglandins. A. Lovastatin.
B. Gastric secretion. B. Dehydrocholic acid.
C. Pepsin secretion. C. Methyl teriary butyl ether.
D. H+, K+, ATPase. D. Chenodeoxycholic acid.
E. Anticholinergic. E. Monoctanoin.

14. An effective antidiarrheal agent 20. A patient who must take verapamil
that inhibits peristaltic movement is: for hypertension and angina has
A. Clonidine. become constipated. Which of the
B. Bismuth subsalicylate. following drugs would be most suitable
C. Oral electrolyte Solution. as a long term laxative?
D. Pectin. A. Aluminum hydroxide
E. Diphenoxylate. B. Diphenoxylate
C. lactulose
15. The approved indication for D. Metoclopramide
misoprostol: E. Mineral oil
A. Reflux esophagitis.
B. Regional ileitis. 21. Your cousin is planning a three-
C. Ulcerative colitis. week trip overseas and asks your
D. Prevention of gastric ulceration in advice regarding medications for
patients using large doses of aspirin traveler's diarrhea. A drug suitable for
like drugs. non-infectious diarrhea is
E. Pathologic hypersecretory conditions A. Aluminum hydroxide
such as Zolinger- Ellison syndrome. B. Bismuth subcitrate
C. Magnesium hydroxide
16. Metoclopramide has antiemetic D. Metoclopramide
properties because it: E. Mineral oil
A. Lowers esophageal sphincter
pressure. 22. Which of the following drugs or
B. Is a central dopamine- receptor drug groups is not useful in the
antagonist. prevention of nausea and vomiting
C. Is a central opioid receptor agonist induced by cancer chemotherapy:
D. Has cholinomimetic properties. A. Dexamethasone
E. Has sedative properties. B. Dronabinol
C. Scopolamine
17. Fomatidine has the following D. Ondansetron
properties: E. Metoclopramide
A. A potent proton pump inhibitor.
B. A potent antiemetic agent.
C. A potent inhibitory effect on cyt P450.

278
23. A patient presents with Zollinger- 28. Bisacodyl frequently can cause:
Ellison syndrome due to a gastrinoma. A. Abdominal cramps
He has two bleeding ulcers and B. Constipation
diarrhea. A drug that irreversibly C. Skin rashes
inhibits the H+/K + ATPase in gastric D. Dizziness
parietal cells is E. Nauseas
A. Cimetidine
B. Cisapride 29. Patients with acute bleeding due to
C. Glycopyrolate ruptured esophageal varices could be
D. Omeprazole managed by:
E. Ondansetron A. i.v. terlipressin
B. i.v. sodium bicarbonate
24. A drug that is recently linked with C. Oral lanzoprazole
some cases of cardiac arrhythmias and D. i.v. hydrocortisone
sudden death is: E. Oral lactulose
A. Aluminum hydroxide
B. Lactulose 30. A patient being cared for by the
C. Loperamide gastroenterology service is being
D. Ranitidine treated with sulfasalazine. Which of the
E. Domperidone following is the most likely purpose for
which it is being given?
25. One recognized advantage of A. Antibiotic-associated
domperidone over metoclopramide as pseudomembranous colitis
a prokinetic agent is: B. E. coli–induced diarrhea
A. More prominent antiemetic action C. Gastric H. pylori infections
B. More powerful stimulant of GIT motility D. Inflammatory bowel disease
C. Less CNS adverse effects. E. NSAID-induced gastric ulcer
D. Less incidence of diarrhea prophylaxis
E. Less cardiac adverse effects

26. A cannabinoid receptors agonist


that is useful for prevention of nausea Answers
and vomiting due to cancer
chemotherapy is: 1D 7A 13 D 19 D 25 C
A. Dronabilone 2A 8D 14 E 20 C 26 A
B. Morphine 3B 9A 15 D 21 B 27 B
C. Diphenoxylate 4D 10 D 16 B 22 C 28 C
D. Naloxone 5D 11 E 17 E 23 D 29 A
E. Ondansetron 6B 12 B 18 A 24 E 30 D

27. A pregnant woman with 28 weeks


gestation complaining of distressing
constipation. Which of the following
drugs can be prescribed safely?
A. Ondansteron
B. Lactulose
C. Senna
D. Magnesium sulphate salt
E. Castor oil

279
280
Part 1
1: Diab
betes Me
ellitus a
and Antid
diabetic
c Drugs

Definittion of DM M: It is a clinical synd drome


charactterized by disturbance in carbo ohyd- Histolog
gy

rates, ffat, & pro oteins metabolism d due to There are


e 4 types off cells in the
e islets
either in ciency or ins
nsulin defic sulin resistaance. of Lange
erhans.

 α cellss (20%) : seecrete glucagon.


Manife
estations:  β cellss (75%) : seecrete insullin and
 Polyyuria, polyydipsia, polyphagia
p , and islet as
ssociated p polypeptide
losss of weightt. (amylinn).
 Com mplicationss of DM e.g. rec urrent  δ cellss (3-5%) : ssecrete
infe
ections, angiopathy
a y, retinoppathy, somattostatin.
nepphropathy, DKA, etc.  F cellss < 2 %: seecrete panc creatic
polype eptides (faccilitate dige
estive
proces ss).
Laboraatory invesstigationss:
 Urinne analysiss for glucosuria & kettonuria.
 Bloood glucose e: fasting and
a 2-hr affter oral 75
5 g glucose
e.
 Glyccated hemmoglobin (H HbA1C): in
ndicates avverage blo
ood sugar level for the past
2-3 months
Norm
mal IGT* DM
Fasting
g BG < 100
0 mg/dl 100-126 > 1226 mg/dl
2 hr aftter oral 75 g glucose < 140
0 mg/dl 140-200 > 2000 mg/dl
HbA1C < 6% 6-6.5% > 6.55%
Elevvated levels
s mean
unco ontrolled DM over
the llast 2-3 mo
onths.

* IGT = Impaired glucose tolerance


t es): it can be controlleed by diet regime
((prediabete
alone.

281
Common types of DM:
 Type 1 DM: (old name: insulin dependent DM)
 Type 2 DM: (old name: non-insulin dependent DM)
 Gestational DM: appears during pregnancy and disappears after labor.

Type 1 (IDDM) Type 2 (NIDDM)


Pathophysiology: Autoimmune condition The pancreas usually
against the pancreatic beta produces some insulin. But
cells. The damaged either the amount is not
pancreas doesn't make sufficient, or the body's
insulin. cells are resistant to it.
Onset (age): < 30 years >30 years (adult-onset)
Body built: Usually undernourished Usually obese
Insulin therapy: Essentially required. Required in 20% of patients
Oral drugs: Not required. Essentially required.

Less common types of DM:


 Latent autoimmune diabetes of adults (LADA): is a condition in which type 1
DM develops in adults.
 Maturity onset diabetes of the young (MODY): is a condition in which type 2
DM develops in children or young adults who develop persistent, asympt-
omatic hyperglycemia without progression to diabetic ketoacidosis.
N.B. Adults with LADA or children with MODY are frequently misdiagnosed as
having type 2 or type 1 DM, based on age rather than etiology.
 Secondary DM: 2ry to drugs, infections, endocrine diseases, etc.

Lines of treatment:
 Diet control.
 Diet + insulin (type 1).
 Diet + oral antidiabetic drugs (type 2).
 Exercise: to enhance peripheral glucose utilization.

█ DIET REGIME

It is the first line of treatment in all types of DM.

 Glycemic Index (GI):


– It describes the capacity of a food to raise blood glucose in normal glucose

282
tole
erant individ
duals.
– Fooods with a high GI may be e associated with ↑ risk of obesity and a the
posstprandial hyperglyca
h aemia, with so ↑ the rissk of T2DM
h such foods may als M.

Calo
oric requirrements of
o patients
s Distribution of foo d elementts
Averrage wt: 30--35 c/kg/d 15 % from
m proteinss
Child
dren or thin: 40--45 c/kg/d 55 % from
m carbohyydrates
Obe
ese: 20--25 c/kg/d 30 % from
m fats

█ INSU
ULIN

Chemistry
 2 peeptide chaains: A (21 aa) and B (30 aa)
linke
ed by 2 dissulfide bon
nds.
 Insuulin is sec
creted as proinsuliin then
cleaaved (within the sto orage gran
nules in
the β cells) in
nto insulinn and connnecting
pepptide “C-peeptide”.
B. Measu
N.B urable C-peptide
C levels
indicate the presence of endoge enously
prodduced insu
ulin and functioning β
β-cells.

Secrettion
 Glucose enterrs β cells → ↑ ATP → c
closure
+
of A
ATP-depen ndent K ch hannels →
opeening of vo
oltage-gateed Ca2+ cha
annels
→ ↑ Ca influx
2+
x → insulin release.

Insulin
n receptors
s
- A tyyrosine kin ptor consistts of 2 extra
nase recep acellular
ubunits and
α-su d 2 transme
embrane β
β-subunits.
- Bind
ding of inssulin to the
e α-subunitts causes
actiivation of the
t β-subu units → activvation of tyrosine
kinaase enzym me which triggers
t a sseries of
acellular efffects that ↑ number of glucos
intra se
trannsporters (especially y GLUT4) o on the celll
mem mbrane → ↑ transport of glucosse into the cell.

283
Up-regulation of insulin receptors: Down-regulation of insulin receptors:
 Insulin deficiency (e.g. in fasting)  Insulin excess (Obesity).
 Drugs: INSULIN SENSITIZERS: Obesity causes continuous
a) Thiazolidinediones: e.g. stimulation of β-cells →
pioglitazone & rosiglitazone. hyperinsulinemia → down regulation
b) Trace elements: e.g. selenium & of insulin receptors.
chromium.  Drugs: e.g. corticosteroids

Pharmacokinetics of insulin
 Endogenous insulin:
- The pancreas releases insulin in the portal vein in small bouts at short
intervals according to blood glucose levels. The liver clears ~ 50% of the
secreted amount; the rest is distributed all over the body and cleared by
receptor-mediated uptake and intracellular degradation.
- The normal blood level of insulin is 5-15 U/ml (fasting) and 60-90 U/ml
(postprandial).
 Exogenous insulin:
- Insulin is not administered orally because it is rapidly destroyed by GIT
enzymes. It is given s.c. (common) or i.v. (in emergencies).
- The plasma half-life of insulin given intravenous is 10-12 min.

Sources of commercial insulin

 Traditional (animal) insulin:


- Prepared from animals (beef and pork).
- POrk insulin differs from human insulin in One aa (alanine in pork and threonine
in human) while bEEf insulin differs from human in thrEE aa.
- Animal insulin may contain traces of animal proteins and proinsulin, so it is more
antigenic and can cause insulin resistance.

 Human insulin:
- It is identical to human insulin and is prepared by recombinant DNA technology
(genetic engineering) from yeasts or bacteria.
- Advantages: Less antigenic and rare development of insulin resistance.

 Insulin analogs:
- Recombinant insulin analogs are now available in which some amino acids in the
"normal" insulin have been switched (e.g. insulin lispro) or replaced (e.g. insulin
glargine), making a “different molecule” so as to get different pharmacokinetic
properties.

284
285
Insulin
n administration
 All iinsulins aree given by s.c. injecttion.
 Reg gular insulin is the only
o type thhat can be
e given i.v. in diabeticc emergencies.
 Thee standard insulin co oncentratioon is 100 units/mL
u (U
U-100). It sshould be injected
i
with
h a standard U-100 syringe.
s

Insulin
n requirem
ment and lo
ong term rregimens
 Gennerally, the
e total daily insulin rrequiremen
nt in units is equal tto 0.55 tim
mes the
persson’s weight in kilo ograms. A conserv vative total daily doose (TDD) of 0.4
unitts/kg/d is given inittially to a newly diagnosed patient; tthe dose is then
adjuusted accoording to th
he blood gglucose levvel.

 Bas
sal-bolus regimen:
r
- GGive long g acting insulin at bed time; plus thre ee daily innjection of
o short
aacting ins sulin beforre each me eal. The lo
ong-acting insulin pro
ovides bas sal level
oof insulin that
t contro
ols blood gglucose du uring night and in-bettween mea als, and
tthe short acting
a insu
ulin control s postpran
ndial hyperrglycemia.
- It is comm monly used for patien nts with Typpe-1 DM and
a in preg gnant wom men with
ddiabetes.

 Twiice-daily biphasic
b in
nsulin reg imen:
- Use biphaasic insulin. The averrage daily requiremeent
is 0.5 unitss/kg. Use the
t two-thiirds rule:
- GGive the 2/3
2 of the TDDT in the
e morning and the 1/3
a
at eveningg.
- It is commmonly used for patien ts with Typpe-2 DM

 ulin pum
Insu mp (artifficial pa ancreas): insulin is
auto
omatically released d accord ding to continuous
meaasurementt of blood glucose
g byy electronic
c sensor.

Methods of adm
ministration
- S.c. injection (using insu
ulin syringe
es).
- Porrtable pen injector (Novo® pen)).

Follow
w up of insulin therapy
By esttimation ofo glucose in urin ne using urine
dipstick cose level using
ks or from capillary blood gluc
portablle glucomeeters.

286
Indicattions of in
nsulin
 Typ
pe 1 DM.
 Typ
pe 2 DM in some con nditions:
- AAfter failurre of oral drugs.
d
- If the patieent of type 2 got “strress condiitions” e.g
g. infectionns, surgery,, or
p
pregnancyy.
 Diabetic ketoa
acidosis: regular
r inssulin is the e only type used i.v.
 Hyp a (insulin + glucose i.vv.): insulin ↑ shift of K+ from blo
perkalemia ood to tissues.

se effects of insulin
Advers
- Hyp
poglycemiia: the mos
st commo n and dangerous sid
de effect.
- Hyppokalemia a: insulin caauses shiftt of K+ from extra- to
o intracelluular fluid.
- Hyppersensitiv vity reactiions: urtica
aria, angio
oedema or anaphylacctic shock.
- Insu
ulin resisttance (see below).
- Loc
cal advers
se effects:

 A
Allergy: att the site of
o injection , especially mal insulin..
y with anim
T
Treatmentt:
- Change the type of insulin.
- Local corticoster
c roids.

 Lipodystrophy: (atrophy or hypertrophy)


of s.c. tisssue after re
epeated inj ections.
T
Treatmentt: rotating thet injectio
on site.

 Local infe
ection.

▌Insuliin resistan
nce (IR)

Definittion: failure
e of the boody cells tto respond d to either endogeno ous or exoogenous
insulin. As a result, larger doses of inssulin are re
equired to give the d esired resp
ponse.

Causes
s and mec
chanism of
o IR:
- Pre-receptorss defect (IImmunologgical): due ation of a ntibodies against
e to forma
insu
ulin.
- Recceptor defeect: due too down-re egulation ofo insulin receptors ee.g. in:
- The metabolic
m syndromee: it is a combination of central obesity,o
hyperc
cholesterollemia, hearrt disease, type2 DMM and IR.
- Pregnaancy.
- Severee infection or stress.
- Drugs: e.g. cortic costeroids .

287
- Post-receptor defect (intracellular): abnormal signal transduction due to genetic
defect.
- Local insulin resistance: due to degradation of insulin in s.c. tissue by proteolytic
enzymes. It is diagnosed by changing the route of administration to i.v. injection.
If there is good response, then the resistance is local.

Treatment of IR: Hepatitis C makes


people three to four times
 Life-style modification: more likely to develop
- Strict diet control. type 2 DM and insulin
- Weight reduction. resistance.
- Physical exercise.

 Correction of any precipitating factor: e.g. infection or stress.


 Metformin: to improve receptor sensitivity and ↓ intestinal glucose
absorption (see later).
 Thiazolidindiones (Insulin sensitizers): e.g. pioglitazone to ↑ insulin
sensitivity (see later).
 Change the site of injection regularly In cases of local insulin resistance

█ ORAL ANTIDIABETIC DRUGS

1. Sulfonylureas

 Sulfonylureas are the most widely used medications for the treatment of T2DM.
 All of the sulfonylureas are well absorbed after oral administration and bind to
plasma proteins.

Classification
 First-generation compounds: chlorpropamide, tolbutamide, acetohexamide.
 Second-generation compounds: glibenclamide, glyclazide and glipizide; they
are up to 200 times more potent than first-generation agents.
 Third-generation compounds: e.g. glimepiride; these compounds may interact
with different cellular proteins than other sulfonylureas.

Mechanism of action
 Pancreatic action:
- Sulfonylureas ↑ insulin secretion by pancreatic β cells. They block K+
channels in β cells, leading to depolarization, increased Ca2+ entry via
voltage-dependent calcium channels, and increased insulin secretion.
- They ↓ serum glucagon, which opposes the action of insulin.

288
 Extrapancreatic action:
- They ↑ insulin receptor sensitivity, may be by increasing the number of
insulin receptors. However, sulfonylureas do not decrease the insulin
requirements of patients with type I diabetes.
- They ↓ hepatic output of glucose (gluconeogenesis).

Therapeutic uses
 Type 2 DM (they are not effective in type 1 DM).
 Chloropropamide may be used in treatment of nephrogenic diabetes insipidus
(it ↑ sensitivity of renal tubules to ADH).

Adverse effects
- Hypoglycemia especially with long acting drugs (chlorpropamide) or in elderly
patients with hepatic or renal dysfunction .. (contraindicated in renal failure).
- Increased appetite and weight gain.
- Pharmacologic failure is common, initially affecting 15%–30% of patients (1ry
failure) and 90% after 6–7 years of therapy (2ry failure). It is due to progressive
decline in β cell function.
- Hepatotoxicity (cholestatic hepatitis) .. (contraindicated in liver disease).
- Allergic reactions.

2. Meglitinides: repaglinide and nateglinide

 They increase insulin secretion by the same mechanism like sulfonylureas


although the binding site on the β-cells is different.
 They have very fast onset and short duration so; they are taken orally just before
meals to control postprandial hyperglycemia.
 They don't contain sulfur, so they can be used in patients allergic to
sulfonylureas.
 Hypoglycemia is the major side effect.
 They should be used with caution in patients with hepatic or renal impairment.

3. Biguanides: metformin

Mechanism of action
 ↓ intestinal glucose absorption and ↓ hepatic gluconeogenesis. It does not
increase insulin secretion (so it doesn’t cause hypoglycemia).
 ↑ insulin receptors sensitivity.

289
Therapeutic uses
 Type 2 DM either alone (in mild cases) or in combination with other drugs.
 To enhance weight loss in obese patients (↓ glucose absorption).
 Recent studies showed that metformin is useful in the treatment of polycystic
ovary syndrome (POS); it lowers serum androgen levels and restores normal
menstrual cycles and ovulation.

Adverse effects
- GIT upset (the most common): anorexia, vomiting, and diarrhea.
- ↓ absorption of vitamin B12: rarely a clinical problem.
- Lactic acidosis: due to increased anaerobic glycolysis especially in patients
with severe renal or hepatic diseases or if taken with alcohol.

4. Thiazolidinediones (insulin sensitizers): Pioglitazone – Rosiglitazone*

Mechanism of action
 They act on nuclear genes called peroxisome proliferator-activated receptor-
gamma (PPAR- γ) present in muscles, adipose tissue, and liver cells leading to:

- They ↑ insulin receptor sensitivity (by about 60%) and ↓ insulin resistance.
- They ↑ number of glucose transporters →↑ glucose uptake.
- They have beneficial effect on serum lipoprotein levels (↓TGs).
 They have slow onset and prolonged duration because their mechanism
involves gene regulation.

Therapeutic uses: To improve insulin resistance in type 2 DM.

Adverse effects
- Hepatotoxicity
- Fluid retention leading to peripheral edema & weight gain. (They should be
avoided in patients with CHF).

*N.B pioglitazone is now the only thiazolidinedione on the market.

5. α-Glucosidase inhibitors (starch blockers): Acarbose – Miglitol

 They act by competitive inhibition of intestinal α-glucosidase enzyme → ↓


digestion & absorption of carbohydrates.
 GIT side effects are common: flatulence, diarrhea, abdominal pain.
 They should be avoided in patients with inflammatory bowel disease.

290
█ NEWER ANTIDIABETIC DRUGS

Incretins are group of metabolic peptides released by gastric and intestinal cells
after eating to stimulate insulin secretion and lower plasma glucose. They are rapidly
inactivated by the enzyme dipeptidyl peptidase 4 (DPP-4). The most important
incretin peptide is glucagon-like peptide-1 (GLP-1).

1. Incretin (GLP-1) mimetics: Exenatide

Exenatide is a synthetic 39-amino acid GLP-1 analog; liraglutide is longer acting


analog and more resistant to metabolism.

Mechanism of action
 They ↑ insulin secretion and ↓ glucagon secretion.
 They slow gastric emptying and ↓ appetite.

Therapeutic uses
 Type 2 DM either alone or in combination with oral drugs. (It should not be given
with insulin).
 They are given parenterally (s.c.) 60 min before breakfast and dinner. (It should
not be given after a meal).

Major adverse effects: nausea, vomiting, and pancreatitis (may be serious).

2. Dipeptidyl peptidase 4 inhibitors (Gliptins): Sitagliptin - Saxagliptin

Mechanism of action
 They inhibit dipeptidyl peptidase 4 (DPP-4), the enzyme responsible for the
proteolysis of the incretin GLP-1.
 DPP-4 inhibitors may also improve β-cell function.

Therapeutic uses
 Type 2 DM either alone or in combination with metformin.
 They are given orally; most common side effects are headache and nausea.

3. Amylin analogs: Pramlinitide

 Pramlintide is a synthetic amylin analog. Amylin is a polypeptide secreted by β-


cells of the pancreas, and it acts in concert with insulin to reduce blood sugar.
 It is injected s.c., typically with insulin.
 Common side effects are hypoglycemia and nausea.

291
Drug interactions with oral hypoglycemic drugs

Drugs potentiate the hypoglycemic Drugs antagonize the hypoglycemic


effect: effect:
 Microsomal enzyme inhibitors.  Microsomal enzyme inducers.
 β-blockers: ↓ hepatic  β-agonists: ↑ hepatic glycogenolysis.
glycogenolysis  Thiazides and diazoxide: they open
 Salicylates: displacement of ATP sensitive K+ channels → ↓ insulin
sulfonylureas from plasma proteins. release and ↓ peripheral glucose
utilization.
 Anti-insulin hormones: e.g. steroids
and glucagon.

█ TREATMENT OF DIABETIC COMPLICATIONS

▌Hypoglycemic coma

Causes:
- Large dose of insulin or sulfonylurea.
- Missed meal while taking insulin or sulfonylureas.
- Vigorous muscular exercise without dietary adjustment.
Manifestations:
- Sympathetic overactivity: tremors, cold sweating, mydriasis, tachycardia, etc.
- Hunger pain, mental confusion, and coma.
Treatment:
 If the patient is conscious or semiconscious → give oral sugar solution.
 If the patient is in deep coma: (1) i.v. glucose 10%; (2) Glucagon 1 mg i.m.

▌Hyperglycemic coma (diabetic ketoacidosis; DKA):

Causes:
- Excess food intake.
- Inadequate insulin administration.
- Infection or stress condition.
Manifestations (DKA):
- Dehydration and dry shrunken tongue.
- Ketosis: smell of acetone in breath and ketones in urine.
- Acidosis (pH < 7.3) → acidotic breathing (rapid and deep).
- Deep coma in late cases.

292
Treatment:
 Fluid replacement: fluid deficit = 4-5L.
– Use normal (0.9%) saline. Ringer’s lactate is also acceptable choice.
– Start with 1–2 L/h for the first 2 hours, followed by 500 mL/h for 6 hours.
– Potassium: is routinely added to the i.v. fluid as 20 mEq KCl/1L.

 Regular insulin:
– Start with a bolus of dose of 0.1 units/kg i.v. followed by an infusion of 0.1
units/kg/hour.
– Insulin infusion should continue until serum electrolytes, pH, and glucose
level are normalized.
– Glucose 5 % is given when blood glucose falls to ~250 mg/dl to prevent
cerebral edema.

 Bicarbonate (HCO3): is not routinely given unless there is severe acidosis


(pH < 7.1).

▌Hyperosmolar hyperglycemic state (HHS):

 HHS was previously termed hyperosmolar hyperglycemic nonketotic coma


(HHNC); however, the terminology was changed because coma is found in fewer
than 20% of patients with HHS.
 It is a life-threatening medical emergency that is more common in old age.
 Pathogenesis: severe hyperglycemia causes ↑ water loss (osmotic diuresis) with
little sodium loss → severe dehydration
with hyperosmolarity. Ketoacidosis is DKA HHS
absent. BG > 250 mg/l > 600 mg/l
pH < 7.3 > 7.3
Treatment: similar to DKA except in: Ketonuria ++ --

 Fluid replacement: fluid deficit


>5L.
– Rapid and aggressive i.v. isotonic saline is the principal treatment. The
patient may require 5-8 L of saline.
– Fluid replacement alone can reduce glucose levels.
 Insulin i.v.i similar to DKA. Insulin used without concomitant vigorous fluid
replacement increases the risk of shock.
 Hypokalemia is usually less marked but potassium may be required early in
the treatment.
 LMWH to prevent thrombotic complications.

293
▌Diabetic neuropathy

Diabetic neuropathy is the most common complication of diabetes mellitus (DM),


affecting as many as 50% of patients with type 1 and type 2 DM. It may be cranial,
sensory, sensory-motor or autonomic. DN is an irreversible process.

Pathogenesis
 Advanced glycated end-products: elevated intracellular levels of glucose cause
a non-enzymatic covalent bonding with cellular proteins, causing microvascular
damage, ischemia, hypoxia, and nerve damage.

 Polyol pathway (sorbitol/aldose reductase pathway): elevated intracellular


glucose is converted into sorbitol via the enzyme aldose reductase. Sorbitol is
toxic to tissues.

Treatment
 Tight, stable glycemic control is the most important factor for slowing
progression of DN.
 For neuropathic pain, and tingling sensation: duloxetine (antidepressant drug)
30 mg/12h orally (first choice) or pregabalin (second choice).
 For diabetic gastroparesis: metoclopramide
 For erectile dysfunction: sildenafil

294
Part 2
2: Med
dical Tre
eatmentt of Obe
esity

█ Bas
sic inform
mation

Leptin
 It is thought to
o play a ke
ey role in thhe regulation of body
y weight.
 It iss produced (synthes sized) by adipose tissue
t and
d acts on satiety centers in
the hypothala amus to ↓ appetite
a (i.e
e. Leptin induces saatiety).
 As such whe en patientss reach a certain peripheral
p fat mass, leptin actts as a
lipoostat to red
duce food d intake.

Ghrelin
n
 Ghrrelin stimu ulates hunnger.
 It is producedd mainly byy the fundu
us of the sttomach an
nd pancreaas.
 Ghrrelin levels ↑ before meals
m and ↓ after me
eals.

The bod
dy-mass in
ndex (BMI) – (Kg/m2)

 Norrmal: 18-25
5
 Ove
erweight: 25-30
0
 Obeese: >30
 Morrbid obesitty: >40

295
▌Medical treatment

1. Orlistat (Xenical®)

 Orlistat is a pancreatic lipase inhibitor. It inhibits digestion and absorption of


dietary fats leading to weight loss.
 Orlistat is normally used for <1 year. Patients fail to lose at least 5% of their
bodyweight within 3 months, orlistat should be discontinued.
 Adverse effects include steatorrhea and flatulence.

2. Lorcaserin (Belviq®)

 It is recently approved selective 5-HT2C receptor agonist in the hypothalamus


leading to weight loss through satiety.
 It is used for long term control of obesity.
 Adverse effects are mild and include mainly headache.

3. Liraglutide (Saxenda®)

 It is an injectable long acting glucagon-like peptide-1 receptor agonist, binding


to the same receptors as does the endogenous hormone GLP-1 that stimulates
insulin secretion.
 It is developed principally for treatment of type 2 DM. In 2015, it was approved
for treatment of morbid obesity in adults.

4. Sibutramine (Meridia®)

 Centrally acting appetite suppressant related to amphetamine (inhibits uptake


of 5-HT and noradrenaline at hypothalamic sites that regulate food intake).
 Adverse effects: hypertension, arrhythmia, insomnia, and headache.
 It has been withdrawn in 2010 from the USA due to an increased risk of
cardiovascular events, but still available in many countries including Egypt and
Middle East.

296
Part 3
3: Thyroid Gla
and and Antithy
yroid Dru
ugs

Genera
al principles
 Thee major hormone
h secreted
s b
by the
thyrroid is thyroxine
t (T4), whiich is
deioodinated in many tissues tto the
morre potent triiodothyrronine (T3)). Both
are bound reversibly y to thyrroxine-
bindding globuulin (TBG)). Only th
he free
(unbbound) frraction en nters cellss and
produces biollogical effe
ects.
 Thee most imp ep in the p rocess
portant ste
of thyroid hormone
h synthesis
s is oxidation (orga anification)) of iodid
des by
perroxidase enzyme
e to
t produc ce molecu ular iodine
e, which tthen attac ches to
tyro
osine to foorm monoo- and diiiodotyrosin ne. T4 is formed b by couplinng of 2
diio
odotyrosine
es, and T3 by couplinng of diiod
dothyrosine
e with monno-iodotyro osine.
 T4 secretion is stimula
ated by thyyroid-stimulating ho
ormone (TS
SH). In turrn, TSH
sec
cretion is in
nhibited by
y T4, formin
ng a negattive feedba
ack loop.
 Thee gland syn
nthesizes T4
T > T3 (20
0:1) but T3
3 is 4-times
s more pottent than T4.
T
 Mosst of the cirrculating T3 is derived
d from peripheral deio
odination o
of T4.
 B-b
blockers an
nd corticos
steroids inhibit periph
heral conve
ersion of T44 into T3.

Preparrations of thyroid ho
ormones
 L-th
hyroxine: a synthetic sodium
salt of T4 tha
at maintain ns normal
T4 aand T3 levels (t ½ is 7 days).
 Liotthyronine:: a synthetic
sod
dium salt off T3 (t ½ is
s 1 day).

anism of action
Mecha a
 T3 & T4 diffuse through
h the cell
memmbranes and bind
b to
cyto
oplasmic receptorrs then
tran
nsported too the nucleus and
mitochondria a, where it interacts
with
h many DNA receptors
(genes) and affect
a their function.
 Mosst of T3 & T4 recepto
ors are fouund in pituitary, liver, kidney, heeart, sk ms
s.
 T3 & T4 are re
esponsible
e for optima al growth and
a develo opment.

297
▌Hypo
othyroidis
sm (myxedema)

 Hyp
pothyroidissm in infants lead
ds to
cretinism (m myxedema with phyysical
andd mental re etardation)..
 It iss treated by replacement witth L-
thyrroxin (T4). The thera apeutic go
oal is
'norrmalizationn' of the TS
SH level.
 Children requ uire more T4T than a adults
to maintain optimal physical and
men ntal develo
opment.
 T3 (Liothyronine) is nott used forr replacem
ment therappy becausse L-thyroxin (T4)
cann maintain normal T4 and T3 levvels, and also
a becau
use T3 has shorter t½
½.

▌Hype
erthyroidiism (thyro
otoxicosi s)

Definittion: It is a clinical sy
yndrome rresults from
m high leve
els of thyro
oid hormon
nes.

Clinica
al types

 Gra
aves’ diseaase:
- It is autoim
mmune dis
sease in wh
hich there are
a abnorm
mal
antibodiess (thyroid stimulating
s g immunog globulins)
activates TSH recep ptors in the
e thyroid gland.
g
- The glandd is diffuse
ely enlargedd and soft.
- The treatm
ment is usually medi cal.
 Toxxic multino
odular goiiter: treatm
ment is usu
ually surgic
cal.
 Lesss commo
on types: e.g.
e subacu
ute (de
The terrm “goiter”
Que
ervain’s) th
hyroiditis. It is due to viral infection. means any thyroid d
swellinng which is
Clinica
al picture neitherr inflammatoory
nor neo oplastic. Th
hyroid
 Mosst manifesstations off hyperthyyroidism re
esemble enlargeement may occur
symmpathetic overactivity be
ecause thyroxin
t with hyypo- or
hyperthhyroidism.
incrreases sen gic -recep
nsitivity off adrenerg ptors to
circ
culating cattecholamin nes.
 Theere are tach
hycardia, arrhythmia
a a, sweating
g, lid lag, ex
xophthalm
mos, etc.

Investiigations
- Meaasuring serrum T3, T4, and TSH (the mostt important test): T4 is ↑ and TS
SH is ↓.
- Asssay for possitive anti-thyroid anttibodies: 90
0% +ve in Grave's d
disease.
- Thyyroid scan for tumorss.

298
Management

There are 2 main lines for treatment:

 Medical treatment: mainly for Grave’s disease.


 Surgical thyroidectomy: mainly for multiple nodular goiter

█ MEDICAL TREATMENT (ANTITHYROID DRUGS)

1. Thiouracil drugs (thioamides)


(Carbimazole – Methimazole - Propylthiouracil)

Pharmacokinetics
 Methimazole is the active metabolite of carbimazole.
 The t1/2 of propylthiouracil is 1.5 hr while the t1/2 of methimazole is 6 hrs.
 The short t½ of these drugs has little effect on their effect because they are
selectively accumulated in the thyroid.
 Propylthiouracil is preferable during pregnancy because it does not cross
placental barrier (because it is strongly bound to plasma protein).

Mechanism of action
 They inhibit oxidation of iodides into iodine by inhibiting peroxidase enzyme.
Consequently, they inhibit iodination of tyrosine and coupling of iodotyrosine to
form iodothyronines.
 Propylthiouracil also inhibits the peripheral conversion of T4 into T3.
 Clinical response appears after 3-4 weeks till the stored hormones are depleted from
the gland (but propylthiouracil has faster effect, so it is used in thyrotoxic crisis).

Doses and duration


 Carbimazole (Neomercazole): start with 30 mg/d till reach euthyroid state (after
~4-8 weeks) then maintain on 15 mg /d for 12-18 months (until the gland
undergo spontaneous remission).
 Propylthiouracil: start with 300 mg/d for 4-8 w then 150 mg/d for 12-18 months.

Adverse effects
– Agranulocytosis & bone marrow depression (<1%): it is the most dangerous
side effect but it is usually reversible (see chapter Blood).
– Hypothyroidism with increased size and vascularity of the gland due to ↑ TSH.
– Hypothyroidism of the infant (fetal goiter) if given during pregnancy (less
common with PTU).

299
– Hepatotoxicity that may be fatal (more with propylthiouracil).
– Hypersensitivity reactions: may require stopping of the drugs.
– There is 50-68% incidence of relapse.

Precautions during thiouracil treatment


 Therapy with thiouracil drugs should continue for 1-2 years and should be
stopped gradually (to prevent relapse).
 Agranulocytosis is prevented by repeated WBC count and by observing early
manifestations of the disease (sore throat & fever).
 If used during pregnancy, propylthiouracil is the drug of choice.

2. Radioactive iodine

Radioactive iodine is an effective oral treatment for thyrotoxicosis caused by toxic


multinodular goiter. It is relatively ‘safe’ and has largely replaced surgery (unless
there is huge gland causing pressure symptoms).

Mechanism of action
 The isotope usually employed is 131I with a t ½ of 8 days.
 Radioactive 131I is selectively accumulated in the thyroid tissue (normal or
metastatic) and emits β rays that destroy the gland. After 6-12 weeks of
administration, the gland will shrink in size and the patient becomes euthyroid.
 Delayed hypothyroidism is the main adverse effect (80%); the majority of
patients will require thyroxin supplementation after 5 years.

Contraindications
– Pregnancy and lactation: 131I crosses placental barrier and excreted in milk.
– Age < 16 years for fear of delayed malignant changes and gonadal damage.
– Thyroid eye disease (exophthalmos; ophthalmopathy): radioiodine may
worsen the condition.

3. -blockers (Propranolol)

 It controls sympathetic overactivity e.g. tachycardia and arrhythmia.


 It ↓ insomnia, and tremors (blocks central and peripheral β2 receptors).
 It inhibits peripheral conversion of T4 to T3.
N.B. If propranolol is contraindicated, give diltiazem to control tachycardia and
arrhythmia (other CCBs are less effective).

300
█ SURGICAL TREATMENT: subtotal thyroidectomy

Indications
 Failure of the medical treatment.
 Multinodular goiter with tracheal compression.
 Presence of malignancy.

Preparation of patient before operation


 Carbimazole: 10 mg t.d.s. 6 weeks before the operation to reach euthyroid
state.
 Potassium iodide: 10-14 days before operation to ↓ size and vascularity of
the gland (see below).
 Propranolol: to control HR and cardiac arrhythmia.
 Sedatives (diazepam): to ↓ anxiety.

Iodides

 Potassium iodides or Lugol’s iodine, 5 drops twice daily is given 10-14 days
before surgery in order to:
– Iodides inhibit synthesis and release of T4 & T3 from the gland by inhibiting
the proteolytic enzymes that release T4 & T3 from thyroglobulin (the main
mechanism).
– They inhibit release of TSH leading to ↓ size and vascularity of the gland.
 Improvement in thyrotoxic symptoms occurs within 2–7 days, but if therapy with
iodides is continued (>2-4 weeks), the beneficial effects disappear and
manifestations of hyperthyroidism reappear (iodine escape).

Adverse effects
– Iodides are secreted in saliva, nasal, and lacrimal secretions causing metallic
taste and irritation of the salivary glands, mucous membranes and gastric
mucosa. They increase lacrimal and nasal secretions.
– Iodine escape if used > 2-4 weeks.
– Iodides increase intraglandular stores of iodine, which may impair uptake of
thiouracil drugs or radioactive iodine by the gland. So, during preoperative
preparation, they must be given after thiouracil drugs (or radioactive iodine),
not before them.
– Allergic reactions: skin rash, drug fever, etc.

301
▌Thyrotoxic crisis (thyroid storm)

It is a sudden severe exacerbation of the manifestations of thyrotoxicosis due to


sudden release of T3 & T4 (medical emergency). It is a common postoperative
complication if the patient was not well-prepared.

Manifestations – High fever with vomiting and sweating.


– Tachycardia, arrhythmia, acute heart failure, and shock.
– Convulsions, coma, and even death from heart failure.
Management
 Intravenous fluids and Paracetamol to control dehydration and fever.
N.B. Aspirin must be avoided, because salicylate displaces bound T4 and T3,
and also because of its uncoupling effect on oxidative phosphorylation renders
the metabolic state even more severe.

 Propranolol (1-2 mg slowly i.v. or 40 mg oral /6 h) to control arrhythmia.


– Esmolol is a short-acting -blocker can be also given by i.v.i.
– If -blockers are contraindicated give diltiazem by i.v. infusion.
 Potassium iodides: 10 drops orally/day to block hormone release and
peripheral conversion of T4 to T3.
 Propylthiouracil: 250 mg/6 hrs orally to block hormone synthesis. It acts more
rapidly than other thiouracil drugs.
 Hydrocortisone: 100 mg i.v./6-8 h to elevates BP and reduces toxemia. It also
blocks peripheral conversion of T4 to T3.
 Plasmaphoresis in severe cases to reduce circulating thyroxin.

▌Management of myxedema coma

Myxedema coma is an end state of untreated hypothyroidism. There is progressive


weakness, hypothermia, hypoventilation, hypoglycemia, hyponatremia, water
intoxication, shock, and death. It carries high mortality and is a medical emergency.

Management
 Hospitalization (ICU): artificial respiration may be required.
 L-thyroxine (T4): 400 μg i.v. initially, followed by 50 μg daily orally. Intravenous
T3 can be used.
 Hydrocortisone: 100 mg i.v./ 6-8 h because the patient usually has associated
adrenal insufficiency.
 Intravenous fluids with caution to avoid excessive volume overload.

302
Part 4: Adrenocortical Steroids

Classification:  Glucocorticoids: have mainly anti-inflammatory action.


 Mineralocorticoids: have mainly Na & water retaining action.
 Sex hormones: are mainly anabolic.

Glucocorticoids

Anti-inflam Na+ retention Duration


potency potency of effect
Short-acting glucocorticoids:
Hydrocortisone (cortisol) 1 1 8-12 h
Prednisone and Prednisolone 4 0.5
Methylprednisolone 5 0.1
Intermediate-acting glucocorticoids:
Triamcinolone 5 0.1 12-36 h
Paramethasone 10 0.01

Long-acting glucocorticoids:
Dexamethasone 30 0.01 48 h
Betamethasone 30 0.01

Aldosterone: 0.3 3000

 Hydrocortisone (cortisol) is the natural (endogenous) glucocorticoid.


 Equivalent dose means 20 mg of hydrocortisone = 5 mg of prednisolone.
 Steroids with high anti-inflam action (e.g. dexamethasone) are better used for
inflammatory conditions e.g. rheumatic fever, while those with high Na & H2O
retention (hydrocortisone) are better used for treatment of hypotension and
shock states (they also have rapid action).

Pharmacokinetics
 All glucocorticoids are completely & rapidly absorbed by all routes.
 80% of hydrocortisone is bound to plasma globulin, 10% to albumin.
 Plasma t ½ varies according to type (60-90 min for hydrocortisone). However, the
effect of glucocorticoids is prolonged due to its effect on gene functions.
 Metabolism is by the liver and excretion is by the kidney.

Mechanism of action
Corticosteroids bind first to cell surface receptors then to cytoplasmic receptors

303
(carrierss), then transported
d to the nucleus, where it interacts with man
ny DNA
recepto ors (steroid
d response
e elementss) and affect their fun
nction.

Pharm
macologica
al effects:

 On m
metabolis
sm: (Cushiing syndro
ome):
- CCarbohydrrate metab
bolism: hyyperglycem
mia (↓
pperipheral glucose uttilization).
- PProtein metabolism
m m: Catabo olic effect → ↓
mmuscle ma ass and thin limbs.
- FFat metabo olism: ↑ lip
polysis withh redistribution of
ffat (moon fa
ace & buffaalo hump).
- OOn water and
a electro olyte balannce: Na+ & water
rretention and
a hypoka alemia.

 Antii-inflamma
atory and anti-imm unologica
al
effe
ects:
- T They inhibbit B cell function n → ↓ an ntigen-
a
antibody re
eaction.
- T nctions → ↓ inflam me
They inhibiit T cell fun ediators annd cytokinee release.
- T They inhibiit macroph hage activitty and stabilize lysos
somal mem mbranes.
- T They inhibiit mast cells → ↓ hista
amine relea ase and ca apillary perrmeability.
- T They inhibiit phospho olipase A2 enzyme → ↓ synthesis of PGs & LTs.

 On C
CVS: Hypeertension due
d to:
- N +
Na & wateer retentionn.
- Increase seensitivity of
o BV and h
heart to cirrculating catecholam
mines.

 Antii-shock efffects: due


e to:
- HHypertensiive and CV VS effects (mention them).
- AAnti-inflam
mmatory ac ction (menttion them).

 Hem
matologicaal effects:
- ↑ RBCs and neutroph hils and ↓ lymphocyttes and eosinophils.
- ↑ coagulation factorss and bloodd choleste
erol.

 On g
growth: Growth
G reta
ardation, w
which is nott prevented by growtth hormon
ne.

 On b
bone: ↓ bo
one matrix and ↑ Ca22+ excretion
n (osteopo
orosis).

Administration
- Drug
g administtration sho
ould follow
w the circa hm: A douuble dose is given
adian rhyth
in th
he morning
g, and a sin
ngle dose is given in the afternoon.

304
- Alternate-day therapy is clinically effective with minimal effect on the adrenal-
hypothalamic-pituitary axis. In this therapy, double the dose of short- or
intermediate-acting glucocorticoids is administered every other day.
- Glucocorticoids should be stopped gradually after long term administration.

Therapeutic uses

 Inflammatory disorders: e.g. iridocyclitis, neuritis, dermatitis, etc.


 Autoimmune disorders: e.g. vasculitis, rh fever, rh arthritis, hemolytic anemia,
nephrotic syndrome, ulcerative colitis, etc.
 Allergic disorders: e.g. anaphylactic shock, urticaria, eczema, bronchial asthma,
allergic conjunctivitis, etc.
 Malignancy: e.g. lymphoma, leukemias, multiple myeloma, etc.
 Shock and hypotension…..Why?
 Organ transplantation: as immunosuppressive to prevent graft rejection.
 Cerebral edema: dexamethasone is used after brain surgery to minimize
inflammatory edema associated with tissue injury.
 Acute hypercalcemia: to enhance Ca2+ excretion.
 As replacement therapy in acute and chronic adrenocortical insufficiency
(Addison’s disease).

N.B. Acute adrencortical insufficiency (acute Addisonian crisis):


- Prolonged corticosteroid therapy produces feedback inhibition of ACTH and
inhibition of endogenous cortisone secretion. When exogenous
corticosteroids are suddenly stopped, severe hypotension and shock occurs.
- Management:
- i.v. fluids (saline).
- Hydrocortisone 100 mg i.v./8h until the patient is stable.
- ACTH: 0.5 mg i.m.

 Stimulation of lung maturation in the fetus:


Lung maturation in the fetus is regulated by fetal secretion of cortisol. When
delivery is anticipated before 34 weeks of gestation, betamethasone is given to
reduce the incidence of respiratory distress syndrome in the infant.
Betamethasone is chosen because its maternal protein binding is less than other
steroids, allowing increased transfer across the placenta to the fetus.

Adverse effects
Most of these side effects occur after long duration of therapy:

305
- Iatrogenic Cushing syndrome: occurs if doses up to 100 mg hydrocortisone are
used daily for > 2 weeks. It is characterized by moon face, buffalo hump, thin
limbs, osteoporosis, hypertension, DM, edema, etc.
- Immune suppression leading to flaring of infections (especially viral and TB).
- Hypertension due to salt & water retention
- Hyperglycemia.
- Peptic ulcer: due to prolonged inhibition of gastroprotective PGs.
- ↑ IOP (Glaucoma): due to ↓ aqueous humor drainage.
- Osteoporosis.
- Growth retardation in children.
- Skin atrophy & hypopigmentation after prolonged topical use.
- Sudden withdrawal after prolonged administration causes acute addisonian
crisis.

Contraindications
- Presence of infections: especially viral
N.B. Uses of corticosteroids in
infection and TB.
presence of T.B:
- DM. - TB meningitis: to prevent
- Hypertension & heart failure: they cause adhesions.
salt and water retention. - TB of the suprarenal gland: to
- Peptic ulcer: they ↓ synthesis of PGE2 and replace hypofunction.
I2 that protect the stomach. - Miliary TB: to ↓ TB toxemia.
- In early pregnancy: may cause cleft palate.

▌Mineralocorticoids

1. Natural mineralocorticoids: Aldosterone

Physiological actions: It binds to specific intracellular receptors in the DCT to


inhibit Na+ excretion (hypernatremia) and stimulates K+ and H+ excretion
(hypokalemia).

2. Synthetic mineralocorticoids:

 Deoxycorticosterone acetate (DOCA): it has mainly mineralocorticoid effect.


 Fluodrocortisone: it has both gluco- and mineralocorticoid effects.

306
307
308
309
▌Hype
ercalcemiia

Causess: In over 90% of caases hyperrcalcemia is i due to either


e hypeerparathyrroidism
or malignancy (eespecially myeloma) . Hypercallcemia norrmally sup presses PTH P and
so PTH
H is therefo
ore the bes
st first test to identity
y the cause
e of hyperccalcemia.

Manag
gement of acute hyp
percalcem
mia
- Saline diuresis: 500-1000 ml/houur plus furrosemide to
t increasse urine flo
ow and
enhance Ca2+ excretion..
- Hyddrocortison
ne: 100 mg hance Ca2++ excretion
g i.v. to enh n.
- Intraavenous bisphosphoonates.
- Hemmodialysis especially
y when ren al failure is
s present.

▌Hypo
ocalcaemia

Causess: Hypopa arathyroidissm (N.B. ttetany occcurs when serum Caa2+ falls < 7mg/dl),
7
chronic
c renal failu
ure, vitamin D deficie
ency, etc.

Treatm
ment
- Diett rich in calcium and low in pho osphate
- Calccium gluco onate: slow
wly i.v. (in a
acute condditions).
- amin D: to ↑ Ca abs
Vita +2
sorption fro om intestin
ne.
- Thia
azide diure etics.

▌Osteo
oporosis

Definittion: it is a condition
n of low bo
one mass that results in fractuures with minimal
trauma
a. It occurss in postme enopausal women (d
due to estro
ogen lack) and in old d age.

Preven
ntion and treatment
t t of osteop
porosis
- Diett rich in calcium.
amin D: to ↑ Ca+2 abs
- Vita sorption frrom intestin
ne
- Sele
ective Esttrogen Re
eceptor M
Modulators en): to ret ain the be
s (Raloxife eneficial
effec
cts of estro
ogen on bone while minimizing
g the risk of
o cancer b
breast and uterus.
- Bisp
phosphonates (e.g. risedron
nate): they preventt bone reesorption (inhibit
osteeoclastic activity) and
d reduce riisk of hip and
a spine fractures.
f T
They are effective
e
in bo
oth men an nd women n for variou
us causes of
o osteopo
orosis.
- Calc
citonin: to
o increase bone
b masss and redu
uce fracture
es.
- Teriiparatide: a recombin
nant form o
of PTH tha
at has been
n recently aapproved fo
or treat-
men
nt of osteop
porosis. It stimulates
s n
new bone fo
ormation and reducess risk of frac
ctures.

310
- Slow e fluoride preparatiion: is a
w release
new
w treatmennt. It may
y reduce rates in
posttmenopausal osteop
porosis.

Part 6
6: Sex Hormon
nes

I. Estro
ogen

 Nattural estro
ogens: estrradiol, estrrone and estriol
e
 Sem
misynthetiic estrogeens: Ethinyyl estradiol and mestrranol.
 Syn
nthetic esttrogens: diethyl
d stilb
bosterol.

Physio
ologic effe
ects
 Normmal develoopment of genital tra
act and breeast.
 Devvelopment of ♀ secon ndary sex characters s.
 Metabolic effe
ects:
- IIncrease bone mass and preveent bone re esorption.
- IIncrease blood gluco ose and TGGs.
- S Salt and water
w retenttion.
 Increase bloodd coagulattion and pl atelet adhesiveness..

Therap
peutic use
es
 C
Contracepptive pills.
 Dysfunctio
onal uterine e bleeding .
 Replacement therapy in ovaria an hypofunnction.
 Postmenopausal sym c vaginitis and osteo
mptoms e..g. atrophic oporosis.
 C
Cancer proostate.

Advers ons: see contracepti


se effects and contrraindicatio c ive pills.

▌Anti--estrogen
ns:

1. Clomiphene citrate (C
Clomid)

 Clom
miphene blocks
b estrogen rece
eptors in hypothalam pituitary → ↑ FSH
mus and p
and LH → stim mulate ovulation.
 It is used to sttimulate ovulation
o i n infertile women
w witth normal p
pituitary fu
unction.
 Adv verse effeccts: Ovaria
an enlargem ment and hot flushess.

311
2. Selective estrogen receptor modulators (SERMs):

 SERMs are ligands for the estrogen receptor that have agonist activity in one
tissue but may have antagonist activity or no activity in another tissue.
 Currently, there are three SERMs: tamoxifen, raloxifene, and toremifene.

Tamoxifen
 It is an estrogen antagonist in the breast but is an agonist in the uterus and
bone.
 It is used in the treatment of advanced, estrogen receptor positive breast
cancer and for primary prevention of breast cancer in women at high risk.
 Adverse effects: tamoxifen increases the risk of endometrial cancer and
thrombotic complications.

Raloxifene
 It is an agonist in bone but has no effect on the uterus or breast.
 It is used for the treatment and prevention of postmenopausal osteoporosis.
 Adverse effects: hot flashes and thrombotic complications.

3. Aromatase inhibitors:

 Aromatase is the enzyme that catalyzes the production of estrogens from


androgenic precursors within the ovary and peripheral tissues.
 Aromatase inhibitors are a new class of oral estrogen synthesis inhibitors.
 Anastrazole and letrozole (Femara) are nonsteroidal competitive inhibitors of
aromatase. Exemestane is a steroidal, irreversible aromatase inhibitor.
 These drugs are used for treatment of postmenopausal women with estrogen-
receptor positive breast cancer who have received two to three years
of tamoxifen and are switched to them for completion of a total of five years of
adjuvant hormonal therapy.

II. Progesterone and progestins

 Natural: progesterone injection


 Synthetic: medroxy progesterone acetate.

Therapeutic uses
 Contraceptive pills.
 Dysfunctional uterine bleeding
 Dysmenorrhea and endometriosis.
 Threatened abortion.

312
Adverse effects
- Breakthrough bleeding.
- Increase risk of birth defects if given in early pregnancy.
- Liver dysfunction.

▌Antiprogesterone: Mifepristone

 It is a competitive blocker of progesterone receptors.


 It is used with PGF2α to induce medical abortion in the first trimester.

III. Androgens and anabolic steroids

 Natural androgens: androsterone and testosterone


 Synthetic androgens: testosterone propionate.
 Anabolic steroids: nandrolone and stanazol.

Uses of androgens and anabolic steroids:


 Chronic debilitating diseases e.g renal failure.
 Chronic refractory anemia: e.g. sickle cell anemia, aplastic anemia.
 Illicit use by athletes: to increase muscle bulk and strength.

Adverse effects
- Reduction in spermatogenesis after stopping.
- Precocious puberty and premature closure of epiphysis in children.
- Cholestatic jaundice.
- Verilizing effects in females.

Contraindications
- Prostatic tumors (benign and malignant).
- Liver diseases
- Children.

▌Antiandrogens

1. 5 α-reductase inhibitors: Finasteride

 It inhibits 5 α-reductase enzyme responsible for conversion of testosterone into


the active form dihydrotestosterone (DHT).
 It is used in:

313
 Treatment of benign prostatic hyperplasia.
 Treatment of male baldness.
 Treatment of hirsutism in females.

2. Testosterone receptor blockers:

- Cyproterone acetate: is a competitive blocker of testosterone receptors. It


is used in male hypersexuality, and hirsutism in females.

- Flutamide: used in cancer prostate.


- Others: Spironolactone is a competitive blocker of both aldosterone and
testosterone receptors. It is used in hirsutism in females.

█ HORMONAL CONTRACEPTIVES

Types of hormonal contraceptives

 Combined preparations (the most effective type): contain both estrogen +


progesterone given from the 5th day of menstruation for 21 days.

 Single entity preparations:


 Progesterone alone (minipills): Don't affect lactation and don't carry risk of
thrombosis but can cause uterine bleeding.
 Estrogen alone (postcoital pills or morning-after pills): It is used within 72 hrs
after sexual intercourse for 5 days.
 Slow release progestins: Medroxyprogesterone acetate (Depo-Provera®) given
i.m. every 3 months. It is suitable for unreliable women.
 Implantable progestin preparation.

Mechanism of action
 They inhibit ovulation by exerting –ve feedback on LH (progesterone) and FSH
(estrogen) secretion.
 Produce endometrial changes and interferes with coordinated contraction of
the cervix, uterus, and fallopian tubes → ↓ sperm transport and fertilization.
 Increase viscosity of cervical mucus to inhibit sperm penetration.

Adverse effects
CVS: the most serious side effects especially in women above 35 years and in
women who are smokers:
- Hypertension and increase risk of myocardial infarction.

314
- Thrombosis and thromboembolic catastrophes.
- Increase TGs levels.

CNS:
- Migraine headache.
- Cerebral hemorrhage (stroke) is 2-10 times higher.
- Mood changes and depression.

GIT:
- Nausea and vomiting.
- Cholecystitis and gall stones.
- Cholestatic hepatitis and hepatotoxicity.

Endocrinal:
- Hyperglycemia and DM.
- Weight gain and edema due to salt and water retention.
- Inhibition of lactation in lactating women.
- Menstrual irregularities: spotting bleeding, breakthrough bleeding, amenorrhea,
and dysmenorrhoea.
- Loss of libido, acne, and hirsutism.

Cancer: increased risk of breast cancer.

Contraindications
- Hypertension or ischemic heart disease (IHD).
- History of embolism, thrombosis or cerebral hemorrhage.
- History of cancer breast or estrogen-dependent neoplasm.
- Migraine headache.
- Chronic liver disease and gall stones.
- Diabetes mellitus.
- Obese, smokers, or women over 35 years.
- Pregnancy.
- Depression.

N.B Causes of failure of contraceptive pills:


 If taken with enzyme inducers e.g. rifampin, phenytoin, etc.
 If woman taking broad spectrum antibiotics e.g tetracyclines (see breakthrough
pregnancy in general pharmacol).
 Paraffin oil (laxative) ↓ intestinal absorption of contraceptive pills.

315
Part 7: Hypothalamic and Pituitary Hormones

█ ANTERIOR PITUITARY HORMONES

▌Growth hormone (GH)

GH promotes longitudinal bone growth and cartilage synthesis through stimulation


of hepatic synthesis of insulin-like growth factors (somatomedins).

Therapeutic uses
- Replacement therapy in children with GH deficiency.
- Illicit use by athletes to increase body mass.

Adverse effects:
- Children may develop scoliosis during rapid growth.
- Peripheral edema and carpal tunnel syndrome.
- Hypothyroidism and gynecomastia.

▌Growth hormone antagonists: Octreotide

- It is a synthetic somatostatin analog.


- It is more potent and has longer duration than somatostatin.

Therapeutic uses
- Acromegaly.
- Hormone-secreting tumors (e.g. insulinoma, glucagonoma, gastrinoma, etc).
- Bleeding esophageal varices (given by i.v.i., it causes VC of splanchnic bl vessels
and controls variceal bleeding with fewer side effects than vasopressin; see GIT).

Adverse effects: Bradycardia and conduction disturbances.

▌Adrenocorticotrophic hormone (ACTH)

ACTH is secreted by the pituitary to stimulate release of glucocorticoids,


meniralocorticoids, and sex hormones. It also stimulates MSH → skin pigmentation.

Therapeutic uses
 To stimulate secretion of endogenous corticosteroids.
 Diagnostic: differentiation between 1ry or 2ry adrenal insufficiency.

316
▌Gonadotropin-releasing hormone (GnRH)

 GnRH is secreted by the hypothalamus to stimulate release of LH and FSH from


anterior pituitary.
 Pulsatile injection of GnRH causes stimulation of LH & FSH but continuous
infusion of GnRH causes inhibition of LH & FSH release.

Therapeutic uses
 Treatment of infertility caused by hypogonadism in both sexes by pulsatile
injection of GnRH (to stimulate LH & FSH).
 Treatment of prostate cancer, endometriosis, and polycystic ovary syndrome by
continuous administration of GnRH (to inhibit LH & FSH).
 In-vitro fertilization (IVF) programs.

▌Follicle-stimulating hormone (FSH)

Therapeutic uses
 Treatment of infertility caused by hypogonadism in both sexes.
 To stimulate ovulation as a part of in-vitro fertilization (IVF) programs.

Adverse effects
- Hyperstimulation syndrome (enlarged ovaries, ascites, fever, embolism, etc.)
- Gynecomastia in men.

▌Human chorionic gonadotropin (hCG)

It is glycoprotein produced by the placenta to stimulate ovarian corpus luteum to


secrete progesterone (like LH). hCG can be used as LH substitute.

Therapeutic uses
 Treatment of infertility caused by hypogonadism in both sexes.
 To stimulate ovulation as a part of in-vitro fertilization (IVF) programs.
 Differentiation between undescended testes (cryptorchidism) and retracted
testes (pseudo-cryptorchidism).

317
█ POSTERIOR PITUITARY HORMONES

▌Antidiuretic hormone (Vasopressin; ADH)

 It acts on 2 types of receptors: V1 and V2:


- V1: present in vascular sm ms → VC and spasm.
- V2: present in the renal collecting tubules → ↑ water reabsorption.

Therapeutic uses
 Desmopressin: synthetic long acting analog given by nasal administration in:
- Diabetes insipidus (cranial type only): its action on renal V2 receptors is 3000
times more potent than on vascular V1 receptors.
- Nocturnal enuresis: by reducing nighttime urine production.
- Hemophilia: because it stimulates hepatic synthesis of factor VIII
and endothelial cells to secrete von Willebrand factor.

 Terlipressin (Glypressin): used by i.v.i. to control acute variceal bleeding.


Octreotide, vasopressin, and terlipressin have been shown to have efficacy in the
control of acute variceal hemorrhage. Terlipressin, however, is the only
medication that has been shown to improve patient survival.

Adverse effects
- Facial pallor and hypertension due to cutaneous VC.
- Coronary spasm.

▌Oxytocin

It causes milk ejection from the breast and uterine contraction at labor.

Therapeutic uses
 Induction and maintenance of labor (10-20 units by i.v.i).
 Control of postpartum hemorrhage.
 To stimulate milk secretion in nursing mothers (nasal spray)

Adverse effects and contraindications


- Rupture uterus in cephalopelvic disproportion (obstructed labor).
- Fetal asphyxia (from uterine spasm).
- Hypertension and cardiac arrhythmia

318
319
Review Questions

1. Classify types of insulin; mention its indications and side effects.


2. Classify types of oral antidiabetic drugs; mention their mechanism of action,
indications and side effects.
3. Classify antithyroid drugs; mention their mechanism of action, side effects, and
precautions of each group.
4. Classify glucocorticoids; mention their therapeutic uses, common side effects
and contraindications.
5. Give full account on pharmacological regulation of bone calcium homeostasis.
6. Classify oral contraceptives; mention their mechanism of action, side effects
and causes of failure.

Short questions:
1. Insulin resistance: definition, mechanism, and management.
2. Mention uses and side effects of sulfonylureas.
3. Mention drug interactions of oral hypoglycemic drugs.
4. Mention therapeutic uses and side effects of vasopressin.
5. Mention side effects (or contraindications) of oxytocin.
6. Mention drug interactions of vit D.
7. Mention treatment of osteoporosis.
8. Mention medical treatment of obesity.
9. Mention antiestrogens and their therapeutic uses.
10. Mention antiandrogens and their therapeutic uses.
11. Mention contraindications of anabolic steroids.
12. Mention side effects (or contraindications) of contraceptive pills.
13. Mention causes of failure of contraceptive pills.

Mention lines of treatment of the following emergency conditions:


1. Diabetic ketoacidosis.
2. Thyrotoxic crisis.
3. Myxedema coma.
4. Acute addisonian crisis.
5. Acute hypercalcemia.

320
Of each of the following questions,
select ONE BEST answer: 5. Which of the following
glucocorticoids is an intermediate-
1. Glucocorticoids are hormonal acting drug?
steroids: A. Cortisone
A. Having an important effect on immune B. Triamcinolone
function C. Butamethasone
B. Having principally salt-retaining D. Prednisolone
activity E. Dexamethasone
C. Having androgenic or estrogenic
activity 6. Immunosupressive effect of
D. Having hypercalcemic activity glucocorticoids is caused by:
E. Having hyperkalemic activity A. Reducing concentration of
lymphocytes and inhibiting function of
2. Correct statements about cortisol tissue macrophages and other
(hydrocortisone) include all of the antigen-presenting cells
following, EXCEPT: B. Suppression of cyclooxygenase II
A. Cortisol is synthesized from expression which results in reducing
cholesterol amount of an enzyme available to
B. ACTH governs cortisol secretion produce prostoglandins
C. Most cortisol is inactivated in the liver C. Activation of phospholipase A2 and
D. Cortisol has equal anti-inflammatory reducing prostaglandin and
and salt-retaining activity leukotriene synthesis.
E. The half-life of cortisol in the D. Activation of angiotensin-converting
circulations is normally about 60 enzyme
hours. E. Suppression of histamine release

3. Correct statements about 7. Indication of glucocorticoids


glucocorticoids include all of the include of the following EXCEPT:
following, EXCEPT: A. Chronic (Addison’s disease) and acute
A. Effects of glucocorticoids are adrenocortical insufficiency
mediated by widely distributed B. Organ transplants (prevention and
glucocorticoid receptors that are treatment of rejection –
members of the superfamily of nuclear immunosuppression)
receptors. C. Inflammatory conditions of bones and
B. Glucocorticoids have high plasma joints (arthritis, bursitis, tenosynovitis).
protein binding D. Hypocalcemia
C. Glucocorticoids have dose-related E. Gastrointestinal diseases
metabolic effects on carbohydrate, (inflammatory bowel disease)
protein, and fat metabolism.
D. Glucocorticoids have pro- 8. Indication for 1,25-dihydroxyvitamin
inflammatory effects. D3 (calcitriol) administration is:
E. Glucocorticoids have catabolic effects A. Vitamin D resistance
in lymphoid and connective tissue, B. Elevated skeletal turnover
muscle, fat, and skin. C. Hypercalcemia of malignancy
D. Hypophosphatemia
4. Which of the following E. Primary hyperparathyroidism
glucocorticoids is relatively a short-
acting drug? 9. Indication for risidronate
A. Prednisolone administration is:
B. Dexamethasone A. Failure of vitamin D formation in skin
C. Triamcinolone B. Hypoparathyroidism
D. Paramethasone C. Elevated skeletal turnover
E. Betamethasone D. Hypophosphatemia

321
E. Metastatic bone disease A. Estrogens
B. Fluorides
10. Correct statements about fluoride C. Parathormone
include all of the following, EXCEPT: D. Bisphosphonates
A. Fluoride is effective for the prophylaxis E. Calcitonin
of dental caries
B. Fluoride is accumulated by bone and 16. Which of the following is an
teeth, where it may stabilize the important effect of insulin?
hydroxyapatite crystal A. Increased conversion of amino acids
C. Subjects living in areas with naturally into glucose
fluoridated water (1-2 ppm) had more B. Increased gluconeogenesis
dental caries and fewer vertebral C. Increased glucose transport into cells
compression fractures than subjects D. Inhibition of lipoprotein lipase
living in nonfluoridated water areas E. Stimulation of glycogenolysis
D. Chronic exposure to very high level of
fluoride results in thickening of the 17. Which of the following agents
cortex of long bones and bony should be administered to achieve
exostoses. rapid control of the severe ketoacidosis
in a diabetic boy?
11. Which one of the following is most A. Regular insulin
likely to be useful in the therapy of B. Glyburide
hypercalcemia? C. Insulin glargine
A. Calcitonin D. NPH insulin
B. Glucocorticoids E. Tolbutamide
C. 1-25 dihydroxy vitamin D3
D. Parenteral infusion of phosphate 18. Which of the following is the most
E. Thiazide diuretics likely complication of insulin therapy?
A. Dilutional hyponatremia
12. Which of the following conditions is B. Hypoglycemia
an indication for the use of calcitonin? C. Increased bleeding tendency
A. Chronic renal failure D. Pancreatitis
B. Hypoparathyroidism E. Severe hypertension
C. Intestinal osteodystrophy
D. Paget’s disease of bone 19. A 24-year-old woman with type 1
E. Rickets diabetes wishes to try tight control of
her diabetes to improve her long-term
13. Which of the following drugs can prognosis. Which of the following
cause rickets in children by increasing regimens is most appropriate?
Vitamin D metabolism? A. Morning injections of mixed insulin
A. Tetracycline lispro and insulin aspart
B. Phenylbutazone B. Evening injections of mixed regular
C. Phenytoin insulin and insulin glargine
D. Ciprofloxacin C. Morning and evening injections of
E. Ibuprofen regular insulin, supplemented by small
amounts of NPH insulin at mealtimes
14. Bone resorption is accelerated by: D. Morning injections of insulin glargine,
A. Estrogens supplemented by small amounts of
B. Fluorides insulin lispro at mealtimes
C. Parathormone E. Morning injection of NPH insulin and
D. Bisphosphonates evening injection of regular insulin
E. Calcitonin
20. Which one of the following drugs
15. Osteonecrosis of the jaw may be an promotes the release of endogenous
adverse effect of: insulin?

322
A. Acarbose 25. Which of the following patients is
B. Pioglitazone most likely to be treated with
C. Glipizide intravenous glucagon?
D. Metformin A. An 18-year-old woman who took an
E. Miglitol overdose of cocaine and now has a
blood pressure of 190/110 mm Hg
21. The combination of metformin and B. A 27-year-old woman with severe
ethanol increases the risk of which of diarrhea caused by a flare in her
the following? inflammatory bowel disease
A. A disulfiram-like reaction C. A 57-year-old woman with type 2
B. Excessive weight gain diabetes who has not taken her
C. Hypoglycemia glyburide for the last 3 d
D. Lactic acidosis D. A 62-year-old man with severe
E. Serious hepatotoxicity bradycardia and hypotension resulting
from ingestion of an overdose of
22. Which of the following drugs is atenolol
most likely to cause hypoglycemia E. A 74-year-old man with lactic acidosis
when used as monotherapy in the as a complication of severe infection
treatment of type 2 diabetes? and shock
A. Acarbose
B. Rosiglitazone 26. In Graves’ disease, the cause of the
C. Glyclazide hyperthyroidism is the production of an
D. Metformin antibody that does which of the
E. Miglitol following?
A. Activates the pituitary thyrotropin-
23. Which of the following drugs is releasing hormone (TRH) receptor and
taken during the first part of a meal for stimulates TSH release
the purpose of delaying the absorption B. Activates the thyroid gland TSH
of dietary carbohydrates? receptor and stimulates thyroid
A. Acarbose hormone synthesis and release
B. Exenatide C. Activates thyroid hormone receptors in
C. Glipizide peripheral tissues
D. Pioglitazone D. Binds to thyroid gland thyroglobulin
E. Repaglinide and accelerates its proteolysis and the
release of its supply of T4 and T3
24. The PPAR-γ receptor that is E. Binds to thyroid-binding globulin
activated by thiazolidinediones (TBG) and displaces bound T4 and T3
increases tissue sensitivity to insulin by
which of the following mechanisms? 27. Methimazole reduces serum
A. Activating adenylyl cyclase and concentration of T3 primarily by which
increasing the intracellular of the following mechanisms?
concentration of cAMP A. Accelerating the peripheral
B. Inactivating a cellular inhibitor of the metabolism of T3
GLUT2 glucose transporter B. Inhibiting the proteolysis of thyroid-
C. Inhibiting acid glucosidase, a key binding globulin
enzyme in glycogen breakdown C. Inhibiting the secretion of TSH
pathways D. Inhibiting the uptake of iodide by cells
D. Regulating transcription of genes in the thyroid
involved in glucose utilization E. Preventing the addition of iodine to
E. Stimulating the activity of a tyrosine tyrosine residues on Thyroglobulin
kinase that phosphorylates the insulin
receptor 28. Though rare, a serious toxicity
associated with the thioamides is
which of the following?

323
A. Agranulocytosis 33. A 62-year-old woman presents with
B. Lupus erythematosus-like syndrome complaints of fatigue, sluggishness,
C. Myopathy and weight gain. She needs to sleep
D. Torsades de pointes arrhythmia several times a day, which is unusual
E. Thrombotic thrombocytic purpura for her. She has been taking T4 for the
(TTP) past 15 yr without significant problems
regarding her energy level. Her recent
29. A 65-year-old man with history is significant for diagnosis of
multinodular goiter is scheduled for a arrhythmia, and she is currently taking
near-total thyroidectomy. Which of the an antiarrhythmic drug. What is the
following drugs will be administered for most likely cause of her current
10–14 d before surgery to reduce the condition?
vascularity of his thyroid gland? A. Amiodarone
A. Levothyroxine B. Lidocaine
B. Liothyronine C. Procainamide
C. Lugol’s solution D. Sotalol
D. Prednisone E. Verapamil
E. Radioactive iodine
34. A 25-year-old woman presents with
30. Which of the following is a sign or insomnia and fears she may have
symptom that would be expected to “something wrong with her heart.” Lab
occur in the event of chronic overdose tests confirm hyperthyroidism. Which
with exogenous T4? of the following is a drug that produces
A. Bradycardia a permanent reduction in thyroid
B. Dry, puffy skin activity?
C. Large tongue and drooping of the A. 131I
eyelids B. Methimazole
D. Lethargy, sleepiness C. Propylthiouracil
E. Weight loss D. Thiocyanate
E. Thyroglobulin
31. When initiating T4 therapy for an
elderly patient with longstanding 35. Glucocorticoids have proved useful
hypothyroidism, it is important to begin in the treatment of which of the
with small doses to avoid which of the following medical conditions?
following? A. Chemotherapy-induced vomiting
A. A flare-up of exophthalmos B. Essential hypertension
B. Acute renal failure C. Hyperprolactinemia
C. Hemolysis D. Parkinson’s disease
D. Overstimulation of the heart E. Type II diabetes
E. Seizures
36. A patient presents with pain and
32. A 27-year-old woman underwent stiffness in his wrists and knees. The
near total thyroidectomy. She was stiffness is worse first thing in the
started on levothyroxine. What morning. A blood test confirms
hormone is produced in the peripheral rheumatoid arthritis. You advise a short
tissues when levothyroxine is course of steroids. Which one of the
administered? following is the most potent anti-
A. Methimazole inflammatory steroid?
B. T3 A. Cortisol
C. T4 B. Dexamethasone
D. TSH C. Fludrocortisone
E. FSH D. Prednisone
E. Triamcinolone

324
37. A 34-year-old woman with 40. Which of the following drugs is
ulcerative colitis has required long- most likely to lower patient’s serum
term treatment with pharmacologic PTH concentration?
doses of a glucocorticoid agonist. A. Calcitriol
Which of the following is a toxic effect B. Cholecalciferol
associated with long-term C. Furosemide
glucocorticoid treatment? D. Gallium nitrate
A. A lupus-like syndrome E. Risedronate
B. Adrenal gland neoplasm
C. Hepatotoxicity 41. The patient began therapy with a
D. Osteoporosis nasal spray containing a protein that
E. Precocious puberty in children inhibits bone resorption. The drug
contained in the nasal spray was which
38. Which of the following drugs is of the following?
most useful for the treatment of A. Calcitonin
hypercalcemia in Paget’s disease? B. Calcitriol
A. Fluoride C. Cinacalcet
B. Hydrochlorothiazide D. Cortisol
C. Pamidronate E. Teriparatide
D. Raloxifene
E. Teriparatide
Answers
39. The active metabolites of vitamin D
act through a nuclear receptor to 1A 10 C 19 D 28 A 37 D
produce which of the following effects? 2E 11 B 20 C 29 C 38 C
A. Decrease the absorption of calcium 3D 12 D 21 D 30 E 39 D
from bone 4A 13 C 22 C 31 D 40 A
B. Increase PTH formation 5B 14 C 23 A 32 B 41 A
C. Increase renal production of 6A 15 D 24 D 33 A
erythropoietin 7D 16 C 25 D 34 A
D. Increase the absorption of calcium 8D 17 A 26 B 35 A
from the GIT 9E 18 B 27 E 36 B
E. Lower the serum phosphate
concentration

325
326
Part 1
1: CNS
S Stimullants

Classiffication:

█ RES
SPIRATOR
RY STIMUL
LANTS (A
ANALEPTIC
CS)

Definittion: drugss that stimu


ulate the d
depressed respiratory
y center (R
RC).

Classiffication:
 Spe
ecific analleptics:
 Naloxon and
a nalorph hine → usedd to treat opiates
o res
spiratory d
depression.
 Flumazenil → used to
o treat ben
nzodiazepin nes toxicity
y.

 Non
n-specific
c analeptic
cs:
 Direct RC stimulatio
on: e.g. Xa
anthines (c
caffeine, theophyllinne) – Etham
mivan -
Heptamino
ol. They an
ntagonize tthe GABA--mediated RC depresssion.
 Indirect RC
C stimulation (reflex)): e.g. Nico
otine and lobeline. T
They stimulate RC
indirectly through
t stiimulation o
of the chem
morecepto
ors in the ccarotid bod
dy.
 Both (direcct and indirect): Nikeethamide - Doxapram
m

Therap es: all case


peutic use es of RC ddepression
n (postanaesthetic, C
CNS depre
essants,
COPD and some premature e babies)

327
Adverse effects
– Tachypnea, tachycardia and hypertension.
– High doses can cause convulsions.

Contraindications
– Epilepsy (to avoid CNS stimulation).
– Severe hypertension, arrhythmia or IHD (to avoid cardiac arrhythmia).
– Thyrotoxicosis.
– Severe bronchial asthma.

█ PSYCHOMOTOR STIMULANTS

Definition: drugs that induce euphoria with increased motor activity.

Classification

 Amphetamine and related drugs: see ANS

 Cocaine
– Cocaine inhibits tissue uptake of catecholamines.
– Behavioral effects of cocaine are very similar to amphetamine.
– Cocaine is used topically as a local anesthetic eye drops.
 Xanthines: see respiratory pharmacology.

 Antidepressants: see later.

█ PSYCHOTOMIMETIC DRUGS (HALLUCINOGENS)

Definition: drugs that affect thought, perception and mood with no effect on
motor activity. They have no clinical applications but important for drug abuse.

Examples and Mechanisms


 LSD (Lysergic acid diethylamide): It stimulates central 5-HT receptors and
affects catecholamine action → severe hallucinations and delusions resembling
acute schizophrenia.

 Cannabis (hashish, marijuana): It acts on cannabinoid receptors in the CNS


(CB1) causing loss of judgment of time & place and dream-like state. It does not
cause physical dependence but psychological dependence. CB2 receptors are
found in peripheral tissue and immune system.

 Phencyclidine: It stimulates -opioid receptors and blocks the NMDA receptors


and may interact with other neurotransmitter systems.

328
Part 2
2: Ana
algesics

Definittion: drugss that reliev


ve or decre
ease pain sensation.
s

▌Class
sification of
o analges
sics:
 Opiioid (narco
otic) analg
gesics
 NSA
AIDs (see chapter 4).
 Ana ntipyretics: e.g. para
algesic an acetamol, Dipyron
D & Nefopam.
N
 Dru f speciffic painfull conditions e.g. ca
ugs used for arbamazep
pine for trig
geminal
neu
uralgia, ergotamine fo
or migraine
e.

█ OPIOID ANAL
LGESICS

 Theey are dru


ugs that decrease p
pain sensa
ation
withhout loss of o conscio ousness annd can ind duce
phyysical depe endence.
 Opiiates: are drugs derived from m opium plant
p
(Pappaver som miniferum).. They in nclude ma ainly
morrphine, cod deine and papaverin e.
 Opiioids: are drugs
d with
h morphinee-like activity.
 Opiiopeptins:: are endo ogenous p peptides with
w
opiooid-like acttivity e.g. en
ndorphins & enkepha alins
 Narrcotics: arre drugs th hat producce narcosiss i.e.
drowsiness or o stupor, with ana lgesia (stu upor
mea ans marke ed impairm ment, but not comp plete
losss of consciousness). These dru ugs are usu ually
adddictive. It is a legal, not a mediccal, term.

Classiffication off opioid drrugs

329
Mechanism of action
 Opioids such as morphine are believed to interact with three major receptors (μ,
δ, κ). Each opioid receptor has distinct subtypes (e.g., μ1, μ2). All three major
receptors are present in high concentrations in the dorsal horn of the spinal cord.
 Interaction with μ-receptors contributes to supraspinal and spinal analgesia,
respiratory depression, sedation, euphoria, decreased GI peristalsis, and
physical dependence (addiction).
 The significance of interaction with κ-receptors is unclear, but it may contribute
to analgesia (through inhibition of release of substance P at dorsal horn).

█ NATURAL OPIOID AGONISTS

1. Morphine

Pharmacokinetics
 Oral absorption: good (bioavailability is 25% due to significant first-pass effect).
However, the analgesic effect is greater when the drug is administered parentrally
 t½ : 4-5 hrs.
 Metabolism: in the liver by conjugation leading to inactive metabolites.
 Excretion: renal (90%) – bile (10% as conjugated morphine).

Pharmacological effects

 Analgesia
 Dose-dependent analgesia (sensory & emotional): consciousness is not
lost and the patient can still locate the source of pain. Analgesia may be
associated with euphoria and decreased anxiety.
 Analgesia results from direct activation of μ and δ receptors in the spinal cord
and possibly higher centers (thalamaus) leading to:
– Activation of descending inhibitory pathways.
– ↓ release of substance-P in pain transmission neurons in the spinal cord.
 The psychic effect results from ↓ NA release in some CNS areas leading to
decrease anxiety and reaction of the patient to pain.
 Morphine and other exogenously administered opioids may also have some
action on peripheral inflamed tissue.

N.B.Morphine can treat all types of pain except itching. Why?


a) Because morphine stimulates histamine release → ↑ itching.
b) Itching is different from pain sensation and has different receptors and
centers.

330
 Euphoria (large doses produce dysphoria).
 Miosis: due to central stimulation of Edinger-Wistphal nucleus. Severe miosis is
indicative of toxic doses.
 Respiratory center depression. This RC depression leads to CO2 retention and
cerebral VD → ↑↑ intracranial tension.
 Cough suppression.
 Vagal stimulation.
 Nausea & vomiting: due to stimulation of chemoreceptor trigger zone (CTZ).

CVS effects:
 Orthostatic hypotension: due to (a) Histamine release; (b) vagal stimulation.
 Bradycardia: due to vagal stimulation.

Smooth ms (Spasmogenic effects):


 Bronchoconstriction: due to: (a) Vagal stimulation. (b) Histamine release.
 Constipation: due to (a) spasmodic non-propulsive contractions of intestinal
smoothms& decreased peristalsis; (b) ↑ intestinal water absorption.
 Spasm of the sphincter of Oddi → ↑ biliary pressure.
 Feeling of urgency with difficult micturition due to ↑ detrusor muscle tone with
spasm of the internal urethral sphincter.
 Uterus: prolongation of labor by unclear mechanism.

Therapeutic uses

 Analgesia: for severe pain e.g. acute MI, cancer, surgery, etc.
 Acute pulmonary edema (cardiac asthma).Why?
– ↓ stress & anxiety of the patient.
– Venodilatation → ↓ VR & preload → ↓ pulmonary congestion.
– Decrease tachypnea caused by the CNS response to hypoxic drive (due to its
depressant effect on RC).

 In anesthesia:
– As adjuvant to anesthetic agents (preanesthetic medication).
– Regional anesthesia (epidural) to achieve long lasting analgesia by its effect
on the spinal cord.

 In severe colic: (morphine combined with atropine). Why?


– Because morphine is spasmogenic and atropine is spasmolytic.
– To counteract muscarinic effects caused by excessive vagal stimulation (e.g.
bronchoconstriction and bradycardia).

331
Preparations and doses
 Morphine sulphate: 10 mg s.c. or i.m. In acute MI it is given 5 mg i.v.
 Intrathecal (epidural) injection: produce long lasting analgesia which is useful for
critically ill patients at risk of RC depression.
 Sustained release preparations &transdermal patches are available.

Adverse effects:

CNS  Tolerance & physical dependence (addiction) with prolonged use:


– Physical dependence can occur within 24 h if given /4 h.
– Tolerance may occur to analgesia and euphoria but not to
respiratory depression.
 ↑↑ intracranial tension.
 RC depression: the most important effect and is dose-dependent.
Resp  Bronchoconstriction.
CVS  Postural hypotension
GIT  Nausea, vomiting, and constipation.
 Increased biliary tract pressure and biliary colic.
Genito-  Urine retention especially in patients with enlarged prostate.
Urinary  Prolongation of labor.
Eye  Miosis is a consistent finding in morphine addiction.

Contraindications & precautions


 Head injury & increased intracranial pressure: Morphine causes respiratory
depression & CO2 retention. The ↑ CO2 causes cerebral VD and ↑ intracranial
tension.
 Respiratory depression.
 Bronchial asthma: Morphine causes bronchoconstriction due to (a) vagal
stimulation; (b) histamine release. It also causes RC depression.
 Biliary colic & gallstones: due to spasm of the sphincter of Oddi → ↑ biliary
pressure.
 Senile enlarged prostate: Morphine ↑ detrusor muscle tone with spasm of the
internal urethral sphincter → feeling of urinary urgency with difficult micturition.
 Hypotension and hypovolemia: because morphine causes postural
hypotension.
 Hepatic damage: Due to: (a) morphine is metabolized by the liver; (b) morphine
increases the risk of hepatic encephalopathy due to marked CNS depression.
 Hypothyroidism and adrenal insufficiency (Addison’s disease). Why?
Because those patients have prolonged and exaggerated response to morphine.

332
 Undiagnosed acute abdominal pain: Morphine masks the pain (which may be
dangerous e.g. appendicitis) and interferes with the correct diagnosis.
 Infants and old patients: are more susceptible to respiratory depression.

▌Chronic opioid toxicity (addiction)

 There are behavioral changes, constipation, itching &miosis.


 Sudden withdrawal (abstinence syndrome):
– Consists of: irritability, nervousness, tremors, hypertension & ms cramps starts
after 6-10 hrs from the last dose - peak effect at 48 hrs - gradually subsides
over 5-10 days.
– Mechanism: chronic administration of opioids ↓↓ endogenous production of
endorphins and NA. Following sudden withdrawal, there is an immediate
deficiency of endogenous opioids with rebound ↑ of NA release.

Treatment of chronic morphine addiction:


– Gradual withdrawal of morphine with substitution by methadone, then
gradual withdrawal of methadone.
– Clonidine: to stimulate central α2 receptors and ↓ NA release.
– Sedatives: e.g. diazepam.

▌Acute opioid toxicity

Manifestations:  Coma with depressed respiration, miosis, and shock.


 Death occurs from respiratory depression.
Treatment:  Gastric lavage.
N.B.
 Establish a patent airway
Opioid blockers are
and artificial respiration if
indicated in acute morphine
needed.
toxicity but they are
 Opioid antagonists: absolutely contraindicated
– Naloxone: pure opioid in chronic morphine
antagonist and can addiction because they
reverse RC depression precipitate severe withdrawal
within minutes (0.4 - 0.8 syndrome.
mg i.v. for 2-3 doses).
– Nalorphine: it is mixed agonist-antagonist (partial blocker).

N.B.
 The duration of opioid antagonists is shorter than morphine. The patient should
be watched carefully because he may go back into coma.
 Care should be taken to avoid withdrawal syndrome.

333
2. Codeine

Similar to morphine with the following differences:


 Has greater oral bioavailability (60%) due to less first-pass effect.
 The analgesic potency is 20% of morphine, but more potent cough suppressant
 It has little euphoric effect.
 Most of the systemic effects of codeine are due to conversion in the liver into
morphine by demethylation (codeine is methylmorphine).

Therapeutic uses
– Analgesic for mild to moderate pain (usually combined with paracetamol).
– As central antitussive (see chapter 7).

Adverse effects: (less than morphine)


– Constipation
– RC depression and addiction liability but it is fewer than morphine.

Morphine Codeine
Oral bioavailability: 25% 60%
Analgesic effects: Strong Weak (20%).
Antitussive effect: Weak Strong
Uses: Mention its 4 uses Analgesic and antitussive

█ SYNTHETIC AND SEMISYNTHETIC OPIOID AGONISTS

1. Heroin and hydromorphone

 They are semisynthetic opioids (heroin is diacetylmorphine).


 They are more potent than morphine but with rapid onset and shorter duration.
 They are not used clinically because they are highly addictive.

2. Meperidine (Pethidine)

Synthetic opioid similar to morphine with the following differences:


 Better absorbed orally and has greater bioavailability than morphine.
 The analgesic potency is 10% of morphine.
 It is used as analgesic alternative to morphine in the following cases:
– Inferior MI because in this case the patient usually has bradycardia.
– It is preferred than morphine during labor because it has short duration
and doesn't prolong labor (little or no spasmogenic action).

334
Adverse effects
– It causes RC depression and addiction liability but weaker than morphine.
– It causes histamine release and bronchoconstriction
– It has weak atropine-like actions → dry mouth, tachycardia, etc.
– It has No GIT, No antitussive, and No vagal stimulant effects.

Morphine Meperidine
Chemistry Natural opioid Synthetic opioid
Bioavailability 25% Greater (50%)
Analgesic effect Strong Weak (10% of morphine)
Spasmogenic effects Present Absent
Autonomic effects Vagal stimulation Atropine-like action
Uses Mention its 4 uses Analgesic only

3. Fentanyl and alfentanil

 They are synthetic derivatives of meperidine. They are the most potent and the
shortest duration opioid agonists.
 They are used as analgesic in severe pain (as long-acting transdermal skin
patch). A transdermal fentanyl 12 microgram patch equates to approximately 30
mg oral morphine daily.

4. Methadone

Synthetic opioid similar to morphine with the following differences:


 The analgesic effect is equal to morphine.
 Has longer duration of action than morphine (t1/2 is 24h).
Uses:
 As analgesic.
 Treatment of chronic opiate addiction and heroin users:
– It can satisfy the craving needs of the patient with less addictive features.
– Methadone withdrawal symptoms are less severe than other opioids.

5. Tramadol

 It has two different mechanisms. First, it binds to the μ-opioid receptor. Second,
it inhibits the reuptake of serotonin and NA.
 Uses: as analgesic for moderate to severe pain, especially musculoskeletal pain
 Adverse effects: It has relatively fewer side-effects than most opioids (but
addiction can occur). It may induce seizures in epileptic patients.

335
6. Diphenoxylate and loperamide (see chapter 8)

█ SEMISYNTHETIC MIXED AGONISTS-ANTAGONISTS (partial agonists)

Nalorphine – Nalbuphine – Pentazocin – Butorphanol

 All these drugs have agonist activity on  receptors and antagonist or partial
agonist activity on  receptors.
 They are used as analgesics alternative to morphine but their analgesic activity
and respiratory depression are less marked than morphine.
 All these drugs (except nalbuphine) increase
systemic and pulmonary vascular resistance N.B.
leading to ↑ cardiac load, so they are, thus,
For opioid analgesics,
contraindicated to relieve pain of acute MI.
potency of the analgesic
 They can lead to withdrawal symptoms if should be considered more
given to opioid addict patients. important than efficacy
because respiratory
depression is dose-
█ SYNTHETIC FULL ANTAGONISTS dependent.

Naloxone and naltrexone

 They are competitive blockers of all opioid receptors.


 Naloxone is given i.v. and has short t½ (~1h) but naltrexone could be
administered orally and has longer t½ (~48 h).
 They can precipitate severe withdrawal syndrome if administered to opioid-
addict patient.

Therapeutic uses of naloxone


 Acute opioid toxicity: given i.v., the adverse respiratory and CVS effects of
opioids are reversed within 1-2 min and lasts for 1-2 hrs.
 It is given during labor to mothers who received opioids to prevent neonatal
respiratory depression, or it can be given to the neonate via the umbilical vein.

336
█ NON
N-OPIOID ANALGES
SICS

1. Ace
etaminophen (Para
acetamoll)

Pharm
macokinetics
 Abssorption is complete and rapid from GIT with
w peak levels afteer 30 min.
 Mettabolism: liver by conjugation
c n. At highh doses, it is convverted into
o toxic
mettabolite (N
N-acetyl-be ne) that is responsible for hepaatotoxicity..
enzoquinon
 Exc
cretion: maainly renal.

Mecha
anism & Ph
harmacolo
ogical effe
ects
 It iss a selectivve Cox III inhibitor sso it inhibitts PGs syn
nthesis in tthe brain only
o and
hass analgesic & antip pyretic acttions witho
out effects on the eenzymes that t are
respponsible fo or synthesis of perip
pheral PGss and so it has no a nti-inflammatory
action.
 It ha
as little orr No effects on the C
CVS, GIT, re
espiratory or platelett functions
s.

Therap
peutic use
es
As anaalgesic andd antipyretic when aspirin is s contraindicated (ee.g. patien
nts with
peptic ulcer, hemmophilia, etc).
e Aceta
aminophen
n can be administer
a red in preggnancy
with greater safetty than asp
pirin.

Advers
se effects
 At therapeuttic doses: acetami nophen iss
well-tolerated but may cause:
c
– Skin rash h& drug fever (a as allergic c
reactions)..
– Long term m use may lead
l to ren
nal failure.

 In ttoxic dose
es: dose-d
dependen
nt hepato--
toxiicity (centtrilobular necrosis): It occurs
s
with
h large dosses (about 15 gm in a adults and
d
m in childrren)
4 gm

Mec
chanism of
o hepatottoxicity
 A
Acetamino
ophen is converted
d to toxic
c
metabolite
e (N-acety
yl-benzoquuinone) inn
tthe liver that need
ds detoxifiication by
y
reduced glutathione
e.

337
 When glutathione store is depleted, the toxic metabolite binds covalently to
cellular proteins producing hepatocellular damage.
 Clinical symptoms of toxicity (e.g. vomiting) occur within 24 hrs but signs of
hepatic damage (e.g. jaundice) occur after 2-6 days.

Treatment of toxicity
The 20 hour IV protocol of
 Gastric lavage with activated charcoal. acetylcysteine
 Sulfhydryl donors (acetylcysteine) to
– First, administer an initial
restore hepatic glutathione. It must be
loading dose of 150 mg/kg IV
started within 8 hours of toxicity. over 60 minutes.
 Hemodialysis: better within the first 12
– Next, administer 12.5 mg
hrs after ingestion. /kg per hour IV for 4 hours.
– Finally, administer 6.25 mg
Nefopam (Acupan) /kg per hour IV for 16 hours.

Mechanism and effects


 It is a central analgesic without antipyretic or anti-inflammatory activity. It is
more potent than NSAIDs.
 The analgesic mechanism is unclear but may be related to inhibition of many
transmitters reuptake or blocking central voltage-gated Na+ channels.
 Nefopam is also used to combat severe hiccups.

Adverse effects
– Precipitation of epileptic convulsions in patients with epilepsy.
– Weak atropine-like actions: dry mouth, urine retention, etc.

Contraindications: history of epilepsy

Dipyrone (Novalgin)

Mechanism and effects


 Analgesic antipyretic that is more potent than aspirin. It has no anti-
inflammatory action.
 Its use was restricted in many countries because of risk of agranulocytosis
which is not dose-dependent.

Adverse effects
– Agranulocytosis: reversible in 10 days after stoppage of the drug but lethal in
10% of cases.
– Allergic reactions and anaphylaxis.
– It can trigger bronchoconstriction in patients with Asthma.

338
Part 3: Sedative-Hypnotic Drugs

 These are drugs that cause sedation and relieve anxiety (anxiolytics), or can
induce sleep. They are used primarily to treat anxiety and insomnia.
 Because there is considerable chemical variation within the group, these drugs
are classified based on their clinical uses rather than on chemical structure.

Drugs with main use as sedatives: Drugs with main use as hypnotics:
 Benzodiazepines  Barbiturates
 Buspirone  Ramelteon
 Chloral hydrate

1. Benzodiazepines

 Benzodiazepines (BDZ) have a great margin of safety over previously available


sedative–hypnotic agents (e.g., barbiturates).
 Most benzodiazepines have qualitatively similar therapeutic actions but differ in
their relative lipid solubility, metabolism, and elimination half-life.

Classification
 Short acting (t½ < 5h): midazolam – triazolam
 Intermediate acting (t½ 5-24 h): alprazolam – lorazepam - clonazepam
 Long acting (t½ > 24 h): diazepam – clorazepate - flurazepam

Pharmacokinetics
– Oral absorption is good and rapid. Highly lipid soluble drugs (e.g., midazolam,
triazolam) have fast onset of action.
– Long acting drugs are metabolized by oxidation (CYP450) into active
metabolites giving them long duration of action (e.g. diazepam).
– Short acting drugs are metabolized by conjugation into inactive metabolites
followed by renal clearance.
– In a patient with liver dysfunction, lorazepam and oxazepam, which are metabo-
lized extrahepatically, are less likely to cause excessive CNS depression.

Mechanism of action
 BDZ have special receptors in the CNS and peripheral tissue.
 By acting on these receptors, BDZ cause allosteric modulation of GABA action
on GABAA receptors resulting in ↑ Cl- conductance and hyperpolarization.
 Six BDZ receptor subtypes have been discovered; subtype 1 is the most widely
expressed and mediates most of the effects of BDZ.

339
Pharm
macologica
al effects
 Redduction off anxiety (anxiolytic
( effect) in
smaall dose producing
p calming effect in
mann & tamingg effect in animals
a
 Hyp ect: in high
pnotic effe her doses.
 Cenntral skele
etal musc cle relaxattion: this
is u
useful sinc
ce increas sed ms to one is a
commmon feature in anx xiety and mmay lead
to h
headache and
a ms pa ain.
 Antticonvulsa
ant effect.
 Acuute amnes sia: after high doses..

peutic use
Therap es
 Anx
xiety disorrders: e.g..
– A
Acute anxiety.
– G
Generalizeed anxiety disorders (GAD).
– S
Social phoobia (social anxiety d
disorder).

BDZ Z are effec


ctive for th
he short-tterm mana agement (<<6 weeks)) of these anxiety
diso
orders (seelective se erotonin-re
euptake inhibitors [S
SSRIs; seee later] are now
connsidered thhe first-lin
ne choicess for the long-term m manageement of anxiety
diso
orders.

 Insoomnia: BDDZ that ha


ave a rapid
d onset an
nd sufficien
nt durationn are wide
ely used
(e.g
g., temazep
pam).

 Antticonvulsaants: Lorazepam (a and diazep pam), give


en by i.v. may be used
u for
initial treatm
ment of status
s ep
pilepticus and dru ug-inducedd seizures. The
devvelopment of toleranc
ce preclud
des their long-term us
se.

 Shoort surgica dures: Sho


al proced orter acting
g benzodia
azepines ((e.g., mida
azolam)
are preferred before durring surgerry or endos
scopy.
 Pre
eanesthetic medicattion.

Advers
se effects
– Seddation, meemory distturbance, dull atten ntion (interfere with leearning ab
bility).
– Tole
erance an nd physica al depende ence (treatted by gradual withd drawal).
– Rebbound inso omnia afte er disconti nuation.
– Hanngover: a state of psychomot
p or depress sion occurrs in the fo
ollowing day after
the use of long acting drugs (i.e. re esidual efffect).
– Apnnea after ra
apid i.v. injjection (flu
umazenil is the antido
ote)

340
Fluma
azenil_
Flumazzenil is a competitiv
c ve antago nist at be
enzodiazep
pine recepttors. It is used to
prevent or reverrse the CN NS effectss from beenzodiazep
pine overd
dose or to o speed
recovery from th he effects of benzo
odiazepines s used in anestheticc and dia agnostic
proced
dures.

Precautions: avvoid BDZ use


u in the ffollowing conditions:
c :
– Drivvers and machine
m woorkers neeeding high attention.
– Pre
egnancy an nd lactatio
on: fetal m
muscular hypotonia anda impairred suckling may
occ
cur.
– Hep patic ence
ephalopathy.
– Resspiratory depression
d .
– Com mbination with otherr CNS dep ressants e.g.
e alcohol.

2. Bus
spirone

 It iss partial agonist on 5--HT1A rece


eptors in th
he midbrainn.
 Sele ective anxiolytic with
hout hypno otic or mus
scle relaxant effect.
 It su uppresses anxiety affter a long delay (2 or more weeks).
 It iss the anxiolytic of cho
oice in the elderly.
 No liability forr drug dependence.
 Adv verse effects include e dizzinesss, headach
he, tachycaardia and nnervousne
ess.

3. Barrbiturates
s

 Barrbiturates have
h been largely re
eplaced by
y the more
e safe benzzodiazepin
nes and
the SSRIs, forr the treatm
ment of an xiety and sleep
s disorders.
 Pheenobarbitaal is a long acting bbarbituratee (~6-8h) used
u as ann anticonv
vulsant;
opental is ultrashort agent (~15
thio 5-20 min) used
u as an
n i.v. generral anesth
hetic.

Mecha
anism of action
a
 Barrbiturates have eithe
er GABA-llike action
n
OR enhance the
t effects s of GABA
A at GABAA
eptors ressulting in ↑ Cl- co nductance
rece e
and
d hyperpolaarization.
 Thee action off barbitura
ates is non
n-selective
e
i.e. increasing
g the dos
se of barb biturates →
genneralized CNS and me epression.
edullary de

341
Therap
peutic use
es
 As sedative and hypn
notics in the treatm
ment of in
nsomnia. They are largely
replaced by benzodiaze
b epines.
 Phe al is used in the treattment of grand mal epilepsy.
enobarbita e
 Phe al is micro
enobarbita osomal en
nzyme ind
ducer. It is
s used in the treatm
ment of
hypperbilirubinaemia in neonates
n (p
physiologic
cal jaundicce) to activvate liver enzymes
(glu
ucuronyl traansferase) and fasten
n metaboliism of bilirubin.
 Thio
opental iss used as short
s naesthetic in short prrocedures..
i.v. an

Advers
se effects
– Phyysical depe
endence.
spiratory and myo
– Res ocardial
pression (in acute toxicity)
dep
enobarbital
– Phe is hepatic
crosomal enzyme inducer.
mic
– Pheenobarbital may increase
porrphyrin synthesis.
s It can
preccipitate th
he sympttoms of
acu
ute intermitttent porph
hyria.

4. Ram
melteon

 Ram
melteon is prescribe
ed for patiients who have difficulty fallinng asleep.. It is a
sele
ective agonist at me
elatonin MMT1 and MT2M recepttors that aare involved
d in the
promotion of sleep and that mainttain the noormal circadian rhyth m.
 Advverse effects include
e dizzinesss and fatig
gue.

5. Chloral hydrrate

 It iss one of thee oldest hyypnotic druugs. It is an


n alcohol derivative.
d
 It prroduces hyypnosis in 30 min annd last for 6 hrs.
 It iss used maainly as a hypnotic i n children n and elderly before short surgical or
den
ntal proced
dures but itts use now
w is very lim
mited.

342
Part 4
4: Sk
keletal Muscle
M Relaxan
nts

Classiffication
 Dru
ugs act on n brain hig ers: e.g. ca
gher cente arisoprodo
ol, benzodi azepines, general
ane
esthetics, anticonvuls
a sant drugs , and antip
parkinsonia
an drugs (ssee later).
 Dru
ugs act on
n neurona al cord: e..g. tizanidine and
al transm ission in the spina
bac
clofen.
 Dru
ugs act onn the neu on: e.g. ne
uromuscullar junctio euromuscuular blocke
ers and
boto
olinum tox
xin.
 Dru
ugs act dirrectly on the
t muscle anisms: e..g. dantrole
e contracttile mecha ene.

1. Carrisoprodo
ol

 Carrisoprodol – (and its metabolitee mebrobamate) – are centraally acting skeletal


musscle relaxxants. Their mechan nism is unclear but seems to be sim milar to
barrbiturates.. They bind
d to GABAAA recepto ors in the CNS
C leadi ng to antia
anxiety,
antiiconvulsan
nt, and ske
eletal musc
cle relaxatio
on.
 Thee drug and its metabo
olite have a
abuse potential, so so
ome countrries limit its
s use.

343
2. Baclofen

 Baclofen is a GABA derivative that selectively stimulates GABAB receptors in the


spinal cord → ↓ release of excitatory transmitters.
 It is used as a skeletal muscle relaxant in neurological spastic conditions.

3. Tizanidine

 Tizanidine is a centrally acting α2 agonist with greater effect on presynaptic α2 in


the spinal cord, so it inhibits neurotransmission and reduces muscle spasm with
minimal effect on blood pressure.

4. Dantrolene

 Dantrolene inhibits Ca2+ release from the sarcoplasmic reticulum of skeletal ms


cells. It acts directly on the muscle contractile mechanisms.
 It is used for emergency management of malignant hyperthermia (1 mg/kg i.v.)

5. Botulinum toxin (Botox)

 Botulinum toxin is a neurotoxic protein produced by the bacterium Clostridium


botulinum. It inhibits the release of ACh from motor nerve terminals leading to
skeletal muscle paralysis.
 It is injected locally to treat local muscle spasm e.g. in cervical dystonia
(spasmodic torticollis), and blepharospasm (uncontrolled muscle contraction or
twitch of the eyelid). It is also used for cosmetic reduction of facial wrinkles.

6. Neuromuscular blockers: see ANS.


Mephensin

Therapeutic uses of skeletal muscle – Mephensin is a selective


relaxants inhibitor of polysynaptic
excitation of the spinal motor
 Neurological spastic conditions such as neurons through stimulation of
multiple sclerosis, back pain, and spine glycine receptors in the spinal
injuries. cord.
 Dantrolene is used for emergency man- – It was historically used as
agement of malignant hyperthermia. antidote for treatment of
 Botolinum toxin is injected locally to relive strychnine poisoning but its
blepharospasm, and for cosmetic clinical use now is very limited
because of serious side
reduction of facial wrinkles.
effects including respiratory
 Neuromuscular blockers are used to depression and hemolysis.
produce muscle relaxation during surgical
procedures.

344
Part 5
5: Antiiepileptiic Drugs
s

█ Basiic inform
mation

 Epilepsy, a chronic disease, occurs in


appproximatelyy 1% of the popu lation. The
cauuse of most cases of epilepsy iss unknown n,
alth
hough som me people develop e epilepsy as
a
the result of brain
b injury
y, stroke, b
brain tumor,
andd drug toxicity. Ge enetic muttations arre
linke
ed to a sm
mall proportion of the e disease.
 Epilleptic seizu
ures resultt from exc
cessive and
d
abn
normal cortiical nerve cell
c activity in the brain
n.
 Thee diagnosiss typicallyy involves ruling outt other conditions thhat cause similar
neuurological symptoms. This m may be confirmed d by braain imagin ng and
elec
ctroencephhalogram (EEG)
( but a normal te
est does not excludee the condition.
 Antiepileptic drugs
d (AED
Ds) are effe
ective for about
a 80%
% of these patients. Lifelong
L
trea
atment mayy be neces
ssary.
 It m
may take we
eeks to es
stablish ade
equate dru
ug plasma levels and
d to determ
mine the
ade
equacy of therapeutiic improve
ement. Lac mpliance iis respons
ck of com sible for
manny treatme
ent failures.
 AED
Ds are mo
ost effectiive and ha
ave the le
east adverse effectts when they are
use
ed as mono
otherapy.
 Add
dition or withdrawal
w of any drrug should
d be gradual, becauuse seizurres may
occ
cur on withdrawal.
 Som
me AEDs are
a terato
ogenic; thiis may ca
all for the reduction or termina
ation of
therrapy during
g pregnanc
cy.

Classiification of
o epileps
sies:

Epilepssies are characteriz


c zed by eitther focall or generalized ab bnormal neuronal
n
discharrges. Drugg selection
n, based o
on seizure classifica
ation, is lissted below
w in the
order o
of general choice.
c

Simple Loc
calized disc
charge; co
onsciousne
ess is 1 . Carbama azepine
partial not altered. 2 . Lamotriggine
seizures
Partial

3 . Valproic acid
Complex x Loc
calized disc
charge tha
at becomes
s
partial widespread; aaccompanied by loss
s of
con
nsciousnesss.

345
Tonic–clo
onic Draamatic convvulsions with
w either jerking 1 . Valproic acid
(grand ma
al) of the extremiities or rigidity of the entire 2 . Lamotriggine
boddy; accomppanied by loss of 3 . Carbama azepine
connsciousnesss.
Generalized

Absence Suddden onsett of altered


d consciouusness 1 . Ethosuximide
seizures

(petit mal)) thatt lasts 10–4


45 secondds, with up to 2 . Valproic acid
hun
ndreds of sseizures peer day; beg
gins in 3 . Clonazeppam
chilldhood or adolescen nce.

Myoclonic Lighhtning-like
e jerks of one or more e 1 . Valproic acid
syndromes extrremities occcurring sin
ngly or in bursts
b 2 . Lamotriggine
of up
u to a hun ndred; accompanied by
alte
eration of c
consciousn ness.

 Status epilep
pticus:
Pro
olonged se
eizure (>20 0 min) of a
any of the types prev viously desscribed; th
he most
com
mmon is liffe-threaten
ning generaalized tonic–clonic sttatus epileepticus.

1. Dip
phenylhyd
dantoin (P
Phenytoin
n)

Mechaanism: it blocks Na a+, K+ andd Ca2+


channeels in the brain (and
d heart) le
eading
to deccrease prropagation n of abn normal
impulse
es. It prooduces so ome degrree of
drowsin
ness.

Therap
peutic use
es
 Parttial and geeneralized seizures
s
 Stattus epileptticus: it sho
ould be givven i.v. in the
t form of fosphenyytoin (prod
drug).
 Ven
ntricular arrrhythmia.

Advers
se effects
– CNS S: Nystagmus, diplo opia, ataxia
a.
– Heppatotoxicityy.
– Miccrosomal enzyme ind duction.
– Bonne marrow depressio on & Mega loblastic anemia
a e to ↓ folicc acid).
(due
– Teraatogenicityy: craniofacial abnorm
malities.
– Gingival hype erplasia: 2ry
2 to incrreased ex xpression of platelett derived growth
facttor (PDGF)).
– Lym
mphadenop pathy.

346
2. Carbmazepine (Tegretol)

Mechanism: it blocks Na+ channels & ↓ excitability of cortical neurons.

Therapeutic uses
 Partial and generalized seizures (grand mal epilepsy).
 Trigeminal neuralgia.

Adverse effects
– CNS: diplopia & ataxia.
– Hepatotoxicity.
– Microsomal enzyme induction.
– Bone marrow depression.
– Congestive heart failure (CHF).

3. Valproic acid (Depakene)

Mechanism: it activates glutamic acid decarboxylase enzyme→ ↑ GABA synthesis.

Therapeutic uses: all types of epilepsy.

Adverse effects
– Sedation
– Microsomal enzyme inhibition
– Teratogenicity.
– Alopecia
– Pancreatitis

4. Ethosuximide (Zarontin)

Mechanism: blocks neuronal voltage-dependent Ca2+ and Na+ channels.

Therapeutic uses: absence seizures (petit mal epilepsy) (1st choice).

Adverse effects
– Sedation
– Vomiting
– Leucopenia

5. Benzodiazepines: Clonazepam and diazepam

Mechanism: allosteric modulation of GABA action to facilitate its effects.

347
Therapeutic uses
 Clonazepam: petit mal epilepsy.
 Diazepam: status epilepticus.

Adverse effects: see before.

6. Barbiturates: Phenobarbitone

Mechanism: enhancement of GABA-mediated inhibition of glutamate excitation.

Therapeutic uses: grand mal epilepsy (contraindicated in petit mal epilepsy).

Adverse effects: see before.

█ NEWER ANTIEPILEPTIC DRUGS

1. Felbamate

Mechanism
 Block glycine site on the N-methyl-D-aspartate (NMDA) receptors.
 Block voltage-dependent Ca2+& Na+ channels.

Therapeutic uses: wide variety of partial and generalized seizures.

Adverse effects
– Hepatotoxicity
– Microsomal enzyme induction.
– Bone marrow depression.

2. Lamotrigine

Mechanism
 Decreases glutamate and aspartate, which are excitatory neurotransmitters
 Blocks sodium channels and high voltage-dependent calcium channels leading
to ↓ excitability.

Therapeutic uses: wide variety of partial and generalized seizures and typical
absence seizures in children and adults.

3. Gabapentin

Mechanism: unknown but may interfere with voltage-dependent Ca2+ channels

348
Therapeutic uses: as adjuvant therapy in wide variety of partial and generalized
seizures.

Adverse effects: headache, nystagmus, dizziness & ataxia.

4. Tiagabine

Potent and specific inhibitor of GABA uptake into glial and other neurons. Thus, it
enhances the action of GABA by decreasing its removal from the synaptic space.

Precautions during antiepileptic therapy


 Proper choice of the antiepileptic drug according to type of epilepsy.
 Start with single drug and if fails, substitute it or add another drug.
 Dose of the drug is adjusted according to:
– Plasma concentration of the drug.
– Patient's response.
 Therapy is given for 2-3 years then withdrawn gradually.

Drugs used for treatment of Status epilepticus


Diazepam (drug of choice) 10 mg i.v. or 500 g/kg rectal (in children) OR:
 Fosphenytoin: 100 mg i.v.
 Phenobarbitone: 200 mg i.v.
 In severe cases:
a) Thiopental i.v.
b) Artificial respiration.

349
Part 6
6: Antiidepress
sant Dru
ugs

█ Basiic inform
mation

 Dep
pression iss a disord
der of mo
ood rather
thann disturban
nce of thought or coognition. It
is p
postulated that depreession is d
due to de-
ency of NA
ficie N and/orr 5-HT in the CNS
while mania results from functtional ex-
cesss. Psychicc depression is cha aracterized
by both emo otional andd biologiccal symp-
tomms.
 Reccent studiees suggestt that overractivity of
posst-synapticc 5-HT2A receptors in some
braiin areas iss involved in the path
hogenesis
of d
depression and psychosis.
 Unipolar dep
pression (major depressive
disoorder): mo
ore commo on, may be e reactive
(70%%) or endoogenous (25%),
( cha
aracterized
by low mood anda loss of interest o or pleasure
in normally enjjoyable acttivities.
 Bipolar dep
pression (manic-de
epressive
order): lesss common, charactterized by
diso
osc
cillating periods
p off depresssion and
mannia. There is strong hereditary
h o
origin.
 Thee therapeuttic effect occurs
o onlyy after 2-3
wee eks of dru
ug adminisstration an d is more
clossely assoc
ciated with
h adaptivee changes
in neuronal re
eceptors an
nd brain neeurotropic
facttors.

▌Class
sification of
o antidep
pressant d
drugs:

 Tric
cyclic anntidepress sants (TC
CA) e.g.
imip
pramine, amitriptyline
e.
 Sele
ective serotonin re euptake iinhibitors
SRI): e.g. flu
(SS uoxetine, sertraline.
s
 Aty
ypical hete ssants: e.g
erocyclic antidepres
a g. maprotiline, trazod
done.
 Monoamine oxidase
o in MAOI) e.g
nhibitors (M g. clorgyline
e, selegilinne.

350
1. Tricyclic antidepressants (TCA)
Imipramine, Desipramine, Clomipramine, Amitriptyline, Nortriptyline

Pharmacokinetics
 They are well absorbed after oral administration. They have large Vd.
 Most TCA have long t1/2 because they are metabolized into active metabolites
and undergo enterohepatic cycling.

Mechanism of action: (inhibition of the amine pump)


 TCA inhibit neuronal reuptake of both 5-HT & NA leading to their accumulation in
synaptic spaces and the brain tissue.
 It has been suggested that improvement of the emotional symptoms is related
to enhancement of 5-HT transmission while improvement of biological
symptoms is related to enhancement of NA transmission.
 Elevation of mood in depressed patients occurs after 2-3 weeks

Therapeutic uses
 Major depressive disorder.
 Nocturnal enuresis in children (imipramine).
 Chronic pain syndromes, neuropathic pain, and prophylaxis of migraine (unclear
mechanism).

Adverse effects
– Sedation is common at the start of therapy but tolerance develops later. It may be
due to antagonism with histamine H1 and/or muscarinic receptors.
– CNS troubles: memory dysfunction, agitation, seizures, and suicidal thoughts.
– Atropine-like action: very common - dry mouth, blurred vision, urine retention, etc.
– Orthostatic (postural) hypotension: due to peripheral α1 receptor blockade.
– Cardiac arrhythmias: tachycardia, widening of QRS, and ↑ QT interval.
– Hepatotoxicity: cholestatic hepatitis.
– Weight gain.
TCA overdose

Drug interactions  Metabolic acidosis


 Atropine-like effects
– Toxic synergism with MAOIs and SSRIs  Cardiac arrhythmia
(irritability and convulsions).
– TCA antagonize the antihypertensive effect of Management

clonidine and methyldopa.  IV NaHCO3 (1st step).


– TCA have additive anticholinergic effect with  IV lidocaine
other drugs having anticholinergic activity.  Dialysis is ineffective

351
2. Selective serotonin reuptake inhibitors (SSRIs)
Fluoxetine, Paroxetine, Sertraline, Citalopram, Escetalopram

 They are the most commonly prescribed antidepressants due to their limited
toxicity. They are also used for some other psychiatric disorders.
 Sertraline is the preferred antidepressant following myocardial infarction as there
is more evidence for its safe use in this situation than other antidepressants.
 When stopping an SSRI the dose should be gradually reduced over a 4 week
period, this reduces the risk of relapse.

Mechanism of action
 They selectively block 5HT reuptake leading to accumulation of 5-HT in brain tissue.
 Their effect appears after 2-3 weeks like other antidepressants.

Therapeutic uses
 Major depressive disorder.
 Obsessive-compulsive disorder (OCD).
 Anxiety disorders (generalized anxiety disorder, social phobia, panic disorder).

Adverse effects
– GIT irritation is the most common side effect. A proton pump inhibitor should be
prescribed if a patient is also taking a NSAID to avoid GIT bleeding.
– Sedation or insomnia at the start of therapy but tolerance develops later.
– Muscle cramps and twitches.
– Sexual dysfunction in up to 40% of patients – the main cause of noncompliance.
– Dangerous “serotonin reaction” may occur if given with MAOIs or TCA
(hyperthermia, muscle rigidity, cardiovascular collapse).

3. Atypical antidepressants

The pharmacological properties of atypical heterocyclic antidepressant agents are


similar to those of TCAs.

Mechanism of action
 Trazodone: blocks mainly 5HT2A receptors in addition to H1, and α1 receptors.
It is highly sedating and can cause postural hypotension.
 Mertazapine: blocks mainly 5HT2A receptors in addition to H1, and α2
receptors. It causes weight gain.
 Maprotiline: selective blocker of NA reuptake. It is highly sedating and can
cause seizures.

352
4. Monoamine oxidase inhibitors (MAOIs)
Clorgyline, Selegiline, Pargyline, Moclobemide

Mechanism of action
N.B. There are 2 isotypes of MAO
 They inhibit MAO enzyme leading to
enzyme:
accumulation of active monoamines (NA,
5-HT, dopamine) in neuronal tissue. MAO-A enzyme
 Most MAOIs are irreversible inhibitors. – Present in the cytoplasm of
Recovery of MAO takes several weeks. neurons (CNS) and peripheral
tissues (e.g. liver).
Moclobemide is a reversible inhibitor.
– It acts non-specifically on NA, 5-
HT, and dopamine.
Therapeutic uses – Clorgyline is a specific inhibitor.
 Major depression: they are not used as MAO-B enzyme
a first-line, but usually reserved as a last – Present mainly in the CNS and
line after other classes of drugs have acts more on dopamine.
failed. – Selegiline is a specific inhibitor.
 Selegiline (selective MAO-B inhibitor) is
used for treatment of Parkinsonism (see later).

Adverse effects
– CNS stimulation: irritability, insomnia, tremors, hyperthermia, convulsions
– Hepatotoxicity: occurs more with the old members.
– Orthostatic (postural) hypotension and sexual dysfunction.

Interactions

 Drug-drug interactions:
– Toxic synergism with tricyclic antidepressants and SSRIs.
– Potentiation of sympathomimetics (including cold remedies & nasal
decongestants).

 Drug-food interactions: Hypertensive crisis (cheese reaction):


– Tyramine is an indirect sympathomimetic present in some food and normally
metabolized by MAO-A in the liver.
– When the patient takes MAO-A inhibitor or non-selective MAOIs, severe
hypertension can occur after eating tyramine-rich food e.g. fermented cheese,
yogurt, beer, herrings.
– Treatment: by giving combined α + β blockers (prazosin + propranolol).

353
Moood Stabiilizing D
Drugs (trreatmen
nt of man
nia and
Part 7
7:
bipo
olar diso
order)

 Sod proate is
dium valp s the onlly specific
c
antiimanic ag gent and is the tre eatment of
o
chooice in the acute stagges.
 hium is the
Lith e drug of choice
c for long-term
m
trea
atment to prevent
p relapse.

Lithium
m carbon
nate

Mecha
anism of action
a
 It ↓ cAMP in neuronal
n ells and ↓ NA release
ce e
→ ↓ neuronal firing.
f
 nhibits many metabo
It in olic processses in the
e
nervve tissue.

Therap
peutic use
es
 Trea
atment off mania (valproate
( is the 1sts
cho
oice).
 Trea
atment of manic-deepressive disorder (bipolar
( de
epression).. It is given
n in the
mannic phase while
w TCA or SSRIs are given in
i the deprressive phaase.

Advers
se effects
 Lith
hium has a very na
arrow thera
apeutic index, monitoring of plasma le
evels is
essential.
 It haas long plaasma half-life being eexcreted entirely
e by the
t kidneyys.
 Toxxicity may be precip pitated by dehydratio on, renal failure,
f diuuretics (especially
thia
azide) or AC
CE inhibito
ors.

– Anoorexia, nau
usea, vomitting and d iarrhea.
– Nepphrogenic diabetes insipidus
i leading to polyurea and
a thirst.
– Hyppotension and
a cardia ac arrhythmmia.
– Thyyroid dysfu
unction
– Fine
e tremors s (coarse trremors are
e seen with
h toxic leve
els).
– Tera
atogenicityy.

354
Part 8
8: Antiipsychotic Drug
gs (Neurroleptics
s)

█ Basiic inform
mation

Centra
al dopamin nergic patthways
and drrugs affectting them:

 Thee term “p
psychosis”” denotes a variety
y of
men ntal disord
ders. Schizzophrenia is a partic
cular
kind
d of psychosis cha aracterized by abnormal
soccial behavioor and failure to reco
ognize what is
reall.
 Com
mmon “po
ositive” symptoms
s include false
f
beliefs (delu
usions), unclear or confu used
thin
nking, a
and auditory hallucinations.
“Neegative” symptoms
s include re
educed so ocial
enggagement, blunted em
motions, a
and inactivity.
 It is suggeested that centrall dopam
mine
ove
eractivity and 5H HT2A rece eptors play
p
imp
portant role
e in the patthogenesiss.

355
▌Classification of drugs

 Typical (classic; old) neuroleptics:


– Phenothiazines: chlorpromazine, promethazine.
– Butyrophenones: haloperidol, droperidol.
– Thioxanthenes: thiothixene, chlorprothixene.

 Atypical (newer) neuroleptics:


– Risperidone– Olanzapine – Clozapine

Mechanism and therapeutic action


 The typical antipsychotic drugs primarily block central D2 receptors in the
mesolimbic and mesocortical areas of the CNS leading to ↓ hallucinations and
delusions but can lead to serious extrapyramidal adverse effects.
 The atypical antipsychotic drugs combine between blockade of D2 (to a lesser
degree) and 5-HT2A receptors, so they have fewer extrapyramidal side effects.
 The typical antipsychotic drugs have greater effect on the control of positive
symptoms (hallucinations and thought disturbances), while atypical drugs have
greater effect on negative symptoms (social withdrawal).

Therapeutic uses
 Schizophrenia & mania (typical and atypical antipsychotics):
– Antipsychotic drugs produce an immediate quieting action. However, their
antipsychotic effects typically take longer time to occur (a week or more).

 Other therapeutic uses of typical antipsychotics:


– Chloropromazine is sometimes used to relieve intractable hiccough and as
anaesthetic adjuvant in certain operations to produce hypothermia.
– Severe nausea or vomiting associated with, e.g., radiation treatment and
cancer chemotherapy. Typical antipsychotic agents have strong antiemetic
activity due to D2-receptor blockade in the chemoreceptor trigger zone (CTZ) of
the medulla. The most commonly used are the phenothiazines chloropromazine
and promethazine

Adverse effects

 CNS:
 Extrapyramidal manifestations:
– These adverse effects are related to a D2-receptor blockade in the basal
ganglia. They occur most likely with old generation drugs.

356
– They in
nclude:
 Parkinsoniian-like synndrome
 Dystonia: sustained muscle co
on-
ttractions cause twiisting mov
ve-
ments or abnormal
a p
postures
 Dyskinesia
a: involunta
ary repetitive
ffacial, lip, and ton ngue mov ve-
ments occ curs with cchronic the
er-
apy. It mayy be irreve
ersible.

 Neurolepttic malignant syndro


ome:
– It is a serious compliication with w
mortality rate 20 0%, occurrs more with
w
old genneration drrugs.
– It consists of autonomic distu
urbance, ms
m rigidity, hypertherm
h mia, and sweating.
– It needs immedia ate stoppin
ng of the an
ntipsychottic therapyy.

 Auttonomic side effects


s:
– α
α-recepto
or blockadde leading to postura sion and seexual dysfunction
al hypotens
– de → atropiine-like actions
Muscariniic blockad

 End
docrine dis
sturbance
e (occurs m
more with
h atypical agents):
– W
Weight gain
– Hyperglycemia and precipitatio
p on of DM.
– G
Gynecoma astia, amen
norrhoea

 Oth
her advers
se effects:
– C
Cardiac arrrhythmia
– C
Cholestaticc jaundice
e
– A
Agranulocytosis (parrticularly asssociated with cloza
apine)

Typic
cal drugs Atypic
cal drugs
Mechanism Blockk mainly D2
2 receptors
s Block D
D2 (less) and
a
5HT2A (more)
Effect More effect on +ve
+ More eeffect on – ve
sympttoms symptooms
Extrapyyramidal side effects
s Comm
mon Less ccommon
Neurole
eptic malig
gnant synd
d Comm
mon Less ccommon
Endocrrinal side effects
e Less c
common Comm
mon
Agranu
ulocytosis Less c
common Comm
mon

357
Part 9
9: Anttiparkins
sonian D
Drugs

Parkinssonism is a movem ment


disorde er charactterized by ms
rigidity, tremors, and posttural
instabillity due too loss of do-
pamine ergic neu urons in sub-
s
stantiaa nigra an nd locus ce-
ruleus resulting in imbala ance
of do opaminergic (inhibittory)
and c cholinergic
c (excitattory)
influencces on thee extrapyraami-
dal sysstem. Rece ently, a he
eroin
substitute (MPTP P) was fo ound
to cau use irreverrsible dammage
of the nnegrostriattal dopaminergic neu
urons and a parkinso
onism-like sstate.

▌Class
sification of
o antipark
kinsonian drugs:

 Dop
paminergiic drugs:
– Dopamine e precurso ors: Levod
dopa (L-do opa).
– COMT inh hibitors: To
olcapone - Entacapo one
– Selective MAO-B in nhibitors: S
Selegiline
– Dopamine e agonists s: Bromocrriptine
– Release of
o dopamin ne and inh
hibition of its
reuptake: Amantadin
A ne

 Anttichloinerg
gic drugs:
– Synthetic atropine substitutes
s s: Benztropine –
Orphenadrrine – Trihex
xyphenidyl

1. Lev
vodopa (L
L-dopa)

Pharm
macokinetics
 Abssorbed rap
pidly from the
t small in
ntestine.
 It has short t1/2 (1-2 h)). This sho ort t1/2 ma
ay producee "on-off p
phenomen non" i.e.
rapiid fluctuation of the clinical sta
ate in the form of su
udden tremmors & immmobility
afte
er short period of recovery.
 Amino acids (e.g. leuc cine & isol eucine) co
ompete with L-dopaa absorptio
on from
the gut so it should
s be taken
t on an empty sttomach.

358
Mecha
anism of action
a
 Dop
pamine can’t cross BBBB but LL-dopa can n. It is
st
con
nsidered the
e 1 line tre
eatment of parkinsonism.
 Morre than 95% 9 of th
he adminiistered do ose is
rapiidly decarboxylated d into do opamine in the
y a small f raction escapes
periipheral tisssues. Only
and
d crosses BBB.
B
 Peripheral deecarboxylattion can b
be minimizzed by
adm
ministration
n of a decaarboxylase
e inhibitor which
can
n't cross BBBB e.g. ca
arbidopa oor bensera azide.
 Suc
ch a combination heelps to red
duce the do
ose of
L-dopa and hence the adverse
a efffects

Advers
se effects
 T: nausea & vomiting (domperid
GIT done is the e drug of choice
c to ttreat this vomiting
v
bec
cause it does not cros
ss the BBB
B. It acts on
o the baree area of thhe barrier).

 CNS
S:
– Mood cha anges, halluucinations and nightm mares.
– Dyskinesia a: involunta
ary movemments of th he head, lip
ps, and tonngue.
– On-off ph henomenon n: rapid fl uctuation of the clinical stat e in the form
f of
sudden tre emors and immobilityy after a shhort period of recoverry due to th
he short
t1/2 of the drug.
d

 Auttonomic: postural
p hy
ypotension
n, arrhythm
mias, mydriasis and aacute rise of
o IOP.

 ownish bod
Bro dy secretio
ons & red u
urine due to
t the meta
abolite hom
movanilic acid.
a

Interac
ctions

 od: amino acids


Foo a (e.g. leucine & issoleucine) compete with
w L-dopaa absorptio
on.
 Doppamine bllockers (a
antipsycho tic drugs and metoc
clopramidee) antagon
nize the
effe
ects of L-do
opa.
 Pyrridoxine (vvitamin B6) enhancess periphera
al decarbox
xylation intto dopamine.
 COMT inhibittors and se
elective MA bitors: mak
AO-B inhib ke synergissm with L-d
dopa.

2. COM
MT inhibiitors (L-D
Dopa adju
uvants): Tolcapone
T - Entacap
pone

Mecha
anism of action
a
 The
ey inhibit COMT
C enzy
yme selecttively & rev
versibly, th
he enzymee that conv
verts L-
dop
pa to 3-O-mmethyldop
pa (3OMD) in the gut and liver.

359
 By inhibiting COMT enzyme, tolcapone increases the efficacy of L-dopa and
stabilizes dopamine levels in the striatum and improves motor function.

Adverse effects
– Same side effects of L-dopa (nausea, vomiting, hallucination, mood changes).
– Tolcapone causes fulminating hepatic necrosis.

3. Selective MAO-B inhibitors: Selegiline

Mechanism of action
 It is a selective MAO-B inhibitor decreasing breakdown of dopamine in the brain.
 Unlike MAO-A inhibitors, it does not precipitate "cheese reaction" since
tyramine is metabolized in the liver by MAO-A.

Adverse effects
– Nausea, vomiting & GIT upset.
– Hallucinations & mood changes.
– Insomnia.

4. Dopamine agonists: Bromocriptine

 It is dopamine (D2) agonist derived from ergot alkaloids.


 It differs from L-dopa in: Faster onset, longer duration, No on-off effect.

Other uses
 To treat hyperprolactinemia and to suppress lactation (dopamine agonists ↓
prolactin secretion from pituitary gland).
 Acromegaly (dopamine agonists ↓ GH secretion from pituitary gland).

Adverse effects
– Nausea, vomiting & GIT upset.
– Hallucinations & mood changes in large doses.
– Postural hypotension.

5. Dopamine release stimulation: Amantadine

 It is an antiviral drug for influenza A2 virus.


 The antiparkinsonian mechanism is unclear but may be due to ↑ dopamine
release, ↓ dopamine reuptake, or direct action on dopamine receptors.

360
Advers
se effects
– Nauusea, vomiiting & GITT upset.
– Halllucinations & mood chhanges in la
arge doses
s.
– Posstural hypootension.
– Skin
n pigmenta ation.

6. Antticholinerrgic drugs s:
Benztroopine – Orrphenadrin
ne – Trihex
xyphenidyl

 Theese drugs have gre ectivity fo


eater sele or
bloc
cking central than pe muscarinic receptors..
eripheral m
 Theey producee moderate
e improvem
ment of tremors and
d sialorrhe
ea but little effect
on m
muscle rigidity.
 Theey are used
d also to co
ontrol acutte drug-ind
duced extrrapyramidaal side effe
ects.
 Theese drugs are con
ntraindicatted in pa
atients with prosta
atic hyperrtrophy,
obsstructive GI disease (e.g.,
( paralyytic ileus) and narrow
w-angle gl aucoma.
 In p
patients suffering
s frrom Parkin
nsonism, there is some
s deg ree of de
ementia
associated wiith atrophy
y of cortica
al neurons.. Anticholin
nergic drug
gs may aggravate
dem
mentia and
d memory loss.

361
Part 1
10: Dru
ugs Used in Alz
zheimer''s Disea
ase

Alzheim
mer is a ne
eurodegene erative dise
ease
charactterized by progres ssive losss of
memorry and de ementia du ue to losss of
cholineergic neuurons and depositio on of
abnormmal prote ein “amylo oid plaqu ues”.
Overstiimulation of
o glutamate recep ptors
may be e responsible for this
s neurode gen-
erative process.
The aim of therrapy is to either impprov-
ing cholinergic transmission within the
CNS or preventinng the exc
citatory acttions
of NMDDA glutamaate receptors in sele
ected
brain areas.

1. Cho
olinestera
ase inhib
bitors: Don
nepezil - Rivastigm
R ine

– The
ey are morre selective
e for ChE e
enzyme in the CNS
– The
ey provide a modest reduction in the rate
e of loss off cognitive functions..
– de effects: nausea, vomiting,
Sid v a
anorexia, and muscle e cramps.

2. NMDA recep
ptor antag
gonist: M
Memantine
e

– It iss a derivative of amantadine


– It prevents ratee of memory loss in b
both vasculaar-associatted and Alzzheimer dem
mentia.
– Sidde effects are rare: confusion,
c agitation, restlessne
ess.

3. Rec
cent trials
s:

 β & γ-secreta
ase inhibittors:
Thee enzymes β & γ-seccretase aree responsiible for forrmation of amyloid proteins
p
Aβ440 and Aββ42 that arre found in
n the brain of Alzhe eimer patieents. Inhib
bition of
thesse enzyme
es reduces depositio n of theses
s amyloid deposits.

 Ibup
profen and indomethacin
The
ese NSAID Ds reduce formation of Aβ42 by inhibition of γ-ssecretase enzyme
e
which is unreelated to their
t COX inhibition. Aspirin and
a corticcosteroids do not
duce this effect.
prod e

 Cop
pper and zinc
z chela
ating agen
nts:
Rem
moval of th
hese metals promote
es dissolutiion of amy
yloid plaquees in brain
n tissue.

362
Part 11: General Anesthetics

General anesthesia is characterized by a loss of consciousness, analgesia,


amnesia, skeletal ms relaxation, and inhibition of autonomic and sensory reflexes.

Balanced anesthesia
 Balanced anesthesia refers to a combination of drugs used to take advantage of
individual drug properties and minimizing their adverse actions.
 In addition to anesthetic drugs and neuromuscular blocking drugs, other drugs
are administered preoperatively, intraoperatively, and postoperatively to ensure
smooth induction, sedation, and smooth recovery (e.g., benzodiazepines, opioids).

Stages and planes of anesthesia


 The stages and planes of anesthesia identify the progression of physical signs
that indicate the depth of anesthesia. Newer, more potent agents progress
through these stages rapidly, and therefore, the stages are often obscured.
 Mechanical ventilation and the use of adjunct drugs also obscure the signs
indicating the depth of anesthesia.
– Stage I: Analgesia and amnesia.
– Stage II: Excitation.
– Stage III: Surgical anesthesia: loss of consciousness.
– Four planes have been described relating to increased depth of anesthesia.
– Plane IV includes maximal pupil dilation, apnea, and circulatory depression.
– The loss of the eyelash reflex and a pattern of respiration that is regular in
rate and depth are the most reliable signs of stage III anesthesia.
– Stage IV: Medullary depression: respiratory and cardiovascular failure.

Mechanism of anesthetic action


Inhalation and IV anesthetic agent interaction with discrete protein binding sites in
nerve endings to activate ligand-gated ion channels:
 GABA-A-receptor chloride channels: anesthetic agents directly and indirectly
facilitate a GABA-mediated increase in chloride conductance to hyperpolarize
and inhibit neuronal membrane activity.
 Ligand-gated potassium channels: anesthetic agents increase potassium
conductance to hyperpolarize and inhibit neuronal membrane activity.
 NMDA receptors: certain anesthetics (e.g., nitrous oxide, ketamine) inhibit
excitatory glutamate gated ion channels.

363
▌Class
sification of
o generall anesthettics

█ INTR
RAVENOU
US ANAES
STHETICS
S

1. Ultrrashort acting barrbiturates


s: thiopen
ntal

Advanttages
 Pro
oduces rap pid and pleeasant indu uction with
h rapid recoovery.
 Doees not incrrease intracranial preessure.
 Useed for indu
uction befoore stronge er anesthettic and for short proccedures.

Disadv
vantages
 Poo or analgesia and ske eletal musccle relaxatio
on.
 Induce cough h, laryngo ospasm, brronchospa asm and apnea
a avooided by atropine
a
andd ready arttificial resp
piration.
 Exttravasationn induces necrosis.
n
 Con ntraindicatted absolu utely in ca
ases of prophyria
p (defect
( of ALA syntthetase)
barrbiturates increase synthesis
s of porphy yrins which
h precipitaates acute
e attack
inducing paraalysis and death.
d

2. Eto
omidate

Advanttages: It iss very similar to thiop


pental but:
 Larg ger marginn between the anaessthetic dos se and the dose prod ducing respiratory
and d cardiovasscular depression tha an with thiopental.
 It iss more rappidly metab bolized tha
an thiopenttal and, therefore, leess likely to
o cause
a prrolonged hangover.
h
 It iss used onlyy as an ind duction age
ent, and is preferablee to thiopeental in pattients at
riskk of circulattory failure
e.

364
Disadvantages
 More likely to cause involuntary movements during induction.
 And to cause postoperative nausea and vomiting.
 Suppresses the adrenal cortex with prolonged use which has been associated
with an increase in mortality in severely ill patients.

3. Propofol

Advantages
 Rapid and pleasant induction with rapid recovery and clarity of mental status.
 Does not increase intracranial pressure.
 Used for induction or maintenance.

Disadvantages
 Respiratory center depressant less than thiopental.
 Markedly decreases the blood pressure.

4. Midazolam (see under Benzodiazepines)

 They are used in conjunction with anaesthetics to sedate the patient.


 The most commonly employed is midazolam.
 Diazepam and lorazepam are alternatives. All three facilitate amnesia while
causing sedation.

Advantages
 Used for induction
 3-4 times as potent as diazepam.
 Does not cause local irritation after injection as diazepam.
 Little cardiovascular or respiratory depression.

Disadvantages
 Light anaesthesia.
 Can cause respiratory depression.
 Anterograde amnesia which lasts for at least 2 hours.

5. Ketamine (Dissociative Anaesthesia)

Advantages
 Profound analgesia and amnesia
 The only intravenous anesthetic that routinely produces cardiovascular
stimulation through central sympathetic stimulation (increases Bl.P. H.R and
C.O.). So, it is beneficial in cases of shock.
 No respiratory depression and has a potent bronchodilator effects.

365
Disadvantages
 Increases intracraneal pressure (due to increased cerebral blood flow).
 Unpleasant dreams.
 Recovery often is accompanied by delirium and psychomotor activity).
 Diplopia and nystagmus may occur due to increased muscle tone.
 Contraindicated in cases of hypertension and stroke.

█ INHALATION ANAESTHETICS

An important characteristic of an inhalation anaesthetic is the speed at which the


arterial blood concentration, which governs the pharmacological effect, follows
changes in the concentration of the drug in the inspired air.

Measure of potency
 Through the minimum alveolar concentration (MAC). It is the anaesthetic (MAC)
alveolar concentration at 1 atmosphere that will produce loss of movement in
50% of subjects exposed to a noxious stimulus.
 Methoxyflurane is highly potent while nitrous oxide is of low potency.

Rapidity of induction
 Gas of little solubility in blood as nitrous oxide has a high blood gas tension, a
small Vd, with a consequent rapid induction and rapid recovery.
 Gas of high solubility in blood e.g. methoxyflurane has a low blood gas tension,
a big Vd, with a slow induction and a slow recovery.

Pharmacological effects
 Inhalation anaesthetics produce descending depression of all brain functions in
the following order: Cortex >Subcortex> Mid-brain > Spinal cord > Medullary
centers.
 The highly developed functions like memory are the first to be lost, but the vital
functions as respiration and circulation are the last.
 With modern balanced anaesthesia no more notable staging. Loss of
consiousness, analgesia and muscle relaxation is produced with a combination
of drugs rather than with a single anaesthetic agent.
 Unconsciousness rapidly produced with an i.v. induction agent (e.g. propofol), to
maintain unconsciousness and produce analgesia with one or more inhalation
agent which might be supplemented with an i.v. analgesic agent, and to produce
muscle paralysis with a neuromuscular blocking drug.
 The following table shows points of difference in between the currently used
inhalation anesthetics:

366
367
Preanesthetic medications (anesthetic adjuvant):

They are drugs given to reduce anxiety, providing analgesia and amnesia, and
prevent salivation, bradycardia, and other side effects of anesthesia.

 Anxiolytics (e.g. diazepam): given at night preoperatively to provide sedation &


amnesia.

 Opioid analgesics (e.g. morphine, fentanyl, or alfentanil):


– They are given 1½ hrs before anesthesia to inforce low potency anesthetics as
nitrous oxide.Alfentanyl may offer faster recovery than fentanyl.

 Anticholinergic drugs (e.g. atropine or scopolamine):


– They are given 1½ hrs before anesthesia to reduce respiratory secretions and
prevent occurrence of aspiration pneumonia.
– They block the reflex vagal stimulation induced by some inhalation anesthetics.

 Muscle relaxants & others drugs: e.g. vasopressors to prevent hypotension


and hypothermics (chlorpromazine) to lower the normal body temperature in
cases of cardiac surgery.

368
Part 12: Local Anesthetics

Classification

There are two classes of local anesthetics: amides and esters. The primary
differences between the two classes are in their relative metabolism (amides have
primarily a hepatic metabolism, whereas esters are metabolized by plasma
cholinesterases) and their potential for allergic reactions (esters more than amides).

Esters Amides
Members Cocaine, procaine, Lidocaine, prilocaine,
tetracaine, benzocaine mepivacaine,
bupivacaine
Metabolism Plasma cholineestrase Liver
Hypersensitivity reactions May occur Rare

Mechanism of action
 Local anesthetics block voltage dependent Na+ channels within the nerve fibers
→↓ nerve conduction.
 In general, small nerve fibers (that carry pain sensation) are more sensitive to
local anesthetics than large fibers (motor and other sensations).
 Myelinated fibers are blocked before non-myelinated fibers of the same
diameter.

Pharmacological properties

Chemistry Anesthetic Duration Onset


potency
Procaine Ester Low Short Moderate
Mepivacaine Amide Moderate Moderate Fast
Prilocaine Amide Moderate Moderate Fast
Lidocaine Amide Moderate Moderate Fast
Tetracaine Ester High Long Moderate

Adjuvants of local anesthetics

 Vasoconstricotors e.g. adrenaline:


– They are added to produce local VC; this will reduce wash of the anesthetic
by the local blood flow and thus prolongs the duration of the anesthetic.

369
– Vasoconstrictors should not be added for ring block of hands, feet, fingers,
toes, and ear pinna to avoid tissue damage.

 Sodium bicarbonate:
– All local anesthetics are weak bases. Addition of bicarbonate to the
anesthetic solution maintains the anesthetic in the non-ionized state and this
increases lipid solubility and enhances penetration of the anesthetic into the
nerve sheath.

Techniques and clinical uses

 Surface (topical) anesthesia: is used for anesthetization of the skin, mucous


membranes, or cornea.

 Treatment of itching caused by insect bites or irritant condition.


 To relieve pain caused by oral, laryngeal, or rectal disorders e.g. piles.
 Corneal anesthesia in ophthalmic surgery.

 Infiltration anesthesia: injecting an anesthetic directly into the subcutaneous


tissue for minor surgical procedures e.g. dental procedures. Epinephrine may be
added to prolong the duration of action.

 Nerve block: injecting the local anesthetic close to the appropriate nerve trunk
proximal to the intended area of anesthesia e.g. radial nerve block or retro-orbital
block for ocular surgery.

 Spinal anesthesia: is used for surgeries of the lower limb or pelvic structures. A
local anesthetic is injected into the subarachnoid space below the terminal end
of the spinal cord (usually between 3rd and 4th lumber vertebra).

 Epidural anesthesia: the anesthetic is infused into the space between the dura
mater and the connective tissue lining the vertebral canal as alternative to
subarachnoid anesthesia.

Adverse effects
– Surface anesthesia: allergic dermatitis.

– Infiltration anesthesia: faulty intravascular injection can lead to:


 CNS excitation: irritability and convulsions
 CVS depression: bradycardia, hypotension, myocardial depression or
cardiac arrest.

– Vasovagal syncope: vasovagal (also called neurocardiogenic) syncope is


usually associated with bradycardia (rather than tachycardia) and pallor (rather

370
than flushing). These differences can be helpful in distinguishing it from
anaphylaxis.

– Spinal anesthesia:
 Spinal shock due to sympathetic outflow paralysis.
 Headache due to CSF leakage.
 Respiratory paralysis.
 Septic meningitis.

Recommendations
 The choice of local anesthetic for infiltration depends on several factors,
including duration of the procedure, need for hemostasis, patient sensitivity to
catecholamines, and patient allergy to local anesthetics.

 Prior to infiltration, the clinician should determine if the patient has any history of
allergy. Patients with a true allergic reaction to a local anesthetic need evaluation
by an allergy specialist.

 Local anesthetic infiltration is contraindicated in the following conditions:


– If the area needed to be anesthetized is so large because the amount of
anesthetic needed exceeds the maximal safe dose.
– If the patient has history of allergy to local anesthesia.

Treatment of spinal shock


 Place the patient in the supine position with the legs elevated.
 Airway support: oxygen and artificial respiration.
 If there are convulsions, give i.v. diazepam.
 If there is hypotension, give i.v. fluids plus vasoconstrictors.
 Cardiopulmonary resuscitation (CPR) is necessary when cardiac function is
interrupted.

371
372
Review Questions

Long questions:
1. Classify opioid analgesics and discuss mechanism, uses, &contraindications of
morphine.
2. Anxiety is a common medical disorder. Classify antianxiety drugs and discuss
in details the differences between barbiturates and benzodiazepines.
3. Depression is an important medical disease. Classify antidepressant drugs;
mention their mechanism, indications and side effects.
4. Classify antiepileptic drugs; mention their mechanism, indications and side
effects.
5. Dopamine is found in many areas in the CNS. Discuss the drugs that act by
inhibiting these central dopamine sites.
6. Dopamine is found in many areas in the CNS. Discuss the drugs that act by
activating these central dopamine sites.

Short questions:
1. Give an account on:
a) Mechanism and treatment of paracetamol toxicity
b) Drug treatment of Alzheimer's disease.

2. Discuss the pharmacodynamic principles underlying the use of:


a) Morphine in acute pulmonary edema.
b) Acetylcysteine in paracetamol toxicity
c) Naloxone in acute opioid toxicity.
d) Selegiline in parkinsonism.

3. Discuss on pharmacological basis of each of the following:


a) Nalorphine is contraindicated to relive pain in addict patient.
b) Reserpine is contraindicated with MAOIs.
c) Anticholinergic drugs are not preferred to treat parkinsonism in old patients.

4. Mention drug interactions of each of the following drugs:


(1) L-Dopa (2) MAOIs

5. Mention the main differences between:


a) Valproic acid and lamotrigine
b) L-Dopa and bromocriptine.
c) Chloropromazine and olanzapine

373
Of each of the following questions, E. Less gastric irritation than other
select ONE BEST answer: NSAIDs

1. Morphine, all are true EXCEPT: 6. Aspirin reduces the synthesis of


A. Acts as an agonist at opioid receptors the following eicosanoids EXCEPT:
(especially μ) in the brain and spinal A. TXA2
cord B. PGE2
B. Causes pupillary constriction by C. PGF2α
stimulation of the Edinger–Westphal D. LTB4
nucleus in the mid-brain E. PGI2
C. Acts as an antihistamine
D. Is subject to presystemic metabolism 7. The following statements about
E. Stimulates the chemoreceptor trigger NSAIDs are correct EXCEPT:
zone A. Diclofenac may cause permanent
platelet dysfunction
2. Which of the following is not true B. Indomethacin may cause neurological
for meperidine? side effects (neurotoxicity).
A. It has less analgesic potency than C. Sulindac is a prodrug
morphine D. They may impair renal function
B. It has less spasmogenic effect than E. Aspirin may displace coumarin
morphine anticoagulants from plasma proteins
C. It can cause histamine release and
bronchoconstriction 8. The following statements about
D. It has greater bioavailability than aspirin are correct EXCEPT:
morphine A. May cause GIT hemorrhage after a
E. It does not cause physical single dose
dependence B. Enteric-coated tablets cause less
gastric bleeding
3. Morphine causes all of the C. May cause metabolic alkalosis in high
following EXCEPT: doses
A. Diarrhoea D. May cause Rye’s syndrome in children
B. Increased intrabiliary pressure E. Its toxicity may require treatment with
C. Histamine release hemodialysis
D. Reduced sensitivity of the respiratory
centre to carbon dioxide 9. Which one of the following statem-
E. Hypotension ents concerning Cox-2 inhibitors is
correct:
4. The following are particularly A. They show greater analgesic activity
sensitive to the pharmacological than traditional NSAIDs
actions of morphine EXCEPT: B. They show anti-inflammatory activity
A. Young children greater than traditional NSAIDs
B. The elderly C. They harm the stomach as do non-
C. Patients with hepatic failure selective cox inhibitors
D. Patients with renal failure D. They increase platelet aggregation
E. Patients with hyperthyroidism E. They are cardio protective

5. The pharmacologic effects of 10. Non-narcotic analgesics are all of


acetylsalicylic acid include: the following drugs EXCEPT:
A. Reduction of high body temperature A. Paracetamol
B. Promotion of platelet aggregation B. Acetylsalicylic acid
C. Reduction of pain by stimulation of C. Codiene
PGs synthesis D. Ketorolac
D. Efficacy equals to that of probenecid E. Dipyrone
as uricosuric agent

374
11. Which one of the following non- 17. The following drug can be given to
narcotic agents inhibits mainly reverse benzodiazepine overdose:
cyclooxygenase (COX) in CNS? A. Cocaine
A. Paracetamol B. Flumazenil
B. Ketorolac C. Buspirone
C. Acetylsalicylic acid D. Picrotoxin
D. Ibuprofen E. Diazepam
E. Celecoxib
18. Indicate the anxiolitic agent, which
12. For which of the following relieves anxiety without causing
conditions could aspirin be used marked sedative effects:
prophylactically? A. Diazepam
A. Noncardiogenic pulmonary edema B. Buspirone
B. Peptic ulcers C. Lorazepam
C. Thromboembolism D. Clorazepate
D. Metabolic acidosis E. Zolpidem
E. Periodontitis
19. Local anesthetics produce:
13. Nefopam: A. Analgesia, amnesia, loss of
A. Is associated with gastrointestinal consciousness
haemorrhage B. Blocking pain sensation without loss
B. Causes miosis of consciousness
C. Causes more respiratory depression C. Alleviation of anxiety and pain with an
than morphine altered level of consciousness
D. Potentiates the dysrhythmogenic D. A stupor or somnolent state
effect of halothane anaesthesia E. Physical dependence
E. Is contraindicated in epilepsy
20. Local anesthetics act by:
14. Indicate the benzodiazepine, which A. Blocking voltage-gated sodium
has the shortest elimination half-life: channels
A. Nitrazepam B. Blocking voltage-gated calcium
B. Alprazolam channels
C. Triazolam C. Blocking voltage-gated potassium
D. Diazepam channels
E. Clorazepate D. Blocking chloride conductance
E. Blocking NMDA receptors
15. Which of the following
benzodiazepines is preferred for elderly 21. Which of the following local
patients? anesthetics has short duration?
A. Clorazepate A. Procaine
B. Clonazepam B. Mepivacaine
C. Triazolam C. Prilocaine
D. Prazepam D. Lidocaine
E. Diazepam E. Tetracaine

16. Which of the following 22. Which drug does not activate
benzodiazepines is preferred for opioid receptors, has been proposed as
patients with liver disease? a maintenance drug in treatment
A. Clorazepate programs for opioid addicts, and with a
B. Nitrazepam single oral dose, will block the effects
C. Lorazepam of injected heroin for up to 48 h?
D. Prazepam (A) Fentanyl
E. Diazepam (B) Nalbuphine
(C) Naloxone

375
(D) Naltrexone 27. A 9-year-old child is having
(E) Propoxyphene learning difficulties at school. He has
brief lapses of awareness with eyelid
23. Which drug is a full agonist at fluttering that occur every 5–10 min.
opioid receptors with analgesic activity Which drug would be effective in this
equivalent to morphine, a longer child without the disadvantages of
duration of action, and fewer excessive sedation?
withdrawal signs on abrupt (A) Clonazepam
discontinuance than morphine? (B) Diazepam
(A) Fentanyl (C) Ethosuximide
(B) Hydromorphone (D) Gabapentin
(C) Methadone (E) Phenobarbital
(D) Nalbuphine
(E) Oxycodone 28. Which antiepileptic drug is most
likely to elevate the plasma
24. Which drug used in the concentration of other drugs
maintenance treatment of patients with administered concomitantly?
tonic-clonic or partial seizure states (A) Carbamazepine
increases the hepatic metabolism of (B) Clonazepam
many drugs including both phenytoin (C) Phenobarbital
and warfarin? (D) Phenytoin
(A) Buspirone (E) Valproic acid
(B) Clonazepam
(C) Eszopiclone 29. With chronic use in seizure states,
(D) Phenobarbital the adverse effects of this drug include
(E) Triazolam coarsening of facial features, hirsutism,
and gingival hyperplasia.
25. A patient with liver dysfunction is (A) Carbamazepine
scheduled for a surgical procedure. (B) Ethosuximide
Lorazepam or oxazepam can be used (C) Zonisamide
for preanesthetic sedation in this (D) Tiagabine
patient without special concern (E) Phenytoin
regarding excessive CNS depression
because these drugs are 30. The mechanism of antiseizure
(A) Actively secreted in the renal proximal activity of carbamazepine is
tubule (A) Block of sodium ion channels
(B) Conjugated extrahepatically (B) Block of calcium ion channels
(C) Eliminated via the lungs (C) Facilitation of GABA actions on
(D) Reversible by administration of chloride ion channels
naloxone (D) Glutamate receptor antagonism
(E) Selective anxiolytics like buspirone (E) Inhibition of GABA transaminase

26. This drug used in the management 31. Which statement about phenytoin
of insomnia facilitates the inhibitory is accurate?
actions of GABA. Its actions are (A) Displaces sulfonamides from plasma
antagonized by flumazenil. proteins
(A) Buspirone (B) Drug of choice in myoclonic seizures
(B) Temazepam (C) Half-life is increased if used with
(C) Eszopiclone phenobarbital
(D) Ramelteon (D) Isoniazid (INH) decreases steady-state
(E) Phenobarbital blood levels of phenytoin
(E) Toxic effects may occur with only
small increments in the dose

376
32. Which of the following drugs is the 37. Which of the following drugs is
most effective in the emergency most likely to be of value in obsessive-
management of malignant compulsive disorders?
hyperthermia? (A) Amitriptyline
(A) Atropine (B) Bupropion
(B) Dantrolene (C) Sertraline
(C) Haloperidol (D) Trazodone
(D) Succinylcholine (E) Venlafaxine
(E) Vecuronium
38. A patient under treatment for a ma-
33. Which drug is most likely to cause jor depression is brought to the
hyperkalemia leading to cardiac arrest emergency department after ingesting
in patients with spinal cord injuries? 30 tablets of imipramine. Which of the
(A) Baclofen following would be LEAST useful?
(B) Dantrolene (A) Administer bicarbonate to correct
(C) Pancuronium acidosis.
(D) Succinylcholine (B) Administer lidocaine to control cardiac
(E) Vecuronium arrhythmias.
(C) Initiate hemodialysis to hasten drug
34. Tolcapone may be of value in elimination.
patients being treated with levodopa- (D) Maintain heart rhythm by electrical
carbidopa because it: pacing
(A) Activates COMT (E) Use intravenous diazepam to control
(B) Decreases the formation of 3-O- seizures
methyldopa
(C) Inhibits monoamine oxidase type A 39. A 36-year-old woman presents with
(D) Inhibits neuronal reuptake of dopamine symptoms of major depression. Drug
(E) Releases dopamine from nerve treatment is to be initiated with
endings sertraline. In your information to the
patient, you would tell her that
35. Concerning the drugs used in (A) Sertraline may take 2 wk or more to
parkinsonism, which statement is become effective
accurate? (B) It is preferable that she take the drug
(A) Dopamine receptor agonists should in the morning
never be used in Parkinson’s disease (C) Muscle cramps and twitches can
before a trial of levodopa occur
(B) Levodopa causes mydriasis and may (D) She should notify you if she
precipitate an acute attack of glaucoma anticipates using other prescription drugs
(C) Selegiline is a selective inhibitor of COMT (E) All of the above
(D) The primary benefit of antimuscarinic
drugs in parkinsonism is their ability to Answers
relieve bradykinesia
(E) Therapeutic effects of amantadine 1C 10 C 19 B 28 E 37 C
continue for several years 2E 11 A 20 A 29 E 38 C
3A 12 C 21 A 30 A 39 E
36. Which drug is an antagonist at 5- 4E 13 E 22 D 31 E
HT2 receptors and can be used for the 5A 14 C 23 C 32 B
management of insomnia? 6D 15 C 24 D 33 D
(A) Estazolam 7A 16 C 25 B 34 B
(B) Flurazepam 8C 17 B 26 B 35 B
(C) Trazodone 9D 18 B 27 C 36 C
(D) Triazolam
(E) Zolpidem

377
378
Part 1
1: Bas
sic Princ
ciples o
of Antim
microbiall Drugs

Chemo otherapy iss the killin


ng of a li ving organism whe
ether being
g bacteria
a, fungi,
protozo
oa, or viruss.
Bacteriia are prokaryotic cells. Some e bacteria are patho ogenic to humans and a can
cause sserious infections; th
he principa
al treatmen
nt of infections is withh antibiotics.

█ CLA
ASSIFICAT
TION OF ANTIMICR
A ROBIAL DR
RUGS

A. Acc
cording to source:

 N
Natural co
ompounds s: e.g. peniicillin, chloramphenic col.
 S
Synthetic compound
c ds: e.g. su
ulfonamide es, quinolon
nes.
 S
Semisynth pounds: e .g. ampicilllin.
hetic comp

B. Acc
cording to the effectt on micro
oorganism
ms:

 B dal agents:: that kills tthe microo


Bactericid organism e.g.
e penicillllin.
 B
Bacteriosttatic agents: arrest g growth of the
t microo organism ee.g. sulfona
amides.

cording to mechanis
C. Acc sm of acti on:

 A
Agents actt by inhibittion of celll wall synthesis: e.g. penicillin.
 Inhibition of
o cell mem
mbrane fu
unction e.g
g. amphote
ericin B and
d azoles.
 A
Agents actt by inhibittion of nuc
cleic acid synthesis:
s e.g. quino
olones.
 A
Agents actt by inhibittion of pro tein synthesis:
‒ By actin
ng on ribos
somal 30 S subunit e.g.
e aminog
glycosidess.
‒ By actin
ng on ribos
somal 50 S subunit e.g.
e macrollides.
 A
Agents actt by inhibittion of bac
cterial meta
abolism: e.g.
e sulfonaamides.

379
D. Acc
cording to antimicro
obial spec
ctrum: es pluralis ((spp.), Latin
Specie n
abbreviation for mmultiple speecies.
 N
Narrow sp
pectrum drrugs:
‒ Drugs affect
a main
nly Gram +
+ve spp. e.gg. benzyl penicillin.
p
‒ Drugs affect
a main
nly Gram –vve spp. e.g
g. aminoglyycosides.
 E
Extended spectrum drugs: ag gents that affect
a Gram
m +ve & G Gram –ve spp..
 B
Broad spe
ectrum dru ugs: agentts act on wide
w range
e of Gram m +ve & Grram –ve
sspp. and otthers (prottozoa) e.g. tetracyclin
nes.

█ COM
MBINATIO
ON OF ANT
TIBIOTICS
S

Indicattions:

 To o
obtain broader spec
ctrum e.g. amoxicillin nic acid → co-amoxic
n + clavulan clav.
 To oobtain syn
nergism e.g. sulfonam mides + trimethoprim m → co-trim
moxazole.
 In m
mixed bactterial infections e.g. d
diabetic fo
oot or peritonitis.
 In s
serious bacterial infe
ections e.g . bacterial meningitis
s or septiceemia.
 To oovercome bacterial resistanc e e.g. TB and pseud domonas i nfection.
 To rreduce toxxicity of one drug byy using smmaller doses of two drrugs.

Results:

 Bac dal → syne rgism: e.g


ctericidal + bactericid g. penicillin
n with aminnoglycosides.
 Bac
cteriostatic ostatic → a
c + bacterio addition: e.g.
e tetracy
yclines withh sulfonam
mides.
 Bac static → anttagonism or synerg
ctericidal + bacterios gism:

 Antagonis sm: when bactericid dal drug acts by in


nhibiting ceell wall sy
ynthesis
(i.e. kills rapidly multiplying
m g bacteria
a e.g. penicillin) is combine ed with

380
bacteriostatic drug (e.g. erythromycin) which arrests the organism and
prevents its multiplication.
 Synergism: in special cases e.g. combination between sulfadiazine +
penicillin in meningococcal meningitis (both drugs attain high concentrations
in CSF when meninges are inflamed).

█ CLINICAL APPROACHES FOR RATIONAL PRESCRIBING OF ANTIBIOTICS

A. Confirm the presence of an infection

 Fever:

 Fever is considered a hallmark of most infectious diseases.


 Fever is defined as repeated oral temperatures > 37.2 ◦C.
 Fever may be present in absence of infection e.g. in autoimmune disorders
and several malignancies.
 Fever may be absent in presence of infection if the immune system is
depressed.

 White blood cell count:

 Normal WBC is 4000-10,000 cells/mm3.


 Bacterial infections are associated with elevated granulocyte counts
(neutrophils, basophils, and eosinophils).
 Viral, TB and fungal infections are associated with elevated lymphocytic count.
 Parasitic infections and allergic reactions are associated with increased
eosinophilic count.

B. Selection of antimicrobial agents

 Identification of the infecting organism:

 Infected body materials (e.g., blood, sputum, urine, wound drainage, etc.) must
be sampled and cultured before initiating treatment.
 Empirical therapy before identification of the organism is necessary in the
following conditions:
‒ In all acutely ill patients with infections of unknown origin.
‒ Infection in a neutropenic patient, or a patient with meningitis
(characteristically described by severe headache, neck rigidity, and sensitivity
to bright lights). In such conditions any delay in the treatment could be fatal.

381
 Patient factors:

 In neonates, the use of chloramphenicol can lead to shock and cardiovascular


collapse (gray baby syndrome) because liver functions are not well developed.
Also, the use of sulfonamides may lead to kernicterus (brain damage) due to
displacement of bilirubin from serum albumin.
 In growing children, the use of fluoiroquinolones can lead to arthropathy (joint
swelling, stiffness and cartilage damage). Also, the use of tetracyclines can bind
to growing bones and teeth resulting in abnormal teeth and bone formation.
 In old age (> 65 years) the incidence of renal toxicity with aminoglycosides is greater
than in younger patients.
 In immunocompromised patients, the use of bactericidal agents is necessary,
as the host’s immune system is not capable of final elimination of the bacteria.
 Pregnancy: many antibiotics cross the placenta and cause adverse effects to
the fetus e.g. aminoglycosides and tetracyclines.

 Tissue penetration:

 The capillary lining in some tissues, e.g. prostate, the vitreous body of the eye,
and brain form natural barriers to drug delivery due to presence of tight
junctions of the capillary wall.
 Lipid soluble antibiotics e.g. chloramphenicol and metronidazole can cross these
barriers in normal conditions. Penicillin is ionized at physiologic pH and cannot
cross these barriers unless inflammation is present.
 Poor perfusion of some area, e.g. diabetic foot, reduces the amount of antibiotic
reaching this area, making treatment is difficult.

C. Determinants of the rational dosing

The dose and frequency of an antibiotic is based on three important properties.


Understanding of these concepts can improve clinical outcomes and decrease the
development of resistance.

 Concentration-dependent killing: Minimum inhibitory


concentration (MIC):
Certain antibiotics (e.g. aminoglycosides) show
enhanced bacterial killing in concentration above The MIC is the lowest
the MIC. Giving these antibiotics by a single large concentration of antibiotic
dose per day achieves high peak levels and cause in body tissues and fluids
that inhibits bacterial
rapid killing of bacteria.
growth.

382
 Time-dependent killing:

By contrast, β-lactam antibiotics, macrolides, clindamycin, and linezolid do not


exhibit this concentration-dependent property; instead, the clinical efficacy depends
on the time of the drug concentration remains above the MIC. So, preparations with
long duration kill more bacteria.

 Post-antibiotic effect (PAE):

The PAE is a persistent bacterial suppression after levels of antibiotic fall below the
MIC. Antimicrobials with long PAE (e.g. aminoglycosides and fluoroquinolones)
usually require one dose per day.

█ ANTIBIOTIC RESISTANCE

The development of antibiotic resistance is a major problem in clinical practice.


Methicillin-resistant Staph aureus (MRSA) and some strains of Mycobacteria
tuberculosis are examples of multidrug-resistant bacteria.

 Innate resistance:

Innate resistance is a feature of a particular species of bacteria e.g Pseudomonas.


The gene(s) of resistance can be transferred between bacteria by transfer of naked
DNA (transformation), by conjugation with direct cell-to-cell transfer of
extrachromosomal DNA (plasmids), or through bacteriophage (transduction).

 Acquired resistance:

Acquired resistance is when bacteria that were sensitive to certain antibiotic


become resistant with time. Mechanisms responsible for this resistance include:
 Production of enzymes that inactivate the drug.
 Alteration of drug binding site.
 Reduction in drug uptake by the organism.
 Development of altered metabolic pathways.

█ ADVERSE EFFECTS OF ANTIMICROBIAL AGENTS

The adverse effects associated with the use of antimicrobial agents include:

 General adverse effects

 Hypersensitivity or allergic reactions: In form of fever, skin rash, arthralgia,


cholestatic jaundice or hemolysis. More serious reactions are agranulocytosis,
bone marrow aplasia or anaphylactic reaction.

383
 Reactions related to alterations in normal body flora, superinfection or vitamin
B deficiency may follow the use of broad-spectrum antimicrobials. It is due to
inhibition of bacterial flora that suppresses commensal micro-organisms which
present in gut or that forms these vitamins, respectively.
 Resistance

 Direct toxic reactions, resulting from high doses or drug interactions, on


hemopoietic system, liver, kidney, GIT, nervous system or CVS.

█ SUPERINFECTION (Opportunistic infection):

Administration of antimicrobials usually alter bacterial flora but with no ill effect in
most cases however, broad-spectrum antibiotics if used for long time may alter or
kill bacterial flora. So, the bacteria and fungi that are normally inhibited by bacterial
flora will multiply leading to superinfection (its early manifestation may by diarrhea).
It is caused by staphylococci, Pseudomonas, proteus, Candida albicans or
Clostridia difficile... etc. Superinfection may be vaginal, oral, pharyngeal or even
systemic infection e.g. staphylococcal enterocolitis, candidiasis or
Pseudomembranous colitis (= antibiotic-associated diarrhea).

Treatment:
Stop the causative agent and give drug, which kill the organisms responsible for
superinfection e.g. staphylococcal enterocolitis, which is treated by metronidazole
or vancomycin orally, antifungal nystatin for candidiasis.

█ GENERAL PRINCIPLES OF THERAPY WITH ANTIMICROBIALS

 Antimicrobials should only be given when necessary and after antimicrobials


susceptibility test whenever possible.
 The pharmacokinetics of the drug should be taken into consideration e.g. the
state of hepatic and renal functions of the patient.
 In serious infection it is better to start with a parentral loading of a bactericidal
agent to avoid emergence of resistant strains by giving adequate dosage for
sufficient duration and adapting proper combination regimens.
 Antimicrobials should be continued for 3 days after apparent cure is achieved to
avoid relapse.

384
Part 2
2: Individuall Classe
es of Anttibiotics
s

A. CEL
LL WALL INHIBITOR
RS

-LACT
TAM ANTIBIOTICS

They sshare a -lactam ring in the


eir molecular
structuure which is required for antibacterrial
activityy.
 Pen nicillin
 Cep phalosporrins
 Mo onobactam ms
 Carrbapenem m

Peniciillins

Antibacterial spe
ectrum:
 Gra
am-positivee cocci, e.g. sstreptococ cci,
pne
eumococci and staph hylococci.
 am-negativve cocci: gonococci a
Gra and menin ngococci.
 Gra
am-positivee bacilli: an
nthrax baccillus, C. diphtheria and clostrid
dia.
 am-negativve bacilli: shigella,
Gra s sa
almonella, pseudomo
p onas…etc.
 Spiirochetes: Treponem ma pallidumm.
 Acttinomyces..

Mecha
anism of action:
a
 Pennicillins bin
nd to pennicillin bin
nding pro oteins
(PBBPs) on th he bacterrial cell w
wall and in nhibit
tran
nspeptidattion reactio
on essentia al for bactterial
cell wall syntthesis.
 Thee next sttep is ac ctivation o of intraceellular
autolytic enzzymes (autolysins) leading to o cell
rup
pture. Thus, the antibacterial efffect of pennicillin
is the resultt of both h inhibitionn of cell wall
synnthesis andd destructio
on of existting cell waall by
autoolysins.
 Pennicillins are
e bactericcidal espe ecially for Gram-positive baccteria whic ch have
thic
ck cell wallss.
 Thee major cau use of resis
stance is th
he producttion of β-la
actamase (penicillina
ase).

385
Pharmacokinetics:
 Absorption of oral penicillins is decreased by food. They must be administered
1hr before or 2 hr after meals.
 Route of administration of a B-lactam antibiotic is determined by the stability of
the drug to gastric acid and by the severity of the infection.
 Oral in moderate infection and acid stable preparations e.g. penicillin V.
 Paraentral in severe infection and acid sensitive preparations e.g. penicillin G
 Penicillins can penetrate to CSF and ocular fluid only during meningitis. They
cross placental barrier but they are not teratogenic.
 Most penicillins are excreted through organic acid secretory system via the
kidney. The renal excretion in proximal tubules could be decreased by co-
administration of probenecid which prolongs its duration of action.

PREPARATIONS OF PENICILLINS:

1.BENZYL PENICILLIN (PENICILLIN G) and RELATED DRUG:

Effectiveness: active against gram-negative and positive cocci, gram-positive


bacilli and spirochetes. It is sensitive to penicillinase enzyme.

Classification:

A. Injectable Short-acting preparation e.g. benzyl penicillin (penicillin-G):

- It is the prototype of all penicillin family.


- Primarily covers gram-positive organisms (Pneumococci, streptococci,
staphylococci (non-penicillinase-producing), Cl. perfringens,C. diphtheria) with
little gram-negative coverage (Gonococci and meningococci[but not bacilli]),
making it ineffective in urinary tract infection.
- Others: Treponema pallidum (syphilis).
- It has some limitations:
 It is acid labile and must be given parentrally.
 It has a short half-life, so frequent injections are required.
 It is easily inactivated by -lactamase enzyme.
 It has narrow spectrum.

B. Injectable Long-acting preparation

i. Penicillin G Procaine:
It is a suspensions of combination of penicillin G with procaine that have longer
duration (12-24 hrs) allowing reduced frequency of injections. So it is injected just
once every day, intramuscularly.

386
ii. Benzathine Penicillin: This preparation produces low blood levels lasting from
few days to 4 weeks depending on the dose. So it is used in chemoprophylaxis
and in the treatment of syphilis.

C. Oral Penicillin V: it is an acid stable form of penicillin G (given orally).

2.PENICILLINASE OR -LACTAMASERESISTANT PENICILLINS (ANTI-STAPH PENICILLINS)


[Methicillin, cloxacillin, dicloxacillin, flucloxacillin, and nafcillin]

- They are effective against -lactamase producingStaphylococci.


- They have low or no activity against other gram-positive and gram-negative
organisms.
- The use of these agents is now declining due to increased incidence of
methicillin-resistant S. aureus (MRSA).

3.EXTENDED SPECTRUM PENICILLINS


[Ampicillin and amoxicillin]

- They maintain gram-positive coverage with extended coverage of more gram-


negative bacteria e.g. Salmonella, H. influenza, Proteus, and Shigella (but not
Pseudomonas).
- They can be given orally (acid resistant) and
by injection (IM and IV). Common organisms
capable of producing
- They are inactivated by -lactamase, so they penicillinase:
are usually combined with clavulanic acid or
sulbactam (-lactamase inhibitors) to cover Staphylococcus aureus,
Escherichia coli,
-lactamase producing bacteria. H. influenzae
[Amoxicillin/clavulanic acid (co-amoxiclav) Pseudomonas aeruginosa
Neisseria gonorrhoeae
- Ampicillin/sulbactam (Unasyn)].
Proteus
- Ampicillin is concentrated in bile so it is Bacteroidesspecies.
effective in gall bladder disease and typhoid
carrier.
- Ampicillinis more active on shigella and H. influenza.
- Pro-drugs of ampicillin e.g. pivampicillin and talampicillin are esters of
ampicillin which themselves are microbiologically inactive but after oral
administration they are de-esterified in the gut mucosa or liver to release
ampicillin to the systemic circulation. They are better absorbed from the gut
than ampicillin itself, so gives higher levels in blood and tissues and they have
no effect on the gut flora and less G.I. upset.
- Amoxicillin has better absorption and tissue penetration than ampicillin. So, it
has no effect on the gut flora and less G.I. upset.

387
- Amoxicillin is more active against Salmonella and Streptococcal fecalis.
- Amoxicillin can penetrate mucoid and purulant sputum, so it is useful in
chronic bronchitis.

4.ANTIPSEUDOMONALPENICILLINS
[Ticarcillin, Azlocillin, and Piperacillin]

- They are broad-spectrum, but should only be used for pseudomonas infection
and ampicillin resistant proteus. Also they have activity against anaerobic
gram negative bacteria e.g. Bacteroids fragilis (a common pathogen in intra-
abdominal sepsis)
- They have synergistic effect when they are used with aminoglycosides.
- They are inactivated by -lactamase so they are combined with clavulanic
acid forming Timentin to cover -lactamase producing bacteria.

PENICILLIN G ANTI-STAPH EXTENDED ANTI-


PENICILLINS SPECTRUM PSEUDOMONAL
PENICILLINS PENICILLINS
Cloxacillin Ampicillin Ticarcillin
Flucloxacillin Amoxicillin Azlocillin
Nafcillin Pipracillin
Spectrum Gram +ve spp. Staphylococci Gram +ve and Pseudomonas
and few Gram Gram –ve and Gram –ve
–ve cocci spp. (except spp.
T. pallidum Pseudomonas)
-lactamase Yes No Yes Yes
susceptibility They are combined with
clavulanic acid or sulbactam
to cover -lactamase producing
bacteria.

Therapeutic uses of penicillins:

A. Treatment: Penicillins may be used in the treatment of:

 Streptococcal infections, e.g. wound sepsis, acute throat infections, subacute


bacterial endocarditis,.. etc.
 Staphylococcal infections of skin, mucous membrane and bone.
 Pneumococcal infections e.g. pneumonia.
 Syphilis (5 million units single dose) and gonorrhoea.
 Meningococcal infections: Penicillin diffuses into the CSF only when the
meninges are acutely inflamed.
 Typhoid and paratyphoid fevers: ampicillin and amoxycillin.

388
 Pse
eudomona as infection
n: (ticarcillin
n or pipera
acillin)
 Acttinomycosis, Anthrax x and H. in fluenza inffections.
 Dip
phtheria, teetanus andd gas gang grene. (Pennicillin may
y be used together with
w the
spe
ecific antito
oxins).

phylaxis: Penicillins
B. Prop P may be ussed prophy
ylactically in the follo
owing cond
ditions:

 To prevent reecurrence of rheuma atic fever: Benzathine


B e penicillinn, 1.2 millio
on units
IM once a mo onth.
 To prevent subacute
s bacterial
b e
endocarditis due to bacteraem mia resultin ng from
opeerative pro
ocedures such
s as deental extraaction, tonsillectomyy…etc. in patients
p
with
h congenittal or acqu
uired valvullar disease
e or immunnocomprom mised patient.

Advers
se effects::

■ Hyp
persensitiivity reactions:
‒ It occurs in ~10% off patients rreceiving penicillin.
p
‒ It occurs with
w all typ pes of peniicillin. Allerrgy is not due
d
to peniciillin itself but to degradattion product
common to t all peniccillin
‒ It is mo ore comm mon after parentera al than oral
o
administraation.
‒ All types of reactio ons, from simple ra ash to acu ute
anaphylax xis and angioedem
a ma, can oc ccur withinn 2
minutes (e ype I hype
early or ty ersensitivityy) or up to 12
days (delaayed or type II) afterr administrration.
‒ True anap phylaxis is rare (1:10’ 000 of cas ses).
‒ Penicillin allergy
a is unpredicta
u able i.e. an n individuaal who toleerated penicillin in
the past may
m develo op allergy l ater on and vice vers sa.

Pre
evention:
 Never give penicillin
n if there iss history off penicillin allergy.
 Test for hypersensittivity

Ma
anagement of anaph hock: see CVS.
hylactic sh

■ Oth
her advers se effects:
‒ Penicillin in
i high dos ses can ca ause neuro otoxicity and seizurees in patien nts with
renal failurre.
‒ Nafcillin iss associate
ed with neu utropenia and
a thrombocytopennia.
‒ Methicillinn causes innterstitial n ephritis (an
nd is no longer used for this re eason).
‒ Co-amoxiclav and flucloxacilli n cause he epatotoxiccity (cholesstatic jaund
dice).

389
Drug in
nteraction
ns:
 Bac cteriostatic drugs s (e.g. te etracyclinee, chloroa amphenico ol, erythro omycin)
inte
erfere with the action n of penicilllin becausse penicillin acts by iinhibiting cell
c wall
synnthesis (i.e. kills rapid
dly multipl ying bacteeria) while bacteriosttatic drugs arrest
the organism and preve ents its muultiplication
n.
 Anttipseudom
monal penic cillins (acid
dic drugs, ―ve
― charge ed) form coomplex with amin-
oglycosides (basic
( drugs, +ve charrged) if they
y are mixed
d in the sam
me infusion
n fluid.

Cepha
alosporins

 Cep
phalosporin
ns also hav
ve a β-lac
ctam ring and
a have
the same mechanisms s of action like penicillins.
 Eacch newer generatiion of ccephalospo orins is
incrreasingly re
esistant to
o β-lactammase.

Pharm
macokinetics:
 Cep
phalosporin
ns are wid
dely distrib
buted in bo
ody fluids;; memberss of the th
hird and
fourrth generattions can penetrate
p tto CSF (exxcept cefop perazone).
 Likee penicillin
ns, most cephalospo orins are excreted
e via the kidnneyand hence the
dosse must be e adjusted in patientt with rena al insufficie
ency. The renal excrretion is
alsoo decrease ed by probenecid.
 Cefoperazonee and ceftrriaxone (thiird generattion) are primarily exxcreted in bile
b and
its sserum leveel is not gre
eatly influe
enced by re enal failure
e.

Antibacterial spe
ectrum An
nd Classiffication:
They a d into 1st, 2nd, 3rd, a
are divided and 4th gennerations. The differrence amo ong the
groupss is markeed by cha anges in a antibacteria
al spectruum. In genneral, the activity
againstt gram-possitive bacteria decreeases from first to third generaation while activity
againstt gram-neg
gative orgaanisms inccreases.

1. FIRSTT-GENERATTION CEPHA ALOSPORINSS:


[Cepha alexin (Kefflex), ceph
hradine (Ve
elosef), ceffadroxil (D
Duricef)].

 Prim
marily cover gram-p positive oorganisms t penicilliin G) with
s (similar to h some
gram-negativ ve covera age (E. coiil, Klebsiellla), making
g them eff ffective in urinary
trac
ct infection
n.
 Theey are sens sitive to β--lactamas se.
 Theey do not penetrate
p to CSF (evven in pres
sence of meningitis).
m .
 Theere is partia
al cross allergenicity between them
t and penicillins.
p

390
2.SECOND-GENERATION CEPHALOSPORINS
[Cefoxitin, cefuroxime (Zinnat), cefaclor].

 They maintain gram-positive coverage with enhanced coverage of gram-negative


bacteria e.g. H. influenza, Neisseria, Proteus, (but not Pseudomonas).
 They are used primarily in the management of urinary and respiratory tract,
bone, and soft-tissue infections and prophylactically in various surgical
procedures.
 They are relatively resistant to -lactamases.
 They do not penetrate to CSF (even in presence of meningitis, except cefuroxime).

3.THIRD-GENERATION CEPHALOSPORINS
[Cefotaxime (Clarofan), Ceftazidine, Cefoperazone (Cefobid), Ceftriaxone (Rocephin)

 They maintain gram-positive coverage with excellent coverage of gram-negative


bacteria(including Pseudomonas)and active against anaerobes.
 They can penetrate to CSF in presence of meningitis (except cefoperazone).
 They are excreted by the kidney, except cefoperazoneand ceftriaxone which are
excreted biliary allowing their use in renal failure.
 They are highly resistant to -lactamase.
 They have long duration (12 hrs) and ceftriaxone has longer duration (24 hrs)

4.FOURTH-GENERATION CEPHALOSPORINS

 Cefepime has a powerful coverage against most Gram-positive and Gram-


negative bacteria (including Pseudomonas) and anaerobes.
 Ceftaroline is sometimes described as “FIFTH GENERATION” drug. .

Therapeutic uses:
 Severe undiagnosed sepsis especially in immunosuppressed patient.
 Treatment of infection of respiratory tract, urinary tract, skin, soft tissue, bones
and joints due to susceptible organisms.
 Gram-negative bacterial meningitis may be treated by cefotaxime (third
generation) and ceftriaxone that reach the C.N.S.
 Biliary infection: 3rd generation (cefoperazone or ceftriaxone).
 Gonorrhoea due to penicillin-resistant Gonococci. It is treated by single IM
injection of ceftriaxone.
 Pseudomonal infection when aminoglycosides are not desirable.

391
1ST GENERA
ATION 2NDD GENERATIION 3RD GE
ENERATION
N 4TH GENE
ERATION

Cephalex xin Ceefoxitin Cefootaxime Cefipim


me
Cephradine Ceefuroxime e Cefooperazonee
Cefadroxxil Ceefaclor Ceftrriaxone
Spectrrum Mainly Grram Ma aintain Gra
am Excellent Wide Grram
+ve spp. and +vve coveragge Gramm –ve +ve andd Gram
few Gramm –ve an
nd enhance ed coverage –ve cov verage
bacilli Grram –ve (inclu
uding including
co
overage Pseudomonas) Pseudomonas
-lacta
amase Yes Yees No No
suscepptibility
Penetrration to No even in No
o (except Yes (except
( Yes
CSF meningitis
s ce
efuroxime) cefop
perazone)
Cross allergy High Mo
oderate Low Low
with pe
enicillin
Elimina
ation Renal Re
enal Rena
al & biliary Renal & biliary

Advers
se effects::

‒ Hyp
persensitivvity reac ctions: 1 0% of penicillin--sensitive persons share
hyp
persensitivity to ceph halosporin
ns. The crooss hypersensitivity with penicillin is
rd th
mucch lower with
w the 3 and 4 ge eneration drugs.
d
‒ Som
me first generatio on cephallosporins are neph hrotoxic especially
y when
adm
ministered with loop diuretics
d o
or aminogly
ycosides.
‒ Somme third generation
g n cephalossporins (e.g
g. cephope
erazone) innhibit the enzyme
e
vitam
min K ep poxide re eductase leading to o hypoproothrombin nemia (co ould be
prevvented by vitamin
v K).
‒ Supperinfectio membranous colitis)): Cephalo
on with C.. difficile (pseudom osporins
are the main n cause off hospital--acquired C. difficile
e colitis, a potentia
ally life-
eatening in
thre nfection.

Pseud
domembra
anous collitis

Pseud domembran nous colitis (= antibioti c-associate


ed diarrhea or C.
le colitis), can occ
difficile cur followwing antibiotic treatment.
When antibiotics are given, most of thee resident in ntestinal ba acteria
are killled with sp paring of soome types like Clostri e. The
ridia difficile
norma ally harmlesss C. difficcile grow raapidly because of lack of
compe etition with
h other florra, and pro oduce toxin ns. These toxins
t
damag ge the inner wall of the
e intestines and cause abdominal pain,
fever aand diarrhe ea. Severe cases
c can b
be life-threatening.

ment:
Treatm
‒ Stop all antibiotics when never possibble.
‒ Orral metroniddazole 250 1st choice).
0 mg /6 hr (1
‒ If m
metronidazoole failed, give
g oral van
ncomycin solution
s 125
5 mg/6 hr

392
Drug interactions:
 If IV ceftriaxone is added to any IV calcium-containing solution (e.g. Ringer’s
or parenteral nutrition) dangerous particulates of ceftriaxone-calcium can result
and precipitate in the lungs and kidney. Separate between IV ceftriaxone and any
IV calcium-containing solution by at least 48 hours.
 Some third generation cephalosporins inhibit the enzyme aldehyde
dehydrogenase. If they are co-administered with alcohol, acetaldehyde
accumulates in blood leading to nausea and vomiting (disulfiram-like reaction).

█ OTHER beta-LACTAM DRUGS

Aztreonam (Azactam)

 Aztreonam is a monobactam that share similarity in structure and mechanism


with penicillin but is highly resistant to β-lactamase.
 Aztreonam is especially effective against β-lactamase producing gram-negative
bacteria. It could be used as alternative to aminoglycosides in severe
respiratory, urinary, biliary, GIT and female genital tracts infections.
 It has no cross-hypersensitivity with penicillins or cephalosporins.
 Most of the Adverse effects reported have been, for example, hematological
effects, nephrotoxicity and pseudomembranous colitis,.

Carbapenems [Imipenem-cilastatin, meropenem]

 They are broad spectrum antibiotics that share similarity in structure and
mechanism with penicillin but they are highly resistant to β-lactamase.
 They are one of the antibiotics of last resort for many bacterial infections.
 The ease of penetration (due to its low molecular weight) and resistance to -
lactamase imparts a broad-spectrum of antimicrobial activity against most
aerobic and anaerobic bacteria (Gram-positive and gram-negative) with the
exception of occasional Pseudomonas strains.
 If imipenem is given alone it is inactivated by dihydropeptidase enzyme in kidney
leading to low urinary excretion and significant renal toxicity in animals, therefore
it is combined with cilastatin (Tienam) to inhibit renal dihydropeptidase enzyme.
 Meropenem does not undergo metabolism by renal dihydropeptidase enzyme.

Adverse effects:

 Blood disorders.
 Seizures in high doses. Meropenem is less likely to provoke seizures.
 G.I.T: nausea, vomiting, etc.

393
█ OTH
HER CELL
L WALL INHIBITORS
S

Vanco
omycin

 Van ncomycin inhibits ce ell wall synnthesis and enhance es cell lysiss.
 It is active agaainst gramm-positive organism ms.
 It iss given byy slow i.v. infusion tto treat se erious MR RSA infec ctions in patients
p
allergic to pennicillins or cephalosp porins.
 Bec cause it iss not abso orbed orallly, it is us sed by thiss route too treat anttibiotic-
ass sociated enterocolit
e tis (C. diffficile colitis
s) and oth
her infectioon by susc ceptible
orga anisms.
 Rap pid infusio
on of vanc comycin m may cause e anaphylactoid reaactions an nd “red
man n” syndroome (skin rash and flu ushing due e to histam
mine releasse).
 Rarrely, high levels of vancomyc
v in may ca ause perm manent aud ditory imp
pairment
(otootoxicity) and
a nephrrotoxicity.

Bacitrracin

 Bac hibits the building of p


citracin inh peptidoglyycan blocks (cell walll units).
 It is most activve against gram-pos sitive bacteria.
 It iss used only topically y in comb bination witth neomyccin or pollymyxin fo or minor
skin n infectionss due to seerious nep hrotoxicity
y.

B. INHIBITORS OF
O BACTE
ERIAL PRO
OTEIN SY
YNTHESIS

Macro olides
[Erythrromycin (p
prototype)), azithrom
mycin, clarrithromycin, roxithrromycin]

Mecha
anism of action:
a
 The
ey bind reversibly
r to the 50S ribo osomal
bunit and inhibit bac
sub cterial prootein synthhesis.
 ey are bactteriostatic in low co
The oncentration
ns and
bac
ctericidal in high conce
entrations.

Antibacterial spe
ectrum:
Macrolids are effective against a numb ber of
organissms, inclu
uding gram m-positivee bacteriaa, e.g.
pneumoncocci, staphyloco
s occi, strep
ptococcus species, C.C diphtheeria, some e gram-
negativve bacteria, e.g. Neisseria, H H. influenzza, and in
ntracellularr microorg
ganisms
(Mycopplasma spe ecies, legio
onella and Chlamydia a).

394
Pharmacokinetics:
 Absorption of erythromycin is affected by food and HCl. To minimize destruction
and enhance absorption, erythromycin is administered as esters salts.
 Newer macrolides are acid stable and can be given orally.
 They reach all body fluids including prostate, placenta and milk but only small
amounts can penetrate to CSF.
 Azithromycin and roxithromycin are concentrated in neutrophils, macrophages
and lung tissue, so they have long t1/2 (given once daily) and they are useful
against intracellular organisms.
 Macrolides are concentrated in liver and excreted primarily in active form via
bile with only low levels found in urine.
 Azithromycin and clarithromycin are converted into active metabolites.

Therapeutic uses:

 Treatment and prevention of infections caused by staphylococci, streptococci


and pneumococci (gram-positive) in penicillin-hypersensitive individuals.
 Eradication of C. diphtheriae from pharyngeal carriers (1st choice).
 Azithromycin and roxithromycin are especially effective against intracellular and
atypical bacteria (chlamedia, mycoplasma, legionella)
 Clarithromycin has activity against Toxoplasma. It is also effective against
Helicobacter pylori in peptic ulcer.
 It is a second line drug for the treatment of gonorrhea and syphilis.
 Treatment of middle ear and sinus infections, since the causative agents, H.
influenza and Strep pneumonia are usually sensitive.

Adverse effects:

‒ Epigastric distress especially with erythromycin.


‒ Erythromycin may cause cholestatic hepatitis probably due to hypersensitivity
reaction to the oral estolate form or hepatoxic effect when drug therapy lasts
longer than 10 days or repeated courses are prescribed.
‒ Erythromycin and clarithromycin inhibit hepatic CYP450– metabolism of some
drugs e.g. warfarin and phenytoin (Azithromycin is devoid of this side effect).
‒ Bacterial resistance if it is used more than one week.
‒ Hypersensitivity reactions: fever, rashes and cholestatic hepatitis.

Interactions:

 Combination of the therapeutic dose of erythromycin with penicillin antagonizes


the bactericidal effect of penicillin.
 Erythromycin decreases CYP450 enzymes, thus increase serum concentration of
theophylline, oral anticoagulants, cyclosporins, and digoxin.

395
Lincos
samides: Clindamycin

 Thee lincosam
mide family
y of antib
biotics inc
cludes linc
comycin aand clinda
amycin.
Clin ndamycin is semisyn nthetic derrivative of lincomycin and is m more poteent than
lincomycin.
 Clin ndamycin is i similar, in the mecchanism and kinetics s, to erythhromycin but has
activity againsst anaerob bic bacterria.
 It iss used as an alterna ative drug for treatment of ana aerobic in nfections. Topical
prep parations are
a used fo or treatme nt of acnee.
 Foo od in the sttomach do oes not intterfere withh the abso
orption of cclindamycin. So it
is completely absorbed after oral a administra ation.
 App proximatelyy 90 % are a plasma a protein bound. Clindamycinn penetratte most
tissues well, including bone. The erefore, bone and joint infecctions cau used by
susceptible organisms respond
r w
well to treattment with clindamyccin.
 It iss associateed with higgh incidencce of diarrh hea and ps
seudomem mbranous s colitis
as side effec ct due to superinfec
s ction by re esistant clostridia inn addition to side
effe ects of erytthromycin.

Amino oglycosides
[strepto
omycin (prrototype), Gentamiciin, tobramycin, amik
kacin, kana
amycin,
neomy ycin]

Mecha
anism of action
a
 Am
minoglycosiides are transportedd across the inner
celll membranne by active transpo
ort system
m present
onlly in Gram
m-negative
e aerobic sspp.
 Insiide the ce
ell, they bin
nd to 30S ribosoma
al subunit
and
d inhibit ba
acterial pro
otein synth
hesis.
 Theey are bac ctericidal. Bacterial
B kkilling is co
oncentra-
tion
n-depende ent i.e., the
e higher the e concentrration, the
more bacteria a are killed,, so they aare better given
g as a
sing
gle large do
ose daily.
 Appost-antibiotic effec ct is also p
present i.e., residual
bac
ctericidal activity is s present after the serum
con
ncentrationn falls belo
ow the MIC C.

Antimicrobial sp
pectrum:

They have narrow spectru um againstt mainly Gram


G ative bacillli, very few
nega w Gram
positive
e cocci, an
nd Mycoba
acteria TB..

396
Pharmacokinetics:
Penicillins and
 All aminoglycosides are not absorbed orally aminoglycosides
(must be given parentrally) and cannot
The antibacterial effects of
penetrate to CSF because they are highly
all β-lactam antibiotics are
polar compounds. synergistic with the amino-
 They can cross the placental barrier and may glycosides. Because penicil-
cause congenital deafness. lin inhibits cell wall synthesis
and facilitate the entry of
 They are excreted unchanged in urine by
aminoglycosides to inside
glomerular filtration (dose adjustment is the bacterial cell.
necessary in renal dysfunction). They become
[Note: these drugs should
more active in alkaline urine.
never be combined in the
same infusion fluid because
Therapeutic uses: the basic aminoglycosides
form inactive complex with
The role for aminoglycosides has decreased
the acidic penicillin]
substantially due to their narrow spectrum and
potential toxicity.

 Streptomycin is currently used only for plague, brucellosis and it is one of the
1st line drugs for TB.
 Gentamicin and tobramycin are used in the treatment of gram-negative
infections e.g. urinary tract and respiratory infections.
 They are used in gram-negative septicemia, usually combined with penicillin
and/or metronidazole.
 Pseudomonas infections: they are more efficient when combined with
antipseudomonal penicillin
 A combination of vancomycin and gentamicin is useful in the treatment of
enterococcal endocarditis.
 Neomycin is used orally for hepatic encephalopathy to suppress intestinal
bacteria that produce ammonia.
 Neomycin is used orally for sterilization of intestine before surgery and
bacillary dysentery.
 Neomycin and gentamycin are used topically for skin and eye infections often in
combination with polymixin B or bacitracin.

Adverse effects:

Aminoglycosides have a narrow therapeutic index; it may be necessary to monitor


serum concentrations and individualize the dose especially in old age.

■ Nephrotoxicity:
‒ It is due to accumulation of aminoglycosides in the renal tubular cells.

397
‒ It ranges from mild reversible effect to severe irreversible toxicity.
‒ It is increased by co-administration of other nephrotoxic drugs e.g.
cephalosporins.

■ Ototoxicity:
‒ It is due to accumulation of aminoglycosides in the endolymph and perilymph
of the inner ear causing damage to the hair cells in the organ of Corti.
‒ It may affect the cochlear (auditory) or vestibular functions.
‒ Deafness may be irreversible.
‒ It is increased by co-administration of other ototoxic drugs e.g. loop diuretics
and in old age and with renal diseases.

■ Neuromuscular block:
‒ Often occurs after direct intraperitoneal or intrapleural application of large
doses of aminoglycosides leading to skeletal muscle weakness.
‒ It is treated by immediate administration of calcium gluconate or neostigmine.

■ Malabsorption:
- Occasionally observed following the oral administration of streptomycin,
neomycin, kanamycin. It is due to binding with bile salts and inhibition of
pancreatic lipase leading to steatorrhoea and diarrhoea.

Drug interactions:

 With antibiotics:
 With cephalosporins nephrotoxicity increases.
 Anti-pseudomonal penicillins, and cephalosporins decrease the antibacterial
effect of gentamicin if combined together in the same syringe (in-vitro)
because penicillins are acidic and aminoglycosides are alkaline.

 Skeletal muscle relaxants:


 Aminoglycosides increase the effect of non-depolarizing NMB agents (see ANS).
It could be reversed by neostigmine and calcium gluconate.
 Aminoglycosides should be administered with great caution during surgery or in
the post-operative period.

 Anticoagulants:
 With oral anticoagulants: oral aminoglycosides impair vitamin K production by
intestinal bacteria potentiating the effect of anticoagulants.
 Heparin precipitate aminoglycosides (avoid their mixing in the same syringe).

 Diuretics: Diuretics e.g. ethacrynic acid, and mannitol potentiate ototoxicity of


aminoglycosides.

398
Tetrac
cyclines
[Oxyte
etracycline
e (prototyp
pe), doxyc
cycline, tig
gecycline]

Mecha
anism of action:
a
 Tetracyclines bind re eversibly to the 30S
3
ribo
osomal subunit
s and
a inhib
bit bacte
erial
prootein synth
hesis.
 Theey are bactteriostatic
c.

Antibacterial spe
ectrum:
Tetracyyclines display broadd-spectru m activity and are efffective agaainst:
 Mosst gram-po ositive, ma
any gram-nnegative ba
acteria andd Brucella.
 Rick kettsia, Co
oxiella, Myccoplasma and Chlam mydia (intra
acellular orrganisms).
 Spirrochetes, Actinomyc
A cines, Proto
ozoa.
 Heliicobacter pylori.
p

macokinetics:
Pharm

 Theese antibiiotics are partiallyy absorbe ed from the stom mach and upper
gasstrointestin
nal tract an
nd the amo ounts rema aining may alter bactterial flora leading
l
to s
super infe ection. Beccause abso orption of doxycycline is rapid d and commplete, it
hass weak effeect on intestinal flora
a.
 Callcium (milk and Ca. C antaciids), mag gnesium (Mg hydro oxide), alu uminum
hyd
droxide an nd iron intterfere witth their absorption since theey form in nsoluble
cheelates with tetracyclin
nes.
 The a body fluids and pllacenta du
ey reach all pid solubiliity but only small
ue to its lip
mounts can
am n penetratee to CSF (DDoxycyclin ne is the most
m lipid--soluble).
 Beccause of th heir chelatting propeerties with calcium th hey tend to o be depoosited in
gro
owing bone es and teetth causing g yellow dis
scoloration n of teeth.
 Thee primary route
r of elimination iis renal. Doxycyclin
D ne is excre eted in sto ool and
doees not acc cumulate in patientss with renal impairme ent.

Therap
peutic use
es:

 Tetracyclines have brooad spectrrum activiity againstt both Grram-positive and


Graam-negatiive organis
sms, but t heir use is
s now declining becaause of inc
creased
resistance an
nd the deve
elopment o
of safer druugs.
 Dox
xycycline is used as s alternatiive to ma
acrolides in Chlamyd
dia, Mycop
plasma,
and
d Legionellla infection
ns.
 The
ey have broader spe ectrum tha
an macrollides again
nst other atypical bacteria
b
(Bo
orrelia, Rickkettsia, Cox
xiella):

399
- Borrelia bu
urgdorferi:: is the cau
use of Lym
me
disease, a sp
pirochetal infectio on
transmitted
d by infectted ticks. T The diseas se
consists of skin rash,, fever, and
d arthritis.
- Rickettsia:: is a gen
nus of sm
mall obligatte
intracellula
ar bacteria
a causing ttyphus an
nd
Rocky Mo ountain sppotted fevver.
- Coxiella burnetii:
b is a sma
all obligatte
intracellula
ar bacteriu
um causing
g Q fever.
 xycycline is used orally (100 m
Dox mg/12h for 3
months) for tre
eatment of acne vulgaris.
 xycycline is effective
Dox e in treatm
ment of cho
olera (300 mg single oral dose)).

Advers
se effects::

- Epiigastric distress (nausea, voomiting, ep


pigastric burning
b and hyperacidity)
a
resulting from m irritation
n of gastrric mucosa a. This ca
an be prevvented by
y giving
tetrracycline after
a meals.
- Yelllow stain ning of bo oth the d deciduous and
perrmanent te eeth and dental enam mel hypoplasia
(inc
creased su usceptibilitty to cariees, as well as
retaardation of
o bone grrowth) bec cause of their
cheelating pro operties with
w calciu um. This can
occcur if tetrac
cyclines arre adminisstered afterr the
fourth month of gestatio on or if the
ey are give
en to
chilldren lesss than 8 years
y of a age, this is a
furtther reasoon for avo oiding the eir use duuring
preegnancy.
- Cho
olestatic hepatitis
h particularly
p y in pregnaant women n and at laarge doses.
- Pho y: skin reaction in the
ototoxicity e form of sunburn,
s occurs wheen a patiennt taking
tetrracyclines is exposed
d to sunlig ht or ultrav
violet rays..
- Sup on with re
perinfectio esistant sttaphylococ cci or C. difficile (C
C. difficile colitis).
Also, decreassed syntheesis of Vit. B12 leads to sore tongue & blaack hairy to ongue
- Terratogenessis, when administere
a ed early in pregnancy.

Contra
aindication
ns:

 Sevvere hepattic disease and renal disease.


 It should be avoided
a in patients w
with peptic ulcer,
 In c
children (le
ess than 8 years
y of ag
ge)
 In p
pregnancy and lactattion.

400
Interac
ctions:

 Avooid simultaaneous inggestion of ddairy products (milk & cheese) , antacids,, or iron
withh tetracycllines becauuse of che
elating actio
on of tetracyclines.
 Tettracycliness decrease vitamin K synthesis in intestinal lumen d due to inhib
bition of
inte
estinal flora
a leading to potentiat
ation of ora
al anti-coag
gulant effecct.

Chlora
amphenic
col

anism of action:
Mecha a

 Chlloramphen nicol binds to the 50S ribo osomal


subbunit and inhibits
i ba
acterial prrotein syntthesis.
 Thee drug is either
e c (more co
bactteriostatic ommon)
or b
bactericida
al dependin
ng on the o organism.

Pharm
macokinetics:

 Duee to its lipo


ophilic natuure, chloraamphenico ol is absorb
bed rapidlly, so there
e is less
alte
eration of bacterial flora wiith less susceptibil
s ity to suuperinfectioon and
disttributed throughout body
b fluidss.
 Theerapeutic le evels can penetrate
p tto the CSF
F and placeenta.
 Chlloramphen nicol is inac
ctivated in the liver by
b glucuronyl transfferase.
 Elim
mination iss mainly re enal. Abou ut 10 % of o the adm ministered drug is excreted
uncchanged (sso it is not useful in U UTI).

peutic use
Therap es:

 Chlloramphennicol has broad


b spec
ctrum against both Gram-neg
G gative and Gram-
sitive spp.., but its us
pos se is limite
ed because of its sev
vere toxicitty.
 It c
could be used as ann alternativve to macrrolides and
d tetracycllines for diseases
cauused by aty
ypical bac
cteria (Myc coplasma, Legionellaa, Rickettsiia, Coxiella
a, etc).
 As an alterna ative to pe enicillin an
nd cephaloosporins fo
or treatmeent of meningitis
(due to H. inflluenza), typ
phoid feve er, and anaerobic in
nfections.
 Use
ed as eye drops
d (0.5%) for eye
e infection
ns.

Advers
se effects::

 mia and bo
Apllastic anem one marrow
w suppres
ssion:
‒ It is dose dependent
d t – rare bu t fatal.
‒ It may occcur weeks or monthss after stop pping of the drug.
‒ It may be a result of the inhibittion of hum
man mitoch hondrial prrotein synthesis.
‒ Managem ment of BM suppressiion: see blood.

401
 Gra
ay baby sy
yndrome:
‒ It occurs in neonate es becausse their live
er
cannot me etabolize chloramphe
c enicol.
‒ It consissts of cyanosis, collapse,
abdomina al distensio
on, and sho
ock.
‒ Mortality is high (40%
%).

 Inhibition of cytochro ome P-450 0 enzyme


es
can
n increase levels of other
o drugss.

C. INHIBITORS OF
O BACTE
ERIAL NU CLEIC AC
CID SYNTH
HESIS

Fluoro
oquinolo
ones
[Ofloxa
acin, norflo
oxacin, cip
profloxacin
n, levofloxacin, moxifloxacin, g
gemifloxac
cin]

Quinoloones are synthetic


s analogues
a of Nalidixic acid. The earlier q
quinolone nalidixic
n
acid did not achieve systtemic anti bacterial levels and d is usefuul only as urinary
antisep
ptic. Fluo orination was fou und to produce
compo ounds with greater antibacteria al activity, achieving
useful d
drug levelss in the blo
ood and tisssues.

Membe
ers:

 Firsst generatioon: Nalidix


xic acid.
 Sec cond generration: Pipemidic aci d.
 Thirrd generattion: Mostt are fluorrinated. Th hey include ciproflooxacin, ofloxacin
(tarrivid), levoffloxacin ettc…
 Fou urth generation: e.g. moxiflloxacin, trovafloxa acin, theyy are like third
genneration witth enhance ed gram-ppositive and
d anaerobees activity..

anism of action:
Mecha a
 The
ey inhibit type
t II DN somerase (DNA gyrrase) that is necess
NA topois sary for
bac
cterial repliication.
 Quiinolones are
a bactericidal. Likke aminoglycosides, they exhib
bit concen
ntration-
dep
pendent killing and a post-antib
biotic effec
ct.
 Ressistance iss due to po
oint mutatio
ons in the target enzy
yme.

Pharm
macokinetics:
 Thee absorptio nesis is ↓ by
on of fluorroquinolon b stomac
ch contentss especially milk,
n and anta
iron acids.

402
 The
ey reach alll body tiss
sues but on
nly small amounts
a can
c reach C
CSF.
 Newwer fluorooquinolone es (e.g. leevofloxacin
n and mo n) accumu
oxifloxacin) ulate in
neu
utrophils and macrop phages, soo they have long t1/2 (given oncce daily) and they
are useful aga
ainst intracellular orrganisms.
 The
ey are excrreted prima
arily uncha
anged in urine,
u so th
hey are useeful in UTIs
s.

Therap
peutic use
es:

 Fluo
oroquinolo
ones are es
specially in
ndicated in
n resistantt infection
ns. Empiric
c use of
thesse agents in minor innfections s hould be discourage
d ed.
 rd
3 generation n e.g. Cipprofloxacin have gre eater activ
vity againsst gram-ne egative
bac cteria and moderate activ ity against gram--positive organism ms and
ana aerobes. Newer
N com
mpounds (m moxifloxacin, trovaflo
oxacin) havve equal activities
a
aga ainst all.
 Levvofloxacin and moxiffloxacin arre known as “respirratory quiinolones” due to
theiir high activity against S. pneumoniia and other o atyppical resppiratory
pathogens (le egionella, mycoplasm
m ma).
 rd
3 g generation are used for most s severe sy ystemic inffections ee.g. GIT in
nfection,
urin nary tract infections
s and prosstatitis, reespiratory infections , skin andd Bone
infeections (ostteomyelitis
s).
 Trea atment of pseudomo onas infec
ction.
 Ciprofloxacin for salmonella; ente eric fever

Advers
se effects::

 GIT
T upset: na
ausea, vommiting (the most common).
 Arth
hropathy (in
( experim mental animmals):
‒ It was mannifested as
s articular d
damage & erosions in weight-b
bearing join
nts.
‒ AAlthough this side effect ha as not be
een reported in hum man, the use of
ffluoroquino
olones is not
n recom n <18 years
mmended in children rs.

 Ten
ndinitis an
nd tendo e: there is increas
on rupture sed risk o
of tendinittis and
spo
ontaneous tendon rupture espe
ecially with
h ciprofloxa
acin. The A
Achilles te
endon is
the most frequently affe
ected.

 CNS
S (1-2%): seizures and
a psychiiatric problems.
Fluo
oroquinolo ones shoulld be used
d cautious
sly in
patiients with epilepsy.
e

 Pro and ↑ ris


n of QT interval a
olongation sk of
torssade de po
ointes.

 Inhiibition of cytochrom s can


me P-450 isozymes
incrrease levels of other drugs.

403
D. INHIBITORS OF
O BACTE
ERIAL ME
ETABOLISM

Sulfon
namides
s and trim
methopriim

Mecha
anism of action:
a

 Baccteria cannnot utilize preformed


p folic acid;; they use
aminobenzzoic acid (PABA) a
P-a as a precursor for
syn
nthesis of dihydroffolic acid d (by the enzyme
dihyydrofolate synthase) e), then too the acttive form
tetrrahydrofoolic acid (b by the enzzyme dihyydrofolate
ductase), necessary
red n for bactterial nucleic acid
syn
nthesis.
 Sullfonamidees are struc
ctural anallogs of PA
ABA. They
com
mpete with h PABA at the enzymme dihydrofolate synth
hase resultting in inhib
bition of
dihydrofolic acid
a synthe
esis.
 Trim
methoprimm acts by y inhibitionn of next step (formation off the activ ve form
olic acid) via inhibitio n of dihydrrofolate reductase ennzyme.
tetrrahydrofo
 Botth sulfonam
mides and trimethop
prim are ba
acteriostatiic drugs.
 zole is a combina
Co--trimoxaz ation of sulfamethoxazole (400 mg) and
trim m (80 mg) to
methoprim t produce
e sequential block of
o folic acid
d synthesis
s.
 Bac cterial resisstance occ
curs by de
evelopmentt of alterna
ative metaabolic path
hway for
folic
c acid syntthesis.

Pharm
macokinetics:

 Moost sulfonam mides are adequatel y absorbed from the e GIT (exceept sulfasallazine).
 Theey reach alll body fluid
ds includin
ng CSF (evven without meningititis).
 Sulfonamidess are metabolized i n the live er and exc creted by the kidne ey. The
aceetylated prroducts ca an precipittate and crystalize
c in acidic urine leaading to
urin
nary stoness (crystalluria).
 Alk
kalinization n of urine can en nhance ac ctivity of sulfonami des and reduce
crysstalluria.

404
Therap
peutic use
es:

 Co-trimoxaz
zole:
 T
Treatment of upper respiratorry tract inffections an
nd bronchhitis cause
ed by S.
p
pneumoniaa and H. in
nfluenza.
 T
Treatment of non-co
omplicated
d UTIs and prostatitiis.
 T
The drug of
o choice for
f treatme ent of Pneeumocystis
s carinii p
pneumonia
a (PCP)
a
and toxop
plasomosiss in AIDS p
patients.

 Oth
her sulfonamide com
mbination
ns:
 Fansidar is a comb bination o
of sulfadox
xine + pyrrimethaminne is used
d as an
a
alternative
e treatme ent of m malaria caused
c by
b chlorroquine-re
esistant
P
Plasmodiu um falcipa
arum mala aria
 S zine is poorly
Sulfasalaz p ab
bsorbed combinatio
c on of sullfapyridine
e + 5-
a
aminosalic
cylic acid used
u to tre
eat ulcerative colitis.
 S
Silver sulffadiazine is used top
pically for the
t treatme
ent of burn
n.

Advers
se effects::

- Hyp
persensitiivity reactions:
- Fever andd rash are e the mosst
common.
- Steven-Jo ohnson syndrome e
is a rare
e, but fata al form o
of
extensive skin an nd mucuss
membrane e lesions s due to o
hypersenssitivity reacttion.

- Cryystalluria: due to precipitation


p n
of ssulfonamid
de metabo
olites in ac
cidic urine leading to
o colic, hem
maturia an
nd even
anuuria.

- Kerrnicterus: sulfonamides causee displacement of biilirubin from


m plasma protein
binding sitess. When th hey are g iven to neonates (or( in the last 2 we
eeks of
pre
egnancy), high
h free bilirubin le
evels in th
he blood can
c cross BBB andd cause
perrmanent brrain damag ge.

- Ane
emia:
 Megalobllastic ane
emia: due to folic acid deficie
ency (can be preven
nted by
c acid supplementattion)
giving folic
 Aplastic anemia
a lea
ading to grranulocyto
openia and thromboccytopenia.
 Hemolytic c anemia in patientss with G-6--PD deficie
ency.

405
E. MISCELLANEOUS AND LESS COMMON ANTIBACTERIAL AGENTS

Metronidazole

 Metronidazole was introduced as an antiprotozoal agent but it is also active


against anaerobic bacteria such as Bacteroides, Clostridia, and some
Streptococci (see antiamebic drugs)
 It is effective in the therapy of pseudomembranous colitis, and serious
anaerobic infections (e.g. sepsis secondary to bowel disease).

Fusidic acid

 Fusidic acid is a narrow-spectrum steroid antibiotic active mainly against Gram-


positive bacteria. It acts by inhibiting bacterial protein synthesis.
 It is used in combination with other antistaphylococcal agents in staphylococcal
sepsis, and topically for staphylococcal infections (e.g. as eye drops).
 It is associated with GIT upset (common), skin eruptions and jaundice.

Streptogramins: Quinupristin and dalfopristin

 Quinupristin and dalfopristin are new class of protein synthesis inhibitors. They
inhibit bacterial protein synthesis by binding to the 50S ribosomal subunit.
 Individually, they exhibit only mild bacteriostatic activity, but combined together as
an intravenous injection, they exert bactericidal activity against drug-resistant
Gram-positive spp. including vancomycin-resistant Enterococci and MRSA.
 Their use is limited to serious bacterial infections when other antibiotics fail.
 Bacterial resistance is still uncommon.
 Both drugs undergo extensive first-pass hepatic metabolism and must be given
as an intravenous infusion.
 Adverse effects include inflammation and pain at the infusion site, arthralgia,
myalgia and GIT upset.

Oxazolidinones: Linezolid

 Linezolid is the first member of this new class of protein synthesis inhibitors
(approved in 2000). They inhibit bacterial protein synthesis by binding to the
50S ribosomal subunit.
 It is active against a wide variety of drug-resistant Gram-positive bacteria such
as vancomycin-resistant Enterococci and MRSA. The drug is also effective
against some anaerobes, such as C. difficile.

406
 Its use is limited to serious bacterial infections when other antibiotics fail.
 Bacterial resistance is still uncommon.
 Adverse effects include thrombocytopenia and GIT upset.

Rifaximin

 It is a poorly-absorbed antibiotic related to rifampicin (see antiTB drugs). It has


activity against many Gram positive and Gram negative bacteria. Like rifampin, it
inhibits bacterial DNA and RNA synthesis.
 It is used orally for hepatic encephalopathy to suppress intestinal bacteria that
produce ammonia.
 It is recently approved to reduce symptoms of abdominal bloating and flatulence
associated with irritable bowel syndrome.

Nifuroxazide (Antinal)

 It is a poorly-absorbed antibiotic related to nitrofurantoin (see UTI). It has


activity against many Gram positive and Gram negative enteropathogenic
bacteria. The exact mechanism is unclear.
 Bacterial resistance is still uncommon.
 It is used orally as intestinal antiseptic in cases of infectious diarrhea.

Polymixins

 The polymixin antibiotics are polymixin B and colistin (polymixin E). They exert
their antibacterial action by disrupting the outer cell membrane.
 They have selective and rapid bactericidal action on Gram-negative bacilli,
especially Pseudomonas and coliform spp.
 They are not absorbed from the GIT.
 Due to their high toxicity, they are used only locally for eye and skin infections.

Antipseudomonal drugs

 Antipseudomonal penicillins: e.g. ticarcillin, piperacillin, etc.


 Monobactams: e.g. aztreonam
 Carbapenems: e.g. imipenem, meropenem.
 3rd and 4th generation cephalosporins: e.g. ceftazidime & cefipime
 Fluorinated quinolones: e.g. ciprofloxacin.
 Tobramycin and gentamycin.

407
CAS
SE STUDY
Y
Part 3
3:
Tre
eatmentt of Urin
nary Trac
ct Infections (U
UTI)

Definittion: infecttion of the urinary tra


act indicate
ed
by presence of o > 10 pus ce ells/HPF in
centrifu
uged urine sample (p pyuria).

Sterrile pyuria: presence e of pus ccells witho


out
orga
anisms. It occurs in some co onditions e.g.
renaal TB, ana algesic nep phropathy, and bladdder
tum
mors.
Baccteriuria: urine
u culturre proves presence of
bactteria in urine
e with or without symp
ptoms of UTTI.

▌Pred
disposing factors

 Fem
males > maales due to
o short uretthra – Children > aduults due to
o bad hygieene.
 Imm
munosupprression e.gg. in cases of diabetees mellitus.
 Pressence of obstructio
on to urin e flow e.g g. congennital abnorrmalities, urethral
stric
ctures, and
d stones.

▌Caus
sative org
ganisms

 E-c
coli is the most
m commmon organ nism (80%
%) in uncom
mplicated iinfection.
 Stap
phylococc hyticus (10
cus saproph 0%).
 Oth
her Gram-n negative bacilli
b e.g.. Proteus, Pseudem
monus & K Klebsiella are the
com
mmonest organisms.
o in complic
cated infec
ction.

▌Clinical picture

 Upp
per UTI (p hritis): infla
pyeloneph ammation of the
rena
al tissue orr pelvis.
Mannifestations: loin pain
n, chills, & fever.

 Low cystitis): in
wer UTI (c nflammatio
on of the bladder
b
& urrethra.
Mannifestations: dysuria (burni ng mictu
urition),
urgeency & freq
quency of urination.

 Unc
complicate ccurs for the 1st
e) UTI: oc
ed (simple
time
e without complicatio
c ons. It is usu
ually lower UTI.

 Com nt) UTI: ussually upp


mplicated (recurren per UTI
with
h complica
ations e.g. renal ston es or anatomical dys
sfunction.

408
▌Laboratory investigations

 Urine analysis: the most common test.


‒ Macroscopic: color, pH, glucose, protein, etc.
‒ Microscopic: WBCs, RBCs, epithelial cells, crystals, casts, ova, etc.

 Urine culture & sensitivity test:


‒ Culture and sensitivity test is essential in recurrent UTI because mixed
infection and drug resistance are common.

‒ Does the in-vitro test necessarily reflect the in-vivo response to drugs?
NO because:
- The antibacterial drug is not excreted in sufficient amount in urine or
excreted in urine but as inactive metabolite.
- Presence of contraindications of the drug e.g pregnancy or children.
‒ Most bacteria causing UTI cause alkaline urine, but strongly alkaline urine is
suggestive of Proteus infection because it is urease +ve and splits urea into
ammonia (NH3) in the urine.

 Plain X-ray & i.v. pyelography: to exclude stone or obstruction.


 Serum creatinine to avoid use of nephrotoxic drugs.

▌Management of UTI

1. Specific treatment: Antimicrobial drugs:

 In lower UTI (usually simple):

‒ The antimicrobial drug should be excreted in large amount in urine.


‒ Drugs used routinely: co-trimoxazole, amoxycillin or nitrofurantoin.
‒ Duration of treatment: 3-5 days.
‒ In some cases, simple UTI could be treated by single large dose of
fosfomycin (3g oral once) but with lower cure rates.

 In upper UTI (usually recurrent):

‒ The antimicrobial drug should be bactericidal of high tissue penetration e.g.


aminoglycosides, fluorinated quinolones or 3rd gen cephalosporins.
‒ Duration of treatment: 10-14 days followed by chemoprophylaxis for 15-30
days to prevent recurrence.
‒ Indications of chemoprophylaxis: recurrent upper UTI and UTI in pregnancy
and children.
‒ Drugs used routinely in chemo-prophylaxis: co-trimoxazole, amoxycillin or
nitrofurantoin.

409
2. Changing the urinary pH: Drug % Renal excretion
of unchanged drug
 Normal urine pH is 5.2-6.5. It is Ampicillin 75-92
possible, by the use of Amoxicillin 60-98
pharmacological agents, to produce Piperacillin 75-90
Ticarcillin 80-98
urinary pH values ranging from ~ 5.0
Aztreonam 65-95
to 8.5. Imipenem 5-40
Meropenem 62-83
 Alkalinization of the urine: Cephalexin 91-100
Ceftriaxone 65-95
Indications: Ciprofloxacin 30-50
 To enhance the activity of Levofloxacin 61-86
sulfonamides & aminoglycosides. Moxifloxacin 20
Gentamycin >90
 To prevent uric acid stones and
Topramycin >90
sulfonamides crystalluria.
Nitrofurantoin 27-56
 To relieve dysuria (burning Co-trimoxazole 50-75
micturition) in some cases of
bladder infection.
 E. coli is inhibited in alkaline medium.

Alkalinizing agents:
 Oral: sodium and potassium citrate salts: citrate is metabolized into
bicarbonate which is excreted in urine.
 Intravenous bicarbonate solution: contains 5% NaHCO3.

 Acidification of the urine:

Indications:
 To enhance the activity of nitrofurantoin, fosfomycin, and hexamine.
 Acidifying agents can lead to dangerous systemic acidosis in cases of renal
or hepatic impairment.

Acidifying agents:
 Oral: ascorbic acid > 2 g/d.
 Intravenous ammonium chloride (NH4Cl) solution.

3. Urinary antiseptics:

 Hexamine (Methenamine)
‒ In acidic urine, hexamine is hydrolyzed into ammonia and formaldehyde.
Formaldehyde is bactericidal and lacks bacterial resistance. Urine must be
acidified (pH below 5.5) to get this effect.
‒ Side effects: Chemical cystitis.

410
 Nitrofurantoin

‒ Nitrofurantoin causes bacterial DNA damage by an unclear mechanism


‒ It is used only as a urinary antiseptic in lower simple UTI against E. coli.
Other urinary tract Gram-negative bacteria are often resistant.
‒ It becomes more active in acidic urine.
‒ It is contraindicated in renal failure.
‒ Adverse effects include hemolytic anemia in patients with G-6-PD
deficiency, peripheral neuritis, and dark brown urine.

 Fosfomycin

‒ Fosfomycin inhibits bacterial cell wall synthesis.


‒ It is a broad spectrum antibiotic against many Gram positive and negative spp.
‒ It is used only as a urinary antiseptic in lower simple UTI.
‒ It becomes more active in acidic urine.

Causes of failure of treatment:

 Improper choice, dose, or duration of drug treatment.


 Bacterial resistance.
 Presence of complications e.g. renal calculi, ureteric stricture, etc.
 Renal failure.
 Immunocompromized patient: e.g. DM

Contraindicated drugs Recommended drugs


UTI with  Nitrofurantoin: absolutely contraindicated  Penicillin
Renal because it is nephrotoxic and needs  Most cephalosporins
Failure acidic urine to act while we cannot acidify
 Co-trimoxazole
urine in renal failure.
 K salts and acidifying agents: absolutely
contraindicated because they can cause
dangerous acidosis or hyperkalemia.
 Aminoglycosides.
UTI in  Sulfonamides → kernicterus.  Penicillin
Pregnancy  Fluorinated quinolones → arthropathy.  Most cephalosporins
and  Aminoglycosides → congenital deafness.
lactation  Tetracyclines → teratogenic and cause
yellow staining of teeth.
 Chloramphenicol → grey baby syndrome.

411
Part 4
4: Che
emotherapy of TB and Leprosy
y

Overview:

 Myccobacteriu culosis, o
um tuberc one of a
nummber of mycobacteria, can lead to o
serious infec ctions of the lungss, genito--
urin
nary tract, skeleton,
s and
a mening ges.
 Worldwide esstimations record 9 m million new
w
cases every year, and approxximately 2
million die of the
t disease each yea ar.
 Treaatment of TB (and other myc cobacteria))
pressents theraapeutic prob blems beca ause:
 Tubbercle bac cilli are either exxtracellularr
(meetabolicallyy activ
ve), in
ntracellularr
(meetabolicallyy inactive) or inact ive inside e
neccrotic case eous material. Howe ever, mostt
of tubercle ba acilli are intracellular with sloww
growwth rate. Resista ant strain ns occurr
natuurally to anny agent given as so le drug.
 Baccterial r
resistance e is common,
partticularly in patients non-adhere
n ent to the treatment
t protocol.
p
 Thee organism m grows slo owly and m may require e 6-24 monnths of treaatment.

Genera
al rules du
uring TB th
herapy:

 Nevver treat TB g: at least two


T by a single drug t of the first line d
drugs, are used to
prevvent emerg gence of reesistant st rains.
 Nevver add a single dru ug to a failling regim
men!
 Treaatment mu ust be conttinued for long period (6-24 moonths) to eeradicate bacilli.
b
 Pooor patient complian nce is the ccommones st cause of
o therapeuutic failure,, so it is
bettter to give drugs in a single daiily dose to
o enhance patient
p commpliance.
 Thee patient must
m be followed up for detection of adve erse effectts of drugs
s and/or
therrapeutic fa
ailure.

█ FIRS
ST LINE ANTI-TUBE
A ERCULOU
US DRUGS
S

 Isonniazide
 Rifa
ampicin (R
Rifampin)
 Ethambutol
 Pyrrazinamide
e
 Streeptomycin
n

412
1. Ison
niazid (IN
NH)

Mecha
anism of action
a

 INHH is an anaalog of pyridoxine (vittamin B6).


 INHH is a proddrug that is
s activated
d inside M. tuberculoosis into
active metab bolite whicch inhibits the synth hesis of mycolic
m
acid, an esse ential compponent of tthe mycob bacterial cell
c wall (b bactericidaal).
 Myycolic acid d is presen
nt only in M
M. tubercullosis, so IN
NH has no o activity on
o other
baccteria or attypical myc
cobacteriaa.
 Ressistance iss due to mutations
m in yme respon
n the enzy nsible for cconversion
n of INH
into
o the activee metabolite.
N.B.
macokinetics
Pharm
IN
NH exerts co ompetitive e
 Abssorption of INH is good d after oral o anntagonism with pyrid doxine
admministrationn. It penettrates mosst body fluuids fo
or enzyme aapotryptoph han
inclluding the CSF. leading to cliinical picturre of
 It a
accumulate es in case eated lesio
ons and can c pyyridoxine deeficiency
(neurotoxicityy).
atta
ack both extra- and intracellula
i ar bacilli.
 INHH is acetyylated in the liver. The rate of
aceetylation is genetically determin
ned (“rapid d acetylators” and “s low acetylators”).

Therap
peutic use
es
 Treatment off TB: INH is adminisstered in combination with oone or morre other
firstt-line drugss to minimize the devvelopmentt of resistance.
 For prophylax xis (close contacts),
c INH is use
ed alone.

Advers
se effects

I Inh ↓ metabolissm of othe


hibition of CYP450 (↓ er drugs, es
specially p
phenytoin).

N Neu
urotoxicityy:
‒ High levells of INH compete
c wwith pyrido
oxine at the enzyme pyridoxal kinase
leading to peripheraal neuropaathy.
‒ It is more common in slow accetylators.
‒ It could bee minimize
ed by co-ad
dministratiion of pyrid
doxine (vittamin B6).

patotoxiciity with jau


H Hep undice (3%%):
‒ It is due to
o accumulaation of toxxic metabo
olites in the liver.
‒ It is more common in rapid accetylators and in ind dividuals ovver 35 years.

H Hyp
persensitivity reactions with rrash and fe
ever (2%).

H Hem nemia in patients


molytic an p witth G-6PD deficiency
d

413
2. Rifa
ampin (Riifampicin
n)

Mecha
anism of action
a

 Rifa ampin sele ectively inh


hibits bactterial DNA--
dep pendent RNA po olymerase e enzyme e
lead ding to ↓ RNA
R synthhesis (huma an enzyme e
is nnot affected d).
 Rifa ampin is bactericida
b al for both h intra- and
d
extracellular M. M tubercu ulosis, and atypical mycobacte
m ria.
 It iss also effec
ctive againnst many G Gram-positive and Gram-nega
G ative bacte
eria.
 Res sistance develops
d rapidly
r if th
he drug is used alon
ne. It is duue to a change in
the polymerasse enzyme e.

Pharm
macokinetics
 Rifa
ampin is absorbed
a orally
o and can reach
h all body tissue an d fluids in
ncluding
the CSF, pleuural and asscetic fluidss.
 It c
can reach TB cavitie es, sputumm and pen acrophage killing intra- and
netrate ma
extracellular TB
T bacilli.
 It eenters ennterohepattic circullation and d induces s hepatic microsom mes to
deccrease the s of other d
e half-lives drugs.
 d through the bile and urine
It iss excreted e. Its metaabolites caause oran
nge-red
disccoloration of urine, stool,
s tearss, and swea
at; the patient should
d be warned.

Therap
peutic use
es

 Treaatment of TB in com
mbination w
with INH and
a pyraziinamide. R
Rifampin and
a INH
are the most effective
e antitubercu lous drugs
s.
 Alth
hough rifam
mpin has activity
a aga
ainst many
y Gram positive and negative bacteria
b
but should be
e used only TB to prev
y against T vent develo
opment of resistance
e.
 Treaatment of leprosy in combination of dap
psone and clofazemiine.
 Useed prophyylacticallyy for indiividuals exposed
e to meninggitis caus
sed by
Men ci or H. influ
ningococc uenzae.

Advers
se effects

‒ Heppatotoxicity (rare): abnormal liver enzy ymes, jaun


ndice, etc. especially
y in old
patiients and patients
p wiith chronic
c liver disea
ase.
‒ Mic
crosomal enzyme
e nduction le
in eading to decrease
d half-lives
h o
of other dru
ugs.
‒ Flu--like symp
ptoms: witth intermitttent therap
py.
‒ Red
d tears and urine: ha
armless, b
but can sta
ain contact lenses.
‒ T upset (na
GIT ausea and vomiting)
v a
and skin ra
ash.

414
3. Ethambutol

Mechanism of action

 Ethambutol inhibits arabinosyl transferase involved in cell wall biosynthesis.


 Ethambutol is specific bacteriostatic for M. tuberculosis. It is active against
intra- and extracellular TB bacilli.
 Relatively less toxic and resistant strains develop slowly.

Therapeutic uses

 Treatment of TB in combination with INH and rifampin to minimize resistance.


 Treatment of TB during pregnancy because it is the least toxic.

Adverse effects

‒ Peripheral neuropathy and visual disturbances (1-5%):


‒ Optic neuropathy manifested by field defects and red-green color
blindness. These effects are reversible on stopping the drug.
‒ There is no specific treatment other than stopping of the drug.

‒ Hyperuricemia and gout: due to ↓ uric acid excretion.


‒ GIT disturbance & hypersensitivity reactions.

4. Pyrazinamide

Mechanism of action

Pyrazinamide is taken up by the macrophages and is converted into the active


product by the acidic medium of the lysosomes inside the macrophages. It inhibits
mycobacterial cell wall functions.

Therapeutic uses: In combination with INH and rifampin to minimize resistance

Adverse effects

‒ Hepatotoxicity is the major side effect.


‒ Hyperuricemia and gout: due to ↓ uric acid excretion.

5. Streptomycin

 It is one of the aminoglycosides. It is bactericidal for extracellular bacilli but it is


ineffective against intracellular bacilli. Why?

 Therapeutic uses: in combination with INH & rifampin to treat resistant

415
pulmonary TB and renal TB (because 50-90% is excreted unchanged via the
kidney). It is given by IM injections.

 Adverse effects: nephrotoxicity, ototoxicity (8th nerve damage), and NM block.

█ SECOND-LINE ANTI-TUBERCULOUS DRUGS

 Para aminosalicylic acid


 Ethionamide
 Cycloserine
 Fluoroquinolones
 Capreomycin

1. Para aminosalicylic acid (PAS)


PAS is an analog of PABA; it works similar to sulfonamides but only against
mycobacteria.

2. Ethionamide
Ethionamide, like isoniazid, blocks the synthesis of mycolic acids. Resistance
develops rapidly. It commonly produces severe GI disturbances.

3. Other drugs: fluoroquinolones, cycloserine, etc.

█ REGIMEN OF TB THERAPY

 Patients with active TB:


‒ Initial phase (first 2-4 months): 4 drugs are used (RIPE): (Rifampin + INH +
Pyrazinamide + Ethmabutol). N.B. Recent guidelines now recommend
giving a 'fourth drug' such as Ethambutol routinely - previously this was only
added if drug-resistance was suspected.
‒ Continuation phase (next 4-6 months): at least 2 drugs are used (INH +
rifampin).

 Patients with latent TB (i.e. patients with +ve Tuberculin skin test and had
history of contact to a person proved to have TB): INH alone for 6 months or dual
Rifampicin + INH for 3 months.

 Patients with meningeal TB: are treated for a prolonged period (12-18 months)
with the addition of steroids.

416
 TB during pregnancy: the only anti-TB drug which is absolutely contraindicated
is streptomycin because of the high risk of congenital deafness. The other first
line anti-TB drugs are safe for use in pregnancy.

 TB with liver disease: INH, rifampin, and pyrazinamide are hepatotoxic but
because of their effectiveness, they should be used depending on monitoring of
liver function tests. In severe liver damage, only one drug can be used.

█ CHEMOTHERAPY OF LEPROSY

1. Dapson

 Structural analogue of PABA and chemically related to sulfonamides.


 Mechanism of action: similar to sulfonamides (bacteriostatic).
 Therapeutic uses: treatment of leprosy with rifampin for 2-5 years.
 Adverse effects:
‒ Nausea, vomiting, skin rash
‒ Hemolysis and methemoglobinemia,
‒ Exacerbation of skin lesion of lepromatous leprosy.

2. Clofazimine

 Mechanism of action: inhibit mycobacteria DNA synthesis (bactericidal).


 Therapeutic uses: in combination with dapson and rifampin to prevent
resistance.
 Adverse effects: abdominal pain, brown urine, atropine-like actions.

3. Rifampin: is the most active antilepromatous drug available.

417
Part 5: Antiviral Drugs

█ ANTI-HERPES VIRUS AGENTS

1. Acyclovir

 Acyclovir is a purine analog that inhibits the activity of viral DNA polymerase.
 It is active against herpes simplex virus (HSV) types I and II, and to a lesser
extent against Epstein–Barr virus, varicella-zoster virus, and cytomegalo
virus (CMV).
 Adverse effects: reversible nephrotoxicity and neurotoxicity

2. Gancyclovir

 Similar to acyclovir but highly active against CMV.


 Adverse effects: reversible neutropenia and thrombocytopenia

█ ANTI-INFLUENZA AGENTS

1. Amantadine and rimantadine

 Amantadine and rimantadine inhibit the uncoating and replication of the viral
RNA in infected cells.
 They are used to treat influenza A infections when administered within the first
48 hours of symptoms, and as prophylaxis during flu season.
 Adverse effects: mild CNS effects (insomnia, nervousness) and some GI
dysfunction. Patients with a history of seizures require close monitoring.

2. Ribavirin

 Ribavirin, a guanosine analog that appears to inhibit viral RNA polymerases; the
mechanism of action is not clear.
 It is used to treat respiratory syncytial virus (RSV) and influenza A and B.
 Adverse effects: Hemolytic anemia and teratogenic in pregnancy.

3. Zanamivir and Oseltamivir (Tamiflu)

 Zanamivir, (administered by inhalation), and oseltamivir, (administered orally), are


neuraminidase inhibitors.
 They are used for the treatment and prophylaxis of acute uncomplicated
influenza infection. The agents are effective against both influenza A and B.
 Abdominal pain and GI dysfunction are common with oseltamivir. Zanamivir
may cause bronchospasm.

418
█ ANT
TI-RETROV
VIRAL DR
RUGS

1. Rev
verse Tra
anscriptas
se Inhibittors (RTIs
s):
■ Nucleosid s (NRTI): zzidovudine
de analogs e, lamivudin
ne, tenofovvir
■ Non-Nucleoside an
nalogs (NN NRTI): efav
virenz, nevirapine

Adv
verse effeccts:
‒ A
All agents:: periphera
al neuropatthy and intteraction with
w CYP4550.
‒ Z
Zidoviodin
ne: myopatthy and an emia.

2. Pro
otease inh
hibitors: ritonavir,
r lo
opinavir, attazanavir

Adv
verse effeccts:
‒ A
All agents:: diabetes, hypertrigl yceridaem
mia and hyp
percholest erolaemia
‒ Ritonavir is
s the mostt potent inh
hibitor of CYP450
C kn
nown.

3. Fus
sion recep
ptor protein inhib
bitors: Enffuvirtide.
‒ It competees with the
e gp41 sub
bunit of the
e HIV-1 virral envelop
pe and prev
vents
ffusion to the cell membrane C D4 receptoors.

4. Inte
egrase inhibitors: Raltegraviir
‒ It is a new
w class of drugs
d that inhibit the
e viral enzyme integra
rase to pre
event
HIV replicaation and viral
v integra
ation into the
t host ce ell.

█ ANT
TI-HEPATIITIS VIRUS
S DRUGS

erferon-alpha and peginterf


1. Inte pha (Pegas
rferon-alp sys)

 Inteerferons arre group of natural ccytokines released


r by
b host cellls in response to
infe
ection. The
ey have se ave antivirral effects and regulate the
everal classses; all ha
immmune functtion.
 Gennetically engineered interferonn-α is used, in commbination w with ribavirin, for
bothh HBV and d HCV. Peginterferonn has long duration (injected on nce-week kly s.c.).

 Anttiviral mec
chanism:
‒ Interferon binds to cell membra ane recepttors to initiate a
sseries of re
eactions le
eading to in
nhibition of
o viral repliication.
‒ T
They prom mote apopttosis of vira
al-infected
d cells.

 Advverse effec
cts:
‒ Flu-like symptoms: muscle
m paiin, fever, and
a fatigue e.
‒ Bone marrrow depres ssion: neuttropenia.
‒ Neuropsyc chiatric effects: deprression, co onvulsions.

419
2. Lamivudine

 It is a nucleoside reverse transcriptase inhibitor (NRTI) used for HBV infections,


providing effective and rapid response in most patients, and slows progression
of liver fibrosis.

3. Sofosbuvir (Sovaldi®)

 It is a nucleotide analog used for treatment of HCV in combination with ribavirin


and interferon.
 It works by inhibition of viral RNA polymerase.

4. Grazoprevir/Elbasvir (Zepatier®)

 It is a recently approved combination for treatment of HCV (genotypes 1, 3, 4).


 Grazoprevir is a NS3/4A protease inhibitor. This protease enzyme enables the C
virus to survive and replicate in host cells. Elbasvir is a NS5A inhibitor. NS5A is a
protein needed by the virus for various stages of infection.
 The effectiveness and adverse effects of sofosbuvir, grazoprevir, and elbasvir are
under current investigation.

█ Management of chronic HCV infection

 Treatment with interferon-α alone gives 10-15% success rate in achieving long
term clearing of plasma hepatitis C RNA. A combination of interferon and
ribavirin gives 50% success rate.
 About 50% of successfully treated patients will relapse despite treatment.
 HCV genotype will give guidance to the length of treatment and response
rate:
‒ Those with genotype 2 and 3 can achieve SVR after 24 weeks as they have
better response to treatment.
‒ In genotype 1 and 4, therapy is continued for 48 weeks due to lower
response rate.
 The following also predict a good long term response to interferon:
‒ Younger age.
‒ Female gender.
‒ Absence of cirrhosis on liver biopsy.
‒ Non-black racial origin.
‒ Low hepatic iron.

 No HCV vaccine is currently available.

420
Part 6
6: Che
emotherapy of Fungal Infectio
ons

Fungal infection iss termed mycosis

▌Type
es of fung
gal infecttions:

 Muc
cocutaneo
ous (supe
erficial)
infe
ections:
‒ Derrmatophytees: cause in
nfection
of skin, hairr, and nails: e.g.
tine
ea capitiss (scalp), tinea
cruris (groinn), tinea pedis
(foo
ot), onycho
omycosis (nails).
‒ Yeaasts: causse infectio
ons of
mooist skin and mucousm
membranes: e.g. Cand ans causing oral, pha
dida albica aryngeal, vvaginal, & bladder
infe
ections.

 Sys ycoses: arre fungal in


stemic my nfections affecting
a internal org
gans. It oc
ccurs in
imm
munocomp
promized patients
p e.g
g. cryptoco
occosis, an
nd aspergiillosis (lung
g).

▌Class
sification
n of antifu
ungal dru
ugs:

A. Druggs for c 
Systemic oles: Fluconazole, Itraaconazole,
Azo
mucoc cutaneous
s drugs Voriconazole.
infectio
ons:  Gris
seofulvin
 Terb
binafine

Topical  Azooles: Ketocconazole, M Miconazole e,


drugs Clottrimazole, Tioconazo
T ole, etc.
 Nystatin
 Terbbinafine.
 Otheer drugs: Tolnaftate,
T Ciclopirox
x,
Nafttifine, Whitfield ointm
ment, Gentiian
viole
et, Castella
ani paint, T
Tincture iod
dine.

B. Druggs for sys


stemic  Azooles: Fluconazole, Itraaconazole,
infectio
ons: Voriconazole.
 Ampphotericin--B
 Fluc
cytosine
 Caspofungin

421
1. Azo
oles
[Ketoc
conazole, Miconazo
ole, Flucon
nazole, Itra
aconazole
e, Voricon azole]

Mecha
anism of ac
ction

 Azo
oles inhibitt fungal cy e P450 ne
ytochrome ecessary fo
or ergoste hesis, a
erol synth
major compon nent of fun
ngal cell m
membrane. This will alter
a memb
brane perm
meability
and
d disrupt itss function.
 The
ey are broaad spectrum fungista atic against many derm
matophytees and can
ndida.

Pharmacokinetic cs
 Abssorption of azoles from
m stomach h if affected
d by food and gastric HCl.
 Flucconazole caan reach th
he CSF with good con ncentration
ns. The othher drugs cannot.
 Flucconazole iss excreted in the urinee mostly unnchanged

Therap
peutic use
es

 Sup
perficial fu
ungal infec
ctions: [ke
etoconazo
ole – itraco
onazole – miconazo
ole]
‒ D
Dermatoph
hytes infecttion of the skin (tinea
a), hair, and
d nails (onnychomyco
osis):
‒ For skin infec
ction: treatmment continued for 2-4
2 weeks..
‒ For hair infecttion: treatm
ment continued for 6-8
6 weeks.
‒ For nail infecttion: treatm
ment contin
nued for 3-6 monthss.

‒ M
Mucocauta
aneous can
ndidiasis: o
oropharyngeal, vulvo
ovaginal, eetc.

 Sys
stemic fun
ngal infecttions: [itra
aconazole – fluconazole – vorriconazole
e]
‒ IItraconazoole (orally or
o IV) is the
e drug of choice
c
ffor systemiic blastommycosis.
‒ FFluconazo ole (orally or
o IV) is the
e drug of choice
c
ffor system
mic candid diasis, andd cryptoco occal
mmeningitiss (because e it the onlyy azole tha
at can
ccross to CS
SF with good concen ntration).
‒ VVoriconazole is th he drug of choicee for
iinvasive as sis of the lu
spergillos ung.
N.B. Itraco
onazole hass
placed ketocon-
largely rep
Advers
se effects azole in m
most uses.

‒ Heppatotoxicity and ↑ of serum tra


ansaminas
ses.
‒ Azo
oles inhibit hepatic CYP450 e enzymes (ffluconazole
e is the leaast among them).
‒ Kettoconazolee causes antianderrogenic efffects: gynnecomastiia and imp potence
e to ↓ gona
due adal steroid
d synthesiss.
‒ Vorriconazole
e causes trransient viisual distu
urbances.

422
2. Amphotericin-B

Mechanism of action
Amphotericin B is polyene macrolide that binds to ergosterol of fungal cell
membranes and forms “pores” that alter membrane stability and allow leakage of
cellular contents.

Pharmacokinetics

 Amphotericin B is polar compound that cannot be absorbed from the GIT or


cross the CSF. It should be administered IV or intrathecal.
 Half-life is 15 days. Dialysis is ineffective in case of toxicity.
 Because of significant toxicity, amphotericin B is available in liposomal form in
which the drug is enclosed in lipid microspheres “liposomes”. These lipid
microspheres bind preferentially to ergosterol in the fungal cell membrane with
lower affinity to mammalian cell membranes.

Therapeutic uses

Amphotericin B has the broadest spectrum of activity. It is used to treat severe


systemic fungal infections, including those caused by Candida albicans,
Histoplasma capsulatum, Cryptococcus neoformans, Coccidioides immitis,
Blastomyces dermatitidis, Aspergillus spp., and Sporothrix schenckii.

Adverse effects

‒ Amphotericin B is highly toxic drug. It causes nephrotoxicity in 80% of patients


which is dose-dependent and may be irreversible.
‒ Chills and fever in 50% of patients.
‒ Seizures and neurotoxicity
‒ Hypokalemia and thrombocytopenia

█ OTHER ANTIFUNGAL DRUGS

Flucytosine

 Flucytosine is actively transported into fungal cells and is converted to the uracil
form 5-fluorouracil (5-FU) which inhibits nucleic acid synthesis. Human cells
lack the ability to convert large amounts of flucytosine into 5-FU.
 It is often used in combination with other antifungal agents (because of rapid
development of resistance) to treat severe systemic fungal infections.
 Adverse effects: Flucytosine is relatively nontoxic; the major adverse effect is
depression of bone marrow at high doses and hair loss.

423
Griseofulvin

 Griseofulvin binds to microtubules and prevents spindle formation and mitosis


in fungi. It is fungistatic and requires long duration of therapy.
 The drug binds to keratin structures and accumulates in skin, hair, and nails.
 Griseofulvin is used orally for long-term therapy of dermatophyte infections of
the hair and nail.
 Adverse effects: hepatotoxicity (liver functions should be checked during
therapy), hypersensitivity reactions (skin rash), and CNS effects.

Terbinafine

 Terbenafine inhibits the fungal enzyme squalene epoxidase. This leads to the
accumulation of the sterol squalene, which is toxic to the organism.
 Like griseofulvin, it accumulates in keratin structures and used orally or topically
for treatment of dermatophyte infections of the hair and nail.

Nystain

 Nystatin is polyene macrolide very similar in kinetics and mechanism to


amphotericin B.
 It is too toxic for parenteral administration and is used only topically.
 It is active mainly against Candida, and is used topically for oralpharyngeal and
vaginal candidiasis.

Caspofungin

 It is large cyclic peptide that disrupts the fungal cell wall resulting in cell death.
 Caspofungin is used by i.v. route for salvage therapy in patients with severe
invasive aspergillosis or esophageal candidiasis who failed to respond to
amphotericin B (second line drug).

Ciclopirox

 Broad-spectrum antifungal effective against dermatophytes and yeasts. The


mechanism is unclear.
 It is used topically for skin and nail infections.

424
Part 7
7: Anttiamoeb
bic Drug
gs

 Thee causativve organis sm for a amoebiasiss is Entamoeba h histolytica. Initial


infe
ection occuurs by ingeestion of th
he cyst form. The infectious cyysts pass into the
coloon, where they deve elop into trrophozoitees. These motile org
ganisms ad dhere to
coloonic epithhelial cells
s. Here, the trophozoites fe eed, multiiply, encyyst and
eveentually pa
ass out in th
he feces “a asymptommatic carrier”.
 The
e motile organism inv
vades the ccolonic muucosa caus
sing “amooebic dyseentery”.
The
ey may also spread to
t the liver causing acute
a “amo
oebic liverr abscess””.

▌Class
sification
n of antia
amoebic d
drugs:

A. Lum
minal amoe ebicidal drugs: thesse agents  Diloxanid
de
destroyy the troph
hozoites off E. histolyttica that  Iodoquinool
eventuaally form in
nto cysts in
n the intesstine.  Paromom mycin

B. Tisssue amoeb bicidal druugs: they ddestroy the e  Emetine aand


invadinng trophoz zoite in the
e tissues (e
e.g. liver) but
b dehydroe emetine
are inefffective ag
gainst the trophozoitees in the  Chloroquuine: active e only
intestin
nal lumen. Their use now is ve ery limited d. against livver infection.

C. Mixe
ed tissue and lumin
nal amoeb
bicides: Meetronidazoole, tinidaz
zole,
orn
nidazole.

425
1. Mixed amebicides: Metronidazole

Structure and mechanism of action

 Metronidazole is 5-nitroimidazole compound. Anaerobic protozoa & bacteria lack


mitochondria; instead, they have ferredoxin oxidoreductase enzyme in the
cytoplasm to generate ATP.
 This enzyme system can transfer electron to the 5-nitro groups of metronidazole
converting it into cytotoxic product. This product causes DNA damage and
inhibits DNA repair.
 Metronidazole is active against anaerobic organisms including anaerobic bacteria:
(e.g. Bacteroids, C. difficile), E. histolytica, G. lamblia, T. vaginalis, B. coli.

Therapeutic uses

 Amebiasis: Metronidazole is the most effective agent available for the treatment
of all forms of amoebiasis except asymptomatic person that excrete cysts.
Metronidazole kills trophozoites but not cysts.
 Urogenital trichomoniasis: 250 mg t.d.s. for 7 days is the treatment of choice.
The other partner should be treated simultaneously.
 Giardiasis: 250 mg t.d.s. for 5 days.
 Balantidiasis: 750 mg t.d.s. for 5 days
 Severe anaerobic infections: e.g. puerperal sepsis, peritonitis, acute ulcerative
gingivitis, etc.

Adverse effects

‒ GIT: nausea, vomiting & metallic taste.


‒ CNS: insomnia, headache, vertigo, parasthesia, ataxia, & seizures.
‒ Blood: bone marrow depression, leukopenia &
thrombocytopenia. N.B. Disulfiram is a drug
used for treatment of chronic
‒ Disulfiram-like reaction: metronidazole
alcoholism. It blocks metab-
causes accumulation of acetaldehyde if alcohol
olism of acetaldehyde by the
is consumed leading to nausea & vomiting. enzyme aldhyde dehydro-
‒ Dark brown urine. genase leading to accumula-
tion of acetaldehyde → nau-
sea, vomiting, flushing, etc.
2. Luminal amebicidal drugs:

 Diloxanide: Is active against both trophozoite and cyst forms in the intestinal
lumen but not in the intestinal wall or extraintestinal tissues. The mechanism is
unknown.

426
 Iodo
oquqinol: it is iod quinolone derivative
dinated q e. Is activve agains
st both
trop
phozoite an nd cyst forrms in the intestinal lumen butt not in thee intestinall wall or
extrraintestinal tissues. The
T mecha anism is un nknown.
Beccause it co ontains iodine, it ca
an cause dermatitis
s and perrsistent diarrhea
(iod
dine intolerance).

 Parromomycin: it is a poorly-abso
p orbable am
minoglycos
side antibi otic. It is used
u as
alte ol and diloxxanide. The antiamoebic mechhanism is unclear
ernative to iodoquino u
but may be dued to alteeration of tthe cell me
embrane permeabilit
p ty and leakage of
cell contents.

Uses o
of luminal drugs
 Theey are used for treatme
ent of asym
mptomatic or
o mild inte
estinal infecction.
 Theey can also
o be added d to metroonidazole in
n acute ammoebic dyssentery as well as
hep
patic absce
ess to erad
dicate cystts in the lum
men which
h may causse relapse.

3. Tiss
sue ameb
bicidal drrugs:

 Emetine and dehydroe


emetine: E
Emetine is a
nt alkaloid
plan d. Dehydro oemetine iis a potennt
deriivative. All have significant toxiicity.

 Chloroquine:: it is a synthetic drug (se


ee
trea
atment of malaria).
m

 Both drugs can


c sed as altternative to
be us
mettronidazole
e in the treatment o of amoebic
live
er abscesss or extraintestinal a
amoebiasis
s,
howwever, their use now is very lim
mited.

427
Part 8
8: Anttimalarial Drugs
s

█ Life cycle of malaria and


a sites
s of drug actions

A. Sexual cycle in the fema


ale mosqu
uito.

e in man an
B. Asexual cycle nd consistts of:

 S e stage: sp
Sporozoite porozoites injected by
y the mosquito rapidlyy enter the liver.

 E
Exo-erythrrocytic (liv
ver) stage
es:
PPre-exo-errythrocytic stage (pri mary liver stage):
SSporozoitees in the liv
ver underg
go multipliccation to form tissue
e schizonnt which
eeventually ruptures and
a release es merozo oites to infect RBCs. During this stage
tthe patientt is asympttomatic.

S
Secondaryy exo-erythhrocytic sta age (persisstent liver stage):
s
S
Sporozoite
e of P. vivaax or P. ovvale form dormant
d hypnozoite es in the liv
ver that
c
can persistt and reacttivated afte
er a latent period cau using relap
pse of mala aria.

 EErythrocyttic stage:: merozoittes liberated from ruptured


r l iver schizont will
invade RB BCs to forrm blood schizont. This bloo od schizo nt will eventually
rrupture to liberate merozoites
m and toxic
c products s causing fever and clinical
rd
mmanifestations of maalaria (everyy 3 day in
n tertiary malaria).
m

 G
Gametocyytes stagee: some mmerozoites develop in
nto gamettocytes wh
hich will
b
be sucked by the mo
osquito to c
complete the
t sexual cycle.

428
▌Type
es of trea
atment:

I. Chem
moprophy
ylaxis: (Killling the pa
arasite before multiiplication inside RB
BCs)

 Cau hylaxis: killing the pa


usal proph arasite in th
he liver:
Proguanil
P a
and primaq
quine.

 Clin
nical prophylaxis: killing the pa
arasite as soon as th
hey reach tthe RBCs:
Proguanil
P a
and chloro
oquine.

II. Suppressive or
o clinical cure: (kill ing the pa
arasite in the
t RBCs))

 Chlooroquine: for
f chloroqquine-senssitive malaria.
 Quinine and mefloquine
m e: for chloro
oquine-res
sistant malaria.
 Arte
emisinin an
nd its analo
ogs: for ch
hloroquine--resistant malaria.
m
 Sulffonamidess and pyrim
methamine : Fansidar (sulfadoxine + pyrim methamine)).

dical cure: (clearing the dorman


III. Rad nt hypnozooite from the liver to prevent re
elapse)
Primaquine
P e + chloroq
quine

IV. Pre
evention off transmis
ssion: (kill ing the ga
ametocytees)
Proguanil,
P primaquin
ne, and pyrrimethaminne.

Chloro
oquine

Mecha
anism of action
a
 Inside RBCss, the malarial
m pa
arasite digest
hem
moglobin in n a vacuole inside th
he parasite
e cell
to acquire amino acids e essential for
mulltiplication..
 Chlo
oroquine enters the
e digestio
on vacuole
e by
simple diffussion. The acidic ppH inside the
vacuole make es chloroq
quine ionizzed and, thus,
t
can
nnot diffuuse outsid de the vacuole and
bec
comes trappped inside
e it.
 Inside the vacuole,
v th
he ionized
d chloroq
quine
reaccts with thet digesttive produ
ucts of he
eme
mollecule and d forms a highly to oxic complex
thatt kills the parasite.
p
 Chlo
oroquine is a blood nticide forr Plasmodium vivax,, P. ovale, and P.
d schizon
mallariae but not
n for P. falciparum
f which is usually
u resiistant.

429
Pharmacokinetics
 Absorption from the GIT is good and complete.
 It binds to melanin-rich tissue e.g. skin, eye, etc.
 Chloroquine has very high Vd (100-1000L/Kg) and slow rate of elimination.

Therapeutic uses

 Malaria (except P. falciparum):


‒ For treatment of acute attack: 1g IV followed by 500 mg after 6, 24, 48h.
‒ For chemoprophylaxis: 500 mg once/week orally.
 As a 2nd line for treatment of amoebic liver abscess: 500 mg daily for 14 days.
 As a 2nd line (after metronidazole) for treatment of giardiasis.
 Treatment of rheumatoid arthritis and SLE: 500 mg/day then 250 mg/day.

Adverse effects

‒ Long term administration:


‒ Skin: pruritus (common), dermatitis, bleaching of hair, and alopecia.
‒ Visual disturbances: corneal deposits, optic atrophy, and retinopathy.
‒ Rapid i.v. injection causes hypotension, ECG changes, and cardiac arrest.
Parenteral administration is better avoided or should be infused slowly.
‒ Hemolysis in G-6PD deficient persons.

Quinine (and Quinidine)

Structure and mechanism of action

 Both quinine and quinidine are derivatives from the park of cinchona tree.
Quinidine is the D-isomer of quinine.
 Quinine is highly active blood schizonticide against the four species of human
malaria parasites, but it is primarily used to treat chloroquine-resistant P.
falciparum, often in combination with doxycycline.
 The exact antimalarial mechanism is unknown.

Therapeutic uses: treatment of chloroquine-resistant P. falciparum.

Adverse effects (more common with i.v. administration)

‒ Cinchonism: tinnitus, ringing in ears, blurred vision, headache, and vomiting.


‒ CVS: hypotension, syncope, cardiac arrhythmias and ↑ QT interval.
‒ Hypoglycemia is important and may be fatal side effect with i.v. administration.
It is due to stimulation of insulin release).

430
‒ ackwater fever (rarre): massivve hemoly
Bla ysis with fever, hemmoglobinure ea, and
darrk urine. Th
he pathoge enesis is u nclear.
‒ Hem molysis inn G-6PD de eficient pe rsons. N.B.
N
‒ Hyppersensitivvity reactions. Although
A qu inine cause
es
uterine conttractions but
b the
WHO
W guidel ines consid
der
Artem
misinin and its ana
alogs pregnancy
p iis NOT a
contraindica uinine.
ation of qu
Structu
ure and mechanism
m m of action
n

 Arteemisinin iss the active compoonent of a Chinese


e
herb
b. The compound contains unusual peroxide e
dge. Cleavvage of th
brid his peroxidde bridge inside the
e
dige
estion vac cuole libeerates oxyygen free e radicals
s
which are leth
hal to the parasite.
p
 Arte
emisinin and
a its anaalogs are very rapidly actingg
bloo
od schizo onticides against all huma an malaria
a
para
asites. The
ey have no o effect on hepatic stages.

Pharm
macokinetics

 Arteemisinin is insoluble and can b e used only orally.


 Arteesunate and
a arteme ether are semisynth
hetic analo ogs with im
mproved solubility
to b
be suitable
e for parenteral adm inistration..

Therap
peutic use
es

 Com
mbination of artemisinin and otther antimalarials is now
n the sttandard tre
eatment
of fa m malaria in nearly alll endemic areas.
alciparum
 IV a
artesunatee is now re
ecommend ded by thee WHO in preference
p to IV quin
nine due
to fe
ewer side effects.

Adverse effects: GI
G disturba
ances and,, rarely, allergic reacttions and hhemolysis..

Mefloq
quine

Structu
ure and mechanism
m m of action
n

 Meffloquine is structurally related tto quinine.


 Stroong blood schizonticide again nst P. falcip
parum by unknown
u mechanism.
 It is given onlyy orally be
ecause parrenteral addministratio
on causes severe irritation.

Therap es: treatme


peutic use ent of chlorroquine-re
esistant falc
ciparum m
malaria.

se effects:: neurotoxicity: head


Advers dache, vertigo, psych
hosis, and cconfusion..

431
Primaquine

Mechanism of action

Primaquine is a hepatic schizonticide (for P. vivax and P. ovale) and gametocide


(in all 4 types of malaria), through an unknown mechanism.

Therapeutic uses

 Radical cure: after successful treatment, it is given orally for 15 days to kill the
dormant hypnozoites in the liver and prevent relapse.
 Causal prophylaxis: but prolonged course of primaquine should be avoided.
 Prevention of transmission: by killing the gametocytes in all 4 types of malaria.

Adverse effects

‒ Methemoglobinemia nearly in all patients.


‒ Severe hemolysis in patients with G-6PD deficiency.
‒ GIT upset, visual disturbance, and leucopenia.

Antifolate drugs
(Pyrimethamine, Proguanil, Fansidar)

Mechanism of action: ↓ folic acid synthesis necessary for DNA synthesis.

Therapeutic uses

 Fansidar is a combination of sulfadoxine + pyrimethamine. It produces


sequential block of folic acid synthesis. It is used to treat the acute attack of
chloroquine-resistant malaria in combination with quinine or artemisinin.
 Proguanil is used for prophylaxis and for prevention of transmission.
 Pyrimethamine is used for clinical cure and for prevention of transmission.

432
Part 9: Anthelmintic Drugs

Helminths can be divided into three main groups:

Nematodes (round worms) Trematodes (flukes) Cestodes (tape worms)

Intestinal nematodes: Taenia saginata


Schistosomes
Ascaris lumbricoides Taenia solium
Ankylostoma duodenale Intestinal flukes: Diphyllobothrium latum
Enterobius vermicularis Heterophyes heterophyes Hymenolepis nana.
Trichuris trichiura Metagonimus yokogawia Hydatid tape worm.
Strongyloides stercoralis Fasciolopsis buski
Trichostrongylus
Liver flukes:
Tissue nematodes: Fasciola hepatica
Filaria.
Lung flukes:
Guinea worm.
Paragonimus westermani
Cutaneous and visceral
Larva migrans.

█ DRUGS EFFECTIVE AGAINST NEMATODES

1. Benzimidazoles
[Albendazole – Mebendazole – Thiabendazole]

Mechanism of action

These agents inhibit microtubule synthesis and glucose uptake by the worms.
The two effects lead to ↓ ATP production and paralysis of the worm.

Therapeutic uses

Benzimidazoles are broad-spectrum anthelmintic drugs.


 For intestinal nematodes: (400 mg once daily on an empty stomach for 2-3 days).
 For tissue nematodes: (400 mg twice daily with a fatty meal for 10 days).
 For hydatid disease: (400 mg twice daily for one month).
 Neurocysticercosis: albendazole is the drug of choice (400 mg twice daily with a
fatty meal for 21 days). It is given with corticosteroids to minimize the immune
reaction caused by the dying organisms.

Adverse effects: usually mild and mainly GIT upsets.

433
2. Pyrantel pamoate

 It causes depolarizing neuromuscular blockade and inhibits ChE enzyme in the


parasite → paralysis of the worm.
 It is used for intraluminal round worms.
 Adverse effects: very few.

3. Diethylcarbamazine (Hetrazan)

 It causes paralysis of microfilaria and alters their surface structure, making them
more susceptible to destruction by host defense mechanisms. The mechanism is
unknown.
 Therapeutic uses: treatment of filariasis (Wucheraria bancrofti and Loa loa).
 Adverse effects: sudden death of the microfilaria can produce severe allergic
reactions e.g. fever, leukocytosis, eosinophilia, edema, rashes, tachycardia and
headache (corticosteroids may be needed to suppress these allergic reactions).

█ DRUGS EFFECTIVE AGAINST TREMATODES AND CESTODES

1. Praziquantel

Mechanism of action
It increases cell membrane permeability of Ca2+. This leads to rapid and prolonged
muscle contraction and paralysis of the worm.

Therapeutic uses
 Treatment of schistosomiasis: it is active against adult worm and all immature
forms in all species (40 mg/kg single dose orally).
 Other flukes: e.g. H. heterophyes, Fasciolopsis buski, Paragonimus westermani.
 Cestodes: T. saginata, T. solium, D. latum, and H. nana.

Adverse effects
 Nausea, vomiting, drowsiness, arthralgia, myalgia and low grade fever.
 Mild elevation of liver enzymes.

Contraindications and cautions:


Praziquantel is contraindicated in ocular cysticercosis, because parasite
destruction in the eye may cause immune reaction and irreversible damage. Some
workers also caution against use of the drug in neurocysticercosis for the same
reason.

434
2. Bith
hinol

 It in
nhibits the parasite re
espiration.
 It iss an altern
native to triclabenda
t azole for Fasciola
F hepatica
h ((sheep live
er fluke
infe
ection) and as an alte
ernative to praziquantel for pulm
monary pa
aragonimiiasis.

3. Niclosamide
e

 It is a second line drug for


f most ta
ape worm (cestodes)) infectionss.
 It aappears too act by inhibition of oxida
ative phos
sphorylatio on and ↓ energy
prod duction.
 It is given as 2g
2 once on n an emptyy stomach.
 Advverse effeccts are minimal becauuse the dru
ug is not absorbed.
a

435
436
Review Questions
1. Mention the antibacterial mechanism of action and the major adverse effects of
each of the following drugs:
 Penicillin
 Clarithromycin
 Aminoglycosides
 Tetracyclines
 Vancomycin
 Isoniazid
 Ethambutol

2. Mention the antiviral mechanism of action and the major adverse effects of each
of the following drugs:
 Interferon-alpha
 Acyclovir
 Amantadine

3. Mention the antifungal mechanism of action and the major adverse effects of
each of the following drugs:
 Itraconazole
 Amphotricin B

4. Mention the antiprorozoal mechanism of action and the major adverse effects of
each of the following drugs:
 Metronidazole
 Chloroquine
 Artemisinin

5. Give a short account on the drug management and drug(s) of best choice in the
following infections:
 Bacterial meningitis
 Community-acquired pneumonia
 Bites (human or animal)
 Typhoid fever
 Pseudomembranous colitis
 Gonorrhea

6. Discuss drug treatment of hepatitis C virus. Mention the mechanism of action


and major adverse effects of interferon-alpha.
7. Discuss drug treatment of amebic liver abscess. Mention the mechanism of each
drug.
8. Discuss drug treatment of chloroquine-resistant P. falciparum malaria. Mention
the mechanism and major adverse effects of each drug.

437
Of each of the following questions,
select ONE BEST answer: 6. Amoxicillin is inferior to ampicillin
for the treatment of the following
1. The penicillin G preparation with infection
the longest duration of action is A. Typhoid
A. Benzathine penicillin B. Tonsilitis
B. Sodium penicillin C. Shigella enteritis
C. Potassium penicillin D. Subacute bacterial endocarditis
D. Procaine penicillin E. Gonorrhoea
E. Pipracillin
7. Which one of the following
2. If a patient gives history of statements about ampicillin is false?
urticaria, itching and swelling of lips A. Its activity is enhanced by sulbactam
following injection of penicillin G, then B. It causes maculopapular rashes
A. He will develop milder reaction C. It is the drug of choice for Listeria
whenever penicillin is injected monocytogenes infection
B. He can be given ampicillin safely D. It eradicates most strains of MRSA
C. He can be given cephalosporins safely E. Pseudomembranous colitis may occur
D. He can be given oral phenoxymethyl with its use
penicillin safely
E. All natural and semisynthetic 8. The mechanism of antibacterial
penicillins are contraindicated for him action of azithromycin involves
A. Binding to a component of the 50S
3. The most important reason for ribosomal subunit
highly restricted use of penicillin G B. Inhibition of translocase activity
injections in present day therapeutics C. Blockade of binding of aminoacyl –
is its tRNA to bacterial ribosomes
A. Narrow spectrum of activity D. Selective inhibition of ribosomal
B. Potential to cause hypersensitivity peptidyl transferases
reaction E. Inhibition of DNA–dependent RNA
C. Short duration of action polymerase
D. Neurotoxicity
E. Nephrotoxicity 9. In the empiric treatment of severe
bacterial infections of unidentified
4. Cefotaxime act by the following etiology, this drug, often used in
mechanism: combination with an aminoglycoside,
A. Inhibition of bacterial protein synthesis provides coverage against many
B. Inhibition of bacterial cell wall staphylococci
synthesis A. Amoxicillin
C. Inhibition of bacterial nucleic acid B. Clavulanic acid
synthesis C. Erythromycin
D. Inhibition of bacterial folic acid D. Nafcillin
synthesis E. Tetracycline
E. Inhibition of bacterial cell division
10. C. difficile colitis is more common
5. Benzathine penicillin injected once complication with the use of:
every 4 weeks for 8 years or more is A. Cephalosporins
the drug of choice for B. Vancomycin
A. Agranulocytosis patients C. Co-trimoxazole
B. Prophylaxis of bacterial endocarditis in D. Meropenem
patients with valvular defects E. Flucoloxacillin
C. Prophylaxis of rheumatic fever
D. Treatment of anthrax 11. Beta – lactamase production by
E. Treatment of sinusitis strains of Haemophilus influenzae,

438
Moraxella catarrhalis, and Neissera 16. Methicillin resistant staphylococci
gonorrhoeae confers resistance do not respond to β-lactam antibiotics
against penicillin G. which one of the because
following antibiotics is most likely to be A. They produce a β-lactamase which
effective against all strains of each of destroys methicillin and related drugs
the above organisms? B. They elaborate an amidase which
A. Ampicillin destroys methicillin and related drugs
B. Ceftriaxone C. They have acquired a penicillin binding
C. Clindamycin protein which has low affinity for β-
D. Erythromycin lactam antibiotics
E. Piperacillin D. They are less permeable to β-lactam
antibiotics
12. A 19-year-old woman with
recurrent sinusitis has been treated 17. Indicate the sulfonamide whose
with different antibiotics on several sodium salt yields a nearly neutral
occasions. During the course of one solution which is suitable for topical
such treatment she developed a severe use in the eye
diarrhea and was hospitalized. A. Sulfadiazine
Sigmoidoscopy revealed colitis, and B. Sulfacetamide
pseudomembranes, were confirmed C. Sulfamerazine
histologically. Which of the following D. Sulfamethizole
drugs, administered orally, is most
likely to be effective in the treatment of 18. Which of the following is not true of
colitis due to C difficile? sulfonamides?
A. Ampicillin A. They are primarily metabolized by
B. Azithromycin acetylation
C. Clindamycin B. They are more likely to produce
D. Metonidazole crystalluria in alkaline urine in which
E. Tetracycline they are less soluble
C. They may exert bactericidal action in
13. The drug of choice for Lyme the urinary tract
disease is: D. Used alone, they have become
A. Doxycycline therapeutically unreliable for serious
B. Sulfonamides infections
C. Penicillin
D. Erythromycin 19. Clavulanic acid is combined with
E. Topramycin amoxicillin because
A. It kills bacteria that are not killed by
14. The drug of choice for anaerobic amoxicillin
infections is: B. It reduces renal clearance of
A. Tetracycline amoxicillin
B. Sulfonamides C. It counteracts the adverse effects of
C. Penicillin amoxicillin
D. Erythromycin D. It inhibits beta lactamases that destroy
E. Metronidazole amoxicillin

15. The drug of choice for typhoid 20. Which toxic effect of
fever is: aminoglycoside antibiotics is most
A. Tetracycline irreversible in nature?
B. Sulfonamides A. Optic neuropathy
C. Penicillin G B. Ototoxicity
D. Ciprofloxacin C. Hepatotoxicity
E. Metronidazole D. Muscle weakness
E. Kidney damage

439
21. Highest incidence of antibiotic C. Oseltamivir
associated pseudo membranous D. Ribavirin
enterocolitis has been noted with the E. Ritonavir
use of
A. Ampicillin 27. The primary mechanism of
B. Chloramphenicol antibacterial action of penicillin
C. Vancomycin involves inhibition of:
D. Clindamycin A. Beta-lactamases
E. Gentamycin B. Cell membrane synthesis
C. N-acetylmuramic acid synthesis
22. Which antibiotic class can cause D. Peptidoglycan cross-linking
teeth staining and dental enamel E. Transglycosylation
hypoplasia if given to small children?
A. Fluoroquinolones 28. Fluid expressed from the penile
B. Cephalosporins chancre of a patient revealed to be
C. Tetracyclines infected with Treponema pallidum, the
D. Aminoglycosides best course of action would be to:
E. Macrolides A. Administer a single oral dose of
fosfomycin
23. Antiviral agents that are active B. Administer a single oral dose of
against cytomegalovirus (CMV) include gentamycin
which of the following? C. Inject intramuscular benzathine
A. Ganciclovir penicillin G
B. Acyclovir D. Treat with oral tetracycline for 7 d
C. Amantadine E. Treat with vancomycin
D. Oseltamivir
E. Ribavirin 29. Which of the following statements
about beta-lactam antibiotics is false?
24. Which one of the following drugs is A. Cephalexin and other first-generation
least likely to be effective in the cephalosporins do not cross the
treatment of esophageal candidiasis, it blood-brain barrier
is used by the oral route? B. Ceftriaxone and nafcillin are both
A. Amphotericin B eliminated mainly via biliary secretion
B. Clotrimazole C. Instability of penicillins in gastric acid
C. Fluconazole can limit their oral absorption
D. Griseofulvin D. Renal tubular reabsorption of
E. Ketoconazole amoxicillin is inhibited by probenecid
E. Ticarcillin has activity against several
25. Which one of the following drugs is gram negative rods
most appropriate for oral use in vaginal
candidiasis? 30. A patient needs antibiotic
A. Clotrimazole treatment for native valve, culture
B. Griseofluvin positive infective enterococcal
C. Fluconazole endocarditis. His medical history
D. Flucytosine includes a severe anaphylactic reaction
E. Nystatin to penicillin G during the last year. The
best approach would be treatment with
26. The antiviral actions of this drug A. Amoxicillin-clavulanate
include inhibition of both RNA and DNA B. Aztreonam
synthesis. The drug is used for the C. Ceftriaxone
treatment of severe respiratory D. Ticarcillin
syncytial virus infections in neonates. E. Vancomycin
A. Amantadine
B. Amprenavir

440
31. If ampicillin and piperacillin are B. Intravenous third-generation
used in combination in the treatment of cephalosporin
infections resulting from Pseudomonas C. Oral amoxicillin
aeruginosa, antagonism may occur. D. Oral ciprofloxacin
The most likely explanation is that E. Oral neomycin
A. Ampicillin is bacteriostatic
B. Ampicillin induces beta-lactamase 36. Clarithromycin and erythromycin
production have very similar spectra of
C. Autolytic enzymes are inhibited by antimicrobial activity. The major
piperacillin advantage of clarithromycin is that it
D. Piperacillin blocks the attachment of A. Does not inhibit hepatic drug-
ampicillin to penicillin-binding proteins metabolizing enzymes
E. The 2 drugs form an insoluble B. Eradicates mycoplasmal infections in
complex a single dose
C. Has greater activity against H pylori
32. Which statement about D. Is active against methicillin-resistant
vancomycin is accurate? strains of staphylococci
A. Active against methicillin-resistant E. Is active against strains of
staphylococci streptococci that are resistant to
B. Bacteriostatic erythromycin
C. Binds to PBPs
D. Hepatic metabolism 37. The primary mechanism of
E. Oral bioavailability resistance of gram-positive organisms
to macrolide antibiotics including
33. A 52 years old patient with signs erythromycin is
and symptoms suggestive of typical A. Changes in the 30S ribosomal subunit
bacterial meningitis. Treatment of this B. Decreased drug permeability of the
patient should be initiated immediately cytoplasmic membrane
with intravenous administration of C. Formation of drug-inactivating
A. Amoxicillin acetyltransferases
B. Cephalexin D. Formation of esterases that hydrolyze
C. Benzylpenicillin G the lactone ring
D. Nafcillin E. Methylation of binding sites on the
E. Piperacillin 50S ribosomal subunit

34. The mechanism of antibacterial 38. A suitable treatment for


action of azithromycin involves: community-acquired pneumonia with
A. Antagonism of bacterial translocase little risk of drug interactions can be
activity achieved with
B. Binding to a component of the 50S A. Azithromycin
ribosomal subunit B. Clindamycin
C. Inhibition of DNA-dependent RNA C. Doxycycline
polymerase D. Erythromycin
D. Interference with binding of E. Vancomycin
aminoacyl-tRNA to bacterial
ribosomes 39. Regarding the toxicity of
E. Selective inhibition of ribosomal aminoglycosides which statement is
peptidyl transferases accurate?
A. Gentamicin and tobramycin rarely
35. If a patient had been scheduled for cause renal damage
elective colonic surgery, optimal B. Ototoxicity includes vestibular
prophylaxis against infection would be dysfunction, which may be irreversible
achieved by the use of C. Ototoxicity is reduced if loop diuretics
A. Intravenous cefoxitin are used to facilitate the renal

441
excretion of aminoglycoside 44. Silver sulfadiazine is clinically used
antibiotics for:
D. Reduced blood creatinine is an early A. Treatment of Rocky Mountain spotted
sign of aminoglycoside nephrotoxicity fever
E. Skin reactions are very rare following B. To prevent infections of skin burns
topical use of neomycin C. Treatment of typhoid fever
D. Treatment of Legionella pneumonia
40. Your 23-year-old female patient is E. Treatment of lead toxicity
pregnant and has gonorrhea. The
medical history includes anaphylaxis 45. Which statement about the
following exposure to amoxicillin. The fluoroquinolones is accurate?
most appropriate drug to use is A. Antacids increase their oral
A. Azithromycin bioavailability
B. Cefixime B. Contraindicated in patients with
C. Ceftriaxone hepatic dysfunction
D. Ciprofloxacin C. Fluoroquinolones are drugs of choice
E. Doxycycline in a 6-year-old child with a urinary
tract
41. In a patient suffering from D. Gonococcal resistance to
pseudomembranous colitis due to C fluoroquinolones may involve changes
difficile with established in DNA gyrase
hypersensitivity to metronidazole the E. Modification of dosage is required in
most likely drug to be of clinical value patients renal impairment.
is
A. Amoxicillin 46. Which adverse effect is most
B. Chloramphenicol common with sulfonamides?
C. Doxycycline A. Stevens Johnson syndrome
D. Levofloxacin B. Hematuria
E. Vancomycin C. Kernicterus in the newborn
D. Neurologic dysfunction
42. Trimethoprim-sulfamethoxazole is E. Skin rash
established to be effective against
which of the following opportunistic 47. Which statement about
infections in the AIDS patient? ciprofloxacin is accurate?
A. Cryptococcal meningitis A. Antagonism occurs if used with
B. Herpes simplex dihydrofolate reductase inhibitors
C. Oral candidiasis B. Ciprofloxacin is active against MRSA
D. Toxoplasmosis strains of staphylococci
E. Tuberculosis C. Most “first-time” urinary tract
infections are resistant to ciprofloxacin
43. A 65-year-old woman has returned D. Organisms that commonly cause ear
from a vacation abroad suffering from infections are highly resistant
traveler’s diarrhea, and her problem E. Tendinitis may occur during treatment
has not responded to antidiarrheal
drugs. A pathogenic gram-negative 48. Supplementary folinic acid may
bacillus is suspected. Which drug is prevent anemia in folate-deficient
most likely to be effective in the persons who use this drug
treatment of this patient? A. Ciprofloxacin
A. Ampicillin B. Levofloxacin
B. Ciprofloxacin C. Linezolid
C. Sulfadiazine D. Clarithromycin
D. Trimethoprim E. Trimethoprim
E. Vancomycin

442
49. Supplementary pyridoxin may prophylaxis against CMV retinitis in
prevent neurotoxicity in persons who AIDS patient is
use this drug A. Fluconazole
A. Ciprofloxacin B. Gancyclovir
B. Isoniazid C. Indinavir
C. Linezolid D. Rifabutin
D. Clarithromycin E. Trimethoprim-sulfamethoxazole
E. Trimethoprim
55. Which of the following statements
50. Which statement regarding about interferon-alpha is false?
ethambutol is correct? A. At the start of treatment, most patients
A. It is contraindicated in pregnancy experience flu-like symptoms
B. Visual adverse effects are very rare B. Therapeutic outcome is low when
C. It inhibits arabinosyl transferase used alone for HCV
involved in cell wall biosynthesis C. It is used in the management of
D. Ocular toxicity of ethambutol is hepatitis B and C
prevented by thiamine D. Lamivudine interferes with its activity
E. Resistance is common and rapid. against hepatitis B
E. Toxicity includes bone marrow
51. Interactions between this drug and suppression
cell membrane components can result
in the formation of pores lined by 56. A 22-year-old man with gonorrhea
hydrophilic groups present in the drug is to be treated with cefixime and will
molecule. need another drug to provide coverage
A. Caspofungin for possible urethritis caused by
B. Flucytosine Clmydia trachomatis. Which of the
C. Griseofulvin following drugs is least likely to be
D. Nystatin effective in non-gonococcal urethritis?
E. Terbinafine A. Azithromycin
B. Ciprofloxacin
52. Which statement about fluconazole C. Co-trimoxazole
is accurate? D. Nitrofurantoin
A. Does not penetrate the blood-brain E. Tetracycline
barrier
B. Drug of choice in treatment of 57. The drug regimen most likely to be
aspergillosis effective in treating severe
C. Induces hepatic drug-metabolizing extraintestinal amebiasis is
enzymes A. Chloroquine
D. Has the least effect of all azoles on B. Diloxanide furoate plus iodoquinol
drug metabolism C. Emetine plus diloxanide furoate plus
E. Oral bioavailability is less than that of chloroquine
ketoconazole D. Chloroquine followed by primaquine
E. Tinidazole plus diloxanide furoate
53. The following is the drug of choice
for invasive aspergillosis 58. Metronidazole is not effective in the
A. Voriconazole treatment of
B. Ketoconazole A. Amebiasis
C. Miconazole B. Infections due to Bacteroides fragilis
D. Fluconazole C. Infections due to Pneumocystis carinii
E. Itraconazole D. Pseudomembranous colitis
E. Trichomoniasis
54. The drug most likely to suppress
herpetic infections and provide 59. Which statement about
antiprotozoal drugs is accurate?

443
A. Chloroquine is an inhibitor of 62. After successful treatment of
plasmodial dihydrofolate reductase malaria, which drug should be given
B. Mefloquine destroys secondary later to eradicate schizonts and latent
exoerythrocytic schizonts hypnozoites in the patient’s liver?
C. Primaquine is a blood schizonticide A. Artesunate
and does not affect secondary tissue B. Fansidar (Pyrimethamine-sulfadoxine)
schizonts C. Chloroquine
D. Artemisinin is not useful for P. D. Primaquine
falciparum malaria E. Quinine
E. Intravenous quinine can cause serious
arrhythmia
Answers
60. Plasmodial resistance to
chloroquine is due to: 1A 14 E 27 D 40 A 53 A
A. Change in receptor structure 2E 15 D 28 C 41 E 54 B
B. Decreased accumulation of the drug in 3A 16 A 29 D 42 D 55 D
the food vacuole 4B 17 B 30 E 43 B 56 D
C. Increased activity of DNA repair 5C 18 B 31 E 44 B 57 E
mechanisms 6C 19 D 32 A 45 D 58 C
D. Increased synthesis of dihydrofolate 7D 20 B 33 C 46 E 59 E
reductase 8A 21 D 34 B 47 E 60 B
E. Induction of drug-inactivating 9D 22 C 35 E 48 E 61 A
enzymes
10 A 23 A 36 C 49 B 62 D
11 B 24 D 37 E 50 C
61. Which drug should be used for
12 D 25 C 38 A 51 D
treatment of the acute attack of P vivax
13 A 26 D 39 B 52 D
malaria but does not eradicate
exoerythrocytic forms of the parasite?
A. Chloroquine
B. Mefloquine
C. Primaquine
D. Pyrimethamine-sulfadoxine
E. Quinidine

444

You might also like